Ailet Past Year Papers (2008 - 2020)

Download as pdf or txt
Download as pdf or txt
You are on page 1of 342

AILET Question Paper 2008

1. The Indian Territory which fell under the states on the eve of Independence was:
(A) Only 20 Percent (B) Over 80 percent (C) 54 percent (D) 46 percent

2. The Supreme Court of India enjoys:


(A) Only original jurisdiction (B) Only appellate jurisdiction
(C) Only advisory jurisdiction (D) All the above types of jurisdictions

3. A money bill can be introduced in Parliament by:


(A) The Finance Minister alone (B) Any minister
(C) Any member of Parliament (D) Any member of the ruling party

4. The most powerful legislature in the world is the:


(A) Indian Parliament (B) U.S. Congress
(C) Swiss Legislature (D) British Parliament

5. In the U.S.A., residuary powers or reserve of powers are:


(A) Left to the federal government (B) Left to the States
(C) Not defined properly (D) Given to local government

6. Members of Rajya Sabha are not associated with


1. Public Accounts Committee
2. Estimates Committee
3. Committee on Public Undertakings
(A) 1 and 2 (B) 3 (C) 2 (D) 1 and 3

7. According to the Constitution, amendments cannot be proposed in either House to


(A) The Annual Financial Statement (B) Appropriation Bill
(C) Demand for Grants (D) All of the above

8. Match the following:


(A) Nagpur session (1920) (B) Madras session
(C) Calcutta session (1928) (D) Lahore session (1929)
(E) Karachi session (1930)
(i) Poorna Swaraj
(ii) Changes in the Constitution of Congress
(III) Resolution of Fundamental Rights and National Economics Policy
(iv) Return of Gandhiji to active politics after 6 years
(v) Independence resolution passed for the first time
A B C D E
(A) iii i ii v iv
(B) i ii iv v iii
(C) ii v iv i iii
(D) i iii ii v iv

Previous Years
Page 212 CLAT & AILET Papers
9. .........are words, which appear innocent, but have a latent defamatory meaning
(A) Libel (B) Slander
(C) Innuendo (D) None of the above

10. Which article of our Constitution provides that the State shall make effective position for securing
right to work?
(A) Article 41 (B) Article 39
(C) Article 21 (D) Article 45

11. Who of the following does not constitute an exception for ‘equality before law’ (Article 14)?
(A) The Foreign Diplomats (B) The Prime Minister
(C) The Governor (D) The President

12. The Constitution of India Describes India as:


(A) A Quasi-Federal Country (B) Union of States
(C) A federation of States and Union territories (D) None of the above

13. Which one of the following Fundamental Rights has been the subject of maximum litigation and
controversies?
(A) Right to Property (B) Right to Equality
(C) Right to Freedom of Religion (D) Right to Freedom

14. The Fundamental Duties of the Indian Citizens:


(A) Formed a part of the original Constitution
(B) Were added by the Forty-Second Amendment
(C) Were added by the Forty-Fourth Amendment
(D) Were defined by the Parliament through a law enacted during the emergency

15. What is contained in the Eleventh Schedule to the Constitution of India?


(A) Forms of oath of affirmation
(B) Provisions regarding disqualification on grounds of defection
(C) Items in respect of which Panchayats shall have powers and authority to function as institutions
of Self-Government
(D) Items on which Municipalities shall have powers and authority to function as institutions of Self-
Government

16. The strength of the Council of Ministers:


(A) Has been fixed by the Constitution
(B) Has been fixed by the Parliament under Representation of People’s Act, 1950
(C) Is determined by the Prime Minister keeping in view the requirement of the time
(D) Is determined by the President

Previous Years
CLAT & AILET Papers Page 213
17. The advisory Powers of the Supreme Court of India imply that:
(A) It renders advice to the Government of India on all constitutional matters
(B) It renders advice to the Prime Minister on legal matters.
(C) IT renders advice to the President on question of law or fact which is of public importance
(D) It has power to render advice to all the above persons

18. The provision under which the Supreme Court can grant ‘special leave’ to appeal against decisions
of lower courts and tribunals is contained in:
(A) Section 10 of Supreme Court Rules (B) Article 136 of Indian Constitution
(C) Article 226 of Indian Constitution (D) Article 139 of Indian Constitution

19. Unliquidated damages mean


(A) Damage to something solid
(B) Damage caused by a firm which has gone in liquidation
(C) Damage to a firm in the hands of receivers
(D) Damage to be assessed by a Court as these are not pre-determined

20. Assault and nuisance are


(A) Wrong under Criminal law (B) Wrong under tort
(C) Wrong under neither (D) Wrong under both

21. ‘A’ by cutting the moorings of a boat in which a man and a women were sitting, caused in them a
fear for life. He is liable for
(A) Criminal use of force (B) Assault
(C) Mischief (D) None of these

22. The display of articles in a show room indicating their prices amounts to
(A) Offer (B) Counter offer
(C) Invitation to an offer (D) Mere advertisement

23. An idol of Lord Krishna in a temple is


(A) A natural person
(B) A legal person
(C) Not a person in the eye of law become only the priest of the temple will be a person
(D) Not a person because no personality can be conferred on God

24. When a court sends some one in judicial custody, what does it mean by judicial custody
(A) Sent to jail (B) Sent to police look up
(C) Sent to home under judicial protection (D) None of these

25. In Criminal law, intention is an essential constituent of an offence. In law of ‘Torts


(A) Intention is relevant (B) Intention is irrelevant
(C) Intention is relevant only in some torts (D) None of these

Previous Years
Page 214 CLAT & AILET Papers
26. The law forbids the number of partner is partnership. According to law, the maximum number of
people who can form a partnership firm is
(A)10 (B) 20
(C) 30 (D) No limit

27. The term of office of a judge of the International Court of Justice is


(A) 10 years (B) 9 years
(C) 7 years (D) 5 years

28. The democratic device, used in constitutional states, by which important political questions,
particularly question relating tot he states of a region, may be referred to the people of the region for
their final decision, is called
(A) Plebiscite (B) Referendum
(C) Recall Vote (D) Quorum

29. Ancient treatise on law?


(A) Vedas (B) Upanishads
(C) Smritis (D) Shrutis

30. Muslim religious foundations


(A) Ulema (B) Jihad
(C) Quzat (D) Wakfs

31. The maximum litigation in Courts is caused by


(A) businessmen (B) Middle class
(C) Government (D) Criminals

32. The first-regular adjudicatory mobile court in the country has been inaugurated at Punbanna village
in:
(A) Haryana (B) Bihar
(C) Andhra Pradesh (D) Punjab

33. Dissolution of Muslim Marriage Act, 1939 provides grounds of dissolution of Muslim marriage to
(A) Spouses (B) Only males
(C) Only females (D) Muslim parents and parents-in-law

34. A married man commits adultery if he has sexual intercourse with a/an
(A) Unmarried women (B) Married woman
(C) Any woman except his wife (D) Unmarried woman without her consent

35. A husband is obliged to maintain his divorced wife


(A) For ever (B) For 20 years
(C) Till the children grow up (D) Till she gets remarried

Previous Years
CLAT & AILET Papers Page 215
36. Match the following:
(i) Default (A) Failure to do something required by law
(ii) Delict (B) A wrongful act
(iii) Derelict (C) A thing thrown away by its owner
(iv) Dictum (D) A judge’s observation

(A) I-A, II- C, III-D, IV-B (B) I-D,II-C,III-B,IV-A


(C) I-C, II-D, III-A, IV-B (D) I-A,II-B,III-C,IV-D

37. You send your servant with a typewriter to be delivered to your friend. The servant takes the typewriter
home and uses it over a period of time. You discovered this a fortnight later and report him for
(A) Cheating (B) Embezzlement
(C) Fraud (D) Forgery

38. India and Britain have signed an ‘Extradition treaty’. Extradition means
(A) Exports with double taxation
(B) Order of Indian Courts will apply to Indians living in the UK
(C) Indian and the U.K. will deport criminals on reciprocal basis to each other
(D) None of above

39. Genocide occurred in all but one case


(A) Bosnia (B) Nazi Germany
(C) Rwanda (D) Gulf War

40. High sea is


(A) Sea lying more than 5 km beyond the coast of a country
(B) Sea lying beyond a distance of 3 miles from the coast of any country
(C) Sea beyond 200 km of a cost of any country
(D) Both (A) & (B)

41. Which Inn of Court called Gandhiji to the Bar?


(A) Inner Temple (B) Middle Temple
(C) Lincoln’s Inn (D) Gray’s Inn

DIRECTIONS: Answer the questions (Q. 42-50), which follow from application of the under-mentioned legal
principle:

42. PRINCIPLE:
Nothing is an offence which is done in the exercise of right of private defence. This right also extends
to lawfully causing the death of the assailant, if the offence which occasions the exercise of the right
of private defence, be –
(i) an assault which reasonably causes the apprehension of death or grievous hurt (very serious
hurt).
(ii) an assault which causes reasonable apprehension of rape.
(iii) an assault which causes reasonable apprehension of kidnapping This right is available for
protecting one’s own body, as well as the body of any other, provided the assault is not self
invited.

Previous Years
Page 216 CLAT & AILET Papers
Decide whether the right of private defence is available in the following situations.

FACTS:
A was holding the birthday party of his daughter. Some of his friends decided to present her with a
car. To create an element of surprise, they decided to kidnap her for few minutes while the party was
in full swing and then get her back in the new car. They put the plan into action. While they were
kidnapping, A got alarmed and asked the guards to open fire. The guards killed all the five friends. A
is
(A) Not protected.
(B) Protected because he did not kill, the guards did.
(C) Not protected because A’s own body was not involved.
(D) Protected because to any ordinary person there would have appeared a reasonable apprehension
of kidnapping.

43. PRINCIPLE:
When a contract has been broken, the party who suffers by such breach is entitled to receive, from
the party who has broken the contract, compensation for any loss or damage caused to him thereby,
which naturally arose in the usual course of things from such breach, or which the parties knew,
when they made the contract to be likely to result from the breach of it. Such compensation is not
given for any remote or indirect loss or damage sustained by reason of the breach.
Decide, whether and to what extent B is entitled to damages in the following situation.

FACTS:
A contracts with B to sell him 1,000 tons of iron at Rs.100/- per tone. B tells A that he needs the iron
for export purposes, and that he would be selling the iron at Rs. 200/- per tonne. A breaks the
contract. When the question comes about damages, A says he will pay only Rs. 5,000/- as damages
because the some variety of iron was available in the market at Rs. 105/- per tonne. B however
contends that he should be given Rs. 1,00,000/- because that was the profit which he would have
made had A fulfilled the contract. B had actually bought the iron at Rs. 110/- and had exported it. B
is
(A) not entitled to damages
(B) entitled to Rs. 10,000 as damages
(C) entitled to rs. 5.000 as damages
(D) entitled to Rs. 1,00,000 as damages

44. PRINCIPLE:
Contract is an agreement freely entered into between the parties.

FACTS:
Tapan was a dealer in mustard oil. The Government of India by an order issued under the Essential
Commodities Act, fixed the price of mustard oil, and also the quantity which a person can buy from
the dealer. Tapan carried on his business under this order for a while, but he refused to pay sales tax
on his sale transactions on the ground that these were not the contracts freely entered into by him.
(A) Tapan would succeed because the price and quantity were not negotiated by him.
(B) Tapan would not succeed because free consent between the parties was there despite the
restriction on price and quantity
(C) He would succeed because the Government under the new order forced him to enter into contracts
(D) Both (A) & (C)

Previous Years
CLAT & AILET Papers Page 217
45. PRINCIPLE:
An occupier is not normally liable to a trespasser except in respect of willful act intended to cause
him harm or done with reckless disregard.
FACTS:
Tony, a richman, had kept a ferocious dog to guard his house. He strictly instructed all his servants
not to go near the dog. Further a special attender was hired to take care of the dog. Visitors were
warned by a prominent warning sign board about this dog.
One day, a 13 year old boy playing in the neighborhood, running after his ball got into the house. The
dog attacked him and killed him. Tony was sued for damages.
(A) Tony was not liable because the boy was trespasser
(B) Tony is not liable because a 13 year old boy ought to have known about the presence of the
ferocious dog
(C) Tony is liable of the negligence of his servant to keep watch on such a ferocious dog during the
day time
(D) Both (A) and (B)

46. PRINCIPLE:
A master will be liable for the wrongful acts of his servants in the course of employment.

FACTS:
Maria was an old widow who opened an account with the Indian Overseas Bank, whereby she
should deposit Rs. 5/- everyday in the bank. Stephen was neighbour who used to collect the amount
and deposit them in the bank. Stephen would get a small commission from the bank for the money
deposited. One day it was discovered that Stephen who had not deposited the money for more than
three months, had vanished with the amount. Maria filed a suit against the Bank.

(A) Bank would not be liable because Stephen was not a employee of the bank.
(B) Bank would not be liable because Stephen was paid commission by the bank for doing its work
(C) Bank would be liable because Stephen was paid commission by the bank for doing its work
(D) None of the above

47. PRINCIPLE: Vishal, a famous artist was requested by Arun, an industrialist to draw the portrait of
his deceased wife and paid Rs. 20,000/- in advance and agreed to pay when the work was completed,
sum of Rs. 2 lakhs. When the portrait was half drawn, Vishal died due to heart attack. His son also
a fine artist completed his father’s work and demanded the money from Arun. Arun refused to pay
and to accept the portrait drawn by Vishal’s son and also demanded the advance to be returned.
(A) Arun cannot demand the advance amount from the son of Vishal, because in normal cases the
son is not responsible for the father’s contractual obligation
(B) Son of Vishal cannot demand the rest of amount from Arun because Arun has not assigned him
the work
(C) Arun can demand the rest of the amount
(D) Both (A) and (B)

Previous Years
Page 218 CLAT & AILET Papers
48. PRINCIPLE: Ignorance of law is not an excuse in India with the practice that every person in India
should be acquainted with the law of the land.

FACTS: Mr. Jackson, a foreigner, came to Delhi in the winter season. He saw the people in Delhi set
fire on road side during night and get their body warm. One night he came out of his hotel and asked
two labourers to cut down a dry tree in Pandara Road and when they agreed he paid them Rs. 150/
- Indian currency for cutting down the tree. They cut and made the long into small pieces and the
foreigner along with the tree cutters set fire and got their body warm. After some time, the Police
Patrol car watched it and arrested the foreigner and two labourers on the spot. The foreigner pleaded
that the tree was dry and he did not know the cutting of tree from road side is a offence in India.
(A) The foreigner shall not be responsible for the offence because he himself has not cut the tree but
got it cut through the Indian people
(B) The foreigner shall be punished because in India cutting of tree from road side is an offence
(C) The pretence that he does not know such act is an offence is not an excuse for a foreigner also
(D) Both (B) and (C)

49. PRINCIPLE:
The occupier of a premises owes a duty of care to all his invitees and visitors.
FACTS:
Devi who was the owner of a big home with a compound wall, constructed an underground tank to
store water. This was covered by jute bags since the work was incomplete. The postman who came
inside to deliver registered letter, fell into this tank and hurt himself. There was also a box on the
outside of the compound wall, where all the mail could be deposited. The injured man filed a suit
against Devi claiming compensation.
(A) Devi is not liable, because she did not invite the postman to her house
(B) Devi is not liable, because the postman could have delivered the letter in the box on the outside
of the compound wall
(C) Devi is not liable because the postman was required to take care of himself
(D) Devi is liable because the postman came into the premises in the course of his duty.

50. PRINCIPLE: Even if the sovereign functions of the State are discharged negligently the State is not
vicariously liable in tort.
FACTS:
A’ was a trader in gold. There he was arrested by Police and was detained in the police lock up after
search. The gold with him along with sundry other things were seized. Later he was discharged. His
possessions seized by the police were returned, except the gold. He moved against the State in
tort. In the words of Supreme Court, “There can be no escape from the conclusion that the Police
Officers were negligent in dealing with the property after it was seized.” One of the Constables was
a Muslim. He fled with gold to Pakistan.
(A) ‘A’ succeeded because the servants of the State were negligent and thus caused injury
(B) ‘A’ failed because the Constable who seized the gold had fled to Pakistan and the gold was not
with the State at all
(C) ‘A’ failed because the acts of search and seizure by the Police Office were part of the sovereign
function of the State.
(D) There was some other relief given to ‘A’

Previous Years
CLAT & AILET Papers Page 219
DIRECTIONS: For questions 51 and 52, select the pair of the words nearest in meaning to the original.

51. FRAYED: FABRIC


(A) Watered : Garden (B) Dilapidated : Building
(C) Frozen : Ice (D) Crumpled : Paper

52. LEVITATE: MAGICIAN


(A) Cook: Mother (B) Argue : Lawyer
(C) Float : Astronaut (D) Sky jump: Parachutists

Fill in the blank with the appropriate option:

53. Judicial decrees may not change the heart, but they can_____ the heartless.
(A) Transform (B) Subdue (C) Diarm (D) Regulate

DIRECTIONS: In the following questions 54 to 57, choose the word which is most nearly the SAME in
meaning to the word and mark it in the Answer Sheet.

54. Charlatan
(A) Monster (B) Pre-historic man
(C) Quack (D) An ex-convict

55. Cavil
(A) Frivolous objection (B) To criticize sharply; reprove
(C) Moving away from centre (D) Working of brain; thought process

56. Cynosure
(A) Able to coexist (B) Brief
(C) Centre of attention (D) Abridgement

57. Consummate
(A) One of zenith of perfection (B) Existing from birth
(C) Constant; always present (D) A history

DIRECTIONS: In the following questions 58 to 62: choose the word which is most nearly the OPPOSITE
in meaning to the word and mark it in the Answer sheet.

58. Equanimity
(A) Resentment (B) Dubiousness (C) Duplicity (D) Excitement

59. Palliate
(A) Apologise (B) Hesitate (C) Wait impatiently (D) Cure completely

60. Obsequious
(A) Brusque (B) Quick-witted (C) Sharp-tongued (D) Luxurious

61. Obstreperous
(A) Critical (B) Unruly (C) Unpleasing (D) Weak

Previous Years
Page 220 CLAT & AILET Papers
62. Endemic
(A) Decorative (B) Frustrating (C) Terrorising (D) Universal

DIRECTIONS: Pick the correct answer choice for filing the blanks in each of the following sentences.

63. I was surprised............


(A) At his letting out the secret (B) Why he let out the secret
(C) That he let out the secret (D) By his letting out the secret

64. It is not possible to............


(A) Mix oil in water (B) Mix oil into water
(C) Mix oil with water (D) Mix oil by water

65. He............and escaped


(A) Forced the door open (B) Forced open the door
(C) Open forced the door (D) The door forced open

66. Let.....finish this work as soon as possible


(A) You and I (B) You and me
(C) I and you (D) Me and you

67. ............the room, the door hit him on the brow.


(A) While entering (B) As he entered
(B) On entering (D) On having entered

DIRECTIONS: For the questions 68 to 72, some of the sentences have errors and some have none. Find
out which part of the sentence has an error.

68. He returned back / when he sensed / that there was danger. / No error
A B C D

69. The progress of those plants / is not being very encouraging / and they are in various stage of decay. /
A B C
No error.
D

70. Not only the bandits robbed / the traveller of his purse / but also wounded him grievously. / No error.
A B C D

71. No less than twenty persons / were killed in / the air crash. / No error.
A B C D

72. The police went through the building / with fine toothcombs / but they found no evidence / No error.
A B C D

Previous Years
CLAT & AILET Papers Page 221
DIRECTIONS: For the questions (Q. 73-75) the first and the last sentence of the paragraph are given.
Other sentences are given in jumbled form. You have to arrange the sentences to form a readable passage.

73. (1) Some people seem to have a compulsive need to work continually, they may be called work-
addicts.
(M) However they never quite reach the top.
(N) There are companies which expect everyone to be at work early and stay back late.
(O) Because of their diligence they tend to get rapid promotion at first.
(P) Work addicts generally end up in such organisations.
(6) [in the original paper sentence 6 was missing]
The correct sequence should be:
(A) PNMO (B) MOPN (C) OMNP (D) NPOM

74. (1) If you feed a dog or tame a bear by hand


(P) or until they succeed in getting it
(Q) and tear and pull at it
(R) they get their teeth into the meat
(S) until they bite a piece off
(6) all out of your hand.
The correct sequence should be:
(A) PSRQ (B) SQRP (C) RPSQ (D) RQSP

75. (1) St. Francis taught that


(P) and that it was easier to be good
(Q) if men would be happy
(R) they must be good
(S) and happy if one were poor
(6) and did not trouble oneself about money
The correct sequence should be:
(A) PQRS (B) RPSQ (C) QRPS (D) PSQR

76. In France, children in pre-school programs spend a portion of each day engaged in a program of
stretching and exercise. Pre-school programs in the United States, however, seldom devote time to
a daily stretching and exercise program. In tests designed to measure cardiovascular fitness, children
in the United States were out-performed by their French counterparts. It can therefore be determined
that children attending pre-school programs in the United States can achieve cardiovascular fitness
only by engaging in a daily school program of stretching and exercise.

Which one of the following is an assumption on which the argument depends?


(A) A daily program of stretching and exercise will allow all children to achieve cardiovascular fitness.
(B) Cardiovascular fitness is integral to one’s overall health.
(C) It has been proven that children who participate in stretching and exercise programs in pre-
school have better cardiovascular fitness than adults.
(D) Stretching and exercise are necessary components of French children’s superior cardiovascular
fitness programs.

Previous Years
Page 222 CLAT & AILET Papers
77. The level of blood sugar for many patients suffering from disease Q is slightly higher than the level of
blood sugar in the general population. Nonetheless, most medical professionals believe that slightly
increasing blood sugar levels is a successful means by which to treat disease Q.

This apparently contradictory argument can best be resolved by which one of the following statements?
(A) Blood sugar levels for patients who have been cured of disease Q are virtually identical to the
level of blood sugar found in the general population.
(B) Many of the symptoms associated with severe cases of disease Q have been recognized in
laboratory animals with experimentally induced high blood pressure, but none of the animals
developed disease Q.
(C) The movement from inactive to advanced states of disease Q often occurs because the virus that
causes Q flourishes during periods when blood sugar levels are slightly low.
(D) The blood sugar level in patients with disease Q fluctuates abnormally in response to changes
in blood chemistry.

78. Medical studies indicate that the metabolic rates of professional athletes are substantially greater
than those of the average person. So, most likely, a person’s speed and strength are primarily
determined by that person’s metabolic rate.

Which one of the following, if true, most strengthens the argument?


(A) Some professional athletes are either faster or stronger than the average person.
(B) Some professional athletes do not have higher metabolic rates than some people who are not
professional athletes.
(C) The speed and strength of people who are not professional athletes are not primarily determined
by choices of diet and exercise.
(D) Drugs that suppress metabolic rates have been shown to have the side-effect of diminishing the
speed and strength of those who are not professional athletes.

79. Since Oscar received extensive training in how to repair motorcycles, he is able to repair many of
their most common mechanical problems. However, Oscar does not understand how internal
combustion engines work. When Oscar was given Lucy’s motorcycle to repair, he was able to fix
the problem, despite the fact that he did not understand what was causing it.

From the above paragraph, which of the following can be properly inferred?
(A) The problem with Lucy’s motorcycle involved its engine
(B) Not all mechanical problems can be repaired only by mechanics who understand how an internal
combustion engine works
(C) At least some good mechanics are able to fix mechanical problems without an understanding of
what is causing them
(D) Oscar’s mechanical training in how to repair motorcycles was incomplete

Previous Years
CLAT & AILET Papers Page 223
80. It cannot be true that the lack of success of third-party candidates in national elections is due to the
difficulties such candidates encounter in securing space on national ballots. Everyone who identifies
him or herself as a supporter of a third party has voted for a major-party candidate in at least one
national election when a third-party candidate was listed on the ballot.

Which one of the following most accurately describes a reasoning flaw in the argument?
(A) The argument overlooks the possibility that the lack of success of third-party candidates in
national elections may be due to the fact that their views on major issues prevent them from
gaining broad support
(B) The argument takes for granted that the media coverage devoted to third-party candidates for
national office is comparable to that devoted to major-party candidates for those same offices
(C) The argument treats as contradictory to some claim evidence that may instead provide support
for that claim
(D) The argument draws its conclusion through the use of a set of facts, not all of which can be true

81. The petrol prices shot up by 7% as a result of the hike in the price of crudes. The price of petrol
before the hike was Rs. 28 per litre. Vawal travels 2400 kilometers every month and his car gives a
mileage of 18 kilometers to a litre. Find the increase in the expenditure that Vawal has to incur due
to the increase in the price of petrol (to the nearest rupee)?
(A) Rs. 270 (B) Rs. 262 (C) Rs. 276 (D) Rs. 272

82. The cost of setting up the type of a magazine is Rs. 1000. the cost of running the printing machine
is Rs. 120 per 100 copies, the cost of paper, ink and so on is 60 paise per copy. The magazines are
sold at Rs. 2.75 each. 900 copies are printed, but only 784 copies are sold. What is the sum to be
obtained from advertisements to give a profit of 10% on the cost?
(A) Rs. 730 (B) Rs. 720 (C) Rs. 726 (D) Rs. 736

83. A sum is divided between A and B in the ratio of 1 : 2. A purchased a car from his part, which
depreciates per annum and B deposited his amount in a bank, which pays him 20% interest per
annum compounded annually. By what percentage will the total sum of money increase after two
years due to this investment pattern (approximately).
(A) 20% (B) 26.66% (C) 30% (D) 25%

84. Two cubes of bronze have their total weight equivalent to 60 kg. The first piece contains 10 kg of pure
zinc and the second piece contains 8 kg of pure zinc. What is the percentage of zinc in the first
piece of bronze if the second piece contains 15 per cent more zinc than the first?
(A) 15% (B) 25% (C) 55% (D) 24%

85. An ant moved for several seconds and covered 3 mm in the first second and 4 mm more in each
successive second than in its predecessor. If the ant had covered 1 mm in first second and 8 m
more in each successive second, then the difference between the path it would cover during the
same time and the actual path would be more than 6 mm but less than 30 mm. Find the time for
which the ant moved (in seconds)
(A) 5 s (B) 4 s (C) 6 s (D) None of these

Previous Years
Page 224 CLAT & AILET Papers
86. The number of natural numbers of two or more than two digits in which digits from left to right are in
increasing order is
(A) 127 (B) 128 (C) 502 (D) 501

87. The Bubna dam has four inlets. Through the first three inlets, the dam can be filled in 12 minutes;
through the second, the third and the fourth inlet, it can be filled in 15 minutes; and through the first
and the first inlet, in 20 minutes. How much time will it take all the four inlets to fill up the dam?
(A) 8 min (B) 10 min (C) 12 min (D) None of these

88. Find the sum of all two-digit numbers that give a remainder of 3 when they are divided by 7.
(A) 686 (B) 676 (C) 666 (D) 656

89. A train travels with a speed of 20 m/s in the first 10 minutes, goes 8.5 kms in the next 10 minutes,
11 kms in the next 10 minutes, 8.5 kms in the next 10 and 6 kms in the next 10 minutes. What is
the average speed of the train in kilometer per hour for the journey described?
(A) 42 kmph (B) 35.8 kmph (C) 55.2 kmph (D) 46 kmph

90. Two solutions of 90% and 97% purity are mixed resulting in 21 litres of mixture of 94% purity. How
much is the quantity of the first solution in the resulting mixture?
(A) 15 liters (B) 12 litres (C) 9 litres (D) 6 litres

DIRECTIONS: For questions (Q. 91-94), some information is provided in the form of statements. On the
basis of that information find the answer to the questions which follow.
Coach Balkishen is trying to put together a team of four players for a tennis tournament.
He has seven players available: males A, B and C and the females M, N, O and P. All players are of equal
ability and there must be at least two males on the team. For a team of four, all players must be able to
play with each other player.
Player B cannot play with player M
Player C cannot play with player P
Player M cannot play with player O

91. If player O is selected and player B is rejected the team will consist of which foursome?
(A) A, C, M and O (B) A, C, N and O
(C) A, C, P and O (D) A, N, P and O

92. If player M is on the team, what other players must be on the team as well?
(A) A, B and N (B) A, C and N
(C) A, C and O (D) A, C and P

93. What statement is false?


(M) Player B and C are never selected together
(N) Player C and O are never selected together
(O) Player C and N are never selected together

(A) Only M (B) Only N


(C) Only O (D) All the three

Previous Years
CLAT & AILET Papers Page 225
94. Which statement must always be true?
(M) If M plays, A plays
(N) If O plays, B plays
(O) If M plays, O plays

(A) Only M (B) Only N


(C) Only O (D) Only M and O

95. The door of my house is towards east direction. From the backside of my house I drove straight 100
metres, then turned towards right and drove for 100 metres, then turned towards left and drove for 50
metres and reached my destination. In what direction am I from the starting point?
(A) North-west (B) West
(C) North (D) East

DIRECTIONS: Each interrogative statement (Qs. 96-97) is followed by two arguments. You are to pick up
one of the following responses as the answer.

96. Should divorce laws be made more favourable to women in India?


(X) Yes, because women are persecuted by their women in India.
(Y) No, because it will disrupt family life.
(A) Argument ‘X’ is forceful (B) Argument ‘Y’ is forceful
(C) Neither ‘X’ nor ‘Y’ is forceful (D) Both ‘X’ and ‘Y’ are forceful

97. Is leisure irrelevant to modern life?


(M) Yes, because life has become too much mechanical today.
(N) No, because without leisure efficiency will fall, body and brain must be given rest.
(A) Only argument ‘M’ is forceful (B) Only argument ‘N’ is forceful
(C) Both ‘M’ and ‘N’ is forceful (D) Neither ‘M’ nor ‘N’ is forceful

98. The island of Bombay was acquired by the East India Company from
(A) Netherlands (B) France
(C) Portugal (D) Spain

99. The outstanding achievement of Todar Mal was in the field of


(A) Military Conquests (B) Revenue Administration
(C) Religious Reforms (D) Art and Architecture

100. The deity most praised in Rigveda is


(A) Indra (B) Agni
(C) Mitra (D) Varuna

101. The Peacock throne of Shahjahan was taken away by


(a) Changez Khan (B) Timur
(C) Nadir Shah (D) Ahmad Shah Abdali

Previous Years
Page 226 CLAT & AILET Papers
102. Shivaji’s Council of Ministers was called
(A) Nava Patnas (B) Ashta Pradhans
(C) Ashta Diggajas (D) Mantri Parishad
(NLUD 2008)
103. Charak was the court physician of
(A) Asoka (B) Kanishka
(C) Chandragupta Maurya (D) Samudragupta

104. Which of the following reasons was mainly responsible for the unpopularity of Muhammad-bin-
Tughlaq?
(A) Bad luck (B) Lack of resources
(C) Subordination of the theologians (D) Transfer of capital from Delhi to Devagiri

105. The empire of the ‘Satavahana kings’ is known by which of the following names at present?
(A) Andhra Pradesh (B) Punjab
(C) Gujarat (D) Uttar Pradesh

106. The High Courts at Calcutta, Madras and Bombay were established under the
(A) Indian High Courts Act, 1911 (B) Government of India Act, 1909
(C) Indian High Courts Act, 1861 (D) Indian High Courts Act, 1865

107. Which of the following rulers was illiterate?


(A) Ala-ud-din Khilji (B) Mohammad-Bin-Tuglak
(C) Akbar (D) Aurangzeb

108. If the original clay sediments are subjected to high temperature and pressure, they successfully
change into rocks, the correct order of which is
(A) Slate, Shale, Schist (B) Slate, Schist, Shale
(C) Shale, State, Schist (D) Shale, Schist, State

109. Exfoliation in granite is regarded as a process of


(A) Chemical weathering (B) Mechanical weathering
(C) Faulting (D) Folding

110. The tree species most commonly used in social forestry is


(A) Gulmohar (B) Mango (C) Pipal (D) Eucalyptus

111. The Hindustan-Tibet road connects


(A) Kalimpong with Lhasa (B) Shimla with Gangtok
(C) Leh and Lhasa (D) Gangtok with Gyangyse

112. The Aswan Dam is constructed at a point where


(A) There are several waterfalls
(B) There are several cataracts
(C) There are several pigeon towers to facilitate the collection of droppings needed to keep the land
productive
(D) The old traditional basic irrigation facility exists

Previous Years
CLAT & AILET Papers Page 227
113. The difference in time per degree longitude between any two places on globe is
(A) 4 minutes (B) 5 minutes
(C) 15 minutes (D) 30 minutes

114. The limit beyond which stars suffer internal collapse is called
(A) Chandrasekher limit (B) Eddington limit
(C) Hoyle limit (D) Fowler limit

115. The best anti-knock compound used in petrol to increase mileage is


(A) Ethyl-magnesium chloride (B) Sodium ethoxide
(C) Zinc ethyl (D) Tetraethyl lead

116. Rainbow is produced when sunlight falls on drops of rain. Which of the following physical phenomena
are responsible for this?
(I) Dispersion
(II) Refraction
(III) Internal Reflection

(A) II and III only (B) I and II only


(C) I, II and III (D) I and III only

117. Which of the following bombs, when dropped on a city, will kill only the inhabitants?
(A) Atom Bomb (B) Hydrogen Bomb
(C) Chemical Bomb (D) Neutron Bomb

118. Which of the following will you use to remove rust stains on cloth?
(A) Alcohol (B) Oxalic acid solution
(C) Kerosene oil (D) Lime

119. “Mach Number” is a term associated with the speed of


(A) Ships (B) Cars
(C) Aeroplanes (D) Light

120. If an object is placed midway between two parallel plane mirrors facing each other, then the number
of images that will appear in mirrors is
(A) Four (B) Infinite
(C) Two (D) Zero because the images will cancel each other

121. The spherical shape of rain drops is due to


(A) Atmospheric friction of air (B) Gravity of spherical earth
(C) Surface tension of rain water (D) Viscosity of rain water

122. What is the colour of pure diamond?


(A) Light green (B) Light yellow
(C) Light blue (D) Light pink

Previous Years
Page 228 CLAT & AILET Papers
123. Geiger counter is an instrument
(A) Determine the heart beat rate (B) Two detect radioactive radiation
(C) To measure intensity of visible light (D) To count the baggage of an airport

124. The film of oil and soapy water owe their brilliant colours to a combination of light reflection and
(A) Refraction (B) Polarisation
(C) Diffraction (D) Interference

125. When a ship enters a sea from a river, its portion under water will
(A) Increase (B) Decrease
(C) Show no change (D) Increase or decrease alternatively

126. For digestion of food, hydrochloric acid is secreted into stomach at a pH value of
(A) 2 (B) 4
(C) 6 (D) 18

127. Which of the following diseases is genetically linked?


(A) Epilepsy (B) Diabetes
(C) Colour blindness (D) Leucoderma

128. An eye defect in which one cannot distinguish between vertical and horizontal lines is called
(A) Myopia (B) Astigmatism
(C) Trachoma (D) Glaucoma

129. The gland which is attached to the digestive system but does not have any role to play in digesting
food is
(A) Salivary gland (B) Liver
(C) Spleen (D) Pancreas

130. A reptile with a four-chambered heart is


(A) Snake (B) Lizard
(C) Crocodile (D) Turtle

131. Ahmedabad-based Pioma Industries is best known for which brand?


(A) Maggi (B) Kissan
(C) Brooke-Bond (D) Rasna

132. Australian cricketer Brett Lee models for which watch company?
(A) Timex (B) HMT
(C) Titan (D) All of these

133. Sunita Williams, renowned astronaut of Indian origin, spent a record _____ days in space.
(A) 195 (B) 185
(C) 200 (D) 201

Previous Years
CLAT & AILET Papers Page 229
134. ‘Bullion’ refers to gold, silver or other precious metals in the form of
(A) Ingots or bars only (B) Bars as well as coins
(C) Coins only (D) Jewellery

135. What does “R” in Maruti Suzuki’s small car WagonR stand for?
(A) Responsible (B) Remarkable
(C) Reliable (D) Recreation

136. Which of the following took over the Anglo-Dutch steel Company Corus?
(A) Arcelor Mittal (B) Jindal Steel
(C) Neelanchal Ispat Nigam Ltd. (D) Tata Steel

137. What is Phishing?


(A) A Government Plan
(B) A stem cell
(C) Fradulent way of acquiring PIN and bank passwords using e-mail
(D) None of these

138. Which organisation is headed by Indian environmentalist R.K. Pachauri who has been awarded the
Nobel Peace Prize, 2007 along with Al Gore?
(A) International Panel on Climate Change (IPCC)
(B) International Pollution Control Board
(C) International Environment Panel
(D) International Panel on Global Warming

139. The Government of India recently decided to halt the Future Trading of which of the following
commodities for the time being?
(A) Jeera (B) Soya Oil
(C) Wheat (D) Gram

140. Which game will be played for the first time in Asian Games, 2010?
(A) Kabaddi (B) Twenty20 Cricket
(C) Squash (D) Kho-Kho

141. Who is the Managing Director of the Delhi Metro Railway Corporation?
(A) B.L. Joshi, Lt. Governor of Delhi (B) Sam Pitroda
(C) Chairman of the Indian Railway Board (D) Shri. E. Sreedharan

142. Which country has recently adopted its new National Anthem?
(A) Israel (B) Bangladesh
(C) Indonesia (D) Nepal

rd
143. Which among the following films has got the ‘Swarna Kamal Award’ for Best Feature Film in 53
National Film Awards?
(A) Parineeta (B) Sringarm
(C) Iqbal (D) Kaalpurush

Previous Years
Page 230 CLAT & AILET Papers
144. Which of the following is/ are included in the UNESCO’s natural heritage site in India?
(1) Keoladeo (2) Kaziranga National Park
(3) Manag Tiger Project` (4) Nanda Devi Biosphere Reserve

(A) 1 and 2 (B) 2 and 3


(C) 3 and 4 (D) All

145. Which of the following is India’s indigenous helicopter weaponised version made its first flight recently?
(A) Dhruv (B) Cheetah
(C) Chetak (D) Rajhans

DIRECTIONS: Passage in this section is followed by a group of questions (Qs. 146-150) to be answered
on the basis of what is stated or implied in the passage. For some questions, more than one of the choices
could conceivably answer the question. However, you are to choose the best answer, that is, the response
that most accurately and completely answers the question and blacken the corresponding space on your
answer sheet.

One of the most prolific authors of all time, Isaac Asimov was influential both in science fiction and in the
popularization of science during the twentieth century, but he is also justly famous for the scope of his
interests. Although the common claim that Asimov is the only author to have written a book in every
category of the Dewey decimal system is untrue, its spirit provides an accurate picture of the man: a
dedicated humanist who lauded the far-reaching power of reason. His most famous work, the Foundation
trilogy, can be read as an illustration of Asimov’s belief in reason and science, but even while he expressed
that belief, science itself was calling it into question.

Foundation describes a time in which a vast Empire spanning the galaxy is on the verge of collapse. Its
inevitable doom is a consequence not of its size, but of the shortsightedness of its leaders. In this environment,
a scientist named Hari Seldon devises an all encompassing plan to help human civilization recover from
the trauma of the Empire’s coming collapse. Using mathematics, Seldon is able to predict the future
course of history for thousands of years, and he takes steps that are geared toward guiding that future in
a beneficial direction. The trope of the benevolent and paternalistic scientist shaping existence from behind
the scenes, present in much of Asimov’s fiction, is never more explicit than in the Foundation series, which
describes with an epic sweep the course and progress of the Seldon Plan.

As naive and, perhaps, self-serving as the conceit of Foundation may seem to contemporary readers, it
retains to some degree its ability of comfort by offering an antidote to the complex and unpredictable nature
of experience. Science in Asimov’s time was, in popular conceptions, engaged in just this pursuit: discerning
immutable laws that operate beneath a surface appearance of contingency, inexplicability, and change.
But even while Asimov wrote, science itself was changing. In Physics, the study of matter at the subatomic
level showed that indeterminacy was not a transitory difficulty to be overcome, but an essential physical
principle. In Biology, the sense of evolution as a steady progress toward better adapted forms was being
disturbed by proof of a past large-scale evolution taking place in brief explosions, of frantic change. At the
time of Asimov’s death, even Mathematics was gaining popular notice for its interest in chaos and
inexplicability. Usually summarized in terms of the so-called ‘butterfly effect’, chaos theory showed that
perfect prediction could take place only on the basis of perfect information, which was by nature impossible
to obtain. Science had dispensed with the very assumptions that motivated Asimov’s idealization of it in
the Seldon Plan. Indeed, it was possible to see chaos at work in Foundation itself: as sequels multiplied

Previous Years
CLAT & AILET Papers Page 231
and began to be tied into narrative threads from Asimov’s other novels, the urge to weave one grand
narrative spawned myriad internal inconsistencies that were never resolved.

146. Which one of the following most accurately expresses the main point of the passage?
(A) Isaac Asimov’s greatest work, the Foundation trilogy, is an expression of the common trope of
the benevolent and paternalistic scientist
(B) Popularizations of science are always to some degree dependent idealizations and simplifications
of that science, as Isaac Asimov’s work demonstrates
(C) The impossibility of the conceit on which Isaac Asimov’s Foundation trilogy is based demonstrates
that Asimov’s fiction was based on imperfect understandings of science.
(D) Isaac Asimov’s idealization of science as revealed in his Foundation series was called into
question by the science of his time, which was increasingly focused on Chaos and indeterminacy

147. Which one of the following statements most accurately expresses the purpose of the final paragraph?
(A) The ultimate failure of the Foundation series as a coherent scientific narrative is discussed
(B) A claim is made about the purpose of Asimov’s writing and then is finally rejected
(C) A key theme of Asimov’s Foundation series is described and discoveries in science that seem
contrary to that theme are outlined
(D) The history of science is used to demonstrate the falsity of a widely believed claim about the
power of human reason

148. The author’s reference to a common claim made about Isaac Asimov, serves to
(A) Demonstrate that many untrue beliefs are held about him
(B) Illustrate the broad scope of his interests and writings
(C) Undermine the claim that he was prolific writer
(D) Substantiate his belief in the power of human reason

149. With respect of the Seldon Plan, the author’s attitude can most properly be described as
(A) Amused at the naive conception of history it implies
(B) Uncertain of the practical impossibility of its application
(C) Ambivalent because of the reliance on human reason it requires
(D) Convinced that it illustrates Asimov’s attitude toward science

150. Which one of the following statements best illustrates the “butterfly effect” as it is described in the
passage’s third paragraph?
(A) A system implemented to predict the weather worldwide for the next century is soon found to be
inaccurate because it was supplied with the incomplete data
(B) Efforts to predict the result of a nuclear reaction fail because of indeterminacy inherent in the
behavior of subatomic particles
(C) The fossil record indicates that certain adaptations found in many organisms appeared soon
after a past catastrophic event
(D) Scientific predictions about the future course of human history are found to be reasonably
accurate once existing social theories are reconciled.

Previous Years
Page 232 CLAT & AILET Papers
AILET Question Paper 2009

Directions: Choose the most appropriate word:

1. He is ___________ of spelling reforms.


(A) a protagonist (B) an advocate (C) an envoy (D) a champion

2. Negotiable : cheque::
(A) Frozen : Asset (B) Inventory : Merchandise
(C) Bank : Money (D) Trade : Tariff

3. Hedger : Shrubbery :: ? : Stick


(A) snuffer (B) cougher (C) Whittler (D) stickler

4. Honour : Governor :: ? : Duke


(A) Excellency (B) Majesty (C) Highness (D) Grace

Directions: The sentences in the middle of a passage have been removed. You are provided with the
beginning and the end of the passage and the other sentences in a jumbled order. You are to choose the
correct order that will make the passage complete and coherent.

5. 1. When Mrs. Bates came down, the room was strangely empty, with a tension of expectancy.
P: Meantime her anger was turned pale with fear.
Q: She took up her sewing and stitched for some time without raising her head.
R: She rushed to the stair door and opened it, listening.
S: The clock struck eight and she rose suddenly, dropping her sewing on the chair.
6. Then she went out, locking the door behind her.
(A) S Q P R (B) Q P S R (C) S R Q P (D) P R S Q

6. 1. Above all.
P: in the present age of light reading
Q: it is well if something heavier is cast now and then
R: of reading hastily and thoughtlessly
S: that is,
6. into the midst of the reading public.
(A) P Q R S (B) P S R Q (C) P R QS (D) Q S R P

Directions: In the following questions, choose the word which is most nearly the OPPOSITE in meaning
to the bold word and mark it in the Answer Sheet.

7. Ductile
(A) virtuous (B) grotesque (C) contentious (D) stubborn

8. The Prime Minister’s radio broadcast galvanised the people’s spirit


(A) destructed (B) frightened (C) distracted (D) dampened

Previous Years
CLAT & AILET Papers Page 233
Directions: In the following questions, choose the word which is most nearly the SAME in meaning to the
bold word and mark it in the Answer Sheet.

9. Obsequious
(A) poor (B) cheerful (C) servile (D) sullen

10. Dialectic
(A) argumentative (B) instructive (C) constructive (C) destructive

Directions: In the following questions, some of the sentences have errors and some have non. Find out
which part of the sentence has an error.

11. He not only denied / having borrowed money from me, / but also having ever met me. / No error.
(A) (B) (C) (D)

12. The running party stood / for implementation of the Bill / and was ready to stake their political
(A) (B) (C)
existence. / No error.
(D)

13. If I were he, / I should not / accept the post. / No error


(A) (B) (C) (D)

Directions: In the question a paragraph with many numbered blank spaces is given. In case of all spaces,
some phrases for filling up, are suggested. The candidate is to pick up the correct response.

____ 14 ____ evening we decided to ____ 15 ____camp as the weather was not encouraging. The wind
was high and - 16 - storm clouds ____ 17 ____ a wild wet night. Moreover we ____ 18 ____ at a spot which
looked ____ 19 ____ for a camp. A level ____ 20 ____ in the lee of a high hill ____ 21 ____some shelter
from the wind; fresh water was near ____ 22 ____ in a storm which flowed across the plain; a ____ 23 ____
of trees provided adequate supplies of fuel; and the dry grass which ____ 24 ____ on the hill side would
enhance the comfort of our beds. Each member of the part ____ 25 ____ a task. Some erected the tens;
others prepared a scanty meal; yet others ____ 26 ____ to the needs of the ponies, now exhausted after
a very strenuous day. As the angry sun sank, the bustle of activity was hushed into silence and each man
____ 27 ____ down to sleep.

14. (A) A the (B) After the (C) In the (D) Before

15. (A) dig (B) fixed (C) plant (D) pitch

16. (A) collecting (B) gathering (C) accumulating (D) moving

17. (A) preceded (B) proceeded (C) predicted (D) proved

18. (A) arrived (B) have arrived (C) would arrive (D) had arrived

19. (A) promise (B) promiseful (C) promising (D) prosperous

20. (A) expense (B) land (C) expanse (D) piece of land

Previous Years
Page 234 CLAT & AILET Papers
21. (A) gave (B) afforded (C) supplied (D) cast

22. (A) in hand (B) on hand (C) at hand (D) of hand

23. (A) group (B) cluster (C) collection (D) grove

24. (A) abundant (B) luxuriant (C) abounded (D) abounded in

25. (A) allotted (B) was allotted (C) had allotted (D) has allotted

26. (A) will attend (B) had attended (C) attended (D) would have attended

27. (A) settled (B) settles (C) laid (D) lay

28. At a particular speed, a bus starts vibrating violently due to the phenomenon of
(A) Pitch (B) Resonance (C) Rumbling (D) None of the above

29. When a bullet is fired upward vertically, it gains in


(A) Speed (B) Acceleration (C) Kinetic energy (D) Potential energy

30. The metal that is most abundant in the earth is


(A) Silicon (B) Iron (C) Aluminium (D) Nickel

31. A motorcycle passing by disturbs radio and TV reception. The disturbance is caused because
(A) Intense sound waves shake the delicate electronic components of the receiver
(B) Metallic parts of the vehicle deflect the radio waves
(C) The sparking in the spark-plug of the vehicle generates electromagnetic radiation
(D) Modern motorcycle contains an electronic ignition system which emits ratio waves

32. Match the following scientists with their contributions


(A) Weismann I. Theory of Mutation
(B) Darwin II. Principle of Independent Assortment
(C) Mendal III. Theory of Evolution
(D) Hugo-de-varis IV. Continuity of Germplasm

(A) I - (A) II - (B) III - (C) IV - (D)


(B) I - (B) II - (C) III - (D) IV - (A)
(C) I - (D) II - (C) III - (A) IV - (B)
(D) I - (D) II - (C) III - (B) IV - (A)

33. Who was the first Sultan of Delhi to introduce the practice of ‘Sijda’?
(A) Balban (B) Muhammad Tughlaq
(C) Alauddin Khilji (D) Firoz Tughlaq

34. Which of the following is rightly regarded as a milestone in the field of education in India?
(A) Sir Charles Wood’s Despatch (B) Stanley’s Despatch
(C) Hunter Commission Report (D) University Commission

Previous Years
CLAT & AILET Papers Page 235
35. A Public Works Department was set up in India by
(A) Lord Dalhousie (B) Lord Ripon
(C) Lord Warren Hastings (D) Lord William Bentinck

36. The Theory of Economics Drain from India to England was propounded by
(A) R. C. Dutt (B) B. G. Tilak (C) Dadabhai Naoroji (D) L. K. Jha

37. The Calcutta session of the Indian National Congress held in September 1920, passed a resolution
which led to the
(A) Non-Cooperation Movement (B) Civil Disobedience Movement
(C) Home Rule Movement (D) Quit India Movement

38. A cyclone is a system of wind in which the wind blows spirally


(A) towards the centre of low pressure
(B) towards central region of high pressure
(C) towards a region of low pressure
(D) outwards from a central region of high pressure

39. Contour bunding is used


(A) to stop the winds in sandy deserts (B) to irrigate desert areas
(C) to prevent erosion in hilly areas (D) None of the above

40. Which of the following is not a closed sea?


(A) Caspian sea (B) Aral sea
(C) Black sea (D) Red sea

41. If Greenwich Mean Time is ahead by 12 hours, the place may be (10 = 4 minutes)
(A) 1800 West (B) 1800 East
(C) 900 West (D) None of the above

42. The orbit of the earth is an ellipse and not a circle. The distance between earth and sun thus varies.
On January 3, earth is closest to the sun (Perihelion). Similarly, earth is said to be at Aphelion,
when it is farthest from the sun on
(A) March, 23 (B) July, 4
(C) December, 23 (D) April, 21

43. What is the meaning of ‘Gilt edged market’?


(A) Market in Government securities (B) Market in smuggled goods
(C) Market of auctioned goods (D) None of the above

44. “Reduction in rate of taxation leads to more than proportionate increase in tax yield”. This law is
known as
(A) Giffins Effect (B) Laffer Effect (C) Gresham’s Law (D) None of the above

45. The VDIS-Voluntary Disclosure of Income Scheme was the brainchild of


(A) P. Chidambaram (B) Ram Jethmalani
(C) Atal Behari Vajpayee (D) Sonia Gandhi

Previous Years
Page 236 CLAT & AILET Papers
46. A company is said to be ‘Sick’ when the accumulated loss at the end of any financial year leads to
erosion of ...... percent of its net wealth.
(A) 100% (B) 75% (C) 50% (D) 25%

47. Gunnar Myradal has dealt with the problem of Asian countries in her book ASIAN DRAMA ‘regarding
(A) poverty (B) modern industries
(C) military dictatorship (D) neo-colonialism

48. Wealth tax on agricultural property is levied by


(A) Central Government (B) State Government
(C) Both Central and State Governments (D) Non of the above

49. One of the following statements was not among the Simon Commission’s recommendations
(A) dyarchy to be abolished in the provinces
(B) reconstitution of the Central Legislature
(C) establishment of the Provincial Public Service Commission for all Provinces
(D) Indian Council is not needed to advice the Secretary of State for India.

50. Which of the following are the principal features of Government of India Act, 1919
1. Introduction of dyarchy in the executive government of the provinces
2. Introduction of separate communal electorate for Muslims
3. Devolution of legislative authority by the Centre to the Provinces
4. Expansion and reconstitution of Central and Provincial Legislature
Codes:
(A) 1, 2 and 3 (B) 1, 2 and 4
(C) 2, 3 and 4 (D) 1, 3 and 4

51. There were ............... Articles and ............ Schedules in the draft of the Constitution of India.
(A) 315, eight (B) 319, nine
(C) 327, ten (D) 317, nine

52. The Indian National Congress asserted in ............. that India would not accept any constitution
made by anyone other then people of India and without outside interference.
(A) 1942 (B) 1936
(C) 1927 (D) 1935

53. Almost all political parties participated and contributed their share in the formation of Indian
Constitution. Which one of the following party was not associated with the Constituent Assembly?
(A) Communist Party of India (B) Indian National Congress
(C) Hindu Mahasabha (D) Scheduled Castes Freedom

54. The expenditure from the Consolidated Fund of India for which the approval of Parliament is not
necessary according to the Constitution of India, is called
(A) Charged Expenditure (B) Extra Expenditure
(C) Special Provision Fund (D) None of these

55. Proclamation of Emergency on the ground of internal disturbance was for the first time made in
(A) 1971 (B) 1972
(C) 1974 (D) 1975

Previous Years
CLAT & AILET Papers Page 237
56. Which one of the following conditions regarding acquisition of citizenship by naturalisation has been
wrongly listed?
(A) He belongs to a country where citizens of India are permitted to acquire citizenship by
naturalisation
(B) He has been residing in India or serving the Government of India for at least 10 years preceding
the date of application
(C) He possesses workable knowledge of an Indian language
(D) He has consistently supported the Indian cause at various national and international forums

57. Which one of the following statements is correct?


(A) The Right to Private Property was incorporated in the Constitution by the Forty-Second Amendment
(B) The Right to Private Property granted by the original Constitution but it has since been removed
from the list of Fundamental Rights
(C) The Right to Private Property was never a Fundamental Right under the Indian Constitution
(D) The Right to Private Property which was granted by the original Constitution has been made
more sacrosanct by the Forty-Forth Amendment

58. Which one of the following Directive Principles is non Gandhian?


(A) Promotion of cottage industries in rural areas
(B) Prohibition of the use of intoxicants except for medicinal purposes
(C) Prevention of slaughter of cow, calves and other milch cattle
(D) Provision o free and compulsory education for all the children up to the age of fourteen years

59. Which one of the following steps cannot be taken by the President during the Financial Emergency?
(A) He can direct the Union and State Governments to observe such canons of financial propriety as
he deems desirable
(B) He can suspend the Fundamental Rights of the Indian citizens
(C) He can order reduction of salaries and allowances of all civil servants
(D) He can order the reduction of the salaries of the Supreme Court and High Court judges

60. A death sentence by a lower court


(A) Must be confirmed by High Court
(B) Must be confirmed by Supreme Court
(C) Is operational if no appeal is made to higher courts
(D) Must be confirmed by the President

61. How many members are required to support the introduction of a No-Confidence Motion in the Lok
Sabha?
(A) Two-thirds of the membership of the House
(B) 50 members
(C) 80 members
(D) 60 members

62. The decision of a High Court is


(A) Binding on other High Courts
(B) Not binding on other High Courts
(C) Occasionally binding on other High Courts
(D) Of no value for other High Courts

Previous Years
Page 238 CLAT & AILET Papers
63. Irresistible impulse is
(A) not covered under insanity
(B) covered under insanity
(C) covered under certain specified circumstances
(D) covered under circumstances leading to certain consequences

64. The Supreme Court in its judgment held that non-payment of minimum wages is a type of forced
labour:
(A) Asiad workers case (B) Minerva Mills case
(C) Lokhandwala Mills case (D) T. Krishnamachari case

65. X duly posts a letter of acceptance to Y. But the letter is lost in transit by the negligence of the Post
Office
(A) There is no contract concluded because the acceptance has not reached the proposer
(B) There is no contract concluded because the proposer had not received the letter
(C) The contract is concluded because the acceptance is complete from the date of despatch not
with standing any delay or miscarriage in its arrival from causes not within the acceptor’s control
(D) None of these

66. Ramesh asks his servant to sell his cycle to him at a price less than that of market price. This
contract can be avoided by the servant on the ground of
(A) fraud (B) mistake
(C) undue influence (D) coercion

67. In book depot, a catalogue of books enlisting the price of each book and specifying the place where
the particular book is available is
(A) an invitation to offer
(B) an offer
(C) an invitation to visit the book shop
(D) just a promise to make available the particular book at a particular place at the listed price

68. Mark the incorrect answer

The main purpose of the Law of Contract is


(A) The satisfaction of human desires in the highest practicable degree
(B) where there are conflicting human interests and desires by establishing a judicial and administrative
system that acts with reasonable degree of uniformity
(C) to do something in accordance with the norm of prescribed law
(D) all of the above

69. Frustration of contract implies


(A) commercial hardship
(B) physical impossibility due to disappearance of the subject matter of the contract or the object
has failed to materialise
(C) both (A) & (B)
(D) neither (A) nor (B)

Previous Years
CLAT & AILET Papers Page 239
70. With the approval of the Parliament the National Emergency can continue for
(A) a maximum period of three years
(B) a maximum period of one year
(C) an indefinite period
(D) a maximum period of six months

71. Libel is addressed to the eye: slander to the ear: State which of the following statement are slander
(A) slanderous words are uttered by the characters on the dramatic stage
(B) slanderous words are uttered by the characters on the cinema screen
(C) abuses recorded in the gramophone disc
(D) both (B) and (C)

72. Choose the correct statement

Doctrine of double jeopardy means


(A) a person should not be punished more than once for the same offence
(B) a person should be punished more than once for the same offence
(C) a man may be put twice in peril for the same offence
(D) a man may commit the same offence twice but will get punishment once only

73. “What cannot be done directly cannot be done indirectly”. This statement epitomises the doctrine of
(A) pith and substance (B) implied powers
(C) ancillary powers (D) colourable legislation

74. The 39th Amendment laid that election of any person to Lok Sabha holding the office of Prime
Minister cannot be challenged before a Court of Law, but only before an authority established by
Parliament. This was declared unconstitutional by the Supreme Court in
(A) Keshvananda Bharti Case (B) Menaka Gandhi Case
(C) Indira Gandhi v. Raj Narain Case (D) None of these
75. The Janata Party Government of Morarjee Desai constituted.........to find out the truth about the excesses
committed by the Indira Gandhi Government during emergency (1975-77)
(A) Shahbano Commission (B) Jagmohan Commission
(C) Shah Commission (D) Nayyar Commission

76. Public holidays are declared under


(A) Negotiable Instruments Act, 1881 (B) Contract Act, 1872
(C) Public Employees Act, 1967 (D) None of the above

77. According to law, the maximum number of people who can form a partnership firm is
(A) 10 (B) 20 (C) 30 (D) No limit

78. A Hindu wife can marry immediately after divorce. A Muslim wife
(A) can also marry immediately
(B) has to wait till period of Iddat (seclusion) is over
(C) has to wait for one year
(D) None of the above

Previous Years
Page 240 CLAT & AILET Papers
79. Sometimes an accused seeks pardon from the court and offers to give evidence against all others
involved in a crime. He is called..........
(A) witness (B) clone (C) approver (D) accomplice

80. Under the Motor Vehicles Act, 1988, the minimum compensation to be awarded in case of death
(vide 1994 amendment) is
(A) Rs. 25,000 (B) Rs. 30,000 (C) Rs. 40,000 (D) Rs. 50,000

81. What is the legal meaning of the word ‘Battery’?


(A) Cells as used in torch, tape recorder etc
(B) Battering a person to death
(C) Actual or intended striking of another person
(D) Assault resulting in, at least, 6 months hospitalisation

82. Match the following:


(A) Malfeasance I. One who falsely pretends to be sick
(B) Malingerer II. Minor offences
(C) Misdemeanor III. Improper performance of legal duty
(D) Misfeasance IV. Doing an unlawful act

(A) I-A II-B III-C IV-D


(B) I-B II-C III-D IV-A
(C) I-C II-D III-A IV-B
(D) I-D II-A III-B IV-C

83. Once appointed, judges of Supreme Court serve till they attain the age of
(A) 62 years (B) 63 years (C) 64 years (D) 65 years

84. Gangaram is a wood-cutter. He earns his livelihood by cutting forest trees in Nainital, Uttranchal.
The State of Uttranchal makes a law prohibiting the cutting of forest trees. Is Gangaram’s Constitutional
right infringed?
(A) Yes. Because he may not like to do any other job to earn his bread and butter
(B) No. Because the Government is sovereign and can make any law
(C) Yes. Because he has a fundamental right to life and livelihood under Article 21 of the Constitution
and the Government cannot snatch away his bread and butter by making such a law.
(D) No. Because the Government has power to make a law for prohibiting the cutting of forest trees
under Article 48A of the Constitution

85. Culpable homicide means causing death


(A) with the intention of causing death.
(B) with the intension of causing such bodily injury as is likely to cause death.
(C) with the knowledge that by such act death is likely to be caused.
(D) all the above

Previous Years
CLAT & AILET Papers Page 241
86. X takes away a girl out of the custody of her lawful guardians. Which of the following statements is
a complete defence if X is charged under section 361 of the Indian Penal Code for kidnapping on the
ground that the girl was below the age of eighteen years when taken away?
(A) the girl was a student in a college and could understand what was right or wrong for her.
(B) the girl was maltreated by the guardians and X promised her a better life.
(C) the girl looked more than 18 years of age and the accused had satisfied himself that she was
more than 18 years of age.
(D) none of these

Directions: Given below is a statement of principle followed by a facts . Apply the principle to the facts
given below and select the most appropriate answer.

87. PRINCIPLE:
The occupier of a premises owes a duty of care to all his invitees and visitors.

FACTS:
Laloo was running a dairy from his house. People used a part of his farm as a short cut to get to a
nearby railway station. Laloo who did not approve of this, put up a notice that “Trespassers will be
prosecuted”. However, since a number of these people were also his customers he tolerated them
.One day a person who was using this short cut was attacked by a bull belonging to the farm. The
injured person filed a suit against him.

DECISION:

(A) Laloo is liable for having kept a bull on his farm.


(B) Laloo is not liable in view of the clear notice against trespassers.
(C) Laloo is liable because in fact he allowed the people to use his premises.
(D) Laloo is not liable to the people other than his customers.

Directions: In each of the following questions, a series of letters or numbers has been arranged in some
sequence. Below each are given alternative responses. Find out the correct response.

88. 2, 6, 14, 11, 15, 23, 20, 24,___ , 29


(A) 31 (B) 29 (C) 28 (D) 32

89. 97, 86, 73,______, 45, 34


(A) 54 (B) 56 (C) 58 (D) 60

90. MOQ, SUM, YAC, ______


(A) FIL (B) DHJ (C) EGI (D) XAD

91. KWZ, MOX, OIV, QET, _____


(A) SAQ (B) SUR (C) RAP (D) SAR

Previous Years
Page 242 CLAT & AILET Papers
Directions: The following two (5) items consist of two Statements, one labelled the ‘Assertion A’ and the
other labelled the ‘Reason R’. You are to examine these two Statements carefully and decide if the
Assertion A and the Reason R are individually true and if so whether the Reason is a correct explanation of
the Assertion. Select your answers to these items using the codes given below and mark your answer-
sheet accordingly.
Code:
(A) Both A and R are true and R is the correct explanation of A .
(B) Both A and R are true but R is not a correct explanation of A.
(C) A is true but R is false.
(D) A is false but R is true.

92. Assertion (A) : A stranger to a contract has no right to enforce it against the parties
to the contract.
Reason (R) : Privity of contract between the parties is essential for enforcement
of contract.

93. Assertion (A) : A finder of a thing has title to it.


Reason (R) : The finder’s title to a thing is superior to that of the owner.

94. Assertion (A) : A minor is not competent to enter into any contract, even for
necessaries.
Reason (R) : For necessaries supplied to a minor, his estate can be made liable
to reimburse.

95. Assertion (A) : A legal right is a legally protected interest.


Reason (R) : An element of advantage is essential to constitute a right.

96. Assertion (A) : Customs to have the force of law must be immemorial.
Reason (R) : Custom represents common consciousness of people.

Direction: In each of the following questions one statements is followed by two arguments (A) and (B), one
supporting and the other against it.
(A) if only argument (A) is strong.
(B) if only argument (B) is strong.
(C) if either (A) or (B) is strong.
(D) if both (A) and (B) are strong.

97. Statement : Should India adopt a Presidential system?

Arguments
(A) Yes-Because our experience of Parliamentary democracy is disappointing
(B) No - Because it will concentrate power in the hands of a few people.

98. Statement : Should the death sentence be abolished?

Arguments
(A) Yes-The death sentence deprives the culprit of all chances of improving his behaviour.
(B) No - Capital punishment restrains criminal tendency.

Previous Years
CLAT & AILET Papers Page 243
Directions: In the following question, a statement is followed by two conclusions (A) and (B) . You have to
assume everything in the statement to be true, consider both the conclusions together, and then decide
which of the two given conclusions logically follow beyond a reasonable doubt from the information given in
the statement.
(A) if only conclusion (A) follows.
(B) if only conclusion (B) follows.
(C) if either (A) or (B) follows.
(D) If neither (A) nor (B) follows.

99. Statement: The greatest advantage of democracy over all other forms of governments is not that
the men who have gone to the top are exceptionally wise, but that since their power depends upon
popular support, they know that they cannot retain their position if they are guilty of more than a
modicum of injustice.

Conclusions:

(A) In a democracy, persons in power cannot act arbitrarily.


(B) Democratic countries cannot have excellent leaders.

100. I go 100 metres towards North from my house, then I turn left and go 200 metres, then turning left I
go 200 metres, then again turning left I go 100 metres. and then turning left again I go 100 metres.
In which direction am I now from my house?
(A) West (B) East (C) North (D) South

Directions: Some information is provided in the form of statements. On the basis of that information find
the answer to the questions which follow.
1. A cube has six sides each of a different colour
2. The red side is opposite black
3. The green side is between red and black
4. The blue side is adjacent to white
5. The brown side is adjacent to blue
6. The red side is face down.

101. The side opposite to brown is


(A) Red (B) Black (C) White (D) Green

102. The four colours adjacent to green are________


(A) Red, Black, Blue and White (B) Red, Black, Brown and Blue
(C) Red, Black, Brown and White (D) Black, Blue. White and Red

103. K is L’s wife’s husband’s brother. M is the sister of K. N is the sister of L. How is K related to M?
(A) Sister-in-law (B) Brother (C) Daughter (D) Wife

104. Atul is the son of Zamir. Alka is the daughter of Aman. Sheela is the Wife of Aman, Mohan is the son
of Sheela. How is Alka related to Mohan?
(A) Sister (B) Uncle (C) Son (D) Father

Previous Years
Page 244 CLAT & AILET Papers
Directions: Five adults A, B, C, D and E are sitting at Bharat Provision Store. In the group, there are one
Manager, one Accountant, one Supplier of provisions. The Accountant, who has a child earns least money.
A. who is married to C’s sister, earns more than the Manager . D is an unmarried lady and does not do any
work. There is one married couple in a group of which B is husband. E is the brother of C and is neither a
Supplier nor an Accountatnt. No lady is a Supplier or Manager. C is neither a Supplier nor an ‘Accountant’.

105. Who is Accountant?


(A) A (B) B (C) C (D) D

106. Who is Supplier?


(A) A (B) B (C) C (D) D

107. Who is the wife of B?


(A) A (B) B (C) C (D) E

108. Who is earning the highest?


(A) A (B) B (C) C (D) D

109. Who is the sister of C?


(A) A (B) B (C) C (D) D

110. It is possible to make a meaningful word with the third, the fourth, and the eleventh letters of word
‘CONTROVERSIAL’ write the first letter of that word.
(A) I (B) N (C) S (D) T

111. If (A) Chip din chunk means student attends class; (B) din sunk dink means Arjuna is student; (C)
jump nink sink means schools are good; (D) dink mup chimp means teacher is good. Then what is
the code for Arjuna?
(A) din (B) sunk (C) dink (D) chunk

Directions: In the following groups one does not belong to that group. Find the odd one.

112. (A) Cheese (B) Saffron (C) Pepper (D) Lard

113. (A) Hansom (B) Victoria (C) Growler (D) Baroque

Directions: The questions in this section are based on the reasoning contained in brief statements or
passages. For some questions, more than one of the choices could conceivably answer the question.
However, you are to choose the best answer, that is, the response that most accurately and completely
answers the question. You should not make assumptions that are by commonsense standards implausible,
superfluous or incompatible with the passage.

Consumer advocate: Businesses are typically motivated primarily by the desire to make as great a profit
as possible, and advertising helps business to achieve this goal. But it is clear that the motive of maximizing
profits does not impel businesses to present accurate information in their advertisements. It follows that
consumers should be skeptical of the claims made in advertisements.

Previous Years
CLAT & AILET Papers Page 245
114. Each of the following if true would strengthen the Consumer advocate’s argument EXCEPT:
(A) Businesses know that they can usually maximize their profits by using inaccurate information in
their advertisements.
(B) Businesses have often included inaccurate information in their advertisement.
(C) Many consumers have a cynical attitude toward advertising.
(D) Those who create advertisements are less concerned with the accuracy than with the creativity
of advertisements.
Science columnist: It is clear why humans have so many diseases in common with cats. Many
human diseases are genetically based, and cats are genetically closer to humans than are any
other mammals except nonhuman primates. Each of the genes identified so far in cats has an exact
counterpart in humans.

115. Which one of the following if true, most weakens the science columnist’s explanation for the claim
that humans have so many diseases in common with cats?
(A) Cats have built up resistance to many of the disease they have in common with humans.
(B) Most diseases that humans have in common with cats have no genetic basis.
(C) Cats have more diseases in common with nonhuman primates than with humans.
(D) Many of the diseases human have in common with cats are mild and are rarely diagnosed.
Psychologist: It is well known that becoming angry often induces temporary incidents of high blood
pressure. A recent study further showed, however, that people who are easily angered are significantly
more likely to have permanently high blood pressure than are people who have more tranquil
personalities. Coupled with the long established fact that those with permanently high blood pressure
are especially likely to have heart disease, the recent findings indicate that heart disease can result
from psychological factors.

116. Which one of the following would, if true, most weaken the psychologist’s argument?
(A) Those who are easily angered are less likely to recover fully from episodes of heart disease than
are other people.
(B) Medication designed to control high blood pressure can greatly affect the moods of those who
use it.
(C) People with permanently high blood pressure who have tranquil personalities virtually never
develop heart disease.
(D) The physiological factors that cause permanently high blood pressure generally make people
quick to anger.
A professor of business placed a case-study assignment for her class on her university’s computer
network. She later found out that instead of reading the assignment on the computer screen. 50 out
of the 70 students printed it out on paper. Thus, it is not the case that books delivered via computer
will make printed books obsolete.

117. Which one of the following, if true, most strengthens the argument?
(A) Several colleagues of the professor have found that, in their non-business courses, several of
their students behave similarly in relation to assignments placed on the computer network.
(B) Studies consistently show that most computer users will print reading material that is more than
a few pages in length rather than read it on the computer screen.
(C) Some people get impaired vision from long periods of reading printed matter on computer screens,
even if they use high quality computer screens.
(D) Scanning technology is very poor, causing books delivered via computer to be full of errors
unless editors carefully read the scanned versions.
Previous Years
Page 246 CLAT & AILET Papers
Directions: Each group of questions in this section is based on a set of conditions. Choose the response
that most accurately and completely answers each questions.

A panel reviews six contract bids - H, J, K, R, S, and T. No two bids have the same cost.
Exactly one of the bids is accepted. The following conditions must hold:

1. The accepted bid is either K or R and is either the second or the third lowest in cost.
2. H is lower in cost than each of J and K.
3. If J is not the fourth lowest in cost, then J is higher in cost than each of S and T.
4. Either R or S is the fifth lowest in cost.

118. Which one of the following could be an accurate list of the bids in order from lowest to highest in
cost?
(A) T, K, H, S, J, R (B) H, T, K, S, R, J (C) H, S, T, K, R, J (D) H, K, S, J, R, T

119. Which one of the following bids CANNOT be the fourth lowest in cost?
(A) H (B) J (C) K (D) R

120. Which one of the following bids CANNOT be the second lowest in cost?
(A) H (B) J (C) K (D) R

Directions for questions 121 and 122: Detectives investigating a city wide increase in burglaries questioned
exactly seven suspects - S, T, V, W, X, Y and Z - each on a different one of seven consecutive days. Each
suspect was questioned exactly once. Any suspect who confessed did so while being questioned.
The investigation conformed to the following:
1. T was questioned on day three.
2. The suspect questioned on day four did not confess.
3. S was questioned after W was questioned.
4. Both X and V were questioned after Z was questioned.
5. No suspects confessed after W was questioned.
6. Exactly two suspects confessed after T was questioned.

121. Which one of the following could be true?


(A) X was questioned on day one. (B) V was questioned on day two.
(C) Z was questioned on day four. (D) W was questioned on day five.

122. If Z was the second suspect to confess, then each of the following statements could be true EXCEPT:
(A) T confessed. (B) T did not confess.
(C) V did not confess. (D) Y did not confess.

123. Find the least number that when divided by 16, 18 and 20 leaves a remainder 4 in each case, but is
completely divisible by 7.
(A) 364 (B) 2254 (C) 2884 (D) 3234

124. The average age of three boys is 15 years. If their ages are in the ratio 3 : 5 : 7, the age of the
youngest boy is
(A) 21 years (B) 18 years (C) 15 years (D) 9 years

Previous Years
CLAT & AILET Papers Page 247
125. A’s salary is first increased by 25% and then decreased by 20%. The result is the same as B’s
salary increased by 20% and then reduced by 25%. Find the ratio of B’s salary to that of A’s.
(A) 4 : 3 (B) 11 : 10 (C) 10 : 9 (D) 12 : 11

126. A man sells 5 articles for Rs.15 and makes a profit of 20%. Find his gain or loss percent if he sells
8 such articles for Rs.8.40.
(A) 2.22% profit (B) 2.22% loss (C) 8% loss (D) 8%profit

127. What is the simple interest for 9 years on a sum of Rs.800 if the rate of interest for the first 4 years
is 8% per annum and for the last 4 years is 6% per annum?
(A) 400 (B) 392 (C) 352 (D) Cannot be determined

128. In Ramnagar Colony, the ratio of school going children to non-school going children is 5 : 4. If in the
next year, the number of non-school going children is increased by 20% making it 35,400. what is
the new ratio of school going children to non-school going children?
(A) 4 : 5 (B) 3 : 2 (C) 25 : 24 (D) None of these

129. Amar and Akbar left Bhubaneshwar simultaneously and travelled towards Cuttack. Amar’s speed
was 15 km/h and that of Akbar was 12km/h. Half an hour later, Anthony left Bhubaneshwar and
travelled in the same direction. Some time later, he overtook Akbar and 90 minutes further on he
overtook Amar. Find Anthony’s speed.
(A) 18 kmph (B) 24 kmph (C) 12 kmph (D) 16 kmph

130. Five boys and three girls are sitting in a row of eight seats. In how many ways can they be seated
so that not all girls sit side by side?
(A) 36,000 (B) 45,000 (C) 24,000 (D) None of these

131. A bag contains 5 red, 4 green and 3 black balls. If three balls are drawn out of it at random, find the
probability of drawing exactly 2 red balls.
(A) 7/22 (B) 10/33 (C) 7/12 (D) 7/11

132. Ajit can do as much work in 2 days as Baljit can do in 3 days and Baljit can do as much in 4 days
as Diljit in 5 days. A piece of work takes 20 days if all work together. How long would Baljit take to
do all the work by himself ?
(A) 82 days (B) 44 days (C) 66 days (D) 50 days

133. “Project Tiger” was launched in


(A) 1973 (B) 1974 (C) 1978 (D) 1981

134. Which of the following fighter air craft was flown by former President Dr. Abdul Kalam?
(A) F-16 (B) MI - 30 (C) Jaguar (D) Sukhoi - 30 MKI

135. The name “ Baichung Bhutia” is associated with


(A) Football (B) Hockey (C) Polo (D) Cricket

136. The present Chairperson of the National Human Right Commission of India is
(A) R.C.Lahoti (B) Rajendra Babu (C) J.S. Verma (D) A.S.Anand

Previous Years
Page 248 CLAT & AILET Papers
137. The present Secretary General of United Nations Organisation is
(A) Condoleeza Rise (B) Shashi Tharoor (C) Ban ki Moon (D) Kofi Annan

138. Who is the author of the book “My Life”?


(A) Bill Clinton (B) Tony Blair (C) Hillary Clinton (D) Dalai Lama

139. “Golden Handshake” is the term associated with


(A) Share Market (B) Investment of Gold
(C) Voluntary Retirement Benefits (D) Marriage without Dowry

140. National Disaster Management Authority is constituted under


(A) Natural Calamities Management Act, 2005.
(B) Disaster Management Act.2005
(C) Planning Commission Decision
(D) Cabinet Decision

141. Indian Airlines (New name: air India) have redesigned their logo which is a graphic wheel. This logo
has been inspired from which one of the following?
(A) Khajuraho Temple (B) Sun Temple, Konark
(C) Mamallapuram Temples (D) Hampi Temples

142. Who has been recently awarded Officer de La Legion Honor, the highest award of France?
(A) Shatrughan Sinha (B) Lata Mangeshkar
(C) Amitabh Bachan (D) Aishwarya Rai

Directions: Each set of questions in this section is based on the passage. The questions are to be
answered on the basis of what is stated or implied in the passage. For some of the questions, more than
one of the choices could conceivably answer the question. However, you are to choose the best answer;
that is, the response that most accurately and completely answers the questions.

Most people acknowledge that not all governments have a moral right to govern and that there are sometimes
morally legitimate reasons for disobeying the law, as when a particular law prescribes behaviour that is
clearly immoral. It is also commonly supposed that such cases are special exceptions and that, in general,
the fact that something is against the law counts as a moral, as well as legal, ground for not doing it; i.e.
we generally have a moral duty to obey a law simply because it is the law. But the theory known as
philosophical anarchism denies this view, arguing instead that people who live under the jurisdiction of
governments have no moral duty to those government to obey their laws. Some commentators have
rejected this position because of what they take to be its highly counter intuitive implications:(A) that no
existing government is morally better than any other (since all are , in a sense, equally illegitimate), and (2)
that, lacking any moral obligation to obey any laws people may do as they please without scruple. In fact,
however, philosophical anarchism does not entail these claims.

First, the conclusion that no government is morally better than any other does not follow from the claim that
nobody owes moral obedience to any government. Even if one denies that there is a moral obligation to
follow the laws of any government, one can still evaluate the morality of the policies and actions of various
governments. Some governments do more good than harm, and others more harm than good, to their
subjects. Some violate the moral rights of individuals more regularly, systematically, and seriously than
others. In short, it is perfectly consistent with philosophical anarchism to hold that governments vary
widely in their moral stature.
Previous Years
CLAT & AILET Papers Page 249
Second, philosophical anarchists maintain that all individuals have basic, non-legal moral duties to one
another-duties not to harm others in their lives, liberty, health, or goods. Even if governmental laws have no
moral force, individuals still have duties to refrain from those actions that constitute crimes in the majority
of legal systems (such as murder, assault, theft, and fraud). Moreover, philosophical anarchists hold that
people have a positive moral obligation to care for one another, a moral obligation that they might even
choose to discharge by supporting cooperative efforts by governments to help those in need. And where
others are abiding by established laws, even those laws derived from mere conventions, individuals are
morally bound not to violate those laws when doing so would endanger others. Thus, if others obey the law
and drive their vehicles on the right, one must not endanger them by driving on the left for , even though
driving on the left is not inherently immoral, it is morally wrong to deliberately harm the innocent.

143. Which one of the following most accurately expresses the main point of the passage?
(A) Some views that certain commentators consider to be implications of philosophical anarchism
are highly counter-intuitive.
(B) Contrary to what philosophical anarchists claim, some governments are morally superior to
others and citizens under legitimate governments have moral obligations to one another.
(C) It does not follow logically from philosophical anarchism that no government is morally better
than any other or that people have no moral duties toward one another.
(D) Even if as certain philosophical anarchists claim, governmental laws lack moral force, people
still have moral obligation to refrain from harming one another.

144. The author identifies which one of the following as a commonly held belief?
(A) In most cases we are morally obligated to obey the law simply because it is the law.
(B) All governments are in essence morally equal.
(C) We are morally bound to obey only those laws we participate in establishing.
(D) Most crimes are morally neutral, even though they are illegal.

145. The author’s stance regarding the theory of philosophical anarchism can most accurately be described
as one of
(A) ardent approval of most aspects of the theory.
(B) apparent acceptance of some of the basic positions of the theory
(C) concerned pessimism about the theory’s ability to avoid certain extreme views
(D) hesitant rejection of some of the central features of the theory

146. By attributing to commentators, the view that philosophical anarchism has implications that are
“counter-intuitive”, the author most likely means that the commentators believe that
(A) the implications conflict with some commonly held beliefs.
(B) there is little empirical evidence that the implications are actually true.
(C) common sense indicates that philosophical anarchism does not have such implications.
(D) the implications appear to be incompatible with each other.

Previous Years
Page 250 CLAT & AILET Papers
147. Which one of the following scenarios most completely conforms to the view attributed to philosophical
anarchists in third paragraph?
(A) A member of a political party that is illegal in a particular country divulges the names of other
members because he fears legal penalties.
(B) A corporate executive choose to discontinue illegally when she learns that the chemicals are
contaminating the water supply.
(C) A person who knows that a co-worker has stolen funds from their employer decided to do
nothing because the co-worker is widely admired.
(D) A person neglects to pay her taxes, even though it is likely that she will suffer severe legal
penalties as a consequence, because she wants to use the money to finance a new business.

148. It can be inferred that the author would be most likely to agree that
(A) people are subject to more moral obligations than is generally held to be the case
(B) government that are morally superior recognize that their citizens are not morally bound to obey
their laws.
(C) one may have good reason to support the efforts of one’s government even if one has no moral
duty to obey its laws.
(D) there are some sound arguments for claiming that most governments have a moral right to
require obedience to their laws.

149. The author’s discussion of people’s positive moral duty to care for one another functions primarily to
(A) demonstrate that governmental efforts to help those in need are superfluous.
(B) suggest that philosophical anarchists maintain that law that foster the common good are
extremely rare.
(C) imply that the theoretical under-pinnings of philosophical anarchism are inconsistent with certain
widely held moral truths.
(D) indicate that philosophical anarchists recognise that people are subject to substantial moral
obligations.

150. In the passage, the author seeks primarily to


(A) describe the development and theoretical under-pinnings of a particular theory.
(B) establish that a particular theory conforms to the dictates of common sense.
(C) argue that two necessary implications of a particular theory are morally acceptable.
(D) defend a particular theory against its critics by showing that their arguments are mistaken.

Previous Years
CLAT & AILET Papers Page 251
AILET Question Paper 2010

SECTION – A : ENGLISH

I. Read the passage and answer the questions following it : Artists should treat their art as art
and take the process of making it as seriously as anyone takes their chosen profession. Great skill
and insight are required in order to create truly original art. Transforming an idea or concept into a
technically thought- provoking or emotion-arousing work of art in any medium is a talent that few
people possess. And there you have the “purist's vision."

Now if an artist wants to create art and never sell it, then he or she never has to worry about how to price
it. That artist can afford to be a "purist," as you put it, produce art free of any encumbrances or concerns
about what the art world or anyone else might think, and avoid "prostituting" or "debasing" that art by
placing dollar values on it. But if you're an artist who wants to sell your art or who has to sell it in order to
survive as an artist, you must use whatever tools are available to figure out how much it's worth and how
best to sell it.

Let's say you're just starting out as an artist, you have little or no experience showing or selling your work,
and in a period of two minutes, you produce a pencil drawing on a piece of paper. You view this drawing as
highly significant in your evolution as an artist and rank its creation as the single most important creative
moment of your life. Consequently, you put a price of $20,000 on it because only for that amount of money
will you agree to part with such an important work of art. This is a "purist's vision" approach to pricing as
opposed to a "realities of the marketplace" approach.

From a business standpoint, you'll have an extremely difficult time selling your drawing, as you won't be
able to justify the $20,000 price to real art buyers in the real art world. You have no track record of selling
art in that price range, and you have few or no shows, critical reviews, or supporting data from outside
sources indicating that your art has that kind of value or collectibility in the marketplace. The overwhelming
majority of art buyers who have $20,000 to spend look for works of art by established artists with documented
track records of showing and selling art in that price range.Your drawing is still highly significant to you, but
what someone is willing to pay for it on the open market is a matter for art buyers to decide. You can price
it however you wish, but you can never force anyone to buy it. That's the way the art business works. So
if you want to sell it, you have to figure out what dollar amount someone is likely to pay for it on the open
market and then price it at that amount. But the tale of your drawing does not end here.

The art world may, one agree with you that the product of your two-minute moment precipitates a major
transformational turning point in your career, and is well worth a $20,000 asking price, but you're going to
have to prove first. Aspects of that drawing will have to be reflected in your art from the moment you created
it onwards, the art world will have to recognize your art both critically and from the marketing standpoints,
and you'll have to successfully produce, show, and sell for many years. Then one day, when your first
retrospective exhibition opens at the Four-Star Museum of Art, that drawing will hang framed and captioned
as the first inspiration for all subsequent work. The art world will then understand and respect its significance,
and a serious collector may well be willing to pay an extraordinary price to own this historically important
document of your career.

Previous Years
Page 152 CLAT & AILET Papers
Returning for a moment to the concept of a purist artist who creates art and never sells it, sooner or later
(hopefully later), that purist will pass on and leave behind a body of work. Unless that artist leaves specific
instructions in his or her will for that body of work to be destroyed, it will become subject to those market
forces that the artist strived for a lifetime to avoid. At the very least, it'll have to be appraised for tax,
donation, or inheritance purposes. In most cases, it eventually comes onto the market either through a
probable sale, an auction, or as represented by a dealer, gallery, or family member. The moral of the story
is that one way or another, someone somewhere at some point in time will use tried and true methods to
realistically price and either sell, donate, trade or otherwise transact any work of art that comes onto the
market in any way, shape, or form. I hope that that person will be you, the artist, and that you'll price your
art according to what the market will bear, sell plenty of it, and have a long and rewarding career.

Answer the following questions indicating your option for each question:

1. The purist's art is


(A) one that arouses emotions.
(B) thought-provoking.
(C) technically sound.
(D) free from any encumbrances or concerns about what the world or anyone else might think of it.

2. The "realities of the marketplace" approach entails.


a) "prostituting" one's art by putting a dollar price on it.
b) gauging the market value of one's art and then putting a price on it.
c) compromising on one's estimation of one's own art as far as its worth in financial terms is
concerned
d) subjecting one's art to the buyer's interpretation of it
(A) Only a (B) only b and d (C) b, c and d (D) only b

3. For an artist to sell his piece of art at a higher price range, he/she has to
(A) get endorsed by fellow artists and art critics.
(B) hold exhibitions or shows.
(C) gradually increase the collectibility of his/her art in the market place.
(D) all of the above.

4. The first piece of an artist that comes out in the market is seen as
(A) his masterpiece.
(B) the most collectible piece in his / her repertoire.
(C) a blueprint to discern the author's unique style and point of view.
(D) the inspiration for all his subsequent works.

5. From the point of view of the purist, the irony as far as pricing art is concerned is that:
a) a piece from his collection is eventually sold at the same price that he had estimated to be its
real worth years earlier.
b) his art is subjected to the same market forces against which he strove his entire life.
c) “realities of the marketplace” is a concept that negates the very attributes that we associate with
art- whim, fancy and imagination.

(A) only a (B) a and b (C) only b (D) a, b and c

Previous Years
CLAT & AILET Papers Page 153
For questions 6 and 7: Each question consists of two capitalized words that have a certain relationship
to each other, followed by 4 pairs of words. Choose the pair that is related to each other in the same way
as the capitalized pair

6. ENCUMBRANCE : BURDEN
(A) Mnemonic : Memory
(B) Blatant : Subtle
(C) Captious : Acrimonious
(D) Feral : Cultured

7. SUBSEQUENT : PREVIOUS
(A) Significant : Inconsequential
(B) Retrospect : Retrospective
(C) Visionary : Seer
(D) Caption : Legend

Choose the right option :

8. The synonym of the word PRECIPITATE in the context of the passage is


(A) Iaunch (B) trigger (C) provoke (D) accelerate

Complete the sentences by filling in the blanks with the correct tenses from the given choices by indicating
your option for each question:

9. I wish you ________ louder as I can't hear what you say.


(A) were speaking (B) spoke (C) would speak (D) speak

10. The audience ________ to take their seats please.


(A) is requested (B) have requested (C) may request (D) are requested

11. It ________all day yesterday before the garden wall collapsed.


(A) rained (B) has been raining (C) had rained (D) was raining

12. Each member of this group ________ guilty.


(A) is (B) are (C) is being (D) were

13. The Prime Minister ________ to visit the hospital tomorrow.


(A) will (B) is (C) must (D) have

14. Last night the radio said that the volcano, Etna, in Sicily ________
(A) will erupt (B) is erupting (C) erupts (D) has to erupt

15. They say that the Princess ________ incognito at the Sheraton.
(A) have stayed (B) is staying (C) stay (D) will have been staying

16. She ________ in Calcutta for a quarter of a century now.


(A) will live (B) lived (C) has been living (D) had been living

Previous Years
Page 154 CLAT & AILET Papers
17. When we went to their home, Sushma ________ some fresh coffee for us.
(A) made (B) has made (C) will make (D) is making

18. Evidence shows that Jackson ________inside the house at the time of the murder.
(A) was (B) is (C) will be (D) has been

II. Choose the word or phrase that best completes the sentence from the given alternatives.
Indicate your option.

19. To answer accurately is more important than


(A) quick finish (B) finish quickly (C) finishing quickly (D) you finish quickly

20. Professional people now-a-days appreciate ________ when it is necessary to cancel an appointment.
(A) Your calling them (B) That you would call them
(C) You to call them (D) You are calling them

21. Who will ________ the children when their mother is in the hospital ?
(A) look after (B) look for (C) look up (D) look about

22. The teacher gives many examples to ________the idea contained in the poem.
(A) bring about (B) bring in
(C) bring forth (D) bring out

23. That magnificent ________ temple was constructed by the Chinese.


(A) eight -centuries-old (B) eight-century' s-old (C) old-eight-centuries (D) eight-century-old

24. Wheat is not native to India and Barley ________


(A) isn't either (B) is either
(C) is neither (D) isn't neither

25. Encounters between people from different countries can results in misunderstandings ________
different concepts about space.
(A) because they (B) is because they
(C) is because their (D) because of their

III. Choose the correct answer which is closest in meaning to the word/ phrase in bold given
in the sentence. Indicate your option.

26. You may think at first that it is queer to talk of having too much paper money and that money is so
nice and useful that you cannot have too much of it.
(A) Ridiculous (B) Absurd (C) Anomalous (D) Odd

27. The government is under no obligation to offer contracts to companies which choose to flout
guidelines.
(A) Condemn (B) Ignore (C) Defy (D) Neglect

28. Indians exhibited a remarkable solidarity at the time of war.


(A) Coalition (B) Co-operation (C) Unification (D) Unity

Previous Years
CLAT & AILET Papers Page 155
29. His impeccable style caught the attention of all critics.
(A) Faultless (B) Inoffensive (C) Upright (D) Harmless

30. The inspector was a vigilant young man.


(A) Intelligent (B) Ambitious (C) Watchful (D) Smart

31. The young is quite sanguine about the result of his competitive examination.
(A) Depressed (B) Pessimistic (C) Anxious (D) Optimistic

32. It was a scurrilous attack on him.


(A) Serious (B) Unjustified (C) Insulting (D) Justified

33. Both parties were amenable to a peaceful settlement of the land dispute.
(A) Unresponsive (B) Responsive (C) Unwilling (D) Doubtful

IV. In the following questions, four alternatives are given for the idiom/phrase marked In bold
in the sentence. Choose the alternative which best expresses the meaning of the idiom/
phrase from the options given. Indicate your option.

34. The neighbour had to pay through his nose for a brand new car.
(A) Take huge loans (B) Pay a reasonable price
(C) Pay an extremely high price (D) Make a quick buck

35. If he phones again, I am going to give him a piece of my mind.


(A) To be nice to him (B) To take advantage of him
(C) To reprimand (D) To support him

SECTION – B : GENERAL KNOWLEDGE

36. The 2009 Nobel Peace Prize was awarded to


(A) Al Gore (B) Jane Adams
(C) Mohammed Yunus (D) Barak Hussein Obama

37. The present Prime Minister of Bangladesh is


(A) Sheik Hasina. (B) Khaleda Zia (C) Ziaur Rahman (D) None

38. The President of South Africa at present is


(A) Nelson Mandela (B) Jacob Zuma (C) Thabo Mbeki (D) Kgalema Motlanthe

39. The former President who committed suicide in May 2009 was
(A) Kim Jong II of North Korea (B) Maumoon Abdul Gayoom of Maldives
(C) Meghavati Sukarnoputri of Indonesia (D) Roh Moo-Hyun of South Korea

40. Who among the following has not won an Oscar?


(A) A.R. Rahman (B) Satyajit Ray (C) Bhanu Athaiya (D) Dev Patel

41. The present Chief Election Commissioner of India is


(A) T.N. Seshan (B) Navin Chawla (C) N.S. Gopalaswami (D) V.S. Sampath

Previous Years
Page 156 CLAT & AILET Papers
42. 76 CRPF jawans were killed recently by the Maoist in an ambush near
(A) Dantewada in Chattisgarh (B) Konarkhurd in Bihar
(C) Gadhchiroli in Maharashtra (D) Karimnagar in Andhra Pradesh

43. The National Dairy Research Insdtute in Haryana cloned a buffalo calf and named it as
(A) Gowri (B) Tusna (C) Garima (D) Shakthi

44. The former Chief Justice of India who did not become the Chairperson of the National Human Rights
Commission is
(A) Justice S. Rajendra Babu (B) Justice M. N. Venkatachalaiah
(C) Justice Lahoti (D) Justice Verma

45. The first sitting Head of the State to be charged with ‘War crimes and crimes against humanity’ by
the International Criminal Court is
(A) President Manuel Zelaya of Honduras (B) President Omar al-Bashir of Sudan
(C) President Siad Bane of Somalia (D) President Martin Torrijos of Panama

46. The former Chief Election Commissioner to be appointed as a Union Minister is


(A) Sukumar Sen (B) T. Swaminathan (C) R.K. Trivedi (D) M.S. Gill

47. India's Imperial Capital was officially shifted from Calcutta to Delhi in
(A) 1912 (B) 1935 (C) 1947 (D) 1872

48. The first Women's University was established in Pune during


(A) 1909 (B) 1928 (C) 1915 (D) 1968

49. How many official languages are mentioned in Schedule VIII to the Constitution of India
(A) 18 (B) 22 (C) 14 (D) 28

50. The present Committee on Centre-State Relations in India is headed by


(A) Justice Punnchi (B) Justice Ahmedi (C) Justice Anand (D) Justice Ruma Pal

51. In India, women were allowed to contest election and to public offices for the first time in
(A) 1909 (B) 1919 (C) 1935 (D) 1926

52. The recently constituted Telangana Committee by the Central Government is headed by
(A) Justice P.V. Reddy (B) Justice Sri Krishna
(C) Justice Venkatachala (D) Justice Banerjee

53. The Committee on Reforming Criminal Justice System in India was headed by
(A) Justice K.T. Thomas (B) Justice Krishna Iyer
(C) Justice V.S. Malimath (D) Justice Mohan

54. The first President of the Constituent Assembly of India established in 1946 was
(A) Rajendra Prasad (B) Sachidananda Sinha(C) Rajagopalachari (D) B.R. Ambedkar

55. The Constitution of India was adopted on


(A) 26 November 1949 (B) 26 January 1950 (C) 26 November 1947 (D) 26 January 1949

Previous Years
CLAT & AILET Papers Page 157
56. The film that won 14 Oscar nominations was
(A) Gandhi (B) Slumdog Millionaire (C) Benhur (D) All about Eve

57. The first Indian to get a medal at the World Boxing Championship was
(A) Jayadev Bisht (B) Suvanjoy Singh (C) Vijender Singh (D) Rajkumar Sangwan

58. At the Beijing Olympics, highest number of medals were won by


(A) China (B) USA (C) Russia (D) Germany

59. If the Anglo-Indian Community is not adequately represented in the election, the President can
nominate to Lok Sabha from that community
(A) Not more than 5 (B) Not more than 2 (C) Not more than 1 (D) None

60. The right to vote in India is a


(A) Constitutional right (B) Legal right (C) Fundamental right (D) None

61. The maximum fine that the Central or State Information Commission can impose on Public Information
Officer for not furnishing the information within the stipulate time is
(A) Rs. 10,000/- (B) Rs. 15,000/- (C) Rs. 25,000/- (D) Rs. 30,000/-

62. Maximum number of gold medals won by an individual at Olympics in any event is
(A) 3 (B) 5 (C) 7 (D) 8

63. The person who promoted the establishment of the International Committee of the Red Cross was
(A) Charles Dickens (B) Henry Dunant (C) Henry Ford (D) Robert Frost

64. The first African to become the Secretary General of United Nations was
(A) Kofi Annan (B) Dag Hammarskjold
(C) Boutros Boutros Ghali (D) U Thant

65. The present UN' Secretary General Ban Ki Moon is from


(A) South Korea (B) North Korea (C) China (D) Mongolia

66. Who said that Swaraj is my birth right?


(A) M.K. Gandhi (B) Balagangadhar Tilak
(C) Jawaharlal Nehru (D) Mohammed Ali Jinnah

67. Which of the following states got separated from India in 1937 ?
(A) Pakistan (B) Sikkim (C) Sri Lanka (D) Burma

68. The first woman Prime Minister was


(A) Indira Gandhi (B) Srimavo Bandaranaike
(C) Golda Meir (D) Elizabeth Domitien

69. The largest country in terms of territory is


(A) Russia (B) Canada (C) USA (D) China

Previous Years
Page 158 CLAT & AILET Papers
70. The phrase 'cloud computing' means
(A) Delivery of hosted services over the internet
(B) Study of clouds to predict rains and thunderstorms
(C) Study of conditions of clouds for 'cloud seeding'
(D) Use of different software for financial accounting

SECTION – C : LEGAL APTITUDE


71. PRINCIPLE:
An assault is an attempt to do a corporeal hurt to another, coupled with an apparent present ability
and intention to do that act. A battery is the intentional and direct application of any physical force
to the person of another.

A was sitting on a chair reading a book. His friend, B decided to play a practical joke on him.
Accordingly, he pulled the chair from under him, as a result of which A landed on the floor.
(A) B's act amounts to a battery.
(B) B's act amounts to an assault.
(C) B' s act amounts to an assault till the time A lands on the floor.
(D) B' s act amounts to neither because there was no intention.

72. PRINCIPLE:
False imprisonment is a total restraint of the liberty of a person, for how so ever short period of time,
without lawful excuse. (Common for Q. No. 72 and 73)

A was driving down a road heading to her house. As she reached close to her house, she found that
a few people led by B, protesting against an unfair law had blocked the road. There was no alternate
road to her house and hence she was stuck there for around 5 minutes.
(A) B and his group are liable for having falsely imprisoned A.
(B) B and his group are not liable for falsely imprisoning A, since they were exercising their right to
protest.
(C) B and his group are not liable for falsely imprisoning A, since they did not totally restrain the
liberty of A.
(D) B and his group are not liable for falsely imprisoning A, since 5 minutes is too short a time.

73. A was suspected of having committed the murder of B. C, a policeman who was investigating into B'
s murder, saw A in a market. He went up to him, caught hold of his hand and prevented him from
going anywhere.
(A) C is liable for having falsely imprisoned A, since to arrest a person, a policeman requires
permission from a Magistrate.
(B) C is not liable for having falsely imprisoned A, since for the offence of murder, a policeman need
not take the permission of a Magistrate to arrest.
(C) C is not liable for having falsely imprisoned A since the restraint was not total.
(D) C is not liable for having falsely imprisoned A since he did not take him to a prison.

Previous Years
CLAT & AILET Papers Page 159
74. PRINCIPLE: A libel is a publication of a false and defamatory statement tending to injure the
reputation of another person without lawful justification or excuse. A slander is a false and defamatory
statement by spoken words or gestures tending to injure the reputation of another.
A wrote a letter to B calling him a cheat. B' s clerk C opened the letter, as he normally did (a fact which
was known to A) and placed it on B's table. B alleges that A has committed libel
(A) B will succeed since A has published a defamatory statement against A
(B) B will not succeed because it was not written in a newspaper
(C) B will not succeed because everyone knew that A was a cheat
(D) B will not succeed since A did not follow up the letter with a speech

75. PRINCIPLE:
A Master is liable for the acts of his Servant as long as he can control the working of his servant.

A owned a taxi agency. She had hired B to drive one of her cars. On January 1, 2010, C called up A's
taxi agency and asked for a car to drop him from his house to his place of work. On the way,
because of the driver's negligence, the car hit a road divider and C was injured. He sued A for
damages.
(A) A is not liable because A was not driving the car
(B) A is not liable because A was not in the car
(C) A is liable because B was employed be her and was in her control
(D) A is not liable because B was driving as per C's instructions

76. PRINCIPLE: A Master is liable to third persons for every such wrong of his servant as committed in
the course of service. For acts committed beyond the scope of employment, the master is liable
only if he has expressly authorised the act. (Common for Q. No. 76, 77 and 78)

A owned a bus and he had hired B to drive it and C to be the conductor. One day, when B had
stepped out of the: bus to have a cup of coffee, C decided to turn the bus around so that it was ready
for its next trip. While doing so, C ran over D's leg, causing major injuries to him. D sued A for
damages.

(A) D will succeed since C was employed by A


(B) D will not succeed since A had not authorised C to drive the bus
(C) D will not succeed since the bus was not on an official trip
(D) D will succeed since turning the bus was in the course of employment

77. A owned a truck and he had hired B to drive it. On one of its trips, C flagged the truck down and
asked to be dropped to a nearby city. B agreed to do so for a small amount of money. The truck met
with an accident en route, in which C was badly injured. C sued A for damages.

(A) A will be liable since B did the act in the course of his employment
(B) A will be liable since he had not instructed B not to pick up passengers
(C) A will nor be liable since taking passengers in a truck had no connection with his business
(D) A will not be liable because B was an experienced driver

Previous Years
Page 160 CLAT & AILET Papers
78. A handed over Rs. 5,000 to her neighbour B, who was an employee of a bank, and asked him to
deposit the money in her account. Instead of doing so, B spent the money. A sues the bank for
damages.

(A) The bank will be liable since B was its employee


(B) The bank will not be liable since B was not authorised to collect money from A
(C) The bank will not be liable since A gave money to B in his capacity as a neighbour and not as an
employee of the bank
(D) The bank will not be liable since this is a criminal act

79. PRINCIPLE:
When a criminal act is done by one person in furtherance of the common intention of himself and
some other persons, each of such person is liable for the act in the same manner as if it were done
by each one of them.

A, B and C decided to commit burglary. They broke into a locked house. However, before they had
finished their work, a domestic servant appeared from the out-house and started shouting. A, B and
C left the house and started running away. They were pursued by a small crowd. A, on being caught
by X, one of the persons pursuing them, stabbed him and ran away. By the time B and C had
disappeared. X died on account of the stab wounds. Later, the police arrested all the three. They
were charged for attempted burglary and murder of X.

(A) Along with A (the person who stabbed X) B and C are also guilty of murder because A stabbed X
in furtherance of common intention.
(B) Along with A, B and C are also guilty of murder because A, at the time of stabbing X, was acting
on behalf of B and C and he wanted to save not only himself but B and C as well.
(C) A alone is guilty of murder because though there was common intention to commit the offence
of burglary, there was no common intention to commit the offence of murder.
(D) None of the above

80. PRINCIPLE:
A person is liable for any damage which is the direct consequence of his/ her unlawful act, as long
as the consequence could have been foreseen by a reasonable person.

During a scuffle, A knocked B unconscious and then placed B at the foot of a hill at night, when the
temperature was around one degree centigrade. B suffered from hypothermia and had to be
hospitalised for a week. B sues A.
(A) B will be liable for the act, since the consequence was a direct and foreseeable consequence of
his act
(B) B will not be liable because he had no intention of causing hypothermia
(C) B will not be liable because he did not know that A would suffer from hypothermia
(D) B will be liable since A was hospitalised

Previous Years
CLAT & AILET Papers Page 161
81. PRINCIPLE: The owner of a land is entitled to the column of air space above the surface ad infinitum.
But, the right is restricted to such height as is necessary for the ordinary use and employment of his
land, and the structure on it.

A had constructed a single-storeyed house on a corner site. He had no intention of building an


additional floor. B his neighbour, who ran an internet parlour got a hoarding made, which protruded
over A's house at a height of around 6 feet above the terrace. A sues B for trespass.
(A) A will succeed since B' s act amounts to trespass
(B) A will not succeed since he was anyway not planning to build an additional storey
(C) A will not succeed since the hoarding is not obstructing him
(D) A will not succeed since B has a right to erect a hoarding

82. PRINCIPLE:
Trespass to land is the wrongful and unwarranted entry upon the land of another.
A purchased a ticket to watch a movie in a theatre. After the show got over, A refused to leave the
theatre. The owner of the theatre sues A for trespass
(A) He will not succeed since A had purchased a ticket
(B) He will not succeed since A has a right to remain wherever he wants to after the show gets over
(C) He will succeed since A's authorisation to remain in the theatre ends with the show
(D) He will not succeed since A did not have the intention to trespass

83. PRINCIPLE:
Negligence is the breach of a duty caused by an omission to do something which a reasonable
person would do or an act which a prudent and reasonable person would not do. An action for
negligence proceeds upon the principle that the person has an obligation or duty on the part of the
defendant, which he/she breaches, leading to damage.
A, a surgeon operated on B. Subsequent to the operation, B complained of pain in his abdomen. On
examination, it was discovered that A had left a mop in B' s stomach while operating.
(A) A will be liable for negligence, since there is a breach of the duty of care
(B) A will not be liable for negligence since it was an accident
(C) A will not be liable for negligence since no real damage was done
(D) A will not be liable for negligence since it is not a reasonable act

84. PRINCIPLE:
A contract requires a proposal and an acceptance of the proposal. It is necessary to make a binding
contract, not only that the proposal be accepted, but also that the acceptance is notified to the
proposer. (Common for Q. No. 84, 85 and 86)

A sent a letter to B stating that he was willing to sell to B, 10 bags of rice at Rs. 20/- each. B made
telephone call to A to inform him, that he had accepted the offer. Just as he was stating his acceptance,
a very noisy aircraft flew over B’ s house. A was not able to hear the acceptance.
(A) There is no contract since the acceptance has not been notified to A
(B) There is no contract since the reply also has to be in writing
(C) This is a valid contract since B conveyed his acceptance to A. It does not matter if A heard it or
not
(D) None of the above

Previous Years
Page 162 CLAT & AILET Papers
85. A sent a letter to B stating that he was willing to sell to B, 10 bags of rice at Rs. 20/- each. B wrote
a letter to A accepting the offer and posted it,
(A) The contract is entered into the moment B posts the letter
(B) The contract is entered into only after A receives and reads the letter
(C) The contract is entered into only after A receives the letter, reads it and then gets back to B
(D) The contract is entered into the moment A makes the offer

86. A sent a letter to B stating that he was willing to sell to B, 10 bags of rice at Rs. 20/- each. B sent
an e-mail to the address mentioned in the letter-head, accepting the offer.
(A) The contract is entered into the moment the mail reaches A’ s server
(B) The contract is entered into only after A sees the e-mail
(C) The contract is entered into when A sent the letter
(D) The contract is entered into the moment B receives an automated delivery receipt

87. PRINCIPLE:
An agreement becomes a contract when it is entered into between two or more people with each
other's free consent. Two or more people are said to consent when they agree to the same thing in
the same sense. Consent is said to be free when it is not caused by coercion, undue influence,
fraud, misrepresentation or mistake.

A went to a doctor to get himself treated. The doctor asked him to sign a “consent form” and said
that unless he signs it, he will not examine him. A signed the form waiving his right to sue the doctor.

(A) This is a valid contract since A understood and consented to the terms
(B) This is not a valid contract since A had not other option, but to sign
(C) This is a valid contract since it is a doctor-patient relationship
(D) None of the above

88. PRINCIPLE:
Where both parties to an agreement are under a mistake as to matter of fact essential to the
agreement, the agreement is void.

A had a piece of land. He believed that the value of the land was Rs. 1,000 per square foot. B knew
that the value of the land was infact Rs. 1,500 per square foot. However, he did not inform A and
purchased the land at Rs. 1,000.
(A) The contract is valid since A should have been more careful
(B) The contract is valid since B paid for the land .
(C) The contract is void since A was mistaken about the cost of the land
(D) The contract is valid since the land has already been purchased

89. PRINCIPLE:
The object of an agreement is lawful unless it is forbidden by law; is of such a nature that, if
permitted, it would defeat the provisions of any law; is fraudulent; involves or implies injury to the
person or property of another person; the court regards it as immoral; it is opposed to public policy.
(Common for Q. No. 89, 90, 91 and 92)

A and B entered into a contract, whereby A agreed to get married to B, if her parents paid A Rs.
1,00,000 before the wedding. B' s parents failed to pay the promised amount. A sues B and her
parents.
Previous Years
CLAT & AILET Papers Page 163
(A) A will succeed because all the requirements of a contract are met
(B) A will succeed since the payment of the amount was the condition precedent for the wedding
(C) A will not succeed since the contract is void, its object being against the law
(D) A will not succeed since B was not a party to the contract

90. A was arrested by the police for committing an offence and was subsequently granted bail by the
Court. One of the conditions imposed by the Court for granting bail was that A arrange for a surety
for Rs. 50,000. A approached B to be a surety. B agreed, but insisted that they enter into a contract
whereby A would deposit Rs. 50,000 into B's account, which would be returned to A by B after the
case was over.
(A) The contract would be void since its effect is to defeat the provisions of the Cr. P.C.
(B) The contract would be void because A cannot enter into a contract when a criminal case is going
on against him
(C) The contract would be void because the Court is not a party to it
(D) The contract would be valid

91. A and B, a Hindu couple were married to each other. Owing to differences between them, they
decided to get divorced. They entered into a contract laying down the conditions that both parties
had to adhere with. One of the terms of the contract was that their children would not be entitled to
claim the ancestral property of A, the husband.
(A) The contract will be void since the children are not a party to the contract
(B) The contract will be void since its terms since it is opposed to the personal laws of the parties
(C) The contract will be valid
(D) The contract will be void since a woman cannot enter into a contract with her husband

92. The BCCI decided to hold an auction to sell IPL teams. 12 bidders registered for the auction.
Unknown to the BCCI, these 12 bidders had entered Into a contract that they would not bid more
than a certain amount.
(A) The contract will be void because BCCI is not a party to it
(B) The contract will be void since the object of the contract is to cause a loss to the BCCI
(C) The contract will be void since the maximum number of parties to a contract as per Indian law is
10
(D) The contract will be valid

93. PRINCIPLE:
Every agreement by which anyone is restrained from exercising a lawful profession, trade or business
of any kind, to that extent is void.

A company entered into a contract with Coca Cola Ltd. to bottle soft drinks produced by Coca Cola.
One of the terms of the contract was that the company would not bottle soft drinks for any other
competitor of Coca Cola during the pendency of the contract.
(A) The contract is void since it restrains the lawful right of the company to engage in business
(B) The contract is void because there is no consider ration being paid
(C) The contract s valid
(D) The contract is valid since the competitor is not a party to the contract

Previous Years
Page 164 CLAT & AILET Papers
94. PRINCIPLE:
An agreement is void to the extent that it restricts absolutely, a party from enforcing his contractual
rights by usual rights by usual proceedings in any ordinary court.

A and B entered into a transaction in Delhi for sale of goods based in Delhi. The contract stated that
in case of a dispute, only civil courts in Chennai would have jurisdiction. Neither A, nor B resided or
carried on business in Chennai.
(A) The contract is valid since it is still an Indian court that has jurisdiction
(B) The contract is void since it ousts the jurisdiction of the competent court
(C) The contract is void since the consent of the court is required before entering into such a
contract
(D) The contract is valid since Chennai courts have expertise with respect to contractual matters

95. PRINCIPLE:
An Agreement, the meaning of which is not certain, or capable of being made certain, is void.
A and B, who were brothers, entered into an agreement which stated that A would sell his entire
share of the ancestral property to B. The agreement did not mention the specific details (like survey
number) of the property.
(A) The contract will be void since it is vague
(B) The contract will be void because it is not possible to ascertain the property which is being
referred to
(C) The contract will be void since it is illegal
(D) The contract is valid since there is clarity about the property sought to be sold

96. PRINCIPLE:
Whoever intending to take dishonestly (with an intention to cause wrongful loss to another or wrongful
gain to himself) any moveable property without that person's consent moves that property in order to
such taking, is said to commit theft. (Common for Q. No. 93 and 97)

A had lent his watch to B for a month. Two days after he had done so, he walked into B' s office to
find the watch on B' s table. He decided to take the watch back. A was prosecuted for theft.
(A) A is not guilty of the offence since he owned the watch.
(B) A is not guilty of the offence since he was not doing the act dishonestly.
(C) A is guilty of the offence since he caused wrongful loss to B.
(D) A is not guilty of the offence since he had lent the watch to B only for a short period of time.

97. A handed over his watch to B for safekeeping. B sells the watch to C, which he was not authorised
to do. B is prosecuted for theft.
(A) B is guilty of theft because he gained from the act.
(B) B is not guilty of theft because he was in possession of the watch.
(C) B is not guilty of theft because A had given him the watch.
(D) B is guilty of theft because caused wrongful loss to A.

Previous Years
CLAT & AILET Papers Page 165
98. PRINCIPLE:
Whoever intentionally puts any person in fear of any injury to that person, or to any other, and
thereby dishonestly induces that person so put in fear to deliver to any person any property, commits
extortion.
A entered B' s house, caught hold of B' s daughter C and threatened to stab her if B did not give him
Rs. 10,000 immediately. B did so. A is prosecuted for extortion.
(A) A has committed theft, not extortion.
(B) A has committed extortion.
(C) A has committed both theft and extortion.
(D) A has not committed any offence.

99. PRINCIPLE:
Theft is robbery if in order to committing of the theft, or in committing the theft, or in carrying away
or attempting to carry away property obtained by theft, the offender, for that end, voluntarily causes
or attempts to cause to any person death or hurt or fear of instant death or instant hurt. (Common
for Q. No. 99 and 100)

A entered B's house to take away her TV. When he was carrying the TV out of the house, he
encountered B near the door. He left the TV behind and ran away.
(A) A has committed theft.
(B) A has committed robbery.
(C) A has committed both theft and robbery.
(D) A has neither committed theft nor robbery.

100. A entered B' s house and was taking away her wallet and leaving the house, when he encountered
B. He dropped the wallet, but shot her while escaping
(A) A has committed robbery.
(B) A has committed theft, but not robbery.
(C) A has neither committed theft, nor robbery.
(D) A has committed both- theft and robbery.

101. RINCIPLE:
Whoever with the intent to cause, or knowing that he is likely to cause wrongful loss or damage to
the public or to any person, causes the destruction of property, or any such change in any property
or in the situation thereof as destroys or diminishes its value or utility, of affects it injuriously,
commits mischief. (Common Q. No. 101 and 102)

A went to B' s house for dinner. He saw that B had a statue made of ice, which was kept in a freezer,
A removed the statue from the freezer for a few minutes, which led to the hand of the statue
changing shape.
(A) A has not committed mischief, since he did not have the intention to change the shape of the
hand of the statue.
(B) A has committed mischief because he should have known that his act was likely to damage the
statue.
(C) A has not committed mischief because the statue did not get destroyed.
(D) A did not commit mischief because the value of the statue has not diminished.

Previous Years
Page 166 CLAT & AILET Papers
102. A lent his laptop to B. When in possession of the laptop, B inserted a pen drive into the laptop.
When he did a virus scan, he realised that the pen drive was infected. Since he urgently required a
file that was on the laptop, he nevertheless opened the files on the pen drive, in the process infecting
the laptop .
(A) A has committed mischief because he has affected the laptop injuriously.
(B) A has not committed mischief because he did not intend to do so.
(C) A has not committed mischief because the laptop continued working.
(D) A has not committed mischief because the laptop was in his possession.

103. PRINCIPLE:
Whoever enters into or upon property in the possession of another with the intent to commit an
offence or to intimidate, insult or annoy any person in possession of such property or having lawfully
entered into or upon such property, unlawfully remains there with an intent to intimidate, insult or
annoy any such person, or with the intent to commit an offence commits criminal trespass.

A went to Delhi Law College to participate in a competition. After participating in the competition, A
hid inside the campus so that he could steal a few books from the Iibrary,
(A) A has committed theft.
(B) A has committed criminal trespass.
(C) A has not committed criminal trespass, because he entered with permission.
(D) A has not committed any offence.

104. PRINCIPLE: A second suit will not be heard on the same facts between the same parties.
(Common for 104 and 105)

Nakshatra files a suit against Chandra for getting possession of a house on the ground: that the
property passed on to her through the will executed by Surya before his death. The suit gets
dismissed as Nakshatra fails to produce the will. Nakshatra files another suit against Chandra to
get the same house from the latter, on the ground that she was entitled to the house as being the
nearest heir of Sljfya.
(A) Nakshatra will succeed as she is the nearest heir of Surya '
(B) Nakshatra will not succeed as the facts in both the cases are the same
(C) Nakshatra will succeed as the facts in the two cases deal with different situations
(D) Nakshtra will succeed as the grounds in the second case were not taken in the' first one owing
to the mistake of her advocate

105. Mr. Sampath went to a party alone in his wife Sunanda's car. He usually used his wife's car after
office hours and his wife never objected to it. At the party, he got drunk. Instead of taking the risk of
driving the car, he requested his friend Mr. Vivek to drive him back home in Sunanda's car. Mr. Vivek
was quite sober since he had moderately consumed alcohol. On the way, Vivek knocked down a
boy and injured his leg. Subsequently, on behalf of the boy a claim for compensation was brought
against Mrs. Sunanda since the car belonged to her and it was registered in her name. The insurance
company refuses to pay compensation because the police report says that the person driving the
car at the time of accident had consumed alcohol.
(A) Sunanda is liable to pay compensation, although she did not authorize Mr. Vivek to drive the car
which caused the accident, yet it was registered in her name
(B) Sunanda is not liable to pay compensation because it is the liability of Mr. Sampath who had
permission to drive the car and on the basis of this permission, he requested Mr. Vivek who was
driving the car at the time of the accident

Previous Years
CLAT & AILET Papers Page 167
(C) Mr. Vivek is liable to compensate the boy fully because he was negligent in driving the car
(D) Sunanda, Sampath and Vivek are all jointly liable to compensate the boy.

SECTION – D : LOGICAL REASONING

106. If it is true that 'men are seldom honest' , which of the following statement conveys the same
meaning?
(A) At least one man is not honest (B) Men are not honest
(C) At least on one occasion men are not honest (D) Sometimes men are honest

107. If it is false that 'men always obey the law', which one of the following statements is doubtful ?
(A) Sometimes men do not obey the law (B) All men obey the law
(C) Men never obey the law (D) Some men obey the law

108. lf it is false that 'all men strive for the happiness of all', which one of the following statements is true
beyond doubt ?
(A) All men seek their own happiness
(B) There are men who do not desire for the happiness of all
(C) No men strive for the happiness of all
(D) There are men who do not strive for the happiness of all

109. From among the given alternatives, identify the missing premise.
1st Premise : If a plane triangle is possible, then perfect straight line is possible
2nd Premise : If perfect straight line is possible, then Euclid is right
3rd Premise :
4th Premise : But Riemann is right.

Conclusion: Therefore a plane triangle is not possible.


(A) If Riemann is right, then Euclid is right (B) If Riemann is wrong, then Euclid is right
(C) If Euclid is right, then Riemann is wrong (D) If Riemann is right, then Euclid is not wrong

110. From among the given alternatives, identify the conclusion of the following argument.
If A is B, then C is not D.
If C is not D, then E is not F.
If E is not F, then G is not H.
Therefore
(A) If G is not H, then A is B (B) If A is B, then G is not H
(C) If G is H, then A is B (D) If A is not B, then G is H

111. From among the given alternatives, identify the conclusion of the following argument.
You cannot taste success in life unless you work hard.
You cannot be happy unless you taste success in life.
Therefore
(A) You can be happy unless you work hard.
(B) You cannot be happy unless you work hard.
(C) You cannot work hard unless you are happy.
(D) You can work hard unless you are happy.

Previous Years
Page 168 CLAT & AILET Papers
112. If the statement "All men desire their own welfare always" is true, then which of the following
statements must be false?
(A) There are men who desire their own welfare sometimes
(B) Some men desire their own welfare always
(C) There are men who always desire their and others' welfare
(D) It is not the case that all men desire their own welfare always

113. From among the alternatives, identify the statement which states the same fact as the statement
'The dual nature of light is an enigma' .
(A) The nature of light is an enigma.
(B) Light has some enigmatic properties.
(C) Two differing arguments regarding the nature of light are obviously wrong.
(D) It is surprising and puzzling that according to one study, light is made up of one type of
constituents and according to another, light is made up another type of constituents.
114. Identify the statement which can be neither proved nor disproved.
(A) I have head-ache.
(B) Yesterday, New York recorded 20°C at 12 noon.
(C) The gang war that broke out last week was due to business rivalry.
(D) History of Kannada literature states that Nripatunga is the earliest poet.

115. Identify the statement which cannot be doubted.


(A) Sense experience provides true knowledge.
(B) It is reasonable to accept science and most unreasonable to believe any religion.
(C) I am alive.
(D) Whenever there is unrest in society, the police will be alert.

116. If it is true that 'good governance ensures law and order', then identify the statement which has to
be accepted along with the given statement.
(A) A healthy society is governed by maximum number of laws.
(B) A strong government uses force to enforce law.
(C) An able government very effectively uses the law to promote peace and prosperity.
(D) Law is indispensable for the society.

117. Identify the statement which cannot be accepted.


(A) Mahabharatha is an ancient epic.
(B) Many Indian languages have their origin in Sanskrit.
(C) According to the heliocentric theory, the Earth is the centre of the solar system.
(D) Indian Constitution has been amended many times.

118. If it is true that 'religious fundamentalism is dangerous to society’, which one of the following statements
is also true?
(A) Religion without reason is dangerous to the society.
(B) Belief in religions is dangerous to the society.
(C) Religious pluralism is dangerous to the society.
(D) Disrespect for other religions is dangerous to the society.

Previous Years
CLAT & AILET Papers Page 169
119. Identify the argument which has to be accepted when the premiss is accepted as true.
(A) A and B are subsets of C.
Therefore A and B must have at least one common element.
(B) Every man desires happiness.
Therefore happiness is desirable.
(C) God is not perceivable.
Therefore God does not exist.
(D) Nothing comes out of nothing.
Therefore something must have existed always.
120. Identify the argument which cannot be accepted.
(A) All Indians are Asians.
All Asians are dark-coloured.
Therefore All dark coloured are Indians.
(B) Amar is a friend of Basava.
Therefore Basava is a friend of Amar.
(C) All positive integers less than 100 have only 2 digits.
Therefore all positive integers with 2 digits are less than 100.
(D) All spinsters are unmarried women.
Therefore all unmarried women are spinsters.

121. Till Nineties, clearly the policy of the government was guided by the philosophy of socialism. Under
its influence, the government tried to exercise control in every field. But today we hear about
disinvestment and privatization. Hence, there is a clear shift from socialistic principles.
Which one of the following assertions, if true, contradicts the passage mentioned above?
(A) That government is the best government which governs least.
(B) Only enterprise in a free society contributes to the real progress.
(C) Not withstanding the alleged merits of laissez-faire implicit in the passage, lack of effective.
control by the state results in the exploitation of many by a few.
(D) The state is guilty of the dereliction of duty.

122. Geographical division of the Earth resulting in the so-called sovereign nations is artificial because it
is man-made, just as division of society on the basis of religion, language, caste, etc., is artificial.
While the so-called intellectuals raise hue and cry about the latter, they maintain divine silence
about the former. Does it not amount to double standard ?
Which of the following statements, if true, is the essence of the passage mentioned above?
(A) Geographical division of the Earth is of divine origin.
(B) Only division of society on the basis of religion, language, etc., is man-made.
(C) The response to geographical and sociological divisions are very different.
(D) Patriotism is a barren concept.
[Option C in the original paper was “The consequences of geographical and sociological divisions are very different”. As none
of the options are matching with the question, option (C) has been changed to fit the sprit of the question.]

123. It is argued that prosperity depends upon production which means more production more
consumption. However, it is not the case. Consumption should be need based. Just as intake of
food in excess of digestion is pointless so also consumption of natural resources beyond requirement
is pointless.

Previous Years
Page 170 CLAT & AILET Papers
Which of the following statements, if true, contradicts the spirit of the passage?
(A) Prosperity without consumption is ideal.
(B) Prosperity without consumption is shallow.
(C) Prosperity is possible when natural resources are not abused.
(D) To increase consumption we cannot have any alibi.

124. It is true that agriculture is the backbone of a nation's economy. But it is fallacious to argue that only
agriculture should receive overriding importance. Agriculture also depends upon other sectors like
generation and distribution of power, irrigation, transportation, etc. If any one sector suffers, its fall-
out is more than obvious.

Which of the following assertions, if true, is the summary of the given passage?
(A) Budget allocation for all fields must be equal.
(B) If adequate budget allocation in any one fiscal year for every sector is not possible, then each
year one sector can be provided with adequate funds.
(C) Budget allocation for each sector should be need based, neither less nor more, i.e.it should be
optimum.
(D) Reversion to ancient method of farming is the only way out.

125. Education is one field which is totally non-remunerative. Still worse, it is a liability. But then the
government is not a money-making institution. The bottom-line of prosperity is primary education.
Hence the government should raise resources elsewhere only to pour it on primary education. But
primary education cannot be provided without higher education. So higher education stands on par
with primary education.

Which of the following statements, if true, contradicts the passage above?


(A) Money should produce money which means that that which does not generate money has no
place.
(B) Education should raise its own resources.
(C) Education should be made as economical as possible.
(D) If the government cannot spread education, private enterprise should step in.

126. 4 statements are given below. Group two of them in such a way that one is logically impossible and
the other is empirically (factually) possible.
1. A plane equilateral triangle can also be a plane right angled triangle. (A)
2. Velocity of light in vacuum is constant. (B)
3. Consumption of fuel at this rate results in reversal to bullock-cart age. (C)
4. A tall man is a man. (D)
(A) A,B (B) B,D (C) C,D (D) A,C

127. 4 statements are given. Group two of them in such a way that one is logically possible and the other
is empirically (factually) highly improbable.
1. No theories of science can explain the origin of the Universe satisfactorily. (A)
2. Solar energy is inexhaustible. (B)
3. Liberalisation and rise in consumerism are directly related. (C)
4. God exists. (D)
(A) A,B (B) A,C (C) B,D (D) A,D

Previous Years
CLAT & AILET Papers Page 171
128. Four statements are given below. One of them can be demonstrated with the help of reason, while
another requires sense experience to ascertain the truth, Identify them.
1. Ravana killed Rama. (A)
2. Area of circle is given by the formula πr2 (B)
3. Methane is poisonous (C)
4. The absolute is pure consciousness (D)

129. How do you evaluate the statement "Autobiography of Mr M.K. Gandhi" is written by Mr. Mahadeva
Desai?
(A) False (B) True (C) Probable (D) Absurd

130. All faculties of understanding, which a human being enjoys, are necessarily limited in terms of
performance. If so, which of the following alternatives is justified by the preceding statement ?
(A) Human knowledge is necessarily imperfect.
(B) Perfection one's own knowledge is a distinct possibility, but man does not know how to employ
his faculties.
(C) When progress in knowledge is linear, one day or the other the ultimate truth will be discovered.
(D) Human knowledge is not necessarily perfect.

131. 'The Universe has necessarily a beginning and an end' and 'the Universe has boundary' together
imply
(A) The universe is finite in space and time.
(B) The universe is infinite in space and time.
(C) The universe is infinite in space but not in time.
(D) The universe is finite in space but not in time.

132. Which of the following alternatives comes very close to the statement, 'Man is necessarily an evil by
nature'?
(A) As a matter of fact, man is an evil by nature.
(B) Man is not an evil by nature, but circumstances tend to make him so.
(C) Man cannot be an evil by nature.
(D) Actually, man is not an evil.

133. Which of the following alternatives is farthest from the statement, 'Sound waves cannot travel in
vacuum' ?
(A) Sound waves require some medium to travel.
(B) Sound waves may travel in vacuum very slowly.
(C) Sound waves can travel only in vacuum.
(D) Sound waves may not travel in vacuum.

134. Which of the following alternatives contradict the statement 'deterministic laws of physics are
characteristically Newtonian'.
(A) Indeterminism and physics cannot go together.
(B) Indeterminism is inherent in Newtonian system.
(C) Determinism in physics is a flaw.
(D) Determinism in physics is only apparent.

Previous Years
Page 172 CLAT & AILET Papers
135. Identify the alternative which best explains the relation between the arguments mentioned below:
Arguments:
1. It is highly probable that every student of law at University level would have done a course on
principles of ethics and Rama being a student of law, in all probability, would have done a
course on principles of ethics.
2. At University level, every faculty aims at super speciality for which emphasis is laid upon structuring
the curricula in such a way that every faculty is treated as a closed system so that no inter-
disciplinary study is acceptable. Principles of ethics being essentially an integral part of
philosophy, most probably is out of any course on law.
(A) Two arguments are mutually independent.
(B) Two arguments are mutually supportive.
(C) Two arguments are supplementary.
(D) They are incompatible.

136. 'Greater than' is said to be transitive (relation) since A is greater than B and B is greater than C imply
A is greater than C. Similarly 'friend of' is said to be symmetric since if A is a friend of B, then B is
a friend of A. If so, what kind of relation is 'analogous'?
(A) Transitive (B) Symmetric
(C) Identical (D) Transitive and symmetric

137. Since an object is identical with itself ‘being identical’ is called a reflexive relation and if a relation is
transitive, symmetric and reflexive, then such a relation is called 'equivalence'. If so, what kind of
relation is 'different'?
(A) Equivalence (B) Inequivalence (C) Asymmetric (D) Transitive

[Option D in the original paper was “Non-Transitive”. As both options (B) and (D) are matching with the question, option (D)
has been changed to “Transitive” to fit the question.]

138. Which of the following having the same form contradicts the statement 'guilty should be punished' ?
(A) Guilty will not be punished (B) Guilty may not be punished
(C) Guilty are not punished (D) Guilty should not be punished

139. If it is true that 'no innocent persons should be punished' and' Socrates is innocent', which of the
following is the correct conclusion?
(A) Socrates is not punished (B) Socrates may not be punished
(C) Socrates should not be punished (D) Socrates will not be punished

140. Identify the alternative which gives true picture of the relation between the arguments listed below:
1. A healthy society is characterised by overall development of every section. As in the case of
human body no organ is less prominent, so also in a society no section is less important. If any
human organ is sick, then the human being is sick. Similarly, if any section of society is sick,
then the society is sick.
2. It is fallacious to think that priority to production of agricultural products alone promotes the
prosperity of farmers, Prosperity has too many discussions. Transportation, communication,
network, sound economic policy, to mention a few are equally important to the economic wellbeing
of farmers. Hence they also deserve consideration.
(A) These two are irrelevant (B) They are interdependent
(C) They are analogous (D) They are compatible

Previous Years
CLAT & AILET Papers Page 173
SECTION – E : MATHEMATICS

141. Which one of the following is same as 30% of 40% of 560?


(A) 60% of 40% of 280
(B) 15% of 80% of 280
(C) 30% of 40% of 280
(D) 15% of 80% of 140

 
= , then the value of  4 + 2y − x  is
x 4
142. If
y 5  7 2y + x 
3 8
(A) (B) 2 (C) 1 (D)
7 7

143. The average height of 10 students in a class is 105 cms. If 20 more students with an average height
of 120 cms join them, what will be the average height be?
(A) 105 cms (B) 110 cms (C) 112 cms (D) 115 cms

144. Anil is half his father's age now. Fifteen years ago, he was one-third of his father's age. How old will
Anil be after five years?
(A) 45 years (B) 40 years (C) 35 years (D) 30 years

145. 10 men can finish the construction of a hut in 8 days. How many men are needed to finish the same
in half a day?
(A) 80 (B) 100 (C) 120 (D) 160

146. India's Republic day in 1996 fell on Friday. Which week day it will be celebrated in the year 2000?
(A) Tuesday (B) Wednesday (C) Thursday (D) Friday

147. An express train travels at an average speed of 120 km per hour, stopping for 4 minutes after every
80 kms. How long will it take the train to reach a destination 960 km away?
(A) 8 hrs 40 mts (B) 8 hrs 44 mts (C) 8 hrs 48 mts (D) 8 hrs 56 mts

148. In a family, the father took one-fourth of the cake and he had three times more than the others had.
The total number of family members is
(A) 3 (B) 7 (C) 10 (D) 12

149. A clock gains 5 minutes every hour. Then the angle traversed by the seconds hand in one minute will
be
(A) 390° (B) 380° (C) 360.5° (D) 360°

150. A right circular cone and a right cylinder have the same radius and the same volume. The ratio of the
height of the cone to that of the cylinder is
(A) 5 : 3 (B) 5 : 2 (C) 1 : 3 (D) 3 : 1

Previous Years
Page 174 CLAT & AILET Papers
AILET Question Paper 2011
SECTION - A : ENGLISH

Read the passage and answer the questions following it. The questions are to be answered on
the basis of what is stated or implied in the passage. For some of the questions, more than one of
the choices could conceivably answer the question. However, you are to choose the best answer;
that is, the response that most accurately and completely answers the question.

Roger Rosenblatt’s book ‘Black Fiction’, manages to alter the approach taken in many previous studies by
making an attempt to apply literary rather than socio-political criteria to its subject. Rosenblatt points out
that criticism of Black writing has very often served as a pretext for an expounding on Black history. The
recent work of Addison Gayle’s passes a judgment on the value of Black fiction by clear political standards,
rating each work according to the ideas of Black identity, which it propounds.

Though fiction results from political circumstances, its author reacts not in ideological ways to those
circumstances, and talking about novels and stories primarily as instruments of ideology circumvents
much of the fictional enterprise. Affinities and connections are revealed in the works of Black fiction in
Rosenblatt’s literary analysis; these affinities and connections have been overlooked and ignored by
solely political studies.

The writing of acceptable criticism of Black fiction, however, presumes giving satisfactory answers to quite
a few questions. The most important of all, is there a sufficient reason, apart from the racial identity of the
authors, for the grouping together of Black authors? Secondly, what is the distinction of Black fiction from
other modem fiction with which it is largely contemporaneous? In the work Rosenblatt demonstrates that
Black fiction is a distinct body of writing, which has an identifiable, coherent literary tradition. He highlights
recurring concerns and designs, which are independent of chronology in Black fiction written over the past
eighty years. These concerns and designs are thematic, and they come form the central fact of the
predominant white culture, where the Black characters in the novel are situated irrespective of whether
they attempt to conform to that culture or they rebel against it.

Rosenblatt’s work does leave certain aesthetic questions open. His thematic analysis allows considerable
objectivity; he even clearly states that he does not intend to judge the merit of the various works yet his
reluctance seems misplaced, especially since an attempt to appraise might have led to interesting results.
For example, certain novels have an appearance of structural diffusion. Is this a defeat, or are the authors
working out of, or attempting to forge, a different kind of aesthetic? Apart from this, the style of certain
Black novels, like Jean Toomer’s Cane, verges on expressionism or surrealism; does this technique
provide a counterpoint to the prevalent theme that portrays the fate against which Black heroes are pitted,
a theme usually conveyed by more naturalistic modes of expressions?

Irrespective of such omissions, what Rosenblatt talks about in his work makes for an astute and worthwhile
study. His book very effectively surveys a variety of novels, highlighting certain fascinating and little-known
works like James Weldon Johnson’s Autobiography of an Ex-Coloured Man. Black Fiction is tightly
constructed, and levelheaded and penetrating criticism is exemplified in its forthright and lucid style.

Previous Years
Page 176 CLAT & AILET Papers
Answer the following questions indicating your response for each question:

1. The author of the passage raises an objection to criticism of Black fiction like that by
Addison Gayle as it
(a) Highlights only the purely literary aspects of such works.
(b) Misconceive the ideological content of such fiction.
(c) Miscalculate the notions of Black identity presented in such fiction.
(d) Replaces political for literary criteria in evaluating such fiction.

2. The primary concern of the author in the above passage is


(a) Reviewing the validity of a work of criticism.
(b) Comparing various critical approaches to a subject.
(c) Talking of the limitations of a particular kind of criticism.
(d) Recapitulation of the major points in a work of criticism.

3. The author is of the opinion that Black Fiction would have been improved had Rosenblatt
(a) Undertaken a more careful evaluation of the ideological and historical aspects of Black Fiction
(b) Been more objective in his approach to novels and stories by Black authors
(c) Attempted a more detailed exploration of the recurring themes in Black fiction throughout its
history
(d) Calculated the relative literary merit of the novels he analyzed thematically

4. Rosenblatt’s discussion of Black Fiction is


(a) Pedantic and contentious (b) Critical but admiring
(c) Ironic and Deprecating (d) Argumentative but unfocused

5. According to the given passage the author would be LEAST likely to approve of which
among the following?
(a) Analyzing the influence of political events on the personal ideology of Black writers
(b) Attempting a critical study which applies sociopolitical criteria to the autobiographies of Black
authors
(c) A literary study of Black poetry that appraises the merits of poems according to the political
acceptability of their themes
(d) Studying the growth of a distinct Black literary tradition within the context of Black history

6. From the following options, which does the author not make use of while discussing Black
Fiction?
(a) Rhetorical questions (b) Specific examples
(c) Comparison and contrast (d) Definition of terms

Each of the following sentences contain four parts, marked a, b, c, and d. One of these parts is
incorrect. Indicate the letter of the incorrect part.

7. The Baby showed a noticeable / distaste for / these kind / of prepared baby food.
a b c d

8. They cannot go camping right now / because / they are taking care of / a three-weeks-old baby.
a b c d

Previous Years
CLAT & AILET Papers Page 177
9. If it / will rain / this afternoon, we will have / to cancel the picnic.
a b c d

10. Are / you familiar of / the latest / scientific developments in the field?
a b c d

Direction: In the following questions, some of the sentences have errors and some have none. Find out
which part of the sentence has an error.

11. Men are apt to / pick up holes / in the views of those whom they do not like. / No error
a b c d

12. The myth that some races /are naturally superior than others /has now been fully demolished. /
a b c
No error
d

13. The steep rise in the price of petroleum products / have affected / the economic development of the
a b c
developing nations. / No error
d

14. Kiran as well as Kamal / are leaving / for Mumbai. / No error


a b c d

15. Much water / has flown / under the bridge since then. / No error
a b c d

16. Everyone of the men present here / has given a day’s pay / as their contribution to the fund / No error
a b c d

17. Found guilty / on murder / the accused was sentenced to death. / No error
a b c d

Complete the sentences with suitable words indicating your option for each question

18. I saw _____movie last night.


(a) the (b) a (c) an (d) one

19. The RBI in consultation with Government of India has _____ working group to suggest measures for
revival of weak public sector banks.
(a) appointed (b) constituted (c) devolved (d) none of these

20. He stepped out and began to walk _____


(a) behind (b) around (c) besides (d) none of these

21. The lady received quick promotions when they saw how _____she was.
(a) lethargic (b) hostile (c) correct (d) hard working

Previous Years
Page 178 CLAT & AILET Papers
22. The shower in the bathroom needs mending as it has started _____
(a) trickling (b) soaking (c) dripping (d) overflowing

23. The person who is looking for sympathy talks _____


(a) petulantly (b) despicably (c) plaintively (d) deftly
24. The very nature of society is inimical _____ freedom.
(a) of (b) to (c) with (d) on

25. Wilbur Wright flew his airplane _____France in 1909.


(a) On (b) Upon (c) Until (d) Over

Direction (Q.Nos. 26 - 35): Fill in the blanks.


If you look very closely at a newspaper photograph you will see that the grey areas are made up of black
dots of various sizes. The lighter the shades of grey, the smaller the dots. In .....(26)..... area the dots are
larger. The process by which this .....(27)..... is achieved is called half-tone screening. First of all a negative
.....(28)..... is made of the picture. That is, the light and dark areas are .....(29)..... so that white becomes
..... (30)..... and black becomes white. This picture is projected .....(31)..... a special screen. The screen is
made of glass and has lines .....(32)..... on it both vertically and horizontally. So the screen is divided up
.....(33)..... tiny squares rather like graph paper. When the light .....(34)..... the negative is shone through
this .....(35)..... it is broken up into dots. This light lands on a metal plate that is coated with chemicals.

26. (a) smaller (b) darker (c) lighter (d) small

27. (a) effect (b) colour (c) tone (d) affect

28. (a) photograph (b) print (c) copy (d) paint

29. (a) reversed (b) removed (c) coloured (d) darkened

30. (a) dark (b) dotted (c) black (d) white

31. (a) into (b) above (c) through (d) on

32. (a) ruled (b) cut (c) reflected (d) deflected

33. (a) from (b) into (c) like (d) in

34. (a) from (b) of (c) on (d) at

35. (a) graph paper (b) framework (c) screen (d) paper

SECTION - B : GENERAL KNOWLEDGE

36. The world’s largest nuclear power station is


(a) Chernobyl (b) Kashiwazki-Kariwa (c) Fukushima-Daiichi (d) Kalpakkam

37. The First International Women’s Day was celebrated on


(a) 17.3.1911 (b) 8.3.1911 (c) 18.3.1911 (d) 9.3.1911
Previous Years
CLAT & AILET Papers Page 179
38. The nature and extent of nuclear accident in power plants is indicated as International Nuclear and
Radiological Event (INRE) and quantified in terms of the highest as
(a) Level 10 Accident (b) Level 12 Accident (c) Level 7 Accident (d) Level 6 Accident

39. The Minority Affairs Minister of Pakistan who was assassinated in March 2011
(a) Salman Taseer (b) Nawaaz Paintal (c) Shahbaz Batti (d) None

40. The present Central Chief Information Commissioner is


(a) Wajhat Habibullah (b) Satyananda Mishra (c) M. L. Sharma (d) Sushma Singh

41. The country that gets highest percentage of electricity from nuclear power stations is
(a) U. S. A. (b) Japan (c) China (d) France

42. The present Chief Election Commissioner of India is


(a) S. Y. Quraishi (b) H. S. Brahma (c) V. S. Sampath (d) Navin Chawl

43. Which of the following Chief Justices of India also became the Vice President of India?
(a) Patanjali Sastri (b) Hidayatullah (c) Sikri (d) None

44. India’s Foreign Exchange Reserve has become the _____ largest in the world.
(a) Eighth (b) Twelfth (c) Fourth (d) Eleventh

45. The first ever woman Secretary-General of SAARC, Ms. Fathimath Dhiyana Saeed is from
(a) Pakistan (b) Bangladesh (c) India (d) Maldives

46. Air-conditioning was invented by


(a) Carrier (b) Voltas (c) A. Volta (d) Sinclair

47. Motor Cycle was invented by


(a) Davidson (b) G. Daimler (c) Benz (d) MacMillan

48. Which of the following has the largest territory?


(a) U.S.A (b) Brazil (c) Canada (d) China

49. The country that recently joined the South Asian Association for Regional Co-operation was
(a) Myanmar (b) Iran (c) Iraq (d) Afghanistan

50. According to the survey conducted in India during 2001-2003, the highest percentage of deaths
were due to
(a) Cancer (b) Heart diseases (c) Asthma (d) HIV-AIDS
51. The 123 Agreement was entered into between
(a) Canada and India (b) France and India (c) USA and India (d) China and India

52. The largest state in India in terms of territory is


(a) Uttar Pradesh (b) Madhya Pradesh (c) Maharastra (d) Rajasthan

53. The first judge who was sought to be impeached under the Constitution was
(a) Justice V.Ramaswami (b) Justice Dinakaran
(c) Justice Sen (d) None
Previous Years
Page 180 CLAT & AILET Papers
54. The state having the maximum number of seats in Rajya Sabha is
(a) Maharastra (b) Uttar Pradesh (c) Tamil Nadu (d) Andhra Pradesh

55. Rajya Sabha members are elected for a period of


(a) 5 years (b) 4 years (c) 2 years (d) 6 years

56. The Supreme Court recently declared the appointment non-est of


(a) Navin Chawla (b) S. C. Mishra (c) P. J. Thomas (d) Both (b) and (c)

57. The maximum number of nominated members to Lok Sabha is


(a) One (b) Two
(c) Three (d) No one can be nominated

58. The special reservation of 4% in favour of Muslims was struck down as unconstitutional by
(a) High Court of Madras (b) High Court of Andhra Pradesh
(c) High Court of Delhi (d) High Court of Kerala

59. The Year Book of Industrial Statistics 2010 released by the UN Industrial Development Organization
ranks India among
(a) Top 12 countries (b) Top 15 countries (c) Top 4 countries (d) Top 10 countries

60. The European Union member states decided to temporarily withdraw the preferential tariff benefits
from August 2010 on the ground that there are significant shortcomings in the implementation of
three UN Human Rights Conventions in
(a) Sri Lanka (b) India (c) Pakistan (d) Nepal

61. The Eleventh South Asian Games was held in


(a) Kathmandu, Nepal (b) Hyderabad, India (c) Dhaka, Bangladesh (d) Colombo, Sri Lanka

62. Dr. Jean Mayer Award for Global Citizenship by the Tufts University, USA, was awarded in 2010 to
(a) Justice Verma (c) Narayana Murthy (b) M. C. Mehta (d) Sanjoy Hazarika

63. The constitutional advisor to the Constituent Assembly was


(a) Rajendra Prasad (b) B. N. Rau (c) B. R. Ambedkar (d) Kamat

64. Census takes place in India


(a) every 20 years (c) every 10 years (b) every 15 years (d) every 5 years

65. INTERPOL, a 147-nation Police Commission, has its Headquarters at


(a) Paris (b) Lyons (c) London (d) Frankfurt

66. Red Cross is an international society for relief of suffering in the time of war or disaster which has
headquarters at Geneva was founded in 1863 on the advocacy of
(a) Herbert Spencer (b) J.H. Dunant (c) W.H. Morgan (d) C.A. Barnord

67. Who among the following was the Chairman of Sixth Pay Commission?
(a) Justice Rathnaval Pandian (b) Justice Venkatachalliah
(c) Justice A.K. Majumdar (d) Justice B.N. Srikrishna

Previous Years
CLAT & AILET Papers Page 181
68. Who among the following was the Chairman of Sixth Pay Commission?
(a) Justice Rathnaval Pandian (b) Justice Venkatachalliah
(c) Justice A.K. Majumdar (d) Justice B.N. Srikrishna

69. Kyoto Protocol which was replaced by Cancun in December 2010 relates to
(a) Climate (b) Water (c) Air (d) Soil

70. Which of the following mountains has oldest rocks in India?


(a) Aravallis (b) Himalayas (c) Siwaliks (d) None

SECTION - C : LEGAL APTITUDE

71. PRINCIPLE:
A master shall be responsible for the wrongful acts of his servants in the course of his employment.
FACTS:
The Syndicate Bank was running a small savings scheme under which its authorised agents would
go round and collect small savings from several people on daily basis. These agents would get
commission, on the deposits so collected. Ananth was one such agent, collecting deposits from
factory workers engaged on daily wages. Though he regularly carried on his business for sometime,
slowly he started appropriating deposits for his personal use and one day he just disappeared. One
Fatima, who had been handing over her savings to him found that nearly for a month before his
disappearance, he was not depositing her savings at all. The Bank, when approached, took the
stand that Ananth was not its regular and paid employee and therefore, it was not responsible for his
misconduct. She files a suit against the Bank
(a) The Bank is not liable, since Ananth was not its salaried employee
(b) The Bank is liable, since Ananth was projected as its employee
(c) The Bank is not liable, since it did not know about misappropriation by Ananth
(d) None of the above

72. PRINCIPLE:
When the parties to an agreement agree on the same thing in the same sense, there arises legally
binding obligations between them.

FACTS: Zaverilal’s antique shop was a well-known shop of the locality. Taradevi, a socialite of the
locality, went to the shop and she was attracted by an earthern jar on display. Zaverilal explained to
her that the jar belonged to Hoysala period; and despite its earthern composition, it was very strong
and almost unbreakable. Taradevi replied to him that she was so captivated by the jar that it was
immaterial to her as to which period it belonged. She bought the jar and came home. She placed the
jar in a prominent place in her drawing room. One of her friends, an art critique, who happened to
visit her, told her that the jar was not at all an Antique, but Taradevi did not bother about it. One day,
it accidentally fell down and broke into pieces. Taradevi took up the matter with Zaverilal that his
both statements were wrong and therefore, he should pay damages to her.

(a) Zaverilal must compensate her, since his both the statements concerning jar were wrong
(b) Zaverilal need not compensate, since, Taradevi was not concerned with its antique value at all
(c) Zaverilal must compensate, because he carelessly made the statements
(d) None of the above

Previous Years
Page 182 CLAT & AILET Papers
73. PRINCIPLE:
Interfering with another’s goods in such a way as to deny the latter’s title to the goods amounts to
conversion and it is a civil wrong.
FACTS:
Ram went to the bicycle stand a park his bicycle and he found the stand fully occupied. Ram
removed a few bicycles in order to rearrange the stand and make some space for his bicycle. He
parked his bicycle properly and put back all the bicycles except the one belonging to Shyam. It was
rather negligent on the part of Ram and he was in fact in a hurry to get into his office. Somebody
came on the way and took away Shyam’s cycle. The watchman of the stand did not take care of it
assuming that the cycle was not parked inside the stand. Shyam filed a suit against Ram for
conversion
(a) Shyam would succeed because Ram’s act led to the stealing of his bicycle
(b) Shyam would not succeed, because Ram did not take away the bicycle himself
(c) Ram would not be held responsible for the negligence of the watchman
(d) None of the above

74. PRINCIPLE:
Damages are payable for breach of contract and the purpose of damages is to compensate for the
loss suffered and not to improve position more than what it would have been if the contract had been
duly performed.

FACTS:
A wanted to buy a house and he contracted with a surveyor S to inspect a particular house and value
it for him. S surveyed the house and valued it for Rs. 10 lakhs. S, however, failed to notice the
defective plumbing system in the house and had he taken note of it, the house would have been
worth only Rs. 8 lakhs. A followed S’ s advice and bought the house for Rs. 8 lakhs and thereafter
spent Rs. 4 lakhs for repairing the plumbing system. He filed a suit against S claiming Rs. 4 lakhs
as damages.

(a) A should get Rs. 4 lakhs, because that was the amount that he had to spend finally to make the
house worth living
(b) A should get Rs. 2 lakhs, since he paid additional Rs. 2 lakhs on account of S ‘s negligence
(c) A should get Rs. 4 lakhs, since that loss was attributable to S’s negligence
(d) None of the above

75. PRINCIPLE:
A right of action cannot arise out of an illegal activity.

FACTS:
A and B were thieves engaged in stealing cars and other vehicles. Once they stole a car; and
while driving off, they had to cross a city. They engaged a driver to drive them through the city,
since they did not know the route inside. The indicator lamp of the car was not working and the
thieves had not realised this and therefore, had not told about it to the driver. While driving, through
the city, the car was hit by another vehicle because of the faulty indicator. In the accident, the
driver was injured and he filed a suit against A and B.

Previous Years
CLAT & AILET Papers Page 183
(a) The driver would lose, because he was driving a stolen car
(b) The driver would win, because he was not a party to the stealing
(c) The driver would win, because he did not know anything about the stealing
(d) None of the above

76. PRINCIPLE:
Law does not take notice of trifles.

FACTS:
A proposes to his neighbour B that they both should go together for a morning walk. B agrees to the
proposal and it is decided that both of them would meet at a particular point at 6 AM from where they
would set off for the morning walk. Inspite of the agreement, B does not turn up. A waits for him at 6
AM every day for a continuous period of seven days. Thereafter he files a suit against B claiming
damages for the agony and mental torture suffered by him. Decide.

(a) B is guilty of breach of contract and is liable to pay damages


(b) There is no intention to enter into legal relations
(c) The matter is too small and the court will refuse to go into it
(d) None of the above

77. PRINCIPLE:
Limitation bars the remedy, not the right.

FACTS:
After the expiry of the period of limitation of three years, debtor Rohan makes a part payment of debt
to creditor Sohan. Sohan then files a suit against Rohan for recovery of the debt after two years from
the date of part payment. Decide.

(a) The part payment extends the period of limitation


(b) The suit is time barred as part payment is made after the expiry of period of limitation
(c) Fresh period of limitation begins from the date of part payment
(d) None of the above

78. PRINCIPLE:
Ignorance of law is no excuse.

FACTS:
A fails to file his income tax returns for ten years. The Income tax department issues to him notice
to show cause why proceedings should not be initiated against him for the recovery of the income
tax due from him with interest and penalty. Advise.

(a) He may take the plea that his advocate had advised him not to file the return as his income was
not taxable
(b) He may request the department to waive the interest and penalty
(c) A must pay the tax due as ignorance of law cannot be ground of defence
(d) None of the above

Previous Years
Page 184 CLAT & AILET Papers
79. PRINCIPLE:
Nobody shall make use of his property in such a way as to cause damage to others. Any such use
constitutes private nuisance, a wrongful act under Law of Torts.
FACTS:
Vasan was owing a house, adjacent to a cluster of houses, owned by Varadan. Varadan was leasing
out these houses whereas Vasan was living in his house. When Vasan was transferred to another
place, he leased out his house to a person suffering from AIDS. Fearing the spread of AIDS, the
tenants moved out of V aradan’ s houses. Varadan requested Vasan to evict AIDS patient and he
offered to fix a suitable tenant for V asan’ s house, if the AIDS patient is evicted. But Vasan refused
by arguing that AIDS would not spread as feared by V aradan’ s tenants. Varadan filed a suit against
Vasan.

(a) Varadan will win, because Vasan knowingly caused him fmancial damage
(b) Varadan will not win, because Vasan could lease his house to whomever he wanted
(c) Varadan will not win, because Vasan should not be held responsible for public mis-perception
(d) None of the above

80. PRINCIPLE:
One has to compensate another for the injury caused due to his wrongful act. The liability to
compensate is reduced to the extent the latter has contributed to the injury through his own negligence.
This is the underlying principle of contributory negligence.

FACTS:
Veerappa owns a farm at a distance of half a furlong from the railway track. He stored in his land the
stacks of dried up straw after the cultivation as is normal in farming. One day when the train was
passing through the track, the driver was negligently operating the locomotive by allowing it to emit
large quantities of spark. The high wind, normal in open fields, carried the sparks to the stacks
stored by Veerappa and the stacks caught fire thereby causing extensive damage. Veerappa filed a
suit against the Railways claiming damages. The Railways while acknowledging liability alleged
contributory negligence on the part of Veerappa.

(a) Veerappa was not liable since his use of land was lawful
(b) Veerappa’s farm being at a reasonable distance from the railway track, he cannot be held
responsible for the high winds
(c) Veerappa should have anticipated the possibility and hence he is liable for contributory negligence
(d) None of the above

81. PRINCIPLE:
A person is entitled to protect his property by using lawful means.

FACTS:
Ramlal is growing valuable vegetables and fruits in his farm and he has fenced the farm to prevent
the cattle from entering into it. In addition he has kept a ferocious dog to chase away intruding
urchins and cattles. Some children were playing in a nearby playground and the ball slipped into the
farm. A boy running after the ball came near the fence and shouted for the ball. But when there was
no response, he managed to creep into the farm to get the ball. The dog which was surreptitiously
waiting attacked the boy and badly mauled him. The boy’s parents filed a suit against Ramlal.

Previous Years
CLAT & AILET Papers Page 185
(a) Ramlal is not liable, since the fence and the dog are lawful means of protecting the property
(b) Ramlal is not liable for the boy trespassing and getting badly injured in that process
(c) Ramlal is liable, since an ordinary barking dog would have sufficed for the purpose
(d) None of the above

82. PRINCIPLE:
A person who commits an unlawful act towards another which can be imputed to him, must repair
the damage which the other person suffers as a consequence thereof.
FACTS:
Mr. Rajender Singh was riding his scooter on the right side of the road which is illegal as per the
Traffic Rules. Mr. Rajesh Chawla was driving his car in the opposite direction. The two vehicles
collided and resulted in loss of Rs. 50,000/- to Mr. Rajender Singh. This includes his medical
expenses and damage to the scooter. In this accident there is no fault on the part of Mr. Rajesh
Chawla.

(a) Mr. Rajendra Singh will not get any amount as damages
(b) Mr. Rajendra Singh will get full compensation
(c) Mr. Rajendra Singh will get part of compensation
(d) None of the above

83. PRINCIPLE:
When there is breach of contract, the resulting damages will have to be paid by the party breaching
the contract to the aggrieved party. However, the parties are free to agree as to damages payable in
advance in case there is breach of contract.

FACTS:
Mr. Ramesh entered into a contract with Mr. Ramakrishna for selling his green Alto Car for Rs. 3
lakhs. Mr. Ramakrishna was to pay Rs. 3 lakhs on or before 25th April 2005 and take possession of
the car. The party failing to honour the contract has to pay Rs. 40,000/- as damages to the other
party.
Mr. Ramakrishna has not performed his part of the contract. Mr. Ramesh is claiming Rs.40,000/-.
(a) Mr. Ramesh can sell the car in the second hand market and if he suffers any loss, then only he
can claim that amount from Mr. Ramakrishna
(b) Mr. Ramakrishna is liable to pay the agreed damages
(c) Mr. Ramesh has to prove that he has suffered Rs. 40,000/- as damages to get it
(d) None of the above

84. PRINCIPLE:
Every person has a right of self defence, if his life is under imminent threat.

FACTS:
Mr. Prashanth threatens Mr. Krishna that he will kill Mr. Krishna. After saying so, Mr. Prashanth
goes to his house saying that he would get his axe

(a) Mr. Krishna will have to run away.


(b) Mr. Krishna will have to go to the Police Station and file a complaint.
(c) Mr. Krishna cannot exercise the right of self defence.
(d) None of the above

Previous Years
Page 186 CLAT & AILET Papers
85. PRINCIPLE:
Whoever does any act so rashly or negligently as to endanger human life or the personal safety of
others is said to have committed an offence.
FACTS:
Mr. Mangeskar owns a Yamaha motorcycle which has very good pick up and speed. He is studying
in the IV semester of Mechanical Engineering degree course. One day it was getting late for the
college as he woke up late in the morning. He got ready and was rushing to the college so that he
would not miss the class. He was riding the motor cycle at a speed of 140 km per hour in Bangalore
city which was crowded. He was very good in riding the motorcycle. People who were using the road
got annoyed scared with the way Mr. Mangeskar was riding the motorcycle.

(a) Mr. Mangeskar has committed an act of rash and negligent driving
(b) Mr. Mangeskar is very good in driving, so there is no need for others to be panicky about his
driving
(c) Mr. Mangeskar is very studious student and he does not want to miss any class in the College
(d) None of the above

86. PRINCIPLE:
When two or more persons agree to do an illegal act, it is criminal conspiracy punishable with
imprisonment.

FACTS:
Mr. Bharath is a student of B.E. in Computer Science. He loves his computer very much. He
considers his computer as his close friend and companion. On 1.4.2006, while interacting with his
computer, he hacked into the Bank account of Mr. Javed and was successful in withdrawing money
from Mr. Javed’s bank account. He did it to please his girl friend.

(a) Mr. Bharath has committed an offence.


(b) Mr. Bharath has committed a cyber crime.
(c) Mr. Bharath has not committed the offence of criminal conspiracy.
(d) None of the above

87. PRINCIPLE:
Contractual liability is completely irrelevant to the existence of liability in tort (civil wrong).

FACTS:
X purchased a bottle of ginger-beer from a retailer. As she consumed more than 3/4 of the contents
of the bottle, she found a decomposed remains of a snail in the bottle. After seeing the remains of a
snail, she fell sick on the thought of what she consumed. She sued the manufacturer of the beer for
negligence, though there is no contractual duty on the part of the manufacturer.

(a) X cannot sue the manufacturer for negligence in the absence of a contract
(b) X cannot sue the retailer
(c) X can sue the manufacturer as he had a duty to take care to see that bottles did not contain any
other substance than the beer and hence liable to have broken that duty
(d) None of the above

Previous Years
CLAT & AILET Papers Page 187
88. PRINCIPLE:
The standard to determine whether a person has been guilty of negligence is the standard of care
which, in the given circumstances, a reasonable man could have foreseen.

FACTS:
The Agricultural University constructed 200 houses for its employees in its premises. Two huge bore
wells were sunk and motors were installed. They did not cover the pump rooms properly. A child, 6
years old, from one of the quarters was playing near the pumphouse. On hearing the noise of the
pump, she was curious to see the motor. She touched the motor that was not covered properly and
three of her fingers were cut.

(a) The parents of the child cannot sue the University on any grounds
(b) Inspite of the child’s act, her parents can successfully sue the University for damages
(c) The University can be made liable only to the extent of the cost of treatment as the child also
contributed to the incident
(d) None of the above

89. PRINCIPLE:
A person is liable for all the injurious consequences of his careless act.
FACTS:
Ram, a snake charmer, was exhibiting his talents to a group of people. One of the snakes escaped
and bit a child who had to be hospitalised for two days for treatment.

(a) Ram is liable to compensate the child’s family for his careless act
(b) Ram is not liable to anything as such things keep happening
(c) Ram is not in a position to compensate as he is poor
(d) None of the above
90. PRINCIPLE:
A contingent contract is a contract to do or not to do something, if some event, collateral to such
contract, does or does not happen.

FACTS:
A agrees to pay B a sum of Rs. 1 lakh if B marries C within a period of six months. B marries C
during the seventh month as the marriage hall was available only during that month. B claims Rs. 1
lakh from A.

(a) B can successfully claim Rs. 1 lakh from A


(b) B cannot claim Rs. 1 lakh from A as B married C after the duration set in the contract
(c) B can claim Rs. 1 lakh as well as special damages from A for having married C
(d) None of the above

91. PRINCIPLE:
Right to carry on any occupation, trade or business is a Fundamental Right under the Indian
Constitution. The State is under an obligation under the Directive Principles of State Policy to
organize agriculture and animal husbandry on scientific lines, and towards that goal, take steps to
prohibit cow slaughter.

Previous Years
Page 188 CLAT & AILET Papers
FACTS:
The State of X passed a legislation totally prohibiting cow slaughter. A, a butcher, trading in meat of
all animals including cows, challenged this legislation as violating the fundamental right to carry on
his business.

(a) The State of X cannot make a law taking away any fundamental right. Such law is null and void
(b) The State can prohibit cow slaughter to organize animal husbandry in scientific lines
(c) The right to carry on any occupation, trade or business is subject to reasonable restrictions.
Banning cow-slaughter is such a restriction to better animal husbandry production and it is
within the power of State
(d) None of the above

92. PRINCIPLE:
The Constitution guarantees freedom of religion and expects the citizen to respect national anthem as
a fundamental duty.

FACTS:
According to the religions tenets of a particular sect, singing the praise of any entity other than God
is forbidden. The child, belonging to that sect, refuses to sing national anthem in the course of
school assembly, in which every child must compulsorily participate. When the disciplinary action
was instituted against the child, the parents challenged the school’s order of singing the national
anthem.

(a) The School can take disciplinary action against the child as it is the fundamental duty to respect
national anthem
(b) The fundamental right to freedom of religion cannot be abridged by a fundamental duty imposed
by the State
(c) The girl is free to follow the religion of her choice and follow its rules. It cannot be said to be
disrespect on the part of the girl if she refuse to sing national anthem
(d) None of the above

93. PRINCIPLE:
All citizens shall have the right to freedom of speech and expression.

FACTS:
X, a famous writer and Novelist criticized another novelist, B stating that: “the novel of B is foolish, he
is a weakman, his Novel is indecent, his mind is impure, he is not of a good character, he should write
decent and good Novel.” Can X be sued for defamation?

(a) He is not liable because he has just expressed his personal views
(b) He is liable to be sued for defamation if his statement, was not true or said in mala fide intention
(c) He cannot be liable because he has fundamental right to freedom of speech and expression
(d) He cannot be sued, because both are writer and novelists and both can criticize each other

Previous Years
CLAT & AILET Papers Page 189
94. PRINCIPLE:
A citizen is expected to take reasonable duty of care while driving on the road and not to cause
injuries to any person. (Common for Q.94, 95 and 96)
FACTS: X, the owner of a car, asked his friend Y to drive the car to his office. As the car was near his
(X’ s) office, it hit a pedestrian P on account of Y’ s negligent driving and injured him seriously. P
sued X for damages. Which one of the following is correct?

(a) X is not liable as it was the negligence of Y.


(b) The liability was solely of Y as X was not accompanying him.
(c) As Y was driving under X’s care and authority, X is liable.
(d) X is not liable under the principle of inevitable accident.

95. The standard of care generally used in cases of negligence is the

(a) skill and care of a professional person


(b) care taken by an intelligent and prudent man
(c) foresight of a prudent man
(d) skill and foresight of an ordinary person of prudence and competence

96. Two persons are said to be joint tort-feasors when

(a) a person on account of his negligence gives opportunity to another for committing a tort
(b) two or more persons are simultaneously involved in committing a wrong
(c) a tort is committed by two or more persons or any one of them when they are engaged in
furtherance of a concerted purpose
(d) two persons together cornmit a tort
97. PRINCIPLE:
The law permits citizens to use force only for protection when necessary against imminent attack.
(Common for Q. 97, 98 & 99)

FACTS:
P with the intention of committing theft entered the house of Q. Q, on seeing him entering, struck
him with a lathi and P fell down unconscious. Thereafter, Q gave him another blow of lathi on his
head which caused his death. On being prosecuted for murder, Q took the plea of private defense.
Which of the following argument is valid?

(a) Since Q was acting in the exercise of right of private defense of his property, he had taken a valid
defence.
(b) Since in the defence of one’s property one cannot cause death of the intruder, Q has no defense.
(c) Q has used excessive force as once P fell unconscious; there was no need for the second blow.
Hence, Q’ s plea of right of private defence will not succeed.
(d) If P committed house breaking in the night, Q has the right to cause death in defense of his
property, and thus Q’s plea should prevail.

98. To whom, among the following, is the right of private defence is available?
1. Only to the defender being a preventive right.
2. An aggressor, while facing action on the part of the defender which is excessive.

Previous Years
Page 190 CLAT & AILET Papers
Select the correct answer using the code given below:
(a) 1 only
(c) 2 only
(b) Both 1 and 2
(d) Neither 1 nor 2

99. X’s farm house in outskirts of Delhi was attacked by a gang of armed robbers. X without informing
the police, at first warned the robbers by firing in the air. As they were fleeing from the farm, he fired
and killed one of them. At the trial -
I. X can avail the right of private defence as he was defending his life and property.
II. X cannot avail the right as he failed to inform the police.
III. X cannot avail the right as he caused more harm than was necessary to ward off the danger.
IV. X can avail of the right as at first he only fires in the air.

(a) I and IV (b) II only (c) II and III (d) IV only

100. PRINCIPLE:
Conspiracy is a combination between two or more persons formed for the purpose of doing either an
unlawful act or a lawful act by unlawful means.
FACTS:
X and Y conspire to poison Z. X in pursuance of the conspiracy procures the poison and delivers it
to Y in order that he may administer it to Z. Y in pursuance of the conspiracy administers the poison
in the presence of X and thereby causes Z’s death. What offence, if any has been committed by X
and Y, respectively? (Common for Q. 100, 101 & 102)

(a) Y has committed the offence of murder and X was an abettor.


(b) Both X and Y have committed the offence of criminal conspiracy.
(c) X has committed the offence of murder and Y was an abettor.
(d) Both X and Y have committed the offence of murder.

101. Suppose X & Y were apprehended by A before administering poison to Z . What is the offence
committed by X & Y ?
(a) X & Y are not guilty of conspiracy as they did not cause death of Z.
(b) Liability of X comes to an end and Y is only liable.
(c) As no offence is committed and none of them are responsible.
(d) X & Y are to be punished as it is immaterial whether illegal act is ultimate object of the agreement
or merely incidental to that object.

102. The gist of the offence of criminal conspiracy is


(a) agreement is necessary between two or more persons to do or cause to be done an illegal act.
(b) a legal act by illegal means followed by an overt act to be done or cause to be done according
to agreement.
(c) both (a) and (b)
(d) None of the above

Previous Years
CLAT & AILET Papers Page 191
103. PRINCIPLE:
Contributory negligence in an accident is a defence to a charge in criminal law.

FACTS:
X, the deceased was negligently crossing the busy road at Connaught place in Delhi while Y’s car
hit him resulting in death of X. What is the liability of Y?
(a) Y is liable for death of X.
(b) Y is partly liable as X contributed to his own death.
(c) Y is absolved from liability based on the principle of contributory negligence.
(d) None of the above
104. PRINCIPLE:
A contract without consideration is void. When at the desire of one party the other party does
something, the consideration is said to flow from the latter to the former.
FACTS:
A’s house was on fire and a child was trapped inside the house. Everyone was shouting for help. A
brave onlooker, hearing the shrieks of child, went inside the house and brought him out. The father
of the child promised to pay the rescuer Rs.10,000. Subsequently, he backtracked his promise. The
rescuer sued the promise for the breach.

(a) The father of the child must pay for the service rendered by the rescuer
(b) The rescuer is not entitled to the payment, since he acted on his own
(c) Commercial considerations cannot be applied to humanitarian instincts
(d) None of the above

105. PRINCIPLE:
A contract procured by coercion is bad under Indian Contract Act. Coercion is defined as committing
or threatening to commit any act forbidden by Indian Penal Code. Attempt to commit suicide is an
offence under India Penal Code.

FACTS:
A wanted his wife B to part with some property given to her by her father. B resisted fearing that her
husband would squander it away. A threatened her that if she does not sign the deed transferring the
property to him (i.e., A), he would commit suicide. B signed the deed. Subsequently, she challenged
the deed on the ground that the deed was bad under law.
(a) The deed is valid under the law.
(b) The transfer of property is complete once the deed ‘is entered between the owner of the property
and the buyer.
(c) The deed is not valid as it was signed under the threat of suicide by her husband which amounts
to coercion and vitiates the contract.
(d) Husband has a right over the property of the wife that was voluntarily transferred to the buyer.
(d) Husband has a right over the property of the wife that was voluntarily transferred to the buyer.

Previous Years
Page 192 CLAT & AILET Papers
SECTION - D: LOGICAL REASONING

Direction: Find out the missing word.

106. Carpet: Floor: : ? : Wall


(a) Wall paper (b) Wall hanging (c) Scenery (d) Oil painting

107. Kaziranga: Rhino : : ? : Lion


(a) Corbett (b) Kanha (c) Gir (d) Sunderbans

Direction: Find out that word number.

108. AOP, CQR, EST, GUV, ?


(a) IYZ (b) HWX (c) IWX (d) JWX

109. 6, 11, 21, 36, 56, ?


(a) 66 (b) 76 (c) 81 (d) 86

110. A father is now three times as old as his son. Five years back, he was four times as old as his son.
The age of the son is
(a) 12 (b) 15 (c) 18 (d) 20

111. A bus for Delhi leaves every thirty minutes from a bus stand. An enquiry clerk told a passenger
that the bus had already left ten minutes ago and the next bus will leave at 9.35 a.m. At what time
did the enquiry clerk give this information to the passenger?
(a) 9.10 a.m. (b) 8.55 a.m. (c) 9.08 a.m. (d) 9.15 a.m.

112. Soaking dried beans overnight before cooking them reduces cooking time. However, cooking without
pre-soaking yields plumper beans. Therefore, when a bean dish’s quality is more important than the
need to cook that dish quickly, beans should not be pre-soaked.

Which one of the following is an assumption required by the argument?


(a) Plumper beans enhance the quality of a dish
(b) There are no dishes whose quality improves with faster cooking
(c) A dish’s appearance is as important as its taste
(d) None of the other ingredients in the dish need to be pre-soaked

113. Everyone in Biba’s neighbourhood is permitted to swim at Barton Pool at some time during each
day that it is open. No children under the age of 6 are permitted to swim at Barton Pool between
noon and 5 P.M. From 5 P.M. until closing, BartonPool is reserved for adults only.
If all the sentences above are true, then which one of the following must be true ?
(a) Few children under the age of 6 live in Biba’s neighbourhood
(b) If Biba’s next door neighbour has a child under the age of 6, then Barton Pool is open before
noon
(c) If most children who swim in Barton Pool swim in the afternoon, then the pool is generally less
crowded after 5 P.M.
(d) On days when Barton Pool is open, at least some children swim there in the afternoon

Previous Years
CLAT & AILET Papers Page 193
Direction: Find out the missing alphabet/number.

114. D, H, L, R, ?
(a) T (b) X (c) I (d) O

115. 3, 7, 15, 31, 63, ?


(a) 92 (b) 115 (c) 127 (d) 131

116. If SIR is written as PSPIPR, then MAN can be written as


(a) PMANP (b) PMPAPN (c) MANP (d) MPANP

117. In a certain code language, 35796 is written as 44887. How is 46823 written in that code?
(a) 57914 (b) 55914 (c) 55934 (d) 55714

118. If day before yesterday was Tuesday, the day after tomorrow will be
(a) Monday (b) Wednesday (c) Friday (d) Saturday

119. Ram is facing North-West. He turns in clockwise direction by 90°, then 180° in the anti-clockwise
direction and then another 90° in the same direction. Which direction is he facing now?
(a) South-West (b) West (c) South (d) South-East

120. One morning after sunrise, Vikram and Shailesh were standing in a lawn with their backs towards
each other. Vikram’s shadow fell exactly towards left hand side. Which direction was Shailesh
facing?
(a) East (b) West (c) North (d) South

121. Zack’s Coffeehouse schedules free poetry readings almost every Wednesday. Zack’s offers half-
priced coffee all day on every day that a poetry reading is scheduled.

Which one of the following can be properly inferred from the information above?
(a) Wednesday is the most common day on which Zack’s offers half-priced coffee all day.
(b) Most free poetry readings given at Zack’s are scheduled for Wednesdays.
(c) Free poetry readings are scheduled on almost every day that Zack’s offers half-priced coffee all
day.
(d) Zack’s offers half-priced coffee all day on most, if not all, Wednesdays.

122. If it is true that neither Rama is a brother of Krishna, nor is he a lecturer, then which of the following
statements is true?
(a) Rama is not a brother of Krishna, but he is a lecturer.
(b) Rama is a brother of Krishna, but he is not a lecturer.
(c) Rama is a brother of Krishna and he is a lecturer.
(d) Rama is not a brother of Krishna and he is not a lecturer.

123. “Make hay while the sunshine” is logically similar to


(a) Sleep when you are tired.
(b) Take the bull by horn.
(c) Losing hope is along everything.
(d) None of the above

Previous Years
Page 194 CLAT & AILET Papers
124. The Constitution has given the right of free speech. People speak freely in order to enjoy this right.
Which of the following has the same logic?
(a) Smuggling is illegal. People indulge in smuggling because they want to violate laws.
(b) People have the right to live, but they die, since they do not want to exercise that right.
(c) Wealth gives power to enjoy material things. People enjoy these things by spending their wealth.
(d) All of the above

125. Literature is the mirror in which the society can see itself. The above statement implies that
(a) no ugly society can produce good literature.
(b) writers are essentially like newspaper correspondents, reporting faithfully what they are observing.
(c) literature enables the people to comprehend the dimensions which they are unaware of.
(d) None of the above

126. It is time that begging must be forbidden in all public places in India. Exploitation of charitable
instincts in general, far from solving the problem of poverty, may infact aggravate it by inducing
laziness. Which of the following, if true, would support the above reasoning?
(a) Destitution is the result of unwillingness to work.
(b) Most beggars are really poor.
(c) All poor people do not beg.
(d) None of the above

127. Child labour is only a reflection of the malaise in the society.


Which of the following, supports the above statement?
(a) Child labour is not abhorred by the public.
(b) The Government is not serious about eliminating child labour.
(c) Exploitation of the poor, weak and unorganized is not uncommon.
(d) All of the above

128. If it is true that ‘good governance ensures law and order’, then identify the statement which has to
be accepted along with the given statement.
(a) A healthy society is governed by maximum number of laws.
(b) A strong government uses force to enforce law.
(c) Law is indispensible for the society.
(d) All of the above

129. If it is true that ‘very rarely do we come across committed film directors’, which one of the following
is false beyond doubt?
(a) We can never come across committed film directors.
(b) No film director is committed.
(c) All film directors are committed, but we can never meet them.
(d) None of the above

130. If it is true that ‘All men are honest’ and ‘All politicians are men’, for which among the following is it
impossible to follow from the given premises?
(a) Some politicians are not honest.
(b) All politicians are honest.
(c) One politician is honest.
(d) All honest persons are politicians.

Previous Years
CLAT & AILET Papers Page 195
131. Consider a set of premises like this: If science and religion clash, then critical thinking and dogma
will clash. Dogma is not acceptable and hence religion. But critical thinking shakes the foundation
of morality.
Identify the conclusion which follows from the premises.
(a) If science is not accepted, then the foundations of morality are not shaken.
(b) If foundations of morality are shaken, then science is acceptable.
(c) If science is accepted, then foundations of morality are shaken.
(d) Though science is accepted, it cannot shake the foundation of morality.

132. If Ramesh likes oranges more than grapes, strawberries more than apples, and apples more than
oranges, then which one of the following is not true?
(a) He likes grapes more than strawberries.
(b) He likes strawberries more than grapes.
(c) He likes apples more than grapes.
(d) He likes strawberries more than apples.
133. “Justice should not only be done, but seen to have been done”. The statement intends to convey
the message that
(a) Court proceedings must be open to public.
(b) Public participation in judicial decision making is essential for timely justice.
(c) Court proceedings must be fair and judgments should be based on legal reasoning.
(d) All court cases must be publicized and reported to the public in their own language.

134. Yoga has become a very popular type of exercise, but it may not be for everyone. Before you sign
yourself up for a yoga class, you need to examine what is it you want from your fitness routine. If
you’re looking for a high-energy, fast-paced aerobic workout, a yoga class might not be your best
choice.

This paragraph best supports the statement that


(a) Yoga is more popular than high-impact aerobics.
(b) Before embarking on a new exercise regimen, you should think about your needs and desires.
(c) Yoga is changing the world of fitness in major ways.
(d) Most people think that yoga isn’t a rigorous form of exercise.

135. Which one of the following, if true, can be empirically tested?


(a) Violence is engineered by power-hungry politicians.
(b) Only poor are driven to violence.
(c) Faith in God alone can contain violence.
(d) Instinct to preserve one’s own self is at the root of violence.

136. Which one of the following, whether true or false, is irrelevant to the issue?
(a) Also, violence is due to the fact there is a tendency in man to enjoy when somebody suffers.
(b) Violence is commonplace in the world because men hardly learn philosophy.
(c) Violence is the overt manifestation of covert desire to be adventurous.
(d) Violence is the result of man’s tendency to assert himself.

137. Which one of the following makes the argument self-contradictory?


(a) The soul has finite existence. (b) Whatever is corruptible is soluble.
(c) What is spatial is material. (d) Motion is not a reality.

Previous Years
Page 196 CLAT & AILET Papers
138. Before Robert Norman worked on ‘Dip and Field Concept’, his predecessor thought that the
tendency of the magnetic needle to swing towards the poles was due to a point attractive. However,
Norman showed with the help of experiment that nothing like point attractive exists. Instead, he
argued that magnetic power lies is lodestone.

Which one of the following is the problem on which Norman and others worked?
(a) Existence of point attractive (b) Magnetic power in lodestone
(c) Magnetic power in needle (d) Swinging of magnetic needle

139. In his study of conduction of heat, James Maxwell observed that quantities which are significant
in such a study are ‘temperature, flow of heat and conductivity’. Further, he noticed that the
mathematical laws of the uniform motion of heat in homogenous media are identical in form with
those of attractions varying inversely as the square of the distance. He thought source of heat,
flow of heat and temperature correspond respectively to centre of attraction, accelerating effect of
attraction and potential. On this ground, Maxwell observed that conduction of heat proceeds by
an action between contiguous parts of a medium.
Which one of the following best explains Maxwell’s method?
(a) Mathematical calculation (b) Analogy
(c) Observation (d) Experiment

140. Henry Poincare argued that in space there are not only rectilinear triangles in which angles equal
to two right angles, but also curvilinear triangles in which angles are less than two right angles.
He, further, maintained that to name the first one straight is to subscribe to Euclidean geometry and
to name the latter straight is tantamount to subscribing to non-Euclidean system.
Which one of the following accurately represents what is at stake?
(a) Acceptability of Euclidean system (b) Acceptability of Non-Euclidean system
(c) Confusion in Geometry (d) Choice of the definition of straight line

SECTION - E: MATHEMATICS

141. Kirti took loan of Rs.800 at the rate of 11% per year for 7 months. After seven months she has to pay
(a) Rs.851 (b) Rs.852 (c) Rs.950 (d) Rs.951

142. 16 men can do a piece of work in 16 days. 4 days after they started the work, 8 more men joined
them. How many days will they now take to complete the remaining work?
(a) 10 days (b) 6 days (c) 8 days (d) 12 days

143. The number of the members of a club is increased by 10% every year. If the initial number of the
members is 500, then what will be the number at the beginning of the third year?
(a) 610 (b) 615 (c) 620 (d) 605

144. A student rides a bicycle at 8 km/hr and reaches the school 2.5 minutes late. The next day he
increases the speed to 10 km/hr and reaches school 5 minutes early. How far is the school from
his house?
(a) 4 kms (b) 8 kms (c) 5 kms (d) 10 kms

Previous Years
CLAT & AILET Papers Page 197
145. Rs. 324 is divided among three friends Sonu, Monu and Hari in the ratio 5: 6 : 7. What is Monu’s
share of money?
(a) 68 (b) 108 (c) 60 (d) 120

146. In an examination, every candidate took either Physics or Mathematics or both. 84% of the
candidates took Physics and candidates who took Mathematics were half of those who took
Physics. The total number of candidates being 1000, how many took both Physics and Mathematics?
(a) 200 (b) 240 (c) 250 (d) 260

147. A company declares a dividend of 12% on Rs.100 shares. A housewife buys such shares and
gets 15% on her investment. At what price she bought the shares?
(a) Rs.80 (b) Rs.85 (c) Rs.125 (d) Rs.76

148. The price of an article is cut by 10%. To restore it to the original value, the new price must be
increased by
1 1
(a) 10% (b) 9 % (c) 11% (d) 11 %
11 9

149. A shopkeeper mixes two varieties of Tea, one costing Rs.40/kg and another Rs.50/kg in the ratio
3 : 2. If he sells the mixed variety of Tea at Rs.48/kg, his gain or loss percent is
(a) 48.4% gain (b) 48.4% loss (c) 10% gain (d) 10% loss

150. The value of a car depreciates every year at the rate of 10% on its value at the beginning of the year.
If the present value of the car is Rs.52,488, its worth four years ago was
(a) Rs.68,232 (b) Rs.68,234 (c) Rs.69,862 (d) Rs.80,000

Previous Years
Page 198 CLAT & AILET Papers
AILET Question Paper 2012

SECTION – A : LEGAL APTITUDE

1. What is minimum duration of stay essential before a person can apply for citizenship of India?
(A) Five years (B) Seven years (C) Nine years (D) Twelve years

2. The Ordinances issued by the Governor are subject to the approval by


(A) State legislature
(B) President
(C) Central Government
(D) Parliament

3. Scheduled Castes and Scheduled Tribes


(A) have been specified by the Presidential Orders issued in consultation with the Governors of the
respective States
(B) have been specified in the Constitution by the Constitution makers in the form of a Schedule
(C) have to be specified and notified by the Parliament after detailed discussions about the backward
nature of these people
(D) are the Castes and Tribes notified by the State Government after detailed discussion as to their
socio-economic conditions

4. Free legal aid for an accused is a


(A) Fundamental right (B) Fundamental duty of the State
(C) Directive Principles of State Policy (D) Discretion of the State

5. The concept of Secular State is that the State will not make any discrimination on the ground of
religion, caste or community against any person professing any particular form of religious faith.
The Secularism is embodied in
(A) Preamble of Constitution
(B) Directive Principles of State Policy
(C) Fundamental Rights
(D) Judicial Interpretation of Fundamental Rights

6. The National Development Council consists of


(A) Members of Planning Commission
(B) Cabinet Ministers and State Chief Ministers
(C) Representatives of Union Territories
(D) All of the above

7. Who allocates portfolios among the Ministers?


(A) Prime Minister
(B) President
(C) President on the recommendation of the Prime Minister
(D) Chairperson of Ruling Political Party

Previous Years
CLAT & AILET Papers Page 199
8. Which Indian language is given the status of ‘Classical language’ by the Central Government?
(A) Sanskrit (B) Telugu (C) Tamil (D) Pali

9. Each member of the Security Council has one vote. Decisions on procedural matters are to be
taken by .............................. permanent members. Here veto does not apply. On all other matters,
there must be nine affirmative votes including those of five permanent members.
(A) 6 (B) 7 (C) 8 (D) 5

10. The first venture of Mahatma Gandhi in all India Politics was
(A) Non-cooperation Movement (B) Dandi March
(C) Champaran Movement (D) Rowlatt Satyagrah

11. The “Right to Information” (RTI) is a


(A) Constitutional Right (B) Statutory Right
(C) Fundamental Right (D) Contractual Right

12. Which of the following committees has recommended measures for banning and controlling ragging
in educational establishments in India?
(A) The Raghavan Committee (B) Jasraj Committee
(C) Narsimhan Committee (D) Soli Sorabjee Committee

13. The Constitution of India does not mention the post of


(A) The Deputy Speaker of the Lok Sabha
(B) The Deputy Speaker of the State Legislative Assembly
(C) The Deputy Chairman of the Rajya Sabha
(D) The Deputy Prime Minister

14. A Judge of the Supreme Court of India or High Court of any State can be removed by the President
of India only
(A) when the Principles of Natural Justice are followed and the alleged misconduct is proved in an
impartial enquiry
(B) if he is satisfied through the report made by the Chief Justice of India that the misconduct of the
judge has been proved
(C) when an address is made by both the Houses of Parliament in the same session asking for his/
her removal on the grounds of proven misbehaviour and incapacity
(D) when the President and the Prime Minister are satisfied that he or she has committed a misconduct
and the same has been proved before a competent authority

15. The Constitution of India is the result of considerable imitation and adaptation rather than originality
because
(A) Makers of Indian Constitution drew much from the American Constitution, Canadian Constitution
and British made Government of India Act, 1935
(B) Makers of Indian Constitution drew much from Swiss Constitution, German Constitution and
Government of India Act of 1919
(C) Makers of Indian Constitution drew much from Constitution of Singapore, Constitution of Sri
Lanka and Government of India Act of 1919
(D) Makers of Indian Constitution drew from Constitution of South Africa, Constitution of Netherlands
and Government of India Act of 1919

Previous Years
Page 200 CLAT & AILET Papers
16. For transfer of accused person from a foreign State to India for any offence within India or relating to
India is done
(A) by previous repatriation agreement between India and concerned foreign country
(B) by instant repatriation agreement between India and concerned foreign country
(C) at the desire of ambassador representing India in concerned foreign country
(D) at the desire of the ambassador who represents concerned foreign country in India

17. Whether a Bill is a Money Bill or not is certified by the ________ and his decision is conclusive
(A) President (B) Chairman of Rajya Sabha
(C) Speaker of Lok Sabha (D) Chairman of Public Accounts Committee

18. __________ seats are reserved in all Panchayats at all levels for women
(A) One-fourth (B ) One-third (C) One-half (D ) Two-third

19. A seat of a M.P. can be declared vacant if he absents himself from the House for a continuous period of
(A) Six months (B) Two months (C) Three months (D) One year

20. What is true with Members of Parliament’s immunity from arrest?


(A) In civil cases while the Legislature is in Session, for 40 days before and after and in criminal
cases no arrest during Parliament Session
(B) In civil cases, while Legislature is in Session and for 40 days before and after and in criminal
cases arrest can be made at any time
(C) In civil cases while the Legislature is in Session, for 20 days before and after and in criminal
cases no arrest during Parliament Session
(D) In civil cases, while Legislature is in Session and for 20 days before and after and in criminal
cases arrest can be made at any time

21. The maximum duration of the zero hour in Lok Sabha can be
(A) One hour (B) Unspecified (C) 3 0 minutes (D) Two hours

22. Dealing with the chapter of citizenship under the Constitution, the Supreme Court has held that
Sonia Gandhi, the President of Congress (I) is not an Italian citizen, but is an Indian citizen because
she is governed by
(A) Article 10, Constitution of India (B) Article 11, Constitution of India
(C) The Citizenship Act, 1955 (D) Article 5, Constitution of India

23. What is true with appointment of Attorney General for India?


(A) He can be appointed by President of India and is qualified to be appointed a Judge of Supreme
Court
(B) He can be appointed by President of India and is qualified to be appointed a Judge of High Court
(C) He can be appointed by a Special Commission and is qualified to be appointed as Law Minister
(D) He can be appointed by Chief Justice of India and is qualified to be appointed as a Judge of
Supreme Court

24. The earliest Codified Laws (Civil and Criminal) that defined and demarcated for first time in India
during 335-345 A.D. was followed in the
(A) Huna reign (B) Sakka reign (C) Mughal reign (D) Gupta reign

Previous Years
CLAT & AILET Papers Page 201
25. Which of the following statements is/are not correct about the Objectives Resolution?
I. It was moved by Jawaharlal Nehru in the Constituent Assembly
II. It called for just right for minorities
III. It formed the basis for the chapter on Fundamental Rights
IV. It called for the establishment of a socialist and secular polity
(A) I and II (B) I, II and III (C) III and IV (D) Only III

26. The right to constitutional remedies in India is available to


(A) only the citizen of India
(B) all persons in case of infringement of any fundamental right
(C) any person for enforcing any rights conferred on them
(D) an aggrieved individual alone

27. Money bills can be introduced in the State Legislature with the prior consent of the
(A) President (B) Governor (C) Speaker (D) Chief Minister

28. While a proclamation of emergency is in operation, the State Government


(A) cannot legislate
(B) can legislate only on subjects in the Concurrent List
(C) can legislate on the subjects in the State List
(D) is suspended

29. A political party is recognised by the Election Commission only if


I. it has been engaged in political activity for a continuous period of five years
II. has returned at least one member of the Lok Sabha for every 25 members of that House or any
fraction of that number elected from that State
III. has polled not less than six per cent of the total number of valid votes polled by all contesting
candidates at the general elections
IV. has contested elections in four or more states in three consecutive general elections

(A) I and II (B) I, III and IV (C) I, II and III (D) I, II, III and IV

30. Electioneering has to stop in a constituency


(A) one day before the election (B) 48 hours before election starts
(C) 36 hours before a poll commences (D) 48 hours before the closing hour of polling

Directions (Q. 31- Q.35): Given below is a statement of principle followed by a factual situation. Apply the
principle to the facts given below and select the most appropriate answer.

31. PRINCIPLE:
In case of a breach of contract, compensation can be awarded for the personal inconvenience
suffered by a party by reason of the breach, which naturally arose in the usual course of things from
such breach, or which the parties knew, when they made the contract to be likely to result from the
breach of it.

Previous Years
Page 202 CLAT & AILET Papers
FACTS:
Sunita and Sushmita bought bus tickets for a journey from Adyar to Mandaveli. The bus was to go
to St. Thomas Mount via Mandaveli. However, the driver mistakenly took a wrong direction and the
two girls were dropped at a distance of 2½ miles from Mandaveli on the highway. With no other
transportation in sight nor a place to stay, the two had to walk 2½ miles at midnight.

Later they filed a case against the bus company and claimed Rs. 5000 as damages for inconvenience
caused in having to walk and Rs. 6500 for Sushmita having fallen ill by catching cold during the
night.

DECISION:
(A) Both the amounts are liable to be paid because Sunita and Sushmita suffered loss for no fault of
theirs.
(B) The bus company is liable to pay both the amounts claimed because the loss was suffered on
account of the fault of the bus company and the inconvenience suffered and illness arose was in
the normal course of things from breach of contract.
(C) The compensation for inconvenience suffered by being forced to walk at night is liable to be paid
by the bus company. However, no compensation for Sushmita’s illness because this was not
expected on account of breach of contract.
(D) The bus company is not liable to pay any amount, because it was the driver’s fault

32. FACTS:
In order to ensure that people live in an amicable atmosphere the Government of India decided to
abolish courts and constituted Dispute Settlement Boards. Further to achieve this objective, the law
stipulated that lawyers should not be allowed to espouse the claims of parties, and instead their
claims be espoused by social workers.

PRINCIPLES:
(I) Any law made by the Parliament that infringes the fundamental rights of the citizens is invalid
and unenforceable.
(II) Freedom to carry on trade or profession of one’s own choice is a fundamental right.
(III) The Parliament is competent to impose reasonable restrictions on the exercise of this right.
(IV)If the restrictions, on fundamental rights imposed by the Parliament, totally removes or nullifies
any fundamental right then it will be construed as an unreasonable restriction.

DECISION:
(A) The law made by the Parliament is valid as it does not infringe any fundamental right.
(B) The law made by the Parliament is valid as even though there is restriction of fundamental right,
such a restriction is reasonable.
(C) The law made by the Parliament is invalid as it constitutes an infringement of fundamental rights
and the restriction imposed is not reasonable.
(D) None of the above answer is correct.

Previous Years
CLAT & AILET Papers Page 203
33. PRINCIPLE:
A right to action cannot arise out of an illegal activity.

FACTS: A and B were thieves engaged in stealing cars and other vehicles. Once they stole a car;
and while driving off, they had to cross a city. They engaged a driver to drive them through the city,
since they did not know the route inside. The indicator lamp of the car was not working and the
thieves had not realised this, and therefore, had not told about it to the driver. While driving, through
the city, the car was hit by another vehicle because of the faulty indicator. In the accident, the driver
was injured and he filed a suit against A and B.

DECISION:
(A) The driver would lose, because he was driving a stolen car.
(B) The driver would win, because he was not a party to the stealing of car.
(C) The driver would win because he did not know anything about the stealing.
(D) None of the above answer is correct.

34. PRINCIPLE: A man would be responsible for all direct consequences of his act, in so far as he could
reasonably foresee them as arising from his act.

FACTS: A ship carrying petroleum was moving on the high sea. On a short halt in a port, the master
of the ship engaged some stevedores to load some metallic planks onto the ship. While loading the
planks, a plank slipped from the hands of stevedore and the spark, emitted thereby, ignited petroleum
vapour and caused considerable damage to the goods. The owner of the goods filed a suit against
the master of the ship.

DECISION:

(A) The master of the ship is not liable, because he was not responsible for the act of stevedore.
(B) The master of the ship is liable, because he is responsible for the acts of stevedore since he
engaged them.
(C) The master is liable, because he should have foreseen the consequences of the stevedore’s act.
(D) None of the above answer is correct.

35. PRINCIPLES:
All minorities, whether based on religion or language shall have the rights to establish and administer
educational institution of their choice.

FACS:
Md. Yusuf wants to establish an educational institution to help the poor person of his community
and to educate the children of the community. Yusuf being a very rich man has no problem regarding
finance for institution. Therefore he applies to state government to grant him permission to establish
and administer the institution. State government rejects his plea on ground that said institution will
create communal tension in the proposed area. Yusuf has following remedies:
(A) He should file a civil suit in District Court.
(B) He should file a special leave petition before Supreme Court.
(C) He has no remedy under the law.
(D) He could file a writ petition either before the High Court or the Supreme Court.

Previous Years
Page 204 CLAT & AILET Papers
SECTION – B : GENERAL KNOWLEDGE

36. What is the sequence in which the Indian economy has undergone the process of convertibility?
I. Rupee floated on trade account
II. Part convertibility under a dual exchange rate system
III. Introduction of exim scrips
IV. Rupee made convertible on current account
(A) I, II, III, IV (B) II, I, III, IV (C) III, II, I, IV (D) IV, I, II, III

37. Which of the following currency exchange rate mechanisms has been adopted by India?
(A) Free Float (B) Managed Float
(C) Floating with a Band (D) Fixed-but-Adjustable

38. The national income estimates of India are prepared by


(A) Ministry of Finance (B) National Sample Survey
(C) Central Statistical Organisation (D) Indian Statistical Institute

39. The ‘Year of Great Divide’ refers to


(A) rapid growth rate in population after 1921 (B) the declining sex ratio after 1921
(C) the slow down in death rate after 1921 (D) all of the above

40. Aurorae are a result of the interaction of earth’s magnetosphere with


(A) the solar wind (B) the ozone layer (C) radio waves (D) none of the above

41. You are standing in a lift. The force on the floor of the lift due to your weight will
(A) become zero when the lift moves up with an acceleration of 9.8 m/s2
(B) become zero when the lift moves down with an acceleration of 9.8 m/s2
(C) become zero when the lift moves with an acceleration of 9.8 m/s2 in any direction
(D) remain unchanged whatever be the value of acceleration of the lift

42. What happens to the level of mercury in a barometer tube when it is taken down a coal mine? It
(A) falls (B) remain unaltered (C) rises (D) rises and then falls

43. A popping sound is heard in the ear when yawning or swallowing. This is caused by
(A) air in the cochlea (B) the opening of the Eustachian tubes
(C) air entering the perilymph (D) the moving of the jaws

44. The body’s thermostat is in the


(A) pineal (B) hypothalamus (C) thyroid (D) pituitary

45. India is divided into how many PIN code zones?


(A) Six (B) Seven (C) Eight (D) Nine
46. Which of the following statements are correct with regard to the Indus Valley civilisation?
I. The people of this civilisation were the earliest to make use of cotton.
II. They did not cremate their dead.
III. A grid system was followed in city planning.
IV. Stone was the chief building material.
(A) I and II (B) II and III (C) III and IV (D) I and III

Previous Years
CLAT & AILET Papers Page 205
47. Which one of the following may be called a ‘political’ cause of the 1857 Revolt?
(A) The subsidiary alliance system of Wellesley
(B) The Religious Disabilities Act of 1856
(C) Low pay and poor prospects of the sepoys
(D) Dalhousie’s annexation policy

48. Who among the following Indian rulers established embassies in foreign countries on modern lines?
(A) Haider Ali (B) Mir Qasim (C) Shah Alam II (D) Tipu Sultan

49. Which one of the following may be called a ‘political’ cause of the 1857 Revolt?
(A) The subsidiary alliance system of Wellesley
(B) The Religious Disabilities Act of 1856
(C) Low pay and poor prospects of the sepoys
(D) Dalhousie’s annexation policy

50. The first time that Mahatma Gandhi tried out his method of satyagraha was
(A) in South Africa against racist policies (B) against indigo planters in Champaran
(C) in the Kheda no-rent campaign (D) at Bardoli

51. The Royal Indian Navy ratings revolted in February 1946 to protest against
(A) the treatment meted out to them
(B) the punishment meted out to the INA officials
(C) the firing on some ratings of the HMIS Talwar
(D) the Government’s suppression of Congress activities

52. he Preamble enshrines certain ideals that were first spelt out in
(A) the speech by Jawaharlal Nehru on the banks of Ravi when he called for Purna Swaraj
(B) the Nehru Report
(C) a resolution adopted at Karachi session of the Indian National Congress
(D) the Objectives Resolution adopted by the Constituent Assembly

53. A very rapid growth in prices in which money loses its value to the point where even barter may be
preferable is known as
(A) inflation (B) hyper-inflation (C) deflation (D) disinflation

54. Unlike the fluid core of the earth, the core of the moon is
(A) plasma (B) volatile gas (C) viscous liquid (D) solid

55. Which of the following theories has no connection with the origin of the earth?
(A) Nebular hypothesis of Laplace (B) Tidal hypothesis of Jeans and Jeffreys
(C) Binary star theory of Lyttleton (D) Convection current theory of Holmes

56. On what does the escape velocity of a body depend?


I. Mass of celestial body
II. The distance from the centre of mass to the escaping object
(A) Only I (B) Only II (C) Both I and II (D) Neither I nor II

57. The blood enters the aorta from


(A) left ventricle (B) left auricle (C) right ventricle (D) right auricle

Previous Years
Page 206 CLAT & AILET Papers
58. Desert plants would be characterised by
(A) vivipary (B) aerenchyma (C) aerial roots (D) sunken stomata

59. Why is sprouted gram considered more nutritious?


(A) Seeds are storehouses of energy
(B) Germinating seeds produce enzymes which are a rich source of protein
(C) Seeds have plenty of amino acids and glucose
(D) Sprouted gram is not more nutritious; it is just an old wives’ tale that it is

60. The edible portion of which of the following vegetables is not a stem?
(A) Potato (B) Sweet potato (C) Ginger (D) Corn of Colocasia

61. What is the venue for the 20th Commonwealth Games in 2014?
(A) Glasgow (B) Beijing (C) London (D) Italy

62. Who among the following leaders topped 2011 Forbes List of “World’s Most Powerful Person”?
(A) Mr. Barak Obama (B) Ms. Pratibha Patil
(C) Mr. Manmohan Singh (D) Mr. Hu Jintao

63. According to the Srikrishna Committee, which is the most workable option out of the six option
suggested by the Committee to resolve the Telengana issue?
(A) Second (B) Fourth (C) Sixth (D) First

64. Who is convicted to death in the Beant Singh assassination case on March 31, 2012?
(A) Kehar Singh (B) Balwant Singh Rajoana
(C) Karnail Singh (D) Dilawar Singh Babbar

65. Which news corporation was recently alleged to have been using illicit hacking into voicemail
messages of prominent people to find stories?
(A) News of the World (NoW) (B) BBC
(C) CNN (D) AajTak

66. Ms. Aung San Suu Kyi, Myanmar’s democracy icon, has been released from ——— of home
arrest.
(A) 16 years (B) 15 years (C) 10 years (D) 20 years

67. Which country became the 193rd member of UN, recently?


(A) South Sudan (B) Montenegro (C) Serbia (D) Tuvalu

68. More than 50 Sikh shrines in India are named after?


(A) Flowers (B) Trees (C) Birds (D) Animals

69. The country that set the record as the one with the longest time without government, in April 2011?
(A) Burundi (B) Belgium (C) Turkey (D) Taiwan

70. It has been the practice to have ——————as the Managing Director of the IMF.
(A) an American (B) a European (C) an Australian (D) an Arab

Previous Years
CLAT & AILET Papers Page 207
SECTION- C : REASONING

71. For one academic year, all the students at a high school were observed. The aim was to test the
hypothesis that studying more increased a student’s chances of earning a higher grade. It turned
out that the students who spent the most time studying did not earn grades as high as did many
students who studied less. Nonetheless, the researchers concluded that the results of the observation
supported the initial hypothesis.
Which one of the following, if true, most helps to explain why the researchers drew the conclusion
described above?
(A) The students who spent the most time studying earned higher grades than did some students
who studied for less time than the average.
(B) The students tended to get slightly lower grades as the academic year progressed.
(C) In each course, the more a student studied, the better his or her grade was in that course.
(D) The students who spent the least time studying tended to be students with no more than
average involvement in extracurricular activities.

72. Educator: It has been argued that our professional organization should take decision about important
issues - such as raising dues and taking political stands - by a direct vote of all members rather than
by having members vote for officers who in turn make the decisions. This would not; however, be the
right way to decide these matters, for the vote of any given individual is much more likely to determine
organizational policy by influencing the election of an officer than by influencing the result of a direct
vote on a single issue.
Which one of the following principles would, if valid, most help to justify the educator’s reasoning?
(A) No procedure for making organizational decisions should allow one individual’s vote to weigh
more than that of another.
(B) Outcomes of organizational elections should be evaluated according to their benefit to the
organization as a whole, not according to the fairness of the methods by which they are produced.
(C) Important issues facing organizations should be decided by people who can devote their full
time to mastering the information relevant to the issues.
(D) An organization’s procedures for making organizational decisions should maximize the power of
each member of the organization to influence the decisions made.

73. Mayor: Local anti-tobacco activists are calling for expanded antismoking education programs paid
for by revenue from heavily increased taxes on cigarettes sold in the city. Although the effectiveness
of such education programs is debatable, there is strong evidence that the taxes themselves would
produce the sought-after reduction in smoking. Surveys show that cigarette sales drop substantially
in cities that impose stiff tax increases on cigarettes.
Which one of the following, if true, most undermines the reasoning in the argument above?
(A) A city-imposed tax on cigarette will substantially reduce the amount of smoking in the city if the
tax is burdensome to the average cigarette consumer.
(B) Consumers are more likely to continue buying a product if its price increases due to higher
taxes than if its price increases for some other reason.
(C) Usually, cigarette sales will increase substantially in the areas surrounding a city after that city
imposes stiff taxes on cigarettes.
(D) People who are well informed about the effects of long-term tobacco use are significantly less
likely to smoke than are people who are not informed.

Previous Years
Page 208 CLAT & AILET Papers
74. Gotera: Infants lack the motor ability required to voluntarily produce particular sounds, but produce
various babbling sounds randomly. Most children are several years old before they can voluntarily
produce most of the vowel and consonant sounds of their language. We can conclude that speech
acquisition is entirely a motor control process rather than a process that is abstract or mental.
Which one of the following is an assumption required by Gotera’s argument?
(A) Speech acquisition is a function only of one’s ability to produce the sounds of spoken language.
(B) During the entire initial babbling stage, infants cannot intentionally move their tongues while
they are babbling.
(C) The initial babbling stage is completed during infancy.
(D) The initial babbling stage is the first stage of the speech acquisition process.

75. Reducing stress lessens a person’s sensitivity to pain. This is the conclusion reached by researchers
who played extended audiotapes to patients before they underwent surgery and afterward while
they were recovering. One tape consisted of conversation; the other consisted of music. Those who
listened only to the latter tape required less anaesthesia during surgery and fewer painkillers afterward
than those who listened only to the former tape.
Which one of the following is an assumption on which the researchers’ reasoning depends?
(A) All of the patients in the study listened to the same tape before surgery as they listened to after
surgery.
(B) Anticipating surgery is no less stressful than recovering from surgery.
(C) Listening to music reduces stress.
(D) The psychological effects of music are not changed by anaesthesia or painkillers.

Directions: In the following questions, each question has a statement followed by two conclusions. Taking
the statement to be true, decide which of the given conclusions definitely follow from the given statement.
Indicate your answer as (A) if only I follows, (B) if only II follows (C) if neither I, nor II follows and (D) if both
I and II follow.

76. Statement : The Supreme Court gave a judgement that the maintenance of old age parents is the
responsibility of the married girls, if they do not have brothers.
Conclusions I : Constitution is always interpreted to help oppressed people out.
II : Before the Supreme Court gave the verdict, a married girl must have denied to pay for the
maintenance to her parents.

77. Statement : Ideas given by our ancestors that were once discarded as uneconomical and unviable,
turn out to be as functional and inevitable in present circumstances.
Conclusions I : In ancient period, ideas were considered either completely functional or totally
infeasible.
II : Ideas cannot change from time to time.

Previous Years
CLAT & AILET Papers Page 209
Direction: In each question below, a statement is given followed by two assumptions numbered I and II. An
assumption is something supposed or taken for granted. You have to consider the statement and the
following assumptions and decide which of the assumptions is implicit in the statement. Mark answer:
(A) If only assumption I is implicit; (B) If only assumption II is implicit;
(C) If neither I nor II is implicit; and (D) If both I and II are implicit.

78. Statement:
“Use ‘M’ Brand shoes. They are available in all sizes and last longer”- an advertisement in the
newspaper ‘A’.
Assumption I:
Some people do not know about ‘M’ brand shoes.
Assumption II:
People generally prefer shoes which last longer.

79. Statement:
Lack of stimulation in the first four or five years of life can have adverse consequences.
Assumption I:
A great part of the development of observed intelligence occurs in the earliest years of life.
Assumption II:
50 per cent of the measurable intelligence at age 16 is predictable by the age of four.

Direction (Q. 80 – Q. 84): Read the following and answer the questions:
Six persons – P, Q, R, S, T and W work in an Organisation. Each of them likes different colours, viz.,
Black, White, Blue, Green, Red and Yellow and their salaries are also different. The person who earns
maximum does not like Blue or White colour. The person who earns the least does not like Black or Red
colour. R likes Yellow colour and his salary is the second maximum. P likes White colour and earns more
than T but less than W. Q likes Black colour who earns less than P but more than T. S likes Blue colour
and T likes Red colour.

80. How many of them do earn more than Q?


(A) One (B) Two (C) Three (D) Data inadequate

81. Who among them earns least?


(A) Q (B) P (C) R (D) S

82. If they are arranged in the descending order according to their salaries, who will occupy the third
position?
(A) P (B) R (C) T (D) Data inadequate

83. W likes the colour


(A) White (B) Green (C) Blue (D) Either White or Green

84. S likes which colour?


(A) White (B) Green (C) Blue (D) Red

Previous Years
Page 210 CLAT & AILET Papers
85. A child crawls 20 feet towards North, turns right and crawls 30 feet, turns right again and crawls 35
feet. He turns left now and crawls 15 feet. He turns left again and crawls 15 feet. Finally he turns to
his left to crawl another 15 feet. How far is he from his starting point and in which direction?
(A) ) 45 feet North-East (B) 30 feet East
(C) 30 feet West (D) 15 feet West

Directions: Read the following directions and answer the questions 86 and 87.
A is the father of C. But C is not his son. E is the daughter of C. F is the spouse of A. B is the brother of C.
D is the son of B. G is the spouse of B. H is the father of G.

86. Who is the son-in-law of H?


(A) C (B) A (C) D (D) B

87. Who is the grand-daughter of A?


(A) H (B) D (C) B (D) E

88. A is older by 4 years to B at one stage. After 16 years of this stage, A will be thrice his present age
and B will be five times his present age. How old would A and B be two years before the initially
indicated stage?
(A) 8 and 4 (B) 10 and 6 (C) 6 and 2 (D) 12 and 8

89. If day before yesterday was Tuesday, the day after tomorrow will be
(A) Monday (B) Wednesday (C) Friday (D) Saturday

90. My brother is 562 days older to me, while my sister is 75 weeks older to him. If my sister was born
on Tuesday, on what day was I born?
(A) Sunday (B) Monday (C) Tuesday (D) Wednesday

91. Ram is facing North-West. He turns in clockwise direction by 90°, then 180° in the anti-clockwise
direction and then another 90° in the same direction. Which direction is he facing now?
(A) South-West (B) West (C) South (D) South-East

92. One morning after sunrise, Vikram and Shailesh were standing in a lawn with their backs towards
each other. Vikram’s shadow fell exactly towards left hand side.
Which direction was Shailesh facing?
(A) East (B) West (C) North (D) South

Direction: In the following number series (Q. No. 93 & 94) only one number is wrong.
Find out that number:

93. 1, 2, 6, 15, 20, 30, 42


(A) 30 (B) 15 (C) 6 (D) 1

94. 2, 5, 10, 17, 26, 37, 50, 64


(A) 50 (B) 17 (C) 26 (D) 64

95. D, H, L, R, ?
(A) T (B) X (C) I (D) O

Previous Years
CLAT & AILET Papers Page 211
96. 3, 7, 15, 31, 63, ?
(A) 92 (B) 115 (C) 127 (D) 131

97. PLANING is coded in a certain language as UFFHSCSA. How will AUTHORITY be coded in the
same language?
(A) FOYBTLNND (B) FYOTBNNLT (C) FBOYTLNTN (D) FBOYTNLTN

98. In certain code ELECTION is written as GLGCVIQN, then VOTER will be coded as:
(A) XOVET (B) VOXET (C) WPUFU (D) VQTGR

99. CHILD : FIRE


(A) Aged : Wisdom (B) Student : Examination
(C) Youth : Adolescence (D) Judge : Dishonour

100. GRAIN : SALT


(A) Shred : Wool (B) Cave : Stone (C) Chip: Glass (D) Blades : Grass

Direction (Q. 101 – 105): Each group of questions is based on a set of conditions. Choose the response
that most accurately and completely answers each question.
Mercotek carried out a study to compare the productivity of its night shift with that of its day shift. Every
week the company’s six crews – F, G, H, R, S and T – were ranked from first (most productive) to sixth
(least productive). There were no ties. For any given week, either G and T were the two night-shift crews or
else S and H were – the four other crews were the day-shift crews for that week. The following relationships
held for every week of the study:
F is more productive than G
R is more productive than S
R is more productive than T
S is more productive than H
G is more productive than T

101. Which of the following could be an accurate ranking of all the crews, in order from first to sixth, for
a given week of the study?
(A) F, G, T, R, S, H (B) F, R, G, T, H, S (C) G, R, T, S, H, F (D) R, F, G, S, H, T

102. If F is ranked third for a given week of the study, then which one of the following could also be true
of that week?
(A) G ranks second (B) H ranks fourth (C) R ranks second (D) S ranks fourth

103. Which one of the following CANNOT be the crew ranked fifth for any given week of the study?
(A) G (B) H (C) R (D) S

104. For any given week of the study, the ranking of all the crews is completely determined if which one
of the following is true?
(A) F ranks second that week (B) G ranks fifth that week
(C) H ranks third that week (D) R ranks third that week
105. If the night-shift crews rank fifth and sixth for a given week of the study, then which one of the
following could also be true of that week?
(A) G ranks fourth (B) H ranks fifth (C) R ranks third (D) S ranks fourth

Previous Years
Page 212 CLAT & AILET Papers
SECTION – D : MATHEMATICS

106. A purse contains some coins consisting of rupees, fifty paise coins and twenty five paise coins. If
coins be in the ratio of 2 : 3 : 10 and their total value is Rs.72, the number of twenty-five paise coins
will be:
(A) 100 (B) 140 (C) 120 (D) 80

107. In an examination, a student who secured 25% of the maximum marks fails by 60 marks but
another candidate who secures 45% of the maximum marks gets 10 marks more than required
passing marks. The maximum number of marks is:
(A) 450 (B) 350 (C) 525 (D) none of these

108. In a tour, I spent every day as many ten rupee (notes) as the number of days I had been away from
the home. My total expenditure was Rs.18,300. How long did I stay away from the home?
(A) 1 month (B) 2 months (C) 3 months (D) 4 months

109. Before 3 years, the average age of a five-member family was 17 years. A baby having been born and
the average of family is now 17 years. The present age of the baby is:
(A) 3 years (B) 2 years (C) 1 year (D) none of these

110. A publisher sells books to retailer at marked price which is 20% above his outlay. If on cash
payment, he allows a discount of 10% on the marked price. The publisher thus gains:
(A) 12% (B) 10% (C) 8% (D) 14%

111. The average age of A, B and C is 25 years. The ratio of their ages is 3 : 5 : 7. Find the age of A
(A) 21 years (B) 18 years (C) 15 years (D) Data Inadequate

112. A certain sum of money was deposited in a bank and it became two-fold in 10 years. What is the
rate of simple interest?
(A) 8% (B) 10% (C) 12% (D) 13%

113. If Rs.80 amounts to Rs.140 in 4 years, what will Rs. 96 amount to in 10 years at the same rate of
interest per annum?
(A) Rs.276 (B) Rs.306 (C) Rs.386 (D) Rs.300

114. A group of workers accepted to do a piece of work in 25 days. If 6 of them did not turn up for the work
and the remaining workers did the work in 40 days, then the original number of workers was
(A) 22 (B) 18 (C) 20 (D) 16

115. Two taps ‘A’ and ‘B’ can fill a water reservoir in 8 and 6 hours respectively. A third tap ‘C’ can empty
the tank completely in 24 hours. How long would it take to fill the empty tank when all the taps are
open?
(A) 4 hours (B) 5 hours (C) 6 hours (D) 3 hours

Previous Years
CLAT & AILET Papers Page 213
SECTION – E : ENGLISH

Directions: In the following questions, some of the sentences have errors and some have none. Find out
which part of the sentence has an error.

116. The number of marks carried by each question / are indicated / at the end of the Question. / No error.
A B C D

117. As much as I admire him for his sterling qualities / I cannot excuse him for / being unfair to his
A B
friends. / No error.
C D

118. Many times the news has been published / in the papers that the end of the world will be certain /
A B
if a nuclear war breaks out. / No error.
C D

119. She reluctantly said that/if nobody else was doing it/she will do it./No error
A B C D

120. Though child marriage/has been banned, /the custom still prevailed among some groups in India./
A B C
No error.
D
Directions: In the following questions, choose the word which is most nearly the SAME in meaning to the
bold word and mark it in the Answer Sheet.

121. LYNCH
(A) Hang (B) Madden (C) Killed (D) Shoot
122. His speech was nothing but a string of platitudes.
(A) grand statements (B) stereo-typed statements
(C) noble sentiments (D) humorous anecdotes

Directions: In the following questions, choose the word which is most nearly the OPPOSITE in meaning
to the bold word and mark it in the Answer Sheet.

123. PERENNIAL
(A) Frequent (B) Regular (C) Lasting (D) Rare

124. My first speech was a fiasco.


(A) success (B) disaster (C) fun (D) joy

Direction: Fill in the blanks.

125. If I ...................his address, I could write to him.


(A) knew (B) had known (C) know (D) will know

Previous Years
Page 214 CLAT & AILET Papers
126. When the morning.................., murder was discovered.
(A) occurred (B) arrived (C) came (D) happened

127. I...................a car to be absolutely necessary these days.


(A) consider (B) regard (C) think (D) agree

128. Do not intrude, they are talking..........................a confidential matter.


(A) on (B) for (C) over (D) in

129. I slept after lunch.........................armed chair.


(A) over (B) into (C) in (D) on

130. He is so ................. that he immediately believed my story of ghosts.


(A) innocent (B) credulous (C) vociferous (D) credible

131. I will help only ......................


(A) if I shall have time (B) if I would have time (C) if I had time (D) if I have time

132. He doesn’t work with hands, he works ....................... the machine.


(A) with (B) by (C) at (D) on

133. Every Shakespearean hero has an internal ...................... in his character


(A) defect (B) weakness (C) fault (D) flaw
Direction (Q. 134 – 135) : In this section, each passage consists of six sentences. The first and the sixth
sentences are given in the beginning. The middle four sentences in each passage have been removed and
jumbled up. These are labelled P, Q, R and S. You are required to find out the proper sequence of the four
sentences and mark accordingly on the Answer Sheet.

134. S1 : Two men held a struggling crazed dog down on a table.


S6 : The men watched him awe-struck.
P : Its mouth was smothered with saliva and a bite from its jaws might cause death.
Q : He put one end of the tube between his lips and lowered the other towards the foam covered
jaws.
R : Beside them stood Louis Pasteur holding a narrow glass-tube in one hand.
S : As the animal writhed, he carefully sucked some of the saliva up the tube.
The proper sequence should be
(A) P R Q S (B) P Q R S (C) S R P Q (D) R P S Q
135. S1 : There is only one monkey we can thoroughly recommend as an indoor pet
S6 : Finally, let me say that no other monkey has a better temper or more winning ways.
P : They quickly die from colds and coughs after the first winter fogs.
Q : It is the beautiful and intelligent Capuchin monkey.
R : The lively little Capuchins, however, may be left for years in an English house without the least
danger to their health.
S : The Marmosets, it is true, are more beautiful than the Capuchins and just as pleasing, but they
are too delicate for the English climate.
The proper sequence should be:
(A) P Q R S (B) Q R P S (C) Q S P R (D) R P S Q

Previous Years
CLAT & AILET Papers Page 215
Direction (Q. 136-145): Fill in the blanks
The Ganga is........(136).......they said, in her giving and her taking away. If you ask her for anything
she.......(137)......it gently. A calm and slow oblivion in...........(138).............. ever-flowing waves. Mridula
hoped it was true. She sat on the stone steps of the Kedar Ghat, ...........(139).................the first rays of
the Sun touch the ripples on the river with............(140)........of metallic gold. Ganga paschim vahini – the
east flowing river ...............(141)................... west at Varanasi like a moody woman. People bathed, said
prayers standing chest ........(142)................ in her water, muttering ..............(143)..............to her and to
the rising sun, repeating endlessly the .................(144)........... of prayer pouring water through their finger
in habits for which may be they never .............(145)........the meaning.

136. (A) kind (B) name (C) flow (D) cruel

137. (A) takes (B) refuse (C) ignore (D) gives

138. (A) their (B) her (C) any (D) only

139. (A) counting (B) persuing (C) dotting (D) watching

140. (A) dullness (B) dampness (C) glints (D) splint

141. (A) steers (B) hears (C) looks (D) turns

142. (A) deep (B) down (C) up (D) fallen

143. (A) appeals (B) pleas (C) invocations (D) considerations

144. (A) culture (B) rituals (C) works (D) deed

145. (A) mentioned (B) heard (C) conceived (D) knew

Directions (Q. 146 – 150): The questions in this section are based on the passage. The questions are to
be answered on the basis of what is stated or implied in the passage. For some of the questions, more
than one of the choices could conceivably answer the question. However, you are to choose the best
answer; that is, the response that most accurately and completely answers the questions.
In principle, a cohesive group - one whose members generally agree with one another and support one
another’s judgments – can do a much better job at decision making than it could if it were non-cohesive.
When cohesiveness is low or lacking entirely, compliance out of fear of recrimination is likely to be strongest.
To overcome this fear, participants in the group’s deliberations need to be confident that they are members
in good standing and that the others will continue to value their role in the group, whether or not they agree
about a particular issue under discussion. As members of a group feel more accepted by the others, they
acquire greater freedom to say what they really think, becoming less likely to use deceitful arguments or
to play it safe by dancing around the issues with vapid or conventional comments. Typically, then, the more
cohesive a group becomes, the less its members will deliberately censor what they say out of fear of being
punished socially for antagonizing their fellow members.
But group cohesiveness can have pitfalls as well: while the members of a highly cohesive group can feel
much freer to deviate from the majority, their desire for genuine concurrence on every important issue often
inclines them not to use this freedom. In a highly cohesive group of decision makers, the danger is not that
individuals will conceal objections they harbour regarding a proposal favoured by the majority, but that they

Previous Years
Page 216 CLAT & AILET Papers
will think the proposal is a good one without attempting to carry out a critical scrutiny that could reveal
grounds for strong objections. Members may then decide that any misgivings they feel are not worth
pursuing – that the benefit of any doubt should be given to the group consensus. In this way, they may fall
victim to a syndrome known as “groupthink”, which one psychologist concerned with collective decision
making has defined as “a deterioration of mental efficiency, reality testing, and moral judgment that results
from in-group pressures”.
Based on analyses of major fiascos of international diplomacy and military decision making, researchers
have identified groupthink behaviour as a recurring pattern that involves several factors: overestimation of
the group’s power and morality, manifested, for example, in an illusion of invulnerability, which creates
excessive optimism; closed-mindedness to warnings of problems and to alternative viewpoints; and
unwarranted pressures toward uniformity, including self-censorship with respect to doubts about the group’s
reasoning and a concomitant shared illusion of unanimity concerning group decisions. Cohesiveness of
the decision-making group is an essential antecedent condition for this syndrome but not a sufficient one,
so it is important to work toward identifying the additional factors that determine whether group cohesiveness
will deteriorate into groupthink or allow for effective decision making.

146. Why does the author thinks that the cohesive group can do a much better job at decision making
than it could if it were non-cohesive?
(A) The members of a highly cohesive group can feel much freer to deviate from the majority.
(B) Individuals will not conceal objections they harbour regarding a proposal favoured by the majority.
(C) Participants in the group’s deliberations are confident that they are members in good standing
and that the others will continue to value their role in the group, whether or not they agree about
a particular issue under discussion.
(D) All of the above.

147. Which one of the following most accurately expresses the main point of the passage?
(A) Despite its value in encouraging frank discussions, high cohesion can lead to a debilitating type
of group decision making called groupthink.
(B) Group members can guard against groupthink if they have a good understanding of the critical
role played by cohesion.
(C) Groupthink is a dysfunctional collective decision-making pattern that can occur in diplomacy
and military affairs.
(D) Low cohesion in groups is sometimes desirable when higher cohesion involves a risk of groupthink
behaviour.

148. A group of closely associated colleagues has made a disastrous diplomatic decision after a series
of meetings marked by disagreement over conflicting alternatives. It can be inferred from the passage
that the author would be most likely to say that this scenario
(A) provides evidence of chronic indecision, thus indicating a weak level of cohesion in general.
(B) indicates that the group’s cohesiveness was coupled with some other factor to produce a
groupthink fiasco.
(C) provides no evidence that groupthink played a role in the group’s decision.
(D) provides evidence that groupthink can develop even in some groups that do not demonstrate an
“illusion of unanimity”.

Previous Years
CLAT & AILET Papers Page 217
149. The passage mentions which one of the following as a component of groupthink?
(A) unjustified suspicions among group members regarding an adversary’s intentions.
(B) strong belief that the group’s decisions are right.
(C) group members working under unusually high stress, leading to illusions of invulnerability.
(D) the deliberate use of vapid, clichéd arguments.

150. It can be inferred from the passage that both the author of the passage and the researchers mentioned
in the passage would be most likely to agree with which one of the following statements about
groupthink?
(A) Groupthink occurs in all strongly cohesive groups, but its contribution to collective decision
making is not fully understood.
(B) The casual factors that transform group cohesion into groupthink are unique to each case.
(C) The continued study of cohesiveness of groups is probably fruitless for determining what factors
elicit groupthink.
(D) On balance, groupthink cannot be expected to have a beneficial effect in a group’s decision making.
*****

Previous Years
Page 218 CLAT & AILET Papers
AILET Question Paper 2013
SECTION – A : ENGLISH

Directions (Q. 1 - 6) : The questions in this section is based on the passage. The questions are to be
answered on the basis of what is stated or implied in the passage. For some of the questions, more than
one of the choices could conceivably answer the question. However, you are to choose the best answer;
that is, the response that most accurately and completely answers the question.
Although the legal systems of England and the United States are superficially similar, they differ
profoundly in their approaches to and uses of legal reasons: substantive reasons are more common
than formal reasons in the United States, whereas in England the reverse is true. This distinction
reflects a difference in the visions of law that prevails in the two countries. In England, the law has
traditionally been viewed as a system of rules; the United States favours a vision of law as an outward
expression of community’s sense of right and justice.
Substantive reasons, as applied to law, are based on moral, economic, political and other
considerations. These reasons are found both “in the law” and “outside the law” so to speak. Substantive
reasons inform the content of a large part of the law: constitutions, statutes, contracts, verdicts, and
the like. Consider, for example, a statute providing that “no vehicles shall be taken into public parks.”
Suppose that no specific rationales or purposes were explicitly written into the statute, but that it
was clear (from its legislative history) that the substantive purpose of the statute was to ensure quiet
and safety in the park. Now suppose that a veterans’ group mounts a World War II jeep (in running
order but without a battery) as a war memorial on a concrete slab in the park, and charges are
brought against its members. Most judges in the United States would find the defendants not guilty
because what they did had no adverse effect on park’s quiet and safety.
Formal reasons are different in that they frequently prevent substantive reasons from coming into
play, even when substantive reasons are explicitly incorporated into the law at hand. For example,
when a document fails to comply with stipulated requirements, the court may render the document
legally ineffective. A Will requiring written witness may be declared null and void and, therefore,
unenforceable for the formal reason that the requirement was not observed. Once the legal rule - that
a Will is invalid for lack of proper witnessing - has been clearly established, and the legality of the
rule is not in question, application of that rule precludes from consideration substantive arguments in
favour of the Will’s validity or enforcement.
Legal scholars in England and the United States have long bemused themselves with extreme
examples of formal and substantive reasoning. On the one hand, formal reasoning in England has led
to wooden interpretations of statutes and an unwillingness to develop the common law through judicial
activism. On the other hand, freewheeling substantive reasoning in the United States has resulted in
statutory interpretations so liberal that the texts of some statutes have ignored.
1. Which one of the following best describes the content of the passage as a whole?
(a) An analysis of similarities and differences between the legal systems of England and the United
States
(b) A re-evaluation of two legal systems with the use of examples
(c) A contrast between the types of reasons embodied in the United States and English legal
systems
(d) An explanation of how two distinct visions of the law shaped the development of legal reasoning.

Previous Years
CLAT & AILET Papers Page 219
2. It can be inferred from the passage that English judges would like to find the veterans’ group discussed
in the second paragraph guilty of violating the statute because
(a) not to do so would encourage others to act as the group did
(b) not to do so would be to violate the substantive reasons underlying the law
(c) the veterans failed to comply with the substantive purpose of the statute
(d) the veterans failed to comply with the stipulated requirements of the statute

3. From the discussion of Wills in the third paragraph it can be inferred that substantive arguments as
to the validity of a Will might be considered under which one of the following circumstances?
(a) The legal rule that a Will be witnessed in writing does not stipulate the format of the V
(b) The legal rule requiring that a Will be witnessed stipulates that the Will must be witnessed in
writing by two people
(c) The legal rule requiring that a Will be witnessed in writing stipulates that the witnessing must be
done in the presence of a judge
(d) A judge rules that the law can be interpreted to allow for a verbal witness to a Will in a case
involving a medical emergency

4. Which one of the following best describes the function of the last paragraph of the passage?
(a) It presents the consequences of extreme interpretations of the two types of legal reasons
discussed by the author
(b) It shows how legal scholars can incorrectly use extreme examples to support their views
(c) It corrects inaccuracies in legal scholars’ view of the nature of two types of legal systems
(d) It suggests how characterisations of the two types of legal reasons can become convoluted and
inaccurate

5. The author of the passage suggests that in English law a substantive interpretation of a legal rule
might be warranted under which one of the following circumstances?
(a) Social conditions have changed to the extent that to continue to enforce the rule would be to
decide contrary to present-day social norms
(b) The composition of the legislature has changed to the extent that to enforce the rule would be
contrary to the views of the majority in the present legislative assembly
(c) The legality of the rule is in question and its enforcement is open to judicial interpretation
(d) Individuals who have violated the legal rule argue that application of the rule would lead to unfair
judicial interpretations

6. The author of the passage makes use of all of the following in presenting the discussion of the
English and the United States legal systems except
(a) Comparison and contrast (b) Generalisation
(c) Explication of terms (d) A chronology of historical development

Directions (Q. 7 - 11): In the following questions, a group of sentences about a single topic are given. One
or more of the sentence(s) is/are grammatically incorrect. You have to identify the incorrect sentence(s).

7. I) It began with acquisitions in information technology and related services sector.


II) In pharmaceuticals, Wockhardt has bought C.P. Pharma of the United Kingdom for $ 10.85
million.
III) Tata Tea has taken over Tetley of the UK, the world’s biggest tea bag maker, for $ 430 million.
IV) With the processes, it has become the world’s second largest tea company.
(a) II and IV (b) IV only (c) II and III (d) I, II and IV

Previous Years
Page 220 CLAT & AILET Papers
8. I) There are two main reasons for that predatory mood.
II) Having established a domestic presence, the component makers are now looking for an
international presence.
III) Second, having improved their productivity, quality and reliability, Indian companies feel more
confident about spreading their wings abroad.
IV) Various other factors are being attributed to this Indian penchant for the takeover game in all
sectors.

(a) I only (b) I and II (c) II only (d) III and IV

9. I) Moving one by one step away from the expected with the graphics and photography can also
create reader’s interest.
II) Try using a conceptual image or photo to highlight your main message versus very first thing to
come to mind when thinking about your product or services.
III) Another form of contrast is in the actual design.
IV) An unusual fold in a brochure or direct mail piece can add excitement.

(a) I and II (b) II and III (c) I and IV (d) No error

10. I) The typeface that you choose for your print project is an important piece of the foremost overall
design process.
II) First, narrow down your choice by selecting the tone you want to present.
III) Typefaces can convey personality.
IV) For instance, if you are in the banking industry you might choose a classic serif font, such as
Garamond, to convey dependability.

(a) I only (b) II only (c) III only (d) IV only

11. I) Readability is crucial.


II) Be sure of the font we choose is legible and logical.
III) With all of the newest and interesting typefaces available today, it is tempting to pick one that
you think looks “cool”.
IV) This can work if you are going for an edgy look that will appeal to a young audience, but your
copy still needs to be easily understood.

(a) I and IV (b) II only (c) III only (d) II and III

Direction (Q. 12 - 15) : In the following questions, choose the word which is most nearly the same in
meaning to the bold word and mark it in the Answer Sheet.

12. Aberration
(a) Deviation (b) Embarrassment (c) Abhorrence (d) Absence

13. Potpourri
(a) Medley (b) Dose (c) Weird (d) Overabundance

14. Imposture
(a) Claim (b) Status (c) Destruction (d) Deception

Previous Years
CLAT & AILET Papers Page 221
15. Parley
(a)Discuss (b) Deliver (c) Sweeten (d) Race

Direction (Q. 16 -19) : In the following questions, choose the word which is most nearly the opposite in
meaning to the bold word and mark it in the Answer Sheet.

16. Protean
(a) Versatile (b) Amateur (c) Dull (d) Cautious

17. Predilection
(a) Antipathy (b) Ignorance (c) Dissonance (d) Disharmony

18. Impalpable
(a) Visible (b) Audible (c) Tangible (d) Fearless

19. Parochial
(a) Dogmatic (b) Dominant (c) Cosmopolitan (d) Niggardly

Direction (Q. 20 - 32) : Fill in the blanks.


20. The event passed ___________ without any untoward incident.
(a) of (b) on (c) off (d) away

21. Please give me __________ to drink.


(a) little water (b) a little water (c) any water (d) some water

22. Her true feelings manifested themselves in her sarcastic asides; only then was her ___________
revealed.
(a) sweetness (b) bitterness (c) anxiety (d) charm

23. The tapeworm is an example of _______________ organism, one that lives within or on another
creature, deriving some or all its nutrients from its host.
(a) a protozoan (b)a parasite (c) a hospitable (d) an autonomous

24. While the disease is in a latent state it is almost impossible to determine its existence by
(a)observation (b) analysis (c) examination (d) estimate

25. Language, culture and personality may be considered independently of each other in thought, but
they are _________ in fact.
(a) pervasive (b) inseparable (c) autonomous (d) immutable

26. The country is ushering ___________ a new era.


(a) into (b) in (c) of (d) over

27. He is a traitor ___________ the country.


(a) for (b) to (c) in (d) of

28. There is no culture in the world __________ absolutely pure.


(a) that which is into (b) that be (c) that is (d) that what is

Previous Years
Page 222 CLAT & AILET Papers
29. He led me ____________ the green lawn to the palatial building.
(a) upon (b) across (c) along (d) on

30. The lease of our premises has ____________ and we have to vacate it.
(a) run out (b) run off (c) run over (d) run down

31. His boss ____________ an explanation of his conduct with his colleagues.
(a) called up (b) called upon (c) called for (d) called off

32. He ____________ his departure for a week as his mother was not well.
(a) put off (b) put up (c) put out (d) put aside

Directions (Q. 33 - 35) : In this section, each passage consists of five sentences. The first sentence is
given in the beginning. The four sentences in each passage have been jumbled up. These are labeled P, Q,
R and S. You are required to find out the proper sequence of the four sentences and mark accordingly on the
Answer Sheet.

33. S1 : A suicide bomber killed 18 people near a hospital of Baghdad, keeping violence on the boil after
Iraq’s landmark election and ahead of an important Shia religious ceremony.
P : The bomber drove his vehicle towards local government offices and a hospital in the town, but
detonated it outside blast walls protecting buildings.
Q: Around 25 people were wounded in the attack, the second suicide car bombing in as many
days.
R: “Looking at the partial result, it appears the Sistani list will have over 50% and Kurdish parties
will come second,” said a Sunni politician.
S : Meanwhile, the two electoral power houses representing Iraq’s Shia and Kurdish communities
are poised to clinch the country’s two top jobs, with results from the landmark January 30 polls
expected anytime.

The proper sequence should be


(a) SRPQ (b) SQPR (c) PQSR (d) QPSR

34. S1 : It should be noted that Lenin, being the head of the government. held no official posts in the
Party Central Committee, but presided over sessions of the Central Committee plenary meetings
and also of the Political Bureau.
P : In performing his party duties, he was assisted by a secretary of the Central Committee, or the
head of the Secretariat.
Q: Officially, such a post did not exist at the time, but in practice one of the secretaries was
expected to
R: That being the case, Lenin not only heads the Council of People’s commissars, but was also
the de facto leader of the Party’s Central Committee.
S: When Lenin’s health deteriorated, the question arose of strengthening the secretariat and an
authoritative leader has to be found who could supervise Perth’s affairs in Lenin’s absence.

The proper sequence should be


(a) PQRS (b) SRPQ (c) SQPR (d) RPQS

Previous Years
CLAT & AILET Papers Page 223
35. S1: There is nothing in the world fascinating than watching a child grow and develop.
P: At first you think of it as just a matter of growing bigger.
Q: Then, as the infant begins to do things, you may think of it as “learning tricks”.
R: In some ways, the development of each child retraces the whole history of the human race,
physically and spiritually, step by step.
S: But, it’s really more complicated and full of meaning than that.

The proper sequence should be


(a) PQSR (b) RPQS (c) PQRS (d) RPSQ

SECTION-B : GENERAL NNOWLEDGE

36. The sun reaches its maximum angular distance from the equator at the
(a) zenith (b)solstice (c) equinox (d) noon time

37. The difference in the duration of day and night increases as one move from
(a) West to East (b) East and West of the prime meridian
(c) Poles to equator (d) Equator to poles

38. When can one record the lowest temperature of air ?


(a) Just before sunrise (b) At midnight
(c) At 3 a.m. (d) At sunrise

39. What is the difference between a geyser and a hot spring ?


(a) Water is ejected explosively in a geyser
(b) Water from a geyser may be cold
(c) Geysers are formed on volcanic mountains
(d) Geysers are more common in cold countries

40. Sher Shah is well-known for his administrative skill, especially his
(a) Market control steps (b) Land revenue system
(c) Mansabdari system (d) Law and order’

41. A monument resembling Taj Mahal was created by Aurangzeb in


(a) Ahmedabad (b) Aurangabad (c) Hoshangabad (d) Daulatabad

42. The maximum work in popularising female education in the nineteenth century was done by
(a) Ishwar Chandra Vidyasagar (b) J.E.D. Bethune
(c) D.K. Karve (d) Raja Ram Mohan Roy

43. The most learned ruler of the Delhi Sultanate who was well versed in various branches of learning
including Astronomy, Mathematics and Medicine was
(a) Iltutmish (b) Alauddin Khalji
(c) Muhammad Bin Tughlaq (d) Sikandar Lodhi

44. Permanent Settlement was introduced to


(a) help the ryots (b) earn a fixed and regular revenue
(c) help the landlords (d) improve the prospects of agriculture

Previous Years
Page 224 CLAT & AILET Papers
45. Who is, presently, the Director of the National Judicial Academy ?
(a) Prof. N.R. Madhava Menon (b) Dr. Balram K. Gupta
(c) Prof. M.P. Singh (d) Prof. K.N.Chandrasekharan Pillai

46. Which of the following is/are listed among the Directive Principles in Part IV of the Constitution of
India?
I) Equal pay for equal work
II) Uniform Civil Code
III) Small family norm
IV) Education through mother tongue at primary level

(a) I, II and Ill (b) l and II (c) II and III (d) I, II and IV

47. Who decides whether a Member of Parliament is subject to any disqualification?


(a) President (b) Speaker
(c) Election Commissioner (d) None of the above

48. The 9th Postal Zone of India covers


(a) Andhra Pradesh (b) Army Post Office
(c) Goa (d) Andaman and Nicobar Islands

49. Regarding No-Confidence Motion


(a) it is expressed against an individual minister or council of ministers
(b) no grounds have to be set out for the motion
(c) not less than 100 members have to support it for the Speaker to grant leave for its introduction
(d) it is always brought against the Prime Minister

50. Who is responsible for the


(a) Government (b) Voters (c) Political parties (d) Election Commission

51. The term fiscal deficit means


(a) total receipts minus expenditure
(b) total receipts minus interest payments on external debt
(c) revenue receipts minus expenditure
(d) revenue receipts minus defence expenditure

52. The functions of the Reserve Bank of India are


I) issuing all notes and coins
II) distributing all notes and coins
III) formulating monetary policy
IV) acting as agent of government in respect of India’s membership of the IMF

(a) I, II and III (b) II and III (c) II, III and IV (d) I, II, III and IV

53. What do you understand by Bear Raid ?


(a) An attempt to bring down the price of strong short selling
(b) Simultaneous buying of shares and debentures in view of getting more values in near future
(c) Higher rate of price paid for particular government share or debentures
(d) Any of the above

Previous Years
CLAT & AILET Papers Page 225
54. The foreign exchange reserves of India include
(a) Gold (b) SDRs (c) Foreign currencies (d) All of these

55. Among the remedies of inflation, we cannot include


(a) better capacity utilisation (b) lowering bank rate
(c) reducing budgetary deficit (d) an efficient public distribution system

56. Which one of the following is the most effective carrier of communications ?
(a) Cables (b) Radio waves (c) Microwaves (d) Optical fibres

57. Which one of the following compounds is used as a sedative ?


(a) Potassium Bromide (b) Calcium Chloride
(c) Ethyl Alcohol (d) Phosphorus Trichloride

58. In a fire siren, a jet of air is directed against a series of evenly spaced holes in a rotating disc. As the
disc speeds up, the tone
(a) maintains constant pitch (b) drops in pitch
(c) increases in wavelength (d) increases in frequency

59. After death, the


I) arteries are usually empty of blood
II) blood in arteries clots
III) veins are empty of blood
IV) veins are full of clots

(a) I and II (b) I only (c) I and IV (d) III only

60. A victim of a road accident is unconscious. Put in correct order the steps in First- Aid.
I) Treating for cardiac arrest
II) Treating for asphyxia
III) Treating for shock
IV) Arrest Haemorrhage
V) Cleanse and cover wounds

(a) I, II, III, IV, V (b) II, I, IV, Ill, V (c) II, III, I, IV, V (d) V, I, Ill, IV

61. Which one of the following international organisations does India disapprove of ?
(a) ASEAN (b) NATO (c) FAO (d) OPEC

62. Which space agency released surprising pictures of the remains of huge river which ran across
Mars at some point of time on January 17, 2013 ?
(a) NASA (b) European Space Agency
(c) Australian Space Agency (d) ISRO

63. Name the Indian woman weight-lifter who in January 2013 won Maiden National Title in National
Weightlifting Championship, 2013.
(a) Binitha Devi (b) Amanpreet Kaur (b) Manpreet Kaur (d) Karnam Malleswari

Previous Years
Page 226 CLAT & AILET Papers
64. Which of the following Indian company got the top rank among Indian companies in Top 500
companies list of Fortune India ?
(a) Indian Oil Corporation (b) ONGC
(c) RIL (d) SAIL

65. The Report of Transparency International India released in December, 2012 places India at _________
rank among 176 nations of the basis of Corruption Percentage Index.
(a) 93rd (b) 94th (c) 95th (d) 96th

66. Who is crowned as Pond’s Femina Ms. India, 2013?


(a) Zoya Afroz (b) Sobhita Dhulipala
(c) Navneet Kaur Dhillon (d) Anukriti Gusain

67. Which Hindi film was awarded 60th National Film Award, 2013 for Best Feature Film
(a) Paan Singh Tomar (b) Barfi
(c) Gangs of Wasseypur (d) Vicky Donor

68. Who is elected as the 266th Pope of the Roman Catholic Church at Rome In March, 2013?
(a) John Paul I (b) John Paul II (c) Benedict XVI (d) Francis

69. Which country joined WTO in August 2012 ?


(a)Russia (b) Belgium (c) Turkey (d) Taiwan

70. Which of the following are included in the definition of Sexual Harassment under the new Sexual
Harassment of Women at Workplace (Prevention Prohibition and Redressal) Bill?
(a) Physical contact and advances (b) Demand or request for sexual favours
(d) Making sexually coloured remarks (d) All of the above

SECTION - C : LEGAL APTITUDE

Directions (Q. 71 - 85) : Given below is a statement of legal principle followed by a factual situation. Apply
the principle to the facts given below and select the most appropriate answer.

71. LEGAL PRINCIPLE: If a person brings anything dangerous on his land which may prove harmful if
escapes, then that person must keep it at his peril. If a man fails to do so then he must be made
responsible to all natural consequences of its

FACTUAL SITUATION : A grows poisonous trees on his own land and lets the projection of the
branches of his trees on the B’s land. B’s cattle die because of nibbling the poisonous leaves.

DECISION :
(a) A is not liable to B because B must have taken due care to control his cattle
(b) A is not liable to B because trees are still on A’s land and there is no escape of dangerous thing
(c) A is liable to B because projection of branches with poisonous leaves amounts to escape
(d) A is not liable to B because he is not acting negligently

Previous Years
CLAT & AILET Papers Page 227
72. LEGAL PRINCIPLE : Master/Principal is vicariously liable for the tort committed by an servant/
agent, in the performance of his duties as an servant/agent.

FACTUAL SITUATION : A gave some cash and cheques to his friend B, who was an employee of
the State Bank of India, to deposit the same in that Bank in the account of A. B misappropriated the
amount. If A sues the Bank for damages, then the Bank is

DECISION :
(a) Liable to pay because it was the employer of B
(b) Liable to pay because the employee did it during business hours and while working as an
employee
(c) Not liable because he turned out to be the friend of the plaintiff’s husband Not liable because
while committing the fraud, he was not acting as the agent or employee of the Bank

73. LEGAL PRINCIPLE : Master / Principal is vicariously liable for the tort committed by an servant /
agent, in the performance of his duties as an servant / agent.

FACTUAL SITUATION : The plaintiff a bullion merchant was arrested by the police on a charge of
purchasing stolen goods. Some of the gold and silver ornaments were seized from the plaintiff and
were kept in the police station custody. The duty constable appropriated the gold ornaments and
escaped to a foreign country. The plaintiff after being acquitted brought an action against the State
for the compensation. In this case, compensation is

DECISION :
(a) Payable as there is misappropriation by the servants of the State
(b) Payable due to the fact that police constable has escaped to a foreign country
(c) Payable by the police constable himself and not by the State
(d) Not payable as the act was committed in discharge of sovereign function

74. LEGAL PRINCIPLES :


1) Joint tort-feasers means joint wrong doers. People can be joint tort-feasors in case of common
action, in fact or in law.
2) Joint tort-feasers are jointly and severally liable.

FACTUAL SITUATION : Two dogs belonging to two different owner acting in concert attacked a
flock of sheep and injured several sheeps. In an action for damages brought against the owners of
the dogs, if one of them puts a defence claiming that he was liable for one half only of the damage,
then wt one of the following statements is legally sustainable in the above case ?

DECISION :
(a) Neither of the owners is liable for damages done by his dog J
(b) Each owner was responsible for one half of the damage
(c) The owners themselves are not joint tort-feasers
(d) None of the above

Previous Years
Page 228 CLAT & AILET Papers
75. LEGAL PRINCIPLES :
1) Negligence is the omission to do something which a reasonable man, guided upon those con-
siderations which ordinarily regulate the conduct of human affairs, would do or doing something
which a prudent or reasonable man would not do.
2) Defendant’s duty of care depends on the reasonable foreseeability of injury which may be caused
to the plaintiff on breach of duty.

FACTUAL SITUATION : The defendants employees of the Municipal Corporation opened a manhole
in the street and in the evening left the manhole open and covered it by a canvass shelter, unattended
and surrounded by warning lamps. The plaintiff, an eight years old boy, took one of the lamps into
the shelter and was playing with it there, when he stumbled over it and fell into the manhole. A violent
explosion followed and the plaintiff suffered burn injuries. The defendants are

DECISION :
(a) Not liable because the injury to plaintiff is not foreseeable
(b) Liable because they should have completed the work before they left
(c) Not liable because they acted reasonably
(d) Liable because they acted unreasonably

76. LEGAL PRINCIPLES :


1) Master / Principal is vicariously liable for the tort committed by an servant / agent, in the performance
of his duties as an servant / agent.
2) Negligence is the omission to do something which a reasonable man, guided upon those
considerations which ordinarily regulate the conduct of human affairs, would do or doing something
which a prudent or reasonable man would not do.

FACTUAL SITUATION : A patient is brought to a hospital maintained by B. The patient is to be


operated upon. As a result of faulty oxygen supply, the patient dies on the operation theatre table,
then

DECISION :
(a) B would not be liable because the surgeon was negligent
(b) B would be liable because there is master and servant relationship between B and the surgeon
(c) B would not be liable because there is no_ master and servant relationship between B and the
surgeon
(d) B would not be vicariously liable because surgery is a highly skilled work on which B would have
no control

77. LEGAL PRINCIPLE: Defamation means publication of a false and derogatory statement about
another person without lawful justification.

FACTUAL SITUATION: A writes a defamatory letter to B containing defamatory remarks in reference


of B in Urdu language. A is aware that B does not know Urdu. B goes to C who knows Urdu and the
letter is read over by C to him. B sues A for defamation.

DECISION :
(a) A is liable
(b) A is not liable because he addresses the letter to B and not to C
(c) A is not liable because there is no publication of defamatory statement
(d) A is not liable because he is unaware that the letter can be read over by someone else to B

Previous Years
CLAT & AILET Papers Page 229
78. LEGAL PRINCIPLES: A private nuisance may consist of :
1) Any interference with a person’s use or enjoyment of land.
2) The act of wrongfully causing or allowing the escape of deletes things into another person’s land
e.g. water, smoke, smell, etc.
FACTUAL SITUATION: D erected a brick grinding machine adjoining the premises of P, a medical
practitioner. The dust from the machine polluted the atmosphere and caused inconvenience to P
and his patients. Here
DECISION:
(a) P cannot stop D because D is carrying on lawful business
(b) D has fundamental right to carry on any kind of business
(c) P can claim compensation because D’s activity amounts to nuisance for P
(d) D can claim compensation from P because P is trying to maliciously prosecute D

79. LEGAL PRINCIPLE: False imprisonment is the confinement of a person without just cause or
excuse. There must be a total restraint of the person and the onus of proving reasonable cause is on
the defendant.
FACTUAL SITUATION: A entered in B’s park where there was an artificial lake for the boating. A
paid Rs. 100 for entering the park and has to pay Rs. 100 at the time of exit. A waited for 30 minutes
but no boat was available. A came out, however, denied to pay Rs. 100 for exit, B did not allow A to
leave the park unless he paid Rs. 100 for exit. A sued B for false imprisonment.
DECISION:
(a) B is guilty of false imprisonment
(b) B is not guilty of false imprisonment
(c) A can lawfully refuse to pay Rs. 100 when no boat was available
(d) A can ask for even Rs. 100 given for entering the park as B’s services are deficient in the park
and can sue B for false imprisonment

80. LEGAL PRINCIPLES: In a suit for malicious prosecution, the plaintiff must prove the following
essentials:
1) That he was prosecuted by the defendant.
2) That the proceeding complained was terminated in favour of the present plaintiff.
3) That the prosecution was instituted against him without any just or reasonable cause.
4) That the prosecution was instituted with a malicious intention, that is, not with the mere inten-
tion of getting the law into effect, but with an intention, which was wrongful in fact.
5) That he suffered damage to his reputation or to the safety of person, or to security of his
property.
FACTUAL SITUATION: A recovered a large sum of money from Railway Co. for personal injuries.
Subsequently, Railway Co. came to know that injuries were not real and were created by doctor B.
Railway Co. prosecuted B for playing fraud on the company, but B was acquitted. B sued Railway
Co. for malicious prosecution. In the light of these facts which of the following statements is true ?
DECISION:
(a) Railway Co. is guilty of malicious prosecution because it acted without reasonable cause
(b) Railway Co. is not guilty of malicious prosecution because the Co. took reasonable care in
determining the facts and honestly believed them to be true
(c) Railway Co. is liable because it acted negligently
(d) None of the above
Previous Years
Page 230 CLAT & AILET Papers
81. LEGAL PRINCIPLE: Attempt is an act done with an intent to commit crime, and forming part of the
series of acts which would constitute actual commission of the crime, if not interrupted.
FACTUAL SITUATION: A intending to murder B by poison purchases poison and mixes the same
with a glass of water. He gave to the bearer to serve B. The bearer while approaching B, loses the
balance and the glass drops out of his tray.
DECISION:
(a) A has not committed any offence
(b) A has committed the offence of murder
(c) A has committed the offence of attempt to murder
(d) A has not committed an offence of attempt to murder because nothing happened to B

82. LEGAL PRINCIPLES:


1) The concept of joint liability comes under Section 34 of IPC which states that “when a criminal
act is done by several persons, in furtherance of the common intention of all, each of such
persons is liable for that act in the same manner as if it were done by him alone.”
2) A person abets an offence, who abets either the commission of an offence, or the commission
of an act which would be an offence, if committed by a person capable by law of committing an
offence with the same intention or knowledge as that of the abettor.
3) A criminal conspiracy takes place when two or more people get together and plan to commit a
crime and then take some action toward carrying out that plan. The action taken does not have
to be a crime itself to further the conspiracy.
FACTUAL SITUATION: X and Y conspire to poison Z. X in pursuance of the conspiracy procures
the poison and delivers it to Y in order that he may administer it to Z. Y in pursuance of the
conspiracy, administer the poison in the presence of X and thereby causes Z’s death. What offences
X and Y have committed ?
DECISION:
(a) Y has committed the offence of murder and X was an abettor
(b) Both X and Y has committed the offence of criminal conspiracy
(c) X has not committed any offence
(d) Both X and Y has committed the offence of m urder

83. LEGAL PRINCIPLES:


1) Whoever, intending to take dishonestly any movable property out of the possession of any
person without that person’ s consent, moves that property in order to such taking, is said to
commit theft.
2) Whoever dishonestly misappropriates or converts to his own use any movable property, shall be
punished with imprisonment of either description for a term which may extend to two years, or
with fine, or with both.
FACTUAL SITUATION: A takes umbrella belonging to Z out of Z’s possession in good faith, believing
at the time when he took it, that the property belongs to himself. His wife points out after some days
that the umbrella does not belong to them but to Z. After coming to know that, A dishonestly keeps
the umbrella.
DECISION:
(a) A is guilty of criminal misappropriation
(b) A is guilty of criminal breach of trust
(c) A is guilty of theft
(d) Both A and his wife are guilty of criminal misappropriation
Previous Years
CLAT & AILET Papers Page 231
84. LEGAL PRINCIPLE: When at the desire of the promisor, the promisee has done or abstained from
doing, or does or abstains from doing, or promises to do or abstain something, such an act or
abstinence or promise is called consideration for the promise.

FACTUAL SITUATION: A daughter promises to give maintenance to her uncle in consideration of


her mother making a gift of certain properties to her. The daughter pleads lack of consideration when
the uncle seeks to enforce the contract. She says that the uncle is a stranger to the consideration
and so he cannot enforce the contract. The daughter

DECISION:
(a) Will succeed because uncle being a stranger to the consideration cannot enforce it
(b) Will not succeed because uncle is a near relative and in such cases consideration is not
necessary
(c) Cannot succeed because consideration might move from any person
(d) None of the above

85. LEGAL PRINCIPLES:


1) Parties to contract should be capable of entering in to contract, only then they can lay the
foundation of a valid contract.
2) Every person is competent to contract who is of the age of majority.

FACTUAL SITUATION: A minor agreed with B to become a tenant of his house and to pay Rs.
1,000/- for the furniture therein. He paid Rs. 800/- in cash and gave a promissory note for the
balance. A occupied the premises and used the furniture for some months and then brought an
action for refund of consideration, in this case

DECISION:
(a) A is liable to pay Rs. 1,000
(b) A is liable to pay remaining Rs. 200
(c) A is liable to refund of Rs. 800
(d) Neither B is liable to refund Rs. 800 nor A is under obligation to pay Rs. 200

86. “Begar” means


(a) Voluntary work without payment (b) Involuntary work without payment
(c) Involuntary work with payment (d) Voluntary work with payment

87. In India, the reckoning date for the determination of the age of the juvenile is the
(a) date of offence (b) date of trial (c) date of judgment (d) date of arrest

88. Which of the following rights is not available to the citizens of India under Article 21 of the Constitution?
(a) Right to Privacy
(b) Right to Die
(c) Right to Health and Medical Assistance
(d) Right to Get Pollution free Water and Air

89. The “Eco-mark” is given by the Ministry of Environment and Forest to the consumer products which
(a) are degradable (b) are environment friendly
(c) conform to the standards of ISI (d) are electricity saver

Previous Years
Page 232 CLAT & AILET Papers
90. The EIA is abbreviated form of
(a) Environment Impact Assessment
(b) Environment Implementation Arrangement
(c) Environment Impact Apparatus
(d) Environmental Institute of Awareness

91. ____________ was the leader of “Chipko Movement”.


(a) Medha Patkar (b) Subhash Gheising
(c) Balasaheb Doeras (d) Sunderlal Bahuguna

92. When two persons descend from a common ancestor but by different wives, they are said to be
related to each other by
(a) Sapinda relationship (b) Half blood
(c) Full blood (d) None of the above

93. “Pacta sunt Servanda” means that


(a) The States are not bound to respect the agreements entered into by them
(b) The agreements entered into by the States will be respected and followed by them in good faith
(c) The States are under no obligation to follow the agreements in good faith
(d) Though the States are bound to recognise an agreement entered into by them but cannot be
compelled to follow it

94. An offence of breaking a divine idol


(a) Blasphemy (b) Salus populi (c) Crime (d) Sacrilege

95. What is Plea Bargaining ?


(a) A conference between opposing lawyers and judge to determine the time a case should take
place
(b) A procedure by which an accused pleads guilty in exchange for a lesser punishment
(c) A conference between opposing lawyers to settle the claim
(d) A conference between the victim and the accused to settle the claim

96. The act by members of a trade union, persuading others not to work is called as
(a) Non-cooperation (b) Picketing (c) Sit-in-strike (d) Strike

97. In India, cyber terrorism is an offence punishable under


(a) Information Technology Act (b) POTA
(c) TADA (d) Indian Penal Code

98. When a judge makes certain remarks in the course of his judgement, which are said “by the way”
and do not have direct bearing on the facts at hand, such remarks are called
(a) Ratio Decidendi (b) Obiter Dictum (c) Observations (d) Comments ordinaralis

99. In Extradition Treaty, extradition means


(a) Order of Indian Court will apply to Indian living elsewhere than India
(b) Export without double taxation
(c) Two countries will deport criminal on reciprocal basis to each other
(d) None of the above

Previous Years
CLAT & AILET Papers Page 233
100. Who is the Chairperson of the 20th Law Commission of India ?
(a) Justice Altmas Kabir (b) Justice P.V. Reddy
(c) Justice A. R. Lakshmanan (d) Justice D. K. Jain

101. __________ of the Constitution of India lays down that Union of India and the States are juristic
person and can sue and be sued.
(a) Article 225 (b) Article 268 (c) Article 300 (d) Article 348

102. A post dated cheque bears _________ date.


(a) past (b) no (c) future (d) present

103. According to the Doctrine of__________, every generation is obliged to preserve its natural and
cultural heritage for the enjoyment of the future generations.
(a) Sustainable development (b) Polluter pay principle
(c) Precautionary principle (d) Inter-generational equity

104. Which of the following is not recommended by the Justice Verma Committee ?
(a) Minimum sentence for a rapist should be enhanced from 7 years to 10 years
(b) The age of juvenile should not be lowered from 18 to 16 years
(c) Life imprisonment must always mean jail for “the entire natural life of the convict”
(d) Death sentence for “rarest of rare cases” of rape

105. High Courts has been constituted in all these States in January, 2013 except
(a) Assam (b) Meghalaya (c) Manipur (d) Tripura

SECTION - D : REASONING

106. Information that is published is part of the public record. But information that a reporter collects, and
sources that he contacts, must be protected in order for our free press to function free of fear.

The above argument is most severely weakened by which one of the following statements ?
(a) Public information is usually reliable
(b) Undocumented evidence may be used to convict an innocent person
(c) Members of the press act ethically in most cases
(d) The sources that a reporter contacts are usually willing to divulge their identity

107. Psychological novels are superior to novels of adventure. Immature readers prefer novels of adventure
to novels with less action and greater psychological depth. The immature reader, who prefers James
Bond’s exploits to the subtleties of Henry James, can be identified easily by his choice of inferior
reading matter.

A criticism of the logic of this argument would be likely to find fault with the author’s
(a) Presupposing the conclusions he wishes to prove
(b) Failure to define “adventure” clearly
(c) Failure to cite possible exceptions to this rule
(d) Hasty generalisation on the basis of a limited specific case

Previous Years
Page 234 CLAT & AILET Papers
108. Many very effective prescription drugs are available to patients on a “one time only” basis. Suspicious
of drug abuse, physicians will not renew a prescription for a medicine that has worked effectively for
a patient. This practice denies a patient her right to health.

Which one of the following is a basic assumption made by the author ?


(a) A new type of medicine is likely to be more expensive
(b) Physicians are not concerned with a patient’s health
(c) Most physicians prescribe inadequate amounts of medicine
(d) Patients are liable to suffer the same ailment repeatedly

109. Vijay claimed that the large dent in the fender of the company-owned vehicle he had borrowed was
caused by the careless act of another motorist, who backed into the car when it was parked in a
public garage. Yet Vijay’s own car has several dents in its fenders, all of which he acknowledges as
having caused by his own careless driving. Therefore, Vijay’s contention that the dent in the formerly
undented company-owned vehicle was caused by the careless act of another person is not true.

The reasoning in this argument is vulnerable because it


(a) Fails to recognise that Vijay could be lying about the dents in his own vehicle
(b) Fails to recognise that the motorist who backed into him simply did not see him
(c) Fails to acknowledge that many such accidents occur in parking garages
(d) Presumes, without justification, that because Vijay has caused similar dents to his own car, he
caused the dent in the company car

110. It takes a good telescope to see the moons of Neptune. I can’t see the moons of Neptune with my
telescope. Therefore, I do not have a good telescope.

Which one of the following most closely parallels the logic of this statement ?
(a) It takes two to tango. You are doing the tango. Therefore, you have a partner
(b) If you have a surfboard, you can surf. You do not have a surfboard. Therefore, you cannot surf
(c) You can write a letter to your friend with a pencil. You do not have a pencil. Therefore, you cannot
write the letter
(d) If you know the area of a circle, you can find its circumference. You cannot figure out the
circumference. Therefore, you do not know the area

Directions (Q. 111 - 114) : In each question below is given statement followed by two assumptions numbered
I and II. An assumption is something supposed or taken for granted. You have to consider the statement
and the following assumptions and decide which of the assumptions is implicit in the statement. Mark
answer:
(a) If only I assumption is implicit
(b) If only assumption II is implicit
(c) If neither I nor II is implicit; and
(d) If both I and II are implicit

111. Statement : As poor people in India prefer and use jaggery rather than sugar, the government
has decided to decontrol and scrap sugar distribution through Public Distribution
System (PDS).
Assumption I : Jaggery is freely available at reasonable price to all poor people.
Assumption II : PDS has lost its utility.

Previous Years
CLAT & AILET Papers Page 235
112. Statement : Unless country ‘X’ achieves total literacy, it cannot achieve its mission of
development.
Assumption I : It is possible to achieve total literacy in country ‘X’.
Assumption II : No development is possible without a proper mission.

113. Statement : Many species of animals on our earth are still not studied scientifically and if we
do not do this work urgently, many species will face extinction.
Assumption I : Earth may lose all types of life very shortly.
Assumption II : It is desirable and possible to study scientifically many animal species.

114. Statement : Herbs can safely be used for treating diseases of human animal species.
Assumption I : Herbs cannot be used for treating diseases of animals.
Assumption II : Herbs treatment is getting popular.

Directions (Q. 115 - 120): Each group of questions is based on a set of conditions. Choose the response
that most accurately and completely answers each question.

At the snack cafe at a party, Ali, Bina, Champak and Diya are eating cookies. There are five kinds of
cookies to choose from - chocolate chip cookies, oatmeal cookies, sugar cookies, peanut butter cookies
and raisin cookies. Each of these four people eats at least two kinds of cookies. Their choices are governed
by the following rules :

At most two of them eat oatmeal cookies


At least two of them eat sugar cookies
Ali does not eat any sugar cookies
Bina and Champak do not eat the same type of cookies Bina eats chocolate chip cookies
Champak eats sugar cookies
No one eats both raisin cookies and sugar cookies
If someone eats raisin cookies, they also eat peanut butter cookies

115. Which one of the following must be true ?


(a) Champak eats chocolate chip cookies
(b) Ali eats chocolate chip cookies
(c) Bina does not eat peanut butter cookies
(d) Diya does not eat raisin cookies

116. If Bina eats exactly three kinds of cookies, which one of the following must be true ?
(a) Champak eats exactly three kinds of cookies
(b) Diya eats only sugar cookies
(c) If Ali eats oatmeal cookies, Diya eats oatmeal cookies
(d) Champak eats oatmeal cookies

117. Which one of the following cannot be true ?


(a) No one eats raisin cookies
(b) Ali and Diya both eat oatmeal cookies
(c) All and Diya both eat chocolate chip cookies
(d) Bina and Champak eat the same number of kinds of cookies.

Previous Years
Page 236 CLAT & AILET Papers
118. Which pair of cookie types could each be eaten by at least three different people ?
(a) Chocolate chip and oatmeal (b) Oatmeal and peanut butter
(c) Chocolate chip and peanut butter (d) Oatmeal and sugar

119. Which pair of cookie types contains a cookie type eaten by exactly two different people ?
(a) Chocolate chip and oatmeal (b) Oatmeal and peanut butter
(c) Chocolate chip and peanut butter (d) Sugar and raisin

120. Which cookie type could be eaten by none of the people ?


(a) Chocolate chip (b) Oatmeal (c) Sugar (d) Raisin

Directions : Read the following directions and answer the questions 121 to 123.

Sunita, Rahul and Suraj are children of Mrs. and Mr. Ahuja. Rani, Rohan and Shyam are children of Mrs.
and Mr. Malik. Shyam and Sunita are married and Arjun and Sohan are their children. Gunjan and Romesh
are children of Mrs. and Mr. Gandhi. Gunjan is married to Suraj and has three children named Rupali, Sonu
and Ravi.

121. How is Rahul related ot Arjun ?


(a) Brother-in-law (b) Cousin (c) Uncle (d) Maternal uncle

122. Rani is Sohan’s


(a) Niece (b) Sister (c) Aunt (d) Sister-in-law

123. Shyam and Suraj are related as


(a) Brother-in-law (b) Cousin (c) Brother (d) Nephew

124. Shyam travels 5 km towards East and turns left and moves 6 km further. He then turns right and
moves 9 km. Finally, he turns once again to his right and moves 6 km. How far is he from the starting
point ?
(a) Brother-in-law (b) Cousin (c) Uncle (d) Maternal uncle

125. Reaching a place of appointment on Friday, I found that I was two days earlier than the scheduled
day. If I had reached on the following Wednesday, how many days late would I have been ?
(a) One day (b) Two days (c) Three days (d) Four days

126. X was born on March, 6, 1993. The same year Independence Day was celebrated on Friday. Find out
the birth day of X.
(a) Thursday (b) Saturday (c) Friday (d) Wednesday

127. Kamala would like to complete all her home work before 10 p.m. in order to watch an important TV
programme. She has 40 minutes assignment in each of her five subjects. What is the latest time at
which she can start and still complete work in time for the programme ?
(a) 6.40 p.m. (b) 6.30 p.m. (c) 7.10 p.m. (d) 7.20 p.m.

Previous Years
CLAT & AILET Papers Page 237
Direction : In the following number series (Q. No. 128 to 131) only one number is wrong.

128. 21, 28, 33, 35, 37, 36


(a) 33 (b) 35 (c) 21 (d) 36

129. 5, 13, 29, 61, 120, 253


(a) 61 (b) 29 (c) 120 (d) 253

130. 0, 7, 28, 63, 124, 215


(a) 7 (b) 63 (c) 28 (d) 215

131. 9, 19, 40, 83, 170, 340, 340


(a) 340 (b) 170 (c) 83 (d) 40

132. In a certain code, TEAMWORK is written as NBFUJQNV and SOME is written as PTDL. How is
PERSON written in that code ?
(a) QDOOPT (b) QDOMNR (c) SFQMNR (d) SFQOPT

133. In certain code, BASKET is written as 5$3%#1 and TRIED is written as 14*#2. How is SKIRT
written in that code ?
(a) 3%*41 (b) 3*%41 (c) 3%#41 (d) 3#4%1

134. Eagle : Swoops : : Duck : ?


(a) Swims (b) Flits (c) Waddles (d) Floats

135. APPLE : 50 : : ORANGE : ?


(a) 61 (b) 63 (c) 60 (d) 69

Directions : In questions 136 to 140, a particular word is given. The word is followed by four words as given
in the alternatives. One of these four words cannot be formed by using the letters given in the question.

136. ENTHUSIASTICALLY
(a) SATIATE (b) HELMINTH (c) SHALE (d) TANTALUS

137. CONCENTRATE
(a) CENTRE (b) CONCERN (c) REASON (d) TREAT

138. INTRANSIGENT
(a) STAIN (b) GRATE (c) TRACE (d) RESIGN

139. PERPETUATION
(a) REPUTE (b) RETAIN (c) PIPETTE (d) PENANCE

140. ESTRANGEMENT
(a) ENTANGLE (b) RNTREAT (c) GERMAN (d) TANGEN

Previous Years
Page 238 CLAT & AILET Papers
SECTION - E : MATHEMATICS

141. The difference between the number of numbers from 2 to 100 which are not divisible by any other
number except 1 and itself and the numbers which are divisible by atleast one more number along
with one and itself.
(a) 25 (b) 50 (c) 49 (d) none of these

142. The average of three prime numbers lying between 47 and 74 is 191/3. The greatest possible sum
between any two out of the 3 prime numbers is
(a) 120 (b) 138 (c) 132 (d) 136

143. Divya purchased 6 mangoes, 10 oranges and 5 apples for a certain amount. With 40% less amount
Anu could purchase 3 mangoes, 5 oranges and 4 apples. What percentage of the total amount did
Divya spend on apples ?
(a) 25% (b) 33.333%
(c) 40% (d) can’t be determined

144. Military camp is having the provisions for 300 people consuming 600 grams daily for 75 days. They
are joined by 60 more men and daily ration was reduced by 100 grams. How long will the provisions
last approximately ?
(a) 61 days (b) 67 days (c) 75 days (d) 88 days

145. A contractor Abhay Singh employed some men to do a piece of work which can be done by 16 men
in 14 days. At the end of 5 days, 7 of the men stopped working and 3 days later half of the remainder
stopped working; the rest finished the work in 5 days. What is the number of men originally employed?
(a) 25 (b) 27 (c) 29 (d) 33

146. Ram is travelling by car at the rate of 40 kmph. After 80 km he rests for 20 minutes. How long
will he take to cover a distance of 240 kms ?
(a) 6 hrs 40 min (b) 6 hrs 11 min (c) 5 hrs 11 min (d) 7 hrs 15 min

147. There is leak in the bottom of a tank. This leak can empty a full tank in 8 hours. When the tank is full,
a tap is opened into the tank which admits 6 litres per hour and the tank is now emptied in 12 hours.
What is the capacity of the tank ?
(a) 28.8 litres (b) 36 litres (c) 144 litres (d) 145 litres

148. A and B walk from X to Y, a distance of 27 km at 5 km/hr and 7 km/hr, respectively. B reaches Y and
immediately turns back meeting A at Z. What is the distance from X to Z ?
(a) 25 km (b) 22.5 km (c) 24 km (d) 20 km

149. Three consecutive positive even numbers are such that thrice the first number exceeds double the
third by 2, the third number is
(a) 10 (b) 14 (c) 16 (d) 12

150. Devendra, a computer accessories vendor gave a discount of 20% on a pen-drive and his profit
reduced from Rs. 200 to Rs. 100. What is Devendra’s cost price ?
(a) Rs. 300 (b) Rs. 200 (c) Rs. 400 (d) None of these

Previous Years
CLAT & AILET Papers Page 239
AILET Question Paper 2014

SECTION - A : ENGLISH

Directions (Q. 1 – 7) : The questions in this section are based on the passage. The questions are to
be answered on the basis of what is stated or implied in the passage. For some of the questions,
more than one of the choices could conceivably answer the question. However, you are to choose the
best answer; that is, the response that most accurately and completely answers the questions.

Under very early common law, all felonies were punishable by death. The perpetrators of the felony
were hanged whether or not a homicide had been committed during the felony. Later, however, some
felonies were declared to be non-capital offences. The common law courts, in need of a deterrent to
the use of deadly force in the course of these non-capital felonies, developed the “felony-murder” rule.
The first formal statement of the rule stated: “Any killing by one in the commission of a felony is guilty
of murder.” The killing was a murder whether intentional or unintentional, accidental or mistaken. The
usual requirement of malice was eliminated and the only criminal intent necessary was the intent to
commit the particular underlying felony. All participants in the felony were guilty of murder-actual
killer and non-killer confederates.

Proponents of the rule argued that it was justified because the felony demonstrated a lack of concern
for human life by the commission of a violent and dangerous felony and that the crime was murder
either because of a conclusive presumption of malice or simply by force of statutory definition.

Opponents of the rule describe it as a highly artificial concept and “an enigma wrapped in a riddle.”
They are quick to point out that the rule has been abandoned in England where it originated, abolished
in India, severely restricted in Canada and a number of other commonwealth countries, is unknown in
continental Europe, and abandoned in Michigan. In reality, the real strength of the opponents’ criticism
stems from the bizarre and of times unfair results achieved when the felony - murder rule is applied
mechanically. Defendants have been convicted under the rule where the killing was purely accidental,
or the killing took place after the felony during the later flight from the scene; or a third party killed
another (police officer killed a citizen or vice versa; or a victim died of a heart attack 15 -20 minutes
after the robbery was over; or the person killed was an accomplice in the felony).

Attacks on the rule have come from all directions with basically the same demand - re-evaluate and
abandon the archaic legal fiction; restrict and limit vicarious criminal liability; prosecute killers for
murder, not non-killers; increase punishment for the underlying felony as a real deterrent; and initiate
legislative modifications. With the unstable history of the felony - murder rule, including its abandonment
by many jurisdictions in this country, the felony - murder rule is dying a slow but certain death.

1. Which one of the following best states the central idea of the passage ?
(a) The felony - murder rule should be abolished.
(b) Some jurisdictions are about to abandon the felony — murder rule.
(c} The felony - murder rule can be unfair.
(d) Supreme Court of the United States.

Previous Years
Page 240 CLAT & AILET Papers
2. The felony - murder rule was developed in order to
(a) deter felonies
(b) deter murders
(c) deter deadly force in felonies
(d) extend the definition of murder to any malicious act resulting in death

3. Arguments in favour of the felony - murder rule may include all of the following EXCEPT
(a) We can infer that anyone undertaking a dangerous felony demonstrates an indifference to
human life.
(b) If the punishment for the use of deadly force whether intended or not is the same, criminals
will be less likely to use deadly force.
(c) Because a life has been taken, the crime is murder by force of statutory definition.
(d) The victim of murder may be an accomplice of the felony.

4. According to the passage, opponents of the felony - murder rule have raised all of the following
objections to the statute EXCEPT
(a) The felony - murder rule results in murder prosecutions of defendants who have not committed
murder.
(b) The felony - murder rule assigns a criminal liability vicariously.
(c) The felony - murder rule is based upon a presumption of malice even if death is wholly
accidental.
(d) The felony - murder rule deters the use of deadly force in non-capital felonies.

5. In which of the following situations would the defendant NOT be liable to the charge of murder
under the felony — murder rule ?
(a) In escaping from an unsuccessful attempt to rob a bank, the defendant crashes his car,
killing an innocent pedestrian in another city.
(b) A bank security officer, pursuing the defendant after a robbery, falls down a flight of stairs
and suffers serious permanent brain and spinal cord injuries.
(c) The driver of the escape car, who has not entered the bank, crashes the car killing the
armed gunman who committed the robbery.
(d) A bank teller, locked safely in the bank vault by the robber, has a stroke and dies.

6. According to the passage, the decline of support for the felony - murder rule is indicted by the
abandoning of the rule in all of the following locations EXCEPT
(a) Continental Europe (b) India
(c) England (d) Canada

7. The author believes that the felony - murder rule is


(a) unconstitutional
(b) bizarre and unfair
(c) a serviceable rule unfairly attacked by the “intelligentsia ”
(d) an unfair equating of intent to commit a felony and intent to commit murder

Directions (Q. 8 - 11) : In the following questions, choose the word which is most nearly the OPPOSITE in
meaning to the bold word and mark it in the Answer Sheet.

8. Ambiguity
(a) lucidity (b) basal (c) dull (d) necessity

Previous Years
CLAT & AILET Papers Page 241
9. Antidote
(a) medicine (b) poison (c) anodyne (d) amity

10. Which is NOT a synonym for ‘accumulation’?


(a) collection (b) conglomeration (c) assemblage (d) collagen

11. Which is NOT a synonym for ‘incline’?


(a) trenchant (b) slope (c) acclivity (d) gradient

Directions (Q. 12 - 15) : In the following questions, choose the word which is most nearly the SAME in
meaning to the bold word and mark it in the Answer Sheet.

12. Aphorism
(a) prune (b) wither (c) aphis (d) proverb

13. Passe
(a) rude (b) old - fashioned (c) modern (d) chic

14. Vituperation
(a) moisture (b) parallel (c) malediction (d) recover

15. Qualm
(a) concavity (b) amplitude (c) misgiving (d) repute

Directions (Q. 16 - 17) : Choose the exact meaning of the idioms/phrases.

16. She exhibited remarkable sang froid during the crisis.


(a) temper (b) irritation (c) composure (d) anger

17. The co-operation and esprit de corps between the soldiers and the officers was directly responsible
for their victory.
(a) bravery (b) loyalty (c) subordination (d) unity

Directions (Q. 18 - 19) : Answer the questions based on the following information.
In each of the question below, four different ways of writing a sentence are indicated. Choose the best
way of writing the sentence.

18. (a) The main problem with the notion of price discrimination is that it is not always a bad thing,
but that it is the monopolist who has the power to decide who is charged what price.
(b) The main problem with the notion of price discrimination is not that it is always a bad thing,
it is the monopolist who has the power to decide who is charged what price.
(c) The main problem with the notion of price discrimination is not that it is always a bad thing,
but that it is the monopolist who has the power to decide who is charged what price.
(d) The main problem with the notion of price discrimination is not it is always a bad thing, but
that it is the monopolist who has the power to decide who is charged what price.

Previous Years
Page 242 CLAT & AILET Papers
19. (a) A symbiotic relationship develops among the contractors, bureaucracy and the politicians,
and by a large number of devices, costs are artificially escalated and black money is
generated by underhand deals.
(b) A symbiotic relationship develops among contractors, bureaucracy and politicians, and
costs are artificially escalated with a large number of devices and black money is generated
through underhand deals.
(c) A symbiotic relationship develops among contractors, bureaucracy and the politicians, and
by a large number of devices costs are artificially escalated and black money is generated
on underhand deals.
(d) A symbiotic relationship develops among the contractors, bureaucracy and politicians, and
by large number of devices costs are artificially escalated and black money is generated
by underhand deals.

Directions (Q. 20 - 23) : In this section, each passage consists of five sentences. The first sentence
is given in the beginning. The four sentences in each passage have been jumbled up. These are
labeled P, Q, R and S. You are required to find out the proper sequence of the four sentences and
mark accordingly on the Answer Sheet.

20. L1: It is significant that one of the most common objections to competition is that it is blind
P: This is important because in a system of free enterprise based on private property, chances
are not equal and there is indeed a strong case for reducing that inequality of opportunity.
Q: Rather it is a choice between a system where it is the will of a few persons that decides
which is to get what and one where it depends at least partly, on the ability and the enterprise
of the people concerned.
R: Although competition and justice may have little else in common, it is as much a
commendation of competition as of justice that it is no respector of persons.
S: The choice today is not between a system in which everybody will get what he deserves
according to some universal standard and one where individual shares are determined by
chance or goodwill.
L6: The fact that opportunities open to the poor in a competitive society are much more restricted
than those open to the rich, does not make it less true that in such a society the poor are
more free than a person commanding much greater material comfort in a different type of
society.

The proper sequence should be


(a) RSQP (b) SRQP (c) PQRS (d) QPSR

21. L1: The chain saw howled as I finished cutting through the branch.
P: The branch crashed to the ground, taking my spectacles with it.
Q: I almost dropped the saw as I shielded my face from the twigs that brushed by.
R: Howard retrieved my glasses and handed them up to me.
5: 1 pulled the saw away, and my husband tugged against the other end of the rope that I had tied
just above the cut.
L6: Are you okay? He asked.

The proper sequence should be


(a) QRPS (b)SQPR (c) SRPQ (d) QSPR

Previous Years
CLAT & AILET Papers Page 243
22. L1: All human beings are aware of the existence of a power greater than that of the mortals — the
name given to such a power by individuals is an outcome of birth, education and choice.
P: Logically, therefore such a power should be remembered in good times also.
Q: Their other philanthropic contributions include the construction and maintenance of religious
places such as temples or gurdwaras.
R: Industrial organizations also contribute to the veneration of this power by participating in activities
such as religious ceremonies and festivities organized by the employees.
5: This power provides an anchor in times of adversity, difficulty and trouble..
L6: The top management/ managers should participate in all such events, irrespective of their personal
choices.

The proper sequence should be


(a) RPSQ (b) QRPS (c) SPRQ (d) SQRP

23. L1: A thorough knowledge of path or course to be followed is essential for achieving success.
P: Seniors must show the path clearly by laying down the precise expectations of the management
in terms of job description, key result areas, and personal targets.
Q: They should also ‘light the path’ by personal examples.
R: Advice tendered or help offered must be objectively evaluated for its effectiveness in achieving the
desired goals.
S: A display of arrogance and a false sense of ‘self-worth’, in order to belittle those who come to
help, prove dysfunctional.
L6: The individuality of each employee must be respected.

The proper sequence should be


(a) RSPQ (b) RPSQ (c) QPSR (d) PQRS

Directions (Q. 24 - 26) : In the following questions, the word at the top is used in four different ways.
Choose the option in which the usage of the word is INCORRECT or INAPPROPRIATE.

24. Passing
(a) She did not have passing marks in mathematics.
(b) The mad woman was cursing everybody passing her on the road.
(c) At the birthday party, all the children enjoyed a game of passing the parcel.
(d) A passing taxi was stopped to rush the accident victim to the hospital.

25. Bolt
(a) The shopkeeper showed us a bolt of fine silk.
(b) As he could not move, he made a bolt for the gate.
(c) Could you please bolt the door?
(d) The thief was arrested before he could bolt from the scene of the crime.

26. Fallout
(a) Nagasaki suffered from the fallout of nuclear radiation.
(b) People believed that the political fallout of the scandal would be insignificant.
(c) Who can predict the environmental fallout of the WTO agreements?
(d) The Headmaster could not understand the fallout of several of his good students at the public
examination

Previous Years
Page 244 CLAT & AILET Papers
Directions (Q. 27 - 35) : Fill in the blanks with the best alternative among the four options:

Around the world, forests are being destroyed at a rate of about thirteen million hectares a year and
deforestation accounts for an estimated 17% - 20% of all global emissions. In addition, forests and
other terrestrial carbon sinks play a ....(27).... role in preventing runaway climate change, soaking up a
full 2.6 Gt of atmospheric carbon every year. The destruction of forests, therefore, not only emits
carbon — a staggering 1.6 Gt a year, which severely …(28).... forests’ capacity to absorb emissions
from other sources - but also drastically....(29).... the amount of forested land available to act as a
carbon sink in the future.
However, the effects of deforestation extend beyond carbon. Rainforests ....(30).... a wide variety of
ecosystems services, from regulating rainfall to purifying groundwater and keeping fertile soil from ....(31)....;
deforestation in one area can seriously damage food production and ....(32).... to clean water in an
entire region. The value of global ecosystem services has been estimated at 33 trillion USD each
year (almost half of global GDP), but these services have been taken for granted without a mechanism
to make the market reflect their value. Rainforests are also a home and ....(33).... of income for a
huge number of people in Africa, Asia, and South America …(34).... this, economic pressures frequently
drive both local communities and national governments in the developing world to ....(35).... these
forests in ways that are unsustainable, clear-cutting vast areas for fuel, timber, mining, or agricultural
land.
27. (a) tough (b) important (c) vital (d) biggest

28. (a) affect (b) diminish (c) increases (d) impairs

29. (a) plagues (b) develops (c) reduces (d) shortens

30. (a) sell (b) offer (c) give (d) provide

31. (a) transforming (b) decoding (c) erupting (d) eroding

32. (a) handiness (b) excess (c) availability (d) access

33. (a) beginning (b) source (c) ways (d) reference

34. (a) despite (b) also (c) in spite (d) apart

35. (a) exploit (b) encompass (c) nurture (d) work

SECTION - B : GENERAL KNOWLEDGE


36. The Aircraft Carrier inducted in to the Indian Navy recently is
(a) INS Virat (b) INS Vikrant (c) INS Vikramaditya (d) None

37. Which of the following countries impeached its Chief Justice recently ?
(a) Bangladesh (b) Pakistan (c) Sri Lanka (d) Maldives

38. With which of the following country India signed an Extradition treaty that came into force during
October 2013 ?
(a) Afghanistan (b) Kenya (c) China (d) Bangladesh
Previous Years
CLAT & AILET Papers Page 245
39. The present Chief Election Commissioner of India is
(a) Dr. Nasim Zaidi (b) V. S. Sampath (c) H. S. Brahma (d) None

40. According to the recent World population data of 2013, the second most populous city in the world
is
(a) New Delhi (b) Tokyo (c) Mexico (d) Beijing

41. The latest country to launch a satellite during January 2013 from its soil and join the club of space-
faring nations is
(a) South Korea (b) North Korea (c) Ukraine (d) Iran

42. Who won the Women’s Singles at the Malaysian Gran Prix Badminton Championship in 2013 ?
(a) Saina Nehwal (b) Juan Gu (c) P. V. Sindhu (d) Ratchanok Intanon

43. Who is the oldest woman tennis player to be ranked No. 1 in the world ?
(a) Venus Williams (b) Chris Evert (c) Martina Navratilova (d) Serena Williams

44. How many High Courts are there in India as on January 2014?
(a) 18 (b) 21 (c) 28 (d) 24

45. Indira Gandhi Prize for Peace, Disarmament and Development for the year 2013 has been conferred
on:
(a) Ms. Angela Merkel (b) Mr. Nelson Mandela (c) Mr. Uhuru Kenyatta (d) Mr. Hamid Karzai

46. Who has won the Golden Shoe Award in Soccer three times ?
(a) Diego Maradona (b) Christiano Ronaldo (c) Lionel Messi (d) Pele

47. The phrase ‘Bitcoins’ refers to


(a) Cryptocurrency introduced by the United States
(b) Cryptocurrency introduced by France
(c) Cryptocurrency introduced by the World Bank
(d) None

48. Who was conferred the Col. C. K. Nayudu Life Achievement Award in Cricket during 2012 ?
(a) Sunil Gavaskar (b) Sachin Tendulkar (c) Dhoni (d) Kapildev

49. The Parliament of India voted to remove


(a) Justice V. Ramaswami (b) Justice P. Dinakaran
(c) Justice Soumitra Sen (d) None

50. The maximum penalty that can be imposed by CIC / SIC under the Right to Information Act, 2005 is
(a) Rs. 5,000 (b) Rs. 25,000 (c) Rs.250 (d) 10,000

51. On January 1, 2013, internet has completed


(a) 26 years (b) 30 years (c) 32 years (d) 28 years

52. Which of the following was declared by the United Nations General Assembly as “International Year
for Water Co-operation” ?
(a) 2011 (b) 2012 (c) 2013 (d) 2014
Previous Years
Page 246 CLAT & AILET Papers
53. How many states are there in the European Union ?
(a) 28 (b) 24 (c) 36 (d) 22

54. How many members can be nominated to the Rajya Sabha by the President ?
(a) 10 (b) 20 (c) 12 (d) 16

55. The highest award for sporting excellence in India is


(a) Arjuna Award (b) Dronacharya Award
(c) Bharat Ratna (d) Khel Ratna

56. Cloves, used as a spice, are derived from which of the following plant parts ?
(a) Seeds (b) Fruits (c) Flower buds (d) Young leaves

57. Which one of the following is included in the World List of Biosphere Reserves by UNESCO?
(a) Kinnaur Region (b) Spiti Valley (c) Nallamalai Hills (d) Sunderbans

58. Who among the following was the first Law Minister of India ?
(a) Jawaharlal Nehru (b) Maulana Abul Kalam Azad
(b) Dr. B.R. Ambedkar (d) T. Krishnamachari

59. Which one of the following countries is not a UN member country?


(a) Greece (b) Taiwan (c) Portugal (d) Australia

60. The main component of India’s import items from China is


(a) Rice (b) Iron ore
(c) Pharmaceuticals (d) Telecom equipments and Machinery

61. Which of the following divides India into Northern and Southern parts ?
(a) Equator (b) Tropic of Cancer
(c) Tropic of Capricorn (d) Arctic Circle

62. With which one of the following countries, India shares maximum length of the border ?
(a) Bangladesh (b) Pakistan (c) China (d) Nepal

63. What is known as Jasmine Revolution ?


(a) Czech uprising (b) Sudan uprising (c) Arab Spring (d) Libyan uprising

64. Which one of the following statements about NATO is not correct ?
(a) NATO has 28 independent member states
(b) The US is an ex-officio member of NATO
(c) NATO is a collective defence organisation in Europe
(d) Turkey is a member of NATO

65. V.R. Krishna lyer was _________ before he became a Judge.


(a) A minister (b) An ambassador (c) A Governor (d) A bureaucrat

66. What does airbag, used for safety of car driver, contain ?
(a) Sodium bicarbonate (b) Sodium azide (c) Sodium nitrite (d) Sodium peroxide

Previous Years
CLAT & AILET Papers Page 247
67. Which colour of heat radiation represents the highest temperature ?
(a) Blood red (b) Dark cherry (c) Salmon (b) Dark cherry
(d) White

68. Who is the founder of facebook?


(a) Jimmy Wales (b) Larry Page (c) Mark Zuckerberg (d) Brian Acton

69. Shashi Kant Sharma was appointed in 2013 as the


(a) Attorney General of India
(b) Comptroller and Auditor General of India
(c) Commissioner of Linguistic Minorities
(d) Chairman of the National Commission for SCs & STs

70. Mist is a result of which one of the following ?


(a) Condensation (b) Evaporation (c) Sublimation (d) Saturation

SECTION - C : LEGAL APTITUDE

Directions (Q. 71 - 97) : Given below is a statement of legal principle followed by a factual situation. Apply
the principle to the facts given below and select the most appropriate answer.

71. LEGAL PRINCIPLE : The occupier of a premise owes a duty of care to all his invitees and visitor.

FACTUAL SITUATION : Radhika’s brother, Akash, had come to visit at her place. After seeing her
wealth, Akash decided to commit theft that night. While he was trying to escape that night he got
electrocuted by the wires which were fixed on the boundary walls. Akash plans to sue Radhika. Will
his claim succeed?

DECISION :
(a) Yes, because in Indian tradition, guests are like Gods.
(b) No, because one has to be himself cautious about his safety.
(c) Yes, because it is the occupiers’ duty to take care of its visitors.
(d) No, because he himself is guilty of theft. He is no longer an invitee or visitor.

72. LEGAL PRINCIPLES :


1. An unlawful intrusion which interferes with one’s person or property constitutes trespass.
2. An easement is the right to use another person’s land for a stated purpose and has been in use
for quite some time. It can involve a general or specific portion of the property.

FACTUAL SITUATION : Vijay wanted to construct a shed on his window to stop the water from
leaking to his house. The shed was constructed but it protruded in Namit’s house. Vijay claims it is
his easementary right. Will Vijay’s claim succeed?

DECISION :
(a) Yes, because he has a reason to do the same.
(b) Yes, because right of easement exists.
(c) No, because there exists no right of easement.
(d) No, because this is encroachment on Namit’s property.

Previous Years
Page 248 CLAT & AILET Papers
73. LEGAL PRINCIPLE : A person is responsible for that which he could have reasonably foreseen or
prevented.

FACTUAL SITUATION : A chemist sold a hair conditioner to Jyoti. The conditioner was locally
manufactured and the contents, harmful chemicals, were listed on the bottle. The chemist, however,
represented to Jyoti that the chemicals used were harmless and beneficial for the hair. On using it,
Jyoti’s hair was badly damaged and she had to get hair treatment done for the same. Jyoti filed a
complaint against the chemist. Will the chemist be liable?

DECISION :
(a) Yes, as he should have informed Jyoti that the ingredients are not known.
(b) Liable because he was aware of the side effects of the ingredients.
(c) Not liable because it was the buyer’s duty to be aware about the product she is buying.
(d) Not liable as it is a natural tendency of shopkeepers to extol the virtues of the product they are
selling.

74. LEGAL PRINCIPLE : Defamation means publication of a statement injuring the reputation of a
person without !awful justification. Such statement must reflect on a person’s reputation and
tends to lower him in the estimation of right thinking members of the society generally or tends
to make them shun or avoid him.

FACTUAL SITUATION : Madam Tussauds Ltd. decided to keep a waxwork exhibition, and
placed an effigy of Babloo Prasad with a gun, in a room adjoining the “Chamber of Horrors”. Mr.
Babloo Prasad had been tried for murder in India and released on a verdict of “not proven guilty”
and a representation of the scene of the alleged murder was displayed in the Chamber of
Horrors. Is it amount to defamation?

DECISION :
(a) No Defamation as Babloo was an accused.
(b) Defamation as his guilt was not proved and he was released by the court.
(c) No defamation as there was not proper publication.
(d) None of the above.

75. LEGAL PRINCIPLE : Every partner is liable, jointly with all the other partners and also severally,
for all acts of the firm done while he is a partner.

FACTUAL SITUATION : A and B started a partnership firm for providing vehicle repairing services.
C approached the firm for getting his car repaired and noticed that only B was present in the
office. C informed the problem, and B started repairing the car. While B was repairing, he filled
petrol instead of oil in the engine. As a consequence, a small blast occurred and damaged the
car. Now, C sued both A and B for the damage so caused. Decide.

DECISION :
(a) Both liable since B was authorised to carry out the work of the firm.
(b) A is not liable since A has not authorised B to do something which was not for the benefit
of the firm.
(c) A is not liable since it was B’s fault and for that only B can be held liable.
(d) A is liable since it was negligence on his part that he was not resent in the office when C
came.

Previous Years
CLAT & AILET Papers Page 249
76. LEGAL PRINCIPLES :
1. No fault liability means liability of a person even without any negligent act on his part and
even if he has taken due care and caution.
2. If a person brings and keeps any dangerous thing on his land, then he is liable for any
damage caused if the thing escapes.
3. No one can be penalized for an Act of God which is unforeseeable and unpredictable.

FACTUAL SITUATION : B owned and managed a company supplying electricity to the nearby
locality. On a particular windy and stormy day, one of the wires snapped and was hanging
down. A, a cyclist who was driving in the night, saw the wire from a distance. There was a
nearby street light with low visibility. He came in contact with the wire and was electrocuted
immediately. His heirs sued A on ground of strict liability. Decide.

DECISION :
(a) A is not liable because B must have stayed indoor on a windy day.
(b) A is not liable because B’s negligence caused him injury.
(c) A is liable because supplying electricity is an inherently dangerous use of land and he
should have been careful.
(d) A is not liable because sudden storm and winds without A’s negligence was an Act of God.

77. LEGAL PRINCIPLE : The master/principal is liable for all acts done by his duly appointed
servant/agent for all acts done by him lawfully in the course of his employment.

FACTUAL SITUATION : A had an agency which used to lend carpenters to people on need
basis. A deputed B to do some repair work in C’s shed. While so doing, B lit up a cigarette and
threw it as soon as he saw someone coming there. The cigarette remaining lit caused a fire and
the shed was reduced to ashes. C sued A and B. Decide.

DECISION :
(a) A is liable as B was his servant.
(b) A is liable as he should have chosen responsible people.
(c) A is not liable as B’s act was not an authorised act.
(d) A is not liable but B is liable.

78. LEGAL PRINCIPLE : Where the parents of a minor child due to their negligence allow the
child an opportunity to commit a tort, the parents are liable.

FACTUAL SITUATION : The father supplied an airgun to his son who was about to turn 18 next
month. After some complaints of mischief, the father took the gun away and placed it in a
corner of their storeroom which was used by the family to store surplus and other unnecessary
stuff. The son took it out of the store and shot A. A sued his father. Is the father liable?

DECISION :
(a) No he took all necessary steps to prevent the son from using the gun.
(b) Yes, he was the one who gave the gun and allowed him to use it by giving an opportunity.
(c) No, the son was almost 17 years and 11 months of age; hence, he could think about his
well being and interest.
(d) Yes, the father was negligent in disposing off the gun.

Previous Years
Page 250 CLAT & AILET Papers
79. LEGAL PRINCIPLES :
1. Medical professionals are not immune from liability in tort on ground of negligence.
2. Services rendered to a patient by a doctor (except when given free of charge) by way of
consultation, diagnosis and treatment fall in the definition of “service” under the Consumer
Protection Act. In case of negligence, the doctors are liable in tort as well as under the
Consumer Protection Act.

FACTUAL SITUATION : A was the only child of his parents. Once he had high fever and his
parents called a doctor at home. This doctor used to work at a respectable hospital in Delhi. The
doctor administered certain medicines and asked the nurse to stay with him for the night and
administer to him a chloroquine injection. This injection was generally not suitable for young
children. The nurse, without prior test, followed instructions of the doctor and gave the injection.
As a result of an allergic reaction, the child died. The parents sued the nurse and the doctor.

DECISION :
(a) Doctor was rendering a “service”; hence liable to pay compensation.
(b) Doctor was not liable as he came to their home to give personal treatment and was not in
the Hospital.
(c) This is not a service; hence not liable.
(d) Only the nurse is liable.

80. LEGAL PRINCIPLES :


1. An act done by the consent of a person above 18 years is not an offence; provide the
offender did not intend to cause death or grievous hurt.
2. Mere pecuniary benefit is not a ‘thing done for a person’s benefit’.

FACTUAL SITUATION : A, poor man, is in dire need of money to pay off his money lenders. A
approaches Z, a doctor, to operate on him to remove one of his kidneys so that he can donate
it to needy people and earn money. The doctor explains to him the risks and thereafter proceeds
to remove his kidney. In the process, some complications develop and A develops an abdominal
tumour. Is Z guilty?

DECISION :
(a) Yes, donating kidney for money is illegal and amounts to trafficking of organs.
(b) Yes, removing kidney for money is not an act protected by this exception.
(c) No, Z performed the operation with A’s consent and fully explained him the risks involved.
(d) No, Z’s act was done for A’s benefit so that he can pay off the money lenders.

81. LEGAL PRINCIPLES :


1. An act done, even if without the consent of a person is not an offence, provided the offender
did not intend to cause death, and the act was done for the person’s benefit, in good faith.
2. Mere pecuniary benefit is not a ‘thing done for a person’s benefit’.

FACTUAL SITUATION : A is in a house which is on fire, with Z, a child. People below hold out
a blanket. A drops the child from the house top, knowing it to be likely that the fall may kill the
child but intending to save him from the fire. Unfortunately, the child is killed. Is A guilty?

Previous Years
CLAT & AILET Papers Page 251
DECISION :
(a) Yes, A had knowledge of his dangerous act. His act was not justified.
(b) Yes, A should have tried a less dangerous alternative.
(c) No, his act was done in good faith to save the child.
(d) No, he had the best of intentions and this was the only alternative.

82. LEGAL PRINCIPLE : Nothing is an offence which is done by any person who is, or who by
reason of mistake of fact, in good faith, believes himself to be bound by law to do it.

FACTUAL SITUATION : A, soldier, fires on a mob by the order of his superior officer, in
conformity with the commands of the law. B is killed due to such firing. Is A guilty of murder?

DECISION :
(a) Yes, he should have taken care to avoid any innocent person from being killed.
(b) No, he is bound by law to do it.
(c) Yes, as he has killed a person.
(d) The superior officer is guilty.

83. LEGAL PRINCIPLE : A person shall not be guilty of contempt of court on the ground that he
has published any matter which interferes with or obstructs or tends to obstruct the course of
justice in connection with any civil or criminal proceeding pending at the time of the publication,
if at that time he had no reasonable grounds for believing that the proceeding was pending.

FACTUAL SITUATION : X delivered a lecture at the local Rotary Club in favour of an accused
who is prosecuted for assaulting a police officer. He said that the accused is a victim of prevailing
corruption in the judiciary and he knows that the accused is going to be punished by the court
for being honest. Is X guilty of contempt of court?

DECISION :
(a) No, not guilty of contempt of court.
(b) Yes, guilty of contempt of court.
(c) Yes, X has dishonoured judiciary.
(d) No, X is an honest man.

84. LEGAL PRINCIPLE : Whoever intending to take dishonestly any movable property out of the
possession of any person without that person’s consent, moves that property in order to such
taking, is said to commit theft.

FACTUAL SITUATION : A, in good faith, believing property belonging to B to be A’s own


property, sells the property to C. Has A committed theft?

DECISION :
(a) No, because he did not have a dishonest intention.
(b) No, he did not move the property.
(c) No, he did not move the property that was in B’s possession.
(d) Yes, the constituent elements of theft are satisfied in this case.

Previous Years
Page 252 CLAT & AILET Papers
85. LEGAL PRINCIPLES :
1. Preparation to commit an offence is not an offence.
2. After one has finished preparation to commit an offence, any act done towards committing
the offence with the intention to commit it, is an attempt to commit the offence which is by
itself an offence.

FACTUAL SITUATION : Manish wanted to kill Nandini and had therefore gone to the market to
buy explosives to plant in her house. Manish kept those explosives in his godown as he
planned to plant them early next morning. But as the explosives were stolen in the night he
could not plant them in Nandini’s house. However, Nandini came to know about Manish’s plan and
therefore wants to file a complaint against him. Will she succeed?

DECISION :
(a) Yes, because he has done something more than mere preparation.
(b) No, because Nandini did not die.
(c) Yes, because there existed a mala fide intention.
(d) No, because mere preparation is no offence.

86. LEGAL PRINCIPLE : Whoever intentionally uses force to any person, without that person’s consent,
in order to committing of any offence, or intending by the use of such force to cause, or knowing it
likely to cause injury, fear, or annoyance to the person to whom the force is used, is said to use
criminal force to that other.

FACTUAL SITUATION : Akash was burning some crackers in his house when his dog got scared
and got unruly which scared his brother Mohsin. Can this be called criminal force?

DECISION :
(a) Yes, because he did it intentionally.
(b) No, because law doesn’t account for trivial things.
(c) Yes, because it led to annoyance of Mohsin.
(d) No, because there was no mala fide intention.

87. LEGAL PRINCIPLE : No person shall be convicted of any offence except for violation of a law in
force at the time of commission of the act charged as an offence, nor subjected to a penalty greater
than which might have been inflicted under the law in force at the time of commission of the offence.

FACTUAL SITUATION : A boy of 16 years was convicted of house trespass and theft. He was
sentenced to imprisonment for 6 months and fine was also imposed. After the judgement, the
Probation of Offenders Act came into force. It provided that a person below 21 years may not
ordinarily be sentenced to imprisonment. Now the boy claims the benefit of this Act. Should he get
it?

DECISION :
(a) No
(b) The rule of beneficial interpretation required that the benefit of ex post facto law can be applied
to reduce his sentence.
(c) A boy below 18 years is a minor and so should not be punished.
(d) None of the above.

Previous Years
CLAT & AILET Papers Page 253
88. LEGAL PRINCIPLE : No one can be punished for the same offence twice.

FACTUAL SITUATION : Aditya is accused of harassing his colleague Nimisha. She complains to
her superior and a departmental inquiry is initiated against Aditya. Aditya tries to intimidate Nimisha
and continues harassing her following which she complains to the police. The departmental inquiry
finds him guilty and terminates his services. Later, he is found guilty by a court and is jailed for 2
years. He claims that he has been punished twice.

DECISION :
(a) He has been punished twice for the same offence.
(b) He has not been punished twice for the same offence.
(c) He has been punished for different offences.
(d) None of the above

89. LEGAL PRINCIPLE : To be held guilty of an offence, one should have done the act that
causes the intended result.

FACTUAL SITUATION : A, with the intention to murder B stabs him repeatedly with a knife. B
is taken to the hospital and is found to be out of danger. Thereafter, due to the doctor’s
negligence, B’s wounds are infected and he requires surgical interventions. During the time of
operation to remove his infected leg, B died on account of administration of general anaesthesia.

DECISION :
(a) A is guilty of murder. -
(b) A is not guilty of murder though he may be guilty of attempt to murder.
(c) A is not guilty of murder but he may be guilty of causing hurt.
(d) The doctor is liable as he was negligent.

90. LEGAL PRINCIPLE : Every agreement, by which any party is restricted absolutely from enforcing
his rights in respect of any contract, by the usual legal proceedings in the ordinary tribunals,
is void to that extent.

FACTUAL SITUATION : Feroz and Pinto entered into an agreement for rendering certain services.
As per the prevailing law, the agreement may be enforced either at Jaipur or Udaipur. The
agreement itself, however, specifies that upon breach, the parties can only approach courts at
Jaipur. Feroz breaches the contract and Pinto, being a resident of Udaipur, would like to sue
him in Udaipur. He challenged the validity of the clause.

DECISION :
(a) Pinto will succeed as Feroz had made him suffer and the law must take his convenience
into account and allow him to sue in Udaipur.
(b) Pinto will succeed as the contrac’t does not allow him to institute any legal proceedings in
any court or tribunal in Udaipur.
(c) Pinto will fail as the contract does not restrain him from instituting legal proceedings in
Jaipur.
(d) Pinto will fail as he was of sound mind while entering into contract and having accepted it,
he cannot now deny his obligation.

Previous Years
Page 254 CLAT & AILET Papers
91. LEGAL PRINCIPLES : A contract is an agreement enforceable by law.

FACTUAL SITUATION : Amita invited Bina to her house for dinner. Bina accepted the invitation
but later did not go. On Bina’s failure to attend, Amita filed a suit against Bina for the price of
non-consumed food. Will the law enforce this agreement?

DECISION :
(a) No, it is a social agreement.
(b) Yes, Amita can recover amount for non-consumed food.
(c) No, as Bina did not accept the invitation in writing.
(d) No, because the law does not take account of trivial things.

92. LEGAL PRINCIPLES :


1. Once a person accepts another’s offer, and signifies such acceptance to the former, a contract
comes into existence between them.
2. Uncertain agreements are void agreements.
3. Rejected offers can be accepted only if renewed.

FACTUAL SITUATION : Bakshi wanted to purchase a particular land. He sent a letter to his cousin,
Dutt, offering him Rs. 4 lakhs for it. Dutt replied that he would not sell it below 5 lakhs. Bakshi
communicated his willingness to pay this amount. Dutt did not sell the land to Bakshi. Bakshi sued
him for breach of contract.

DECISION :
(a) Dutt is liable because once he communicates an offer to Bakshi and Bakshi accepts it, a
contract comes into existence.
(b) Dutt is liable as he has misled Bakshi by his actions.
(c) Dutt is not liable because he has rejected the offer by giving a counter offer which is also not
specific.
(d) Dutt is not liable as there is no legally enforceable contract.

93. LEGAL PRINCIPLE : An agreement is void if the court regards it as opposed to the public policy.

FACTUAL SITUATION : Sunita, while her husband Shankar was alive, promised to marry Neel in
the event of Shankar’s death. Subsequently, Shankar died, but Sunita refused to marry Neel. Neel
sues Sunita for damages for breach of promise.

DECISION :
(a) Sunita is liable as she is to bound to marry Neel.
(b) She is liable to compensate Neel for breach of promise.
(c) Neel can marry someone else.
(d) She is not liable as the contract is opposed to public policy and so void.

Previous Years
CLAT & AILET Papers Page 255
94. LEGAL PRINCIPLE : An agreement is void if its object is unlawful.
FACTUAL SITUATION : Sunil had a rich uncle who owned prime property in Chennai and had lot of
money in the bank. Being the only heir, Sunil was sure that he would inherit the property. One day,
the uncle called him to his room and announced that he planned to marry again. This angered Sunil
and he plans to murder his uncle so he hired Anuj, a murderer and entered into a contract with him
to kill his uncle. Sunil agreed to pay As. 10 lakhs to Anuj and even paid 5 lakhs as advance. The
following night Anuj entered the uncle’s house intending to kill him. On reaching there, he realised
that Sunil’s uncle was already dead so he left without doing anything. Next day, after post mortem
report, it transpired that Sunil’s uncle had died due to heart attack. Now, Sunil wants to recover the
advance from Anuj. Will he succeed?

DECISION :
(a) Yes
(b) No
(c) Anuj is liable to return the amount as the act was done by him.
(d) None of the above.

95. LEGAL PRINCIPLES :


1. Consideration must be of value in the eyes of law.
2. Consideration is not real if it is illusory.

FACTUAL SITUATION : Arjun received summons to appear at a trial as a witness on behalf of


Bitu, the accused. Bitu promised to pay him a sum of Rs. 1,000 for his trouble. On default by
Bitu, Arjun filed a suit to recover the said sum. Will he succeed?

DECISION :
(a) No, as the consideration is of no value in the eyes of law.
(b) Yes, as the contract is supported by consideration.
(c) No, as there is no consideration for the promise.
(d) Yes, as he appeared before court only after Bitu agreed to pay him the amount of Rs.
1,000.

96. LEGAL PRINCIPLE : A minor is not competent to contract.

FACTUAL SITUATION : Deep, a 9th standard student realizes that he being a minor, he is not
permitted by law to execute a contract, appoints Mandeep as his agent to conclude purchase
of a land to gift it to his mother on her birthday. Mandeep accordingly prepares the papers for
the transaction but at the last minute the seller who had agreed to sell it now refuses to sell it
contending that he does not wish to sell the land to a minor. Deep seeks to enforce the
contract against the seller.

DECISION :
(a) Deep can enforce the contract - since Mandeep is his agent, Deep is deemed to have
personally entered into a contract.
(b) Deep cannot enforce the contract - only Mandeep can, since seller has entered into the
contract with Mandeep.
(c) Deep cannot enforce the contract since he is a minor.
(d) Deep can neither appoint an agent nor enforce the contract since he is a minor.

Previous Years
Page 256 CLAT & AILET Papers
97. LEGAL PRINCIPLES :
1. Acceptance must be given only by the person to whom the offer is made.
2. Communication of acceptance to a person who did not make the offer does not bind the
offeror.

FACTUAL SITUATION : Pal sold his business to Sam without disclosing it to his customers.
Mani, an old customer sent an order for goods to Pal by name. Sam, the new owner, executed
the order. Mani refuses to accept the goods from Sam as he intended to deal only with Pal. In
a suit by Sam against Mani:

DECISION :
(a) Sam cannot recover as Mani never intended to deal with him.
(b) Can recover the price as he had supplied goods only against the order made by Mani.
(c) Sam cannot recover as it was only an invitation to offer by Mani on which no acceptance
can be given by Sam.
(d) Sam can recover as the price of the goods as an offer once accepted results in a contract.

98. The main objectives of the Judicial Standards Accountability Bill 2010 is
1. Is a legislation which aims to increase accountability of the higher judiciary in India
2. It seeks to devise a new “complaint procedure” under which any person may be able to file a
complaint in writing against any judge of a superior court
3. The issue of Judicial Standards must be seen in the context of Article 124(4) of the Constitution
4. The Bill seeks to provide a straight jacketed definition of misbehaviour in Clause 2(k)

Select the correct code:


(a) 1 and 2 (b) 1, 2 and 3 (c) only 4 (d) 3 and 4

99. On April 2013, the Supreme Court held that the modification of a well known cancer fighting drug is
not a patentable new invention against which pharmaceutical firm?
(a) GlaxoSmithkline (b) Novartis (c) Ranbaxy (d) Cipla

100. Criminal Law (Amendment) Act, 2013 does not provide that:
(a) A rape convict can be sentenced to a term not less than 20 years
(b) Death sentence to repeat offenders
(c) Stalking and voyeurism are bailable offences
(d) Acid attack convicts can get a 10 year jail term

101. In India, Uniform Civil Code is applicable in the State of:


(a) Goa (b) Nagaland (c) Jammu & Kashmir (d) Maharashtra

102. NOTA is introduced in the voting machine as one among the options based on
(a) Representation of Peoples Act, 1950
(b) Representation of Peoples Act, 1951
(c) Decision of the Supreme Court of India
(d) Direction of the Election Commission

103. How many duties are provided under Part - IV A of the Constitution?
(a) 10 (b) 11 (c) 12 (d) 08

Previous Years
CLAT & AILET Papers Page 257
104. The Inter State Council has been constituted based on the recommendation of
(a) Second Administrative Reforms Commission
(b) National Commission to Review the Working of the Constitution
(c) Sarkaria Commission
(d) Punchhi Commission
105. Which of the following is not a constitutional body ?
(a) Finance Commission (b) State Public Service Commissions
(c) Election Commission (d) Planning Commission

SECTION - D : REASONING

106. Rahul : One would have to be blind to the reality of moral obligation to deny that people who believe
a course of action to be morally obligatory for them have both the right and the duty to pursue that
action, and that no one else has any right to stop them from doing so.

Richa : But imagine an artist who feels morally obliged to do whatever she can to prevent works of art
from being destroyed confronting a morally committed anti-pornography demonstrator engaged in
destroying artworks he deems pornographic. According to your principle that artist has,
simultaneously, both the right and duty to stop the destruction and no right whatsoever to stop it.

Which of the following, if substituted for the scenario invoked by Richa, would preserve the force of her
argument?
(a) a medical researcher who feels a moral obligation not to claim sole credit for work that was
performed in part by someone else confronting another researcher who feels no such moral obligation.
(b) a manufacturer who feels a moral obligation to recall potentially dangerous products confronting
a consumer advocate who feels morally obliged to expose product defects.
(c) an architect who feels a moral obligation to design only energy-efficient buildings confronting, as
a potential client, a corporation that believes its primary moral obligation is to maximise shareholder
profits.
(d) a health inspector who feels morally obliged to enforce restrictions on the number of cats a
householder may keep confronting a householder who, feeling morally obliged to keep every
stray that comes along, has over twice that number of cats.

107. Between 1951 and 1963, it was illegal in the country of Geronia to manufacture, sell, or transport
any alcoholic beverages. Despite this prohibition, however, the death rate from diseases related to
excessive alcohol consumption was higher during the first five years of the period than it was during
the five years prior to 1951, Therefore, the attempt to prevent alcohol use merely made people want
and use alcohol more than they would have if it had not been forbidden.

Each of the following, if true, weakens the argument EXCEPT:


(a) Many who died of alcohol-related diseases between 1951 and 1963 consumed illegally imported
alcoholic beverages produced by the same methods as those used within Geronia.
(b) Death from an alcohol-related disease generally does not occur until five to ten years after the
onset of excessive alcohol consumption.
(c} The death rate resulting from alcohol-related diseases increased just as sharply during the ten
years before and ten years after the prohibition of alcohol as it did during the years of prohibition.
(d) Between 1951 and 1963, among the people with pre-existing alcohol-related diseases, the
percentage who obtained life-saving medical attention declined because of a social stigma
attached to excessive alcohol consumption.
Previous Years
Page 258 CLAT & AILET Papers
108. Unless they are used as strictly temporary measures, rent-control ordinances (municipal
regulations placing limits on rent increase) have several negative effects for renters. One of
these is that the controls will bring about a shortage of rental units. This disadvantage for
renters occurs over the long run, but the advantage - smaller rent increases - occurs immediately.
In many municipalities, especially in all those where tenants of rent-control units have a secure
hold on political power and can get rent-control ordinances enacted or repealed, it is invariably
the desire for short-term gain that guides those tenants in the exercise of that power.

If the statements above are true, which one of the following can be properly inferred from them?
(a) It is impossible for landlords to raise rents when rent controls are in effect.
(b) In many municipalities, rent-control ordinances are repealed as soon as shortages of rental
units arise.
(c) In many municipalities there is now, or eventually will be, a shortage of rental units.
(d) in the long term, a shortage of rental units will raise rents substantially.

109. A government’s proposed 8 percent cut in all subsidies to art groups will be difficult for those
groups to absorb. As can be seen, however, from their response to last year’s cut, it will not
put them out of existence. Last year there was also an 8 percent cut, and though private fund-
raising was very difficult for the art groups in the current recessionary economy, they did
survive.

The reasoning in the argument is flawed because the argument


(a) Relies without warrant on the probability that the economy will improve.
(b) Overlooks the possibility that the cumulative effect of the cuts will be more than the arts
group can withstand.
(c) Equates the mere survival of the arts groups with their flourishing.
(d) Does not raise the issue of whether there should be any government subsidies to arts
groups at all.

110. Health insurance insulates patients from the expense of medical care, giving doctors almost
complete discretion in deciding the course of most medical treatments. Moreover, with doctors
being paid for each procedure performed, they have an incentive to over-treat patients. It is
thus clear that medical procedures administered by doctors are frequently prescribed only
because these procedures lead to financial rewards.

The argument uses which one of the following questionable techniques?


(a) Assigning responsibility for a certain result to someone whose involvement in the events
leading to that result was purely coincidental.
(b) Inferring the performance of certain actions on no basis other than the existence of both
incentive and opportunity for performing those actions.
(c} Presenting as capricious and idiosyncratic decisions that are based on the rigorous
application of well-defined principles.
(d) Depicting choices as having been made arbitrarily by dismissing without argument reasons
that have been given for these choices.

Previous Years
CLAT & AILET Papers Page 259
Directions (Q. 111 - 112) : Each question contains six statements followed by four sets of combinations of
three. Choose the set in which the combinations are logically related.

111. (1) Some buildings are not skyscrapers.


(2) Some skyscrapers are not buildings.
(3) No structure is a skyscraper.
(4) All skyscrapers are structures.
(5) Some skyscrapers are buildings.
(6) Some structures are not buildings.

(a) (1),(3),(5) (b) (2),(4),(6) (c) (6),(4),(1) (d) (1),(3),(6)

112. (1) All bins are buckets. (2) No bucket is a basket.


(3) No bin is a basket. (4) Some baskets are buckets.
(5) Some bins are baskets. (6) No basket is a bin.

(a) (2),(4),(5) (b) (1),(3),(2) (c) (3),(4),(6) (d) (1),(2),(6)

Directions (Q. 113 -114) : From the alternatives, choose the one which correctly classifies the four sentences
as a

F: Fact : If it relates to known matter of direct observation, or an existing reality or something known to
be true.
J: Judgment : If it is an opinion or estimate or anticipation of common sense or intention.
I: Inference : If it is a logical conclusion or deduction about something based on the knowledge of facts.

113. (1) If democracy is to survive, the people must develop a sense of consumerism.
(2) Consumerism has helped improve the quality of goods in certain countries.
(3) The protected environment in our country is helping the local manufacturers.
(4) The quality of goods suffers if the manufacturers take undue advantage of this.

(a) IJFJ (b) JFJI (c) IJJF (d) 1FJJ

114. (1) Everyday social life is impossible without interpersonal relationships.


(2) The roots of many misunderstandings have been cited in poor relations among individuals.
(3) Assuming the above to be true, social life will be much better if people understand the importance
of good interpersonal relations.
(4) A study reveals that interpersonal relations and hence life in general can be improved with a little
effort on the part of individuals.

(a) FJIJ (b) JFIF (c) FIFJ (d) IFFJ

Previous Years
Page 260 CLAT & AILET Papers
Directions (Q. 115 -120) : Each group of questions is based on a set of conditions. Choose the response
that most accurately and completely answers each question.

There are five flagpoles lined up next to each other in a straight row in front of a school. Each flagpole
flies one flag (red, white, or blue) and one pennant (green, white, or blue). The following are conditions
that affect the placement of flags and pennants on the poles:
On a given flagpole, the pennant and the flag cannot be the same colour.
Two adjacent flagpolescannot fly the same colour flags.
Two adjacent flagpoles cannot fly the same colour pennants.
No more than two of any colour flag or pennant may fly at one time.

115. If the 2nd and 5th pennants are blue, the 2nd and 5th flags are red, and the 3rd flag is white, then
which one of the following must be true ?
(a) Two of the flags are white.
(b) Two of the pennants are white.
(c) The 4th pennant is green.
(d) If the 1st flag is white, then the 1st pennant is green.

116. If the 1st flag is red and the 2nd pennant is blue, then which one of the following is NOT necessarily
true ?
(a) The second flag is white.
(b) If the 5th flag is red, then the 3rd flag is blue.
(c) If the 4th pennant is green, then the 1st pennant is white.
(d) If the 1st and 5th flags are the same colour, then the 3rd flag is blue.

117. If the 1st and 3 rd flags are white and the 2nd and 4th pennants are blue, then which one of the
following is false ?
(a) The 4th flag is red.
(b) The 1st pennant is green.
(c) The 3rd pennant is not red.
(d) The 5th pennant is green.

118. If the 1st and 4th flags are blue, and the 3rd pennant is white, then which one of the following
must be true ?
(a) If the 1st pennant is green, then the 5th pennant is white.
(b) If the 5th pennant is white, then the 1st pennant is green.
(c) The 2nd flag is red.
(d) The 5th flag is red.

119. If the 2nd flag is red and the 3rd flag is white, and the 4th pennant is blue, then which one of the
following must be true ?
(a) If the 5th flag is white, then the two of the pennants are blue.
(b) If the 1st flag is white, then the 2nd flag is white.
(c) If the 1st pennant is blue, then the 5th pennant is green.
(d) if the 1st pennant is green, then the 5th flag is not blue.

Previous Years
CLAT & AILET Papers Page 261
120. If the 1st flag and the 2nd pennant are the same colour, the 2nd flag and the 3rd pennant are the same
colour, the 3rd flag and the 4th pennant are the same colour, and the 4th flag and the 5th pennant are
the same colour, then which one of the following must be true ?
(a) The 1st pennant is white. (b) The 2nd flag is not white.
th
(c) The 5 flag is red. (d) The 3rd pennant is blue.

121. Nicky, who is Ronald’s daughter, says to Irene, “Your mother Rita is the youngest sister of my
father, who is the third child of Sylvester”. How is Sylvester related to Irene ?
(a) Maternal uncle (b) Father (c) Grandfather (d) Father-in-law

122. Pointing to a lady, a person says to his friend, “She is the grandmother of the elder brother of my
father”. How is the girl in the photograph related to the man?
(a) Niece (b) Sister (c) Aunt (d) Sister-in-law

123. Pointing towards a boy, Aruna said to Pushpa, “The mother of his father is the wife of your grandfather
(mother’s father)”. How is Pushpa related to that boy ?
(a) Sister (b) Niece (c) Cousin (d) Wife

124. The front door of Kiran house is towards the south. From the backside of her house she walks 50
metres straight then turns towards the left and walks 100 metres and after that turns right and stops
after walking 100 metres. Now Kiran is facing which direction ?
(a) East (b) South (c) West (d) North

125. If Northwest becomes south and southwest becomes east and all the other directions change in the
similar manner, then what will be the direction for north?
(a) Southeast (b) Northeast (c) North (d) Southwest

126. A policeman goes 20 km east and then turning to the south he goes 30 km and then again turns to
his left and goes 10 km. How far is he from his starting point?
(a) 30 km (b) 20 km (c) 10 km (d) 40 km

Directions (Q. 127 - 130) : In the following number series, find out the missing number:

127. 31, 32, 36, 45, ?


(a) 55 (b) 56 (c) 62 (d) 61

128. 1, 3, 2, 5, 3, 7, 4, 9, 5, 11, 6, ?
(a) 10 (b) 11 (c) 13 (d) 9

129. 300, 296, 287, 271, ?, 210


(a) 246 (b) 250 (c) 244 (d) 261

130. 12, 15, 19, ?, 30, 37


(a) 25 (b) 21 (c) 23 (d) 24

Previous Years
Page 262 CLAT & AILET Papers
Directions (Q. 131-132) : Each question is followed by two statements, I and II. Answer each question
using the following instructions.
Choose (a) : If the question can be answered by using statement I alone but not by using II alone.
Choose (b) : If the question can be answered by using Statement II alone but not by using I alone.
Choose (c) : If the question can be answered by using either statement alone.
Choose (d) : If the question can be answered by using both the statements together but not by either
statement.

131. In a cricket match the ‘man of the match’ award is given to the player scoring the highest
number of runs. In case of tie, the player (out of those locked in the tie) who has taken the
higher number of catches is chosen. Even thereafter, if there is a tie, the player (out of those
locked in the tie) who has dropped fewer catches is selected. Aakash, Biplab and Chirag who
were contenders for the award dropped at least one catch each. Biplab dropped 2 catches
more than Aakash did, scored 50, and took 2 catches. Chirag got two chances to catch and
dropped both. Who was the ‘man of the match’ ?
I. Chirag made 15 runs less than both Aakash and Biplab.
ll. The catches dropped by Biplab are 1 more than the catches taken by Aakash.

132. Four friends, A,B,C and D got the top four ranks in a competitive examination, but A did not get
the first, B did not get the second, C did not get the third, and D did not get the fourth rank. Who
secured which rank ?
I. Neither A nor D were among the first 2.
II. Neither B nor C was third or fourth.

Directions (Q. 133 - 134) : These four (4) items consist of two statements, one labeled as the
‘Assertion (A)’ and the other as ‘Reason (R)’ you are to examine these two statements carefully and
select the answers to these items using the codes given below:

Codes :
(a) Both A and R are individually true and R is the correct explanation of A.
(b) Both A and R are individually true but R is not the correct explanation of A.
(c) A is true but R is false.
(d) A is false but R is true.

133. Assertion (A) :


The phenomenon of nuclear fission generates great energy.

Reason (R) :
The process in which a nucleus is broken into two parts is called nuclear fission.

134. Assertion (A) :


The price of a stock is determined on the basis of demand and supply of the stock.

Reason (R) :
The value of the Sensex increases whenever there is a heavy demand for the stocks which
form the Sensex.

Previous Years
CLAT & AILET Papers Page 263
Directions (Q. 135 — 136) : The question given below has a statement followed by two conclusions I and
II. Consider the statement and the following conclusions. Decide which of the conclusions follow from the
statement. Mark answer as:
(a) If conclusion I follows
(b) If conclusion II follows
(c) If both conclusions I and II follows
(d) If neither conclusion I follows nor 11 follows

135. Statement :
A degree in law is of the most wanted degrees by youth in India.

Conclusions :
I. A degree in law guarantees a good profession.
II. A degree in law is the first choice for youth in India.

136. Statement :
Any student who is caught red-handed using unfair means discredits his parents and teachers.

Conclusions :
I. Such students try to show that their teachers don’t teach properly in the class.
II. Stringent actions must be taken against such students.

Directions (Q. 137 -140) : In the following set of analogies, one word is missing. Find it out from the given
options.

137. Errata : ? : : Flaws : Jewels


(a) Manuscripts (b) Books (c) Literature (d) Prints

138. inoculation : ?: : Exposure : Toughening


(a) Immunity (b) Punctuality (c) Vulnerability (d) Contagious

139. Steel : Rails :: Alnico . ?


(a) Aircraft (b) Machinery (c) Silverware (d) Magnets

140. Conscience : Wrong : : Police : ?


(a) Thief (b) Law (c) Discipline (d) Crime

Previous Years
Page 264 CLAT & AILET Papers
SECTION - E : MATHEMATICS

141. Mohan credits 15% of his salary into his bank and spends 30% of the remaining amount on household
articles. If cash on hand is Rs. 2,3801-, what is his salary ?
(a) Rs. 5,000 (b) Rs. 4,500 (c) Rs. 4,000 (d) Rs. 3,500

142. The average marks of a student in ten papers are 80. If the highest and the lowest score are not
considered the average is 81. If his highest score is 92, what is the lowest score ?
(a) 55 (b) 60 (c) 62 (d) 61

143. Ten years ago, Sunil was half of Sudip’s age. If the age of both at present is in the ratio of 3:4, what
will be the total of their present age?
(a) 20 years (b) 30 years (c) 35 years (d) 45 years

144. A and B started a business with a total capital of Rs. 30,000. At the end of the year, they shared the
profit in the ratio of their investments. If their capitals were interchanged, then A would have received
175% more than what he actually received. Find out the capital of B.
(a) Rs.20,000 (b) Rs. 22,000
(c) Rs. 21,000 (d) Rs. 23,000

145. The ratio in which Aman and Bimal have contributed to the capital of a company is 3 : 4. Bimal has
invested his capital for only 3 months and has received half as much profit as Aman, at the end of
the year. Find out for how much time has Aman invested his capital in the company.
(a) 8 months (b) 14 months (c) 15 months (d) 1 year

146. Two whole numbers whose sum is 64 can be in the ratio of ?


(a) 7 : 2 (b) 7: 6 (c) 3 : 1 (d) 8 : 7

147. In a call centre, 6 employees working for 10 hours complete a certain task. They started working at
11:00 am. This continued till 5:00 pm and after that, for each hour one more employee is added till
the work gets completed. At what time will they complete the work ?
(a) 7:10 pm (b) 8:00 pm (c) 7:35 pm (d) 6:35 pm

148. 72% of students in a class took Physics and 44% took Mathematics. If each student took Physics
or Mathematics and 40 took both, the total number of students in the class would be
(a) 200 (b) 240 (c) 250 (d) 320

149. Amar is twice as fast as Rohit and Rohit is thrice as fast as Chanda is. The journey covered by
Chanda in 42 minutes will be covered by Amar in
(a) 14 min 25 sec (b) 7 min (c) 28 min 37 sec (d) 54 min 35 sec

150. P, Q and R are three consecutive odd numbers in ascending order. If the value of three times P is 3
less than two times R, find the value of R.
(a) 5 (b) 7 (c) 9 (d) 11

Previous Years
CLAT & AILET Papers Page 265
AILET Question Paper 2015

SECTION - A : ENGLISH to be private.’ Throughout the development of Anglo-


American law, the individual has never possessed
Directions (Q. 1 - 6) : The questions in this section absolute dominion over property. Land becomes
are based on the passage. The questions are to be clothed with a public interest when the owner devotes
answered on the basis of what is stated or implied his property to a use in which the public has an
in the passage. For some of the questions, more interest in support of this position the chairman of
than one of the choices could conceivabiy answer the board of the Wilde Lake Shopping Centre in
the question. However, you are to choose the best Columbia, Maryland said :
answer; that is, the response that most accurately
and completely answers the questions. The only real purpose and justification of any of these
centres is to serve the people in the area-not the
The Constitution ofthe United States protects both merchants, not the developers, not the architects.
property rights and freedom of speech. At times The success or tailure of a regional shopping centre
these rights conflict. Resolution then requires a will be measured by what it does for the people it
determination as to the type of property involved. lf seeks to serve.
the property is private and not open to the general
public, the owner may absolutely deny the exercise These doctrines should be applied when
of the right of free speech thereon. On the other accommodation must be made between a shopping
hand, if public land is at issue, the First Amendment centre owner’s private property rights and the public’s
protections of expression are applicable. However, right to free expression. lt is hoped that when the
the exercise of free speech thereon is not absolute. Court is asked to balance these conflicting rights it
Rather it is necessary to determine the will keep in mind what Justice Black said in 1945:
appropriateness of the forum. This requires that “When we balance the constitutional rights of owners
consideration be given to a number of factors of property against those of the people to enjoy (First
including: character and normal use of the property, Amendment) freedom(s) ..... we remain mindful of
the extent to which it is open to the public, and the the fact that the latter occupy a preferred position.”
number and types of persons who frequent it. lf the
forum is clearly public or clearly private, the 1. In which one of the following cases would the
resolution of the greater of rights is relatively straight owner of the property probably be most free
forward. to restrict the freedom of speech ?
(a) an amusement park attended by five
ln the area of quasi-public property, balancing these million people each year owned by a
rights has produced a dilemma. This is the situation multinational company.
when a private owner permits the general public to (b) a small grocery shopping mall owned by
use his property. When persons seek to use the a husband and wife
land for passing out handbills or picketing, how is a (c) an enclosed shopping mall owned by a
conflict between property rights and freedom of single woman
expression resolved ? (d) an eight-unit residential apartment
building owned by a large real estate
The precept that a private property owner surrenders company
his rights in proportion to the extent to which he
opens up his property to the public is not new. In
1675, Lord Chief Justice Hale wrote that when private
property is “affected with a public interest, it ceases

Previous Years
Page 266 CLAT & AILET Papers
2. A conflict between property rights and 6. All other things being equal, the courts must
freedom of speech might arise in all of the (a) favour First Amendment rights over
following situations, EXCEPT property rights
(a) protestors carrying signs outside a (b) favour property rights over First
cinema in an enclosed shopping mall Amendment rights
(b) a disgruntled employee passing out (c) treat property rights and First Amendment
leaflets in front of a hairdresser’s salon rights equally
(c) a religious order soliciting funds and (d) protect property rights of the owners
converts in the swimming pool area ot a
condominium Directions (Q. 7 - 11) : For each of the following
(d) a candidate for mayor handing out flyers words below, a context is provided. From the
in front of his opponent’s headquarters alternatives given, pick the word or phrase that is
closest in meaning in the given context.
3. According to the passage, an owner’s
freedom to deny freedom of speech on his 7. Alphanumeric : The inclusion of
property is determined by all of the following alphanumeric features in cellular phones has
EXCEPT made sending SMSs very convenient.
(a) whether or not the land is open to the (a) using only alphabets
public (b) using digital codes
(b) the nature of and the usual use of the (c) using OSCOLA
property (d) using both letters and numerals
(c) the type of persons who frequents the land
(d) the nature of character of the owner 8. Oligarchy : Mrigank argued that all political
parties, including those which profess
4. We can infer from the passage that the author democratic values become the instruments
believes that shopping malls in America
of their leaders who eventually become a self-
(a) should be in the service of the people who
interested and self-satisfied oligarchy.
frequent them
(a) rule of a mob or crowd
(b) have a right to prohibit distribution of
(b) small group of people having control of
advertising handbills
state
(c) have a right to control any distributed
(c) an iron-rule of democrats
materials
(d) a socialist rule of activists
(d) should permit any charitable solicitations
9. Perestroika : In the 1980’s, the “state
5. According to the passage, the idea that a
socialist’ tradiiion became totally discredited
property owner’s rights decline as the property
as Gorbachev’s programme of perestroika
is more used by the general public
revealed the fundamental failures of the
(a) is peculiar to recent Supreme Court
planned economies of the Communist bloc.
decisions
(a) closing off all entries
(b) is attested by a three-hundred-year-old
(b) manifesto of failure
opinion
(c) opening up
(c) conflicts with the idea that property
(d) nagging policies
affected with a public interest ceases to
be private
(d) is now universally accepted in Great
Britain and in Canada

Previous Years
CLAT & AILET Papers Page 267
10. Bequeath : In consequences, they 17. Ante-diluvium
bequeathed to their followers no clear vision (a) old time (b) up-to-date
of the economics of socialism, that is state (c) time period (d) against dualism
ownership and planning the means of
production, distribution and exchange, and 18. To play truant
others had to advocate market socialism, the (a) to make a narrow escape
state regulation of capital rather than state (b) to run away from work without permission
ownership of planning. (c) to be clever
(a) hand down or pass on (d) none of these
(b) a powerful will of the people
(c) a method of governing 19. To see red
(d) to gjve in promise (a) to find fault with
(b) to be very angry
11. Redundancy : Communication is a subtle, (c) to criticise others
complex and continuously fluctuating (d) to victimise someone
process, affected by a multitude of factors
both external and internal, and two such 20. To flog a dead horse
elements are noise and redundancy. (a) to do a thing in vain
(a) that part of message that is predictable (b) to act in a foolish way
or conventional (c) to criticize strongly
(b) that word or phrase which is surprising (d) try to revive interest in a subject that is
(c) that meaning of a word which is sparingly out of date
used
Direction (Q. 21 - 25) : In this section, each passage
(d) that message which has no clear meaning
consists of four/five sentences. The sentences in
each passage have been jumbled up. These are
Directions (Q. 12 - 15) : In the following questions, labelled P, Q, R, S and T. You are required to find
choose the word which is odd one out. out the proper sequence of the sentences and mark
accordingly on the Answer Sheet.
12. (a) bedlam (b) anarchy
(c) anatomise (d) insurrection
21. P : You couldn’t have asked for a nicer, more
respectable-looking fellow than Jack, the
13. (a) miscreant (b) defendant
day he arrived in town for grandpa’s
(c) accused (d) plaintiff
funeral.
Q : He’d just arrived that morning, got the
14. (a) assault (b) extortion
word that the old Judge was gone’ he said.
(c) sub-poena (d) battery
R : He rode into the yard on his motorbike -
15. (a) barrister (b) attorney the one he’d sold last summer, which
(c) juror (d) advocare wasn’t a patch on the big red one he’d
bought later.
Direction (Q. 16 - 20) : Choose the exact meaning S : He was glad he was in time for the
of the idioms/phrases. services. He sat with Paresh all through
the funeral; he even cried - or if it wasn’t
16. Alpha and Omega crying, it was a darned good show.
(a) related to science
(b) the beginning and the end The proper sequence should be
(c) A to Z (a) QRPS (b) QRSP
(d) none of these (c) SPQR (d) PRQS

Previous Years
Page 268 CLAT & AILET Papers
22. P : Anything to do with his parents had R : At the time, l took this to mean, simply,
always been vague to Manish. “Give your very best to every piece.”
Q : He had been too young to remember S : A deep-chested, powerful man with a
them; he only knew the story of the fatal rugged, gentle face, Enesco looked at me
weekend at the Airondack camel and the across the violin he held under his chin,
overturned canoe. and shook his bow.
R : Then, as the years swept by and the
tragedy faded into time, they began taking The proper sequence should be
him there; and the lake again became a (a) QPRS (b) QSPR
quite lake in the foothills’ a camping place (c) SPRQ (d) SPQR
where the three of them had a good time.
S : Grandpa had still owned the camp a long 25. P : On the one hand, I want very much for
while afterwards he and Grandma hadn’t someone else to clean our house, as
gone there. neither I nor my husband, Ed, has shown
any aptitude for it.
The proper sequence should be Q : No one but me, for instance, should have
(a) PQRS (b) QRSP to clean up the dental floss heaped up
(c) PQSR (d) QPSR like spaghetti near the wastebasket where
l toss it each night, never catching on that
23. P : The role of Western values in floss is not something that can be thrown
contemporary Indian society is a subject with a high degree ot accuracy.
on which I have pondered for years. R : On the other hand, l’d feel guilt in flicting
Q : Moreover, various stakeholders of our such distasteful drudgery on another
company - employees, investors, human being.
customers and vendors - come from S : Have always wanted and not wanted a
across the globe. cleaning person.
R : An organisation is representative of
society, and some of the lessons that I The proper sequence should be
have learnt from the West regarding values (a) SPRQ (b) RQPS
are, I think, applicable to us as a nation. (c) RPQS (d) PQSR
Here are some of them:
S : I come from a company that is built on Direction (Q. 26 - 35) : Fill in the blanks with the
strong values. most appropriate word:
T : In dealing with them over the years, I have
come to appreciate several aspects of the 26. When you are living with your ––––––– values
West’s value system. and principles, you can be straight forward,
honest and –––––––
The proper sequence should be (a) inherited, distinct
(a) PQSTR (b) PSQTR (b) core, upfront
(c) SQTRP (d) SQRPT (c) innate, durable
(d) cultural, perceptive
24. P : “To play great music,” he said, “you must
keep your eyes on a distant star.”
Q : Eleven years old, I was taking a violin
lesson with Georges Enesco, my
teacher, in his Paris studio.

Previous Years
CLAT & AILET Papers Page 269
27. The quality of ––––––– between individuals 33. But ––––––– are now regularly written not just
and the organisation for which they work can for tools but well-established practices,
be ––––––– to the benefit of both parties. organisations and institutions not all of which
(a) life, conceptualised seem to be ––––––– away
(b) interactions, improved (a) reports, withering
(c) service, evaluated (b) stories, trading
(d) work, better (c) books, dying
(d) obituaries, fading
28. Genetic engineering in humans should be
used to ––––––– diseases, not to ––––––– 34. In this context, the ______ of the British
genetic uniformity. Labour Movement is particularly ______
(a) treat, foster (b) eradicate, cater (a) affair, weird
(c) cure, generate (d) avoid, promote (b) activity, moving
(c) experience, significant
29. Stating that the ATM operations are ––––––– (d) atmosphere, gloomy
losses, the nation’s largest bank, the State
Bank plans to _____ the management of 35. Indian intellectuals may boast if they are so
some of its ATMs. inclined of being ______ to the most elitist
(a) generating, resource among the intellectual _______ of the world
(b) generating, close (a) subordinate, traditions
(c) incurring, outsource (b) heirs, cliques
(d) reporting, tighten (c) ancestors, societies
(d) heir, traditions
30. Most journalistic writing could do with –––––
–– of paragraphs to make the prose more SECTION - B :
______
(a) suppression, legible
GENERAL KNOWLEDGE
(b) removal, argumentative
(c) simplification, abstruse 36. Which State has become first to fix minimum
(d) deletion, succinct educational qualification for panchayat polls?
(a) Kerala (b) Gujarat
31. Angered by bureaucrat’s ––––––– comments, (c) Rajasthan (d) Karnataka
the reporter insisted for a more –––––––
response. 37. Which country was chosen to chair
(a) redundant, repetitive Partnership in Population and Development
(b) tactless, immediate (PPD) for the year 2015 ?
(c) circumlocutions, direct (a) India (b) China
(d) sarcastic, beneficial (c) SriLanka (d) Ghana
32. One of the most productive researches ––––
––– in contemporary neurosciences is 38. Recently, the Prime Minister has launched
devoted to ––––––– maps of human the “Give it Up” campaign for voluntarily giving
consciousness. up
(a) trajectories, reconnoitring (a) use of tobacco products
(b) designs, enunciating (b) use of plastic
(c) paradigms, elucidating (c) LPG subsidy
(d) declensions, obfuscating (d) black money

Previous Years
Page 270 CLAT & AILET Papers
39. Who bagged the best actress award at the 46. Which Country has launched the “Let Girls
62nd National Film Awards ? Learn” initiative aimed at educating 62 million
(a) Priyanka Chopra (b) Vidya Balan girls around the World?
(c) Kangana Ranaut (d) Rani Mukherji (a) United States (b) New Zealand
(c) India (d) Australia
40. Simona Halep, who has won the lndian Wells
47. Which among the following parties declared
Open Tennis Women's Singles Title 2015,
victory in the 2015 lsrael Legislative Election?
hails from which country ?
(a) Zionist Union (b) Likud
(a) Belgium (b) Romania
(c) Yesh Atid (d) Joint List
(c) Switzerland (d) Sweden
48. Which of the following measures the value of
41. The winner of prestigious Dadasaheb Phalke a currency against a basket of other
Award 2014 is currencies?
(a) Mohan Lal (b) Shashi Kapoor (a) Real Exchange Rate
(c) Mani Ratnam (d) Girish Karnad (b) Effective Exchange Rate
(c) Real Effect Exchange Rate
42. The Bandung Conference was an important (d) Nominal Exchange Rate
step towards the creation of
(a) Non-Alignment Movement 49. Which is the only regenerative organ in
(b) United Nations human body?
(c) European Union (a) Brain (b) Liver
(d) SAARC (c) Pancreas (d) Lungs

43. Recently, the Union Government has 50. What is the name of the innovative Council
appointed Amitabh Bachchan as the brand proposed in the Railway Budget -2015 to
ambassador for its campaign against which promote innovation ?
disease ? (a) Navachar (b) Rail Tech
(a) Swine Flu (b) Tuberculosis (c) Kayakalp (d) Sankalp
(c) Hepatitis B (d) Diabetes 51. Lysosomes, which are known as suicidal
bags, are produced by which organelle ?
44. The World's first hydrogen powered tramcar (a) Mitochondria (b) Golgi body
has rolled off in which country ? (c) Ribosome (d) Peroxisome
(a) China (b) Japan
(c) U. S. (d) Russia 52. India's rank in the recently released World
Press Freedom Index (WPFI) 2015 is
45. Which is the only Central University in lndla (a) 120 (b) 135
which has Prime Minister as its Chancellor? (c) 136 (d) 140
(a) Banaras Hindu University
(b) Indira Gandhi National Open University 53. Who among the following has been named
(c) Rajiv Gandhi University as new heir of Mysore Royal Family ?
(d) Visva Bharti University (a) Yaduveer Gopal Raj Urs
(b) Chaduranga Kantharaj as Urs
(c) Aditya Gurudev Urs
(d) Chandra Shekar Urs

Previous Years
CLAT & AILET Papers Page 271
54. Who among the following is often called the 61. Which among the following States has
"Greenest Chief Minister of India" ? w o n t h e 1 0 th N a t i o n a l Aw a r d fo r
(a) Pawan Kumar Chamling Excellence work in Mahatma Gandhi
(b) Anandiben Patel National Rural Employment Guarantee
(c) Prakash Singh Badal Act ?
(d) Nabam Tuki (a) Karnataka (b) West Bengal
(c) Haryana (d) Madhya Pradesh
55. Which part of the World is referred to as the
Fertile Crescent ? 62. Who is the newly appointed Chairman of the
(a) Latin Americas (b) South East Asia empowered Committee of State Finance
(c) Middle East (d) Scandinavia Ministers on Goods & Services Tax (GST) ?
(a) J. K. George (b) K. M. Mani
56. Which of the following banks launched (c) Amit Mitra (d) Saurabh Patel
'Pocket', India's first digital bank on mobile
phones ? 63. What is the code name of the military
(a) Axis Bank (b) ICICI Bank operation in Yemen against Shia Houthi
(c) HDFC Bank (d) SBl Group?
(a) Operation Destruction Storm
57. India based Rickey Kej has won the Grammy (b) Operation Decisive Storm
Award for his album (c) Operation Desert Storm
(a) Morning Phase (d) Operation Black Star
(b) Beyonce
(c) The Lonely Hours 64. Who was sworn in as the President of Sri
(d) The Winds of Samsar Lanka in January 2015 ?
(a) Mahinda Deshapriya
58. Recently, UNICEF and which football legend (b) Mahinda Rajapaksa
has launched protection fund named "7" to (c) Gotabhaya Rajapaksa
protect the World's most vulnerable (d) Mithripala Sirisena
youngsters ?
(a) David Beckham 65. Who is the new Chief Election
(b) Lionel Messi Commissioner?
(c) Christiano Ronaldo (a) H.S. Brahma (b) Vinod Zutshi
(d) Zinedine Zidane (c) Nasim Zaidi (d) R. Balakrishnan
59. According to the latest data, which among
66. A minor planet is named after which one of
the following States has attracted maximum
the following Indian Legends ?
Foreign Direct Investment (FDl) in the
(a) Viswanathan Anand
country? (b) Sachin Tendulkar
(a) Gujarat (b) Tamil Nadu (c) A. B. Rahman
(c) Maharashtra (d) Karnataka (d) Milkha Singh

60. Which one of the following countries will hold 67. Who won the title Miss India 2015 ?
the presidency of the BRICS New (a) Aafreen Rachel (b) Vartika singh
Development Bank for the first six years ? (c) Koyal Fana (d) Aditi Arya
(a) India (b) China
(c) South Africa (d) Russia

Previous Years
Page 272 CLAT & AILET Papers
68. _______, the ‘founding father’ and ‘architect’ at Rudra started a website named ‘ricky-
of modern Singapore passed away on 23 thakur-is-a-jerk.com’. She created this
March 2015. website so as to warn other girls about ‘Ricky
(a) Mr. Lee Kuan Yew Thakur’. The real Ricky Thakur files a suit for
(b) Mr. Lee Hsien Loong defamation. Decide.
(c) Mr. Lim Hng Kiang DECISION :
(d) Mr. Lee Li Lian (a) Kiara shall be held liable for defamation
as she published a statement which was
69. Election Commission (EC) has announced injurious to Ricky's reputation.
to launch Electoral Roll Authentication (b) Rudra shall be held liable as he had led
Mission (ERAM) and Purification Drive. Kiara into thinking that he was Ricky
What is the aim of the mission ? Thakur and moreover, it was his fault in
(a) To enlist all the eligible voters in voters the first place that made Kiara create this
list website.
(b) To create awareness about the voting right (c) Kiara cannot be held liable as she had
(c) To weed out bogus voters from the voters actually been referring to Rudra and not
list the real Ricky Thakur.
(d) (a) and (c) (d) Kiara cannot be held liable as her act was
done in good faith as she intended to
70. What is the name of the party of the warn other girls.
Australian Prime MinisterTony Abbott that
saw a confidence vote on spilling the top 72. LEGAL PRINCIPLE : Whoever stores a
leadership in February 2015 ? substance which could cause damage on
(a) Liberal Party escape shall be absolutely liable (i.e. liable
(b) Australian Greens even when he has exercised necessary care)
(c) Labour Party for any damage caused by the escape of the
(d) Palmer United Party substance.
FACTUAL SITUATION: Union Carbide India
SECTION - C : LEGAL APTITUDE Limited (UCIL) manufactured methyl
isocyanate, an extremely toxic gas. Due to
Directions (Q. 71 - 95) : Given below is a statement a storm, the gas that was being stored in
of legal principle followed by a factual situation. Apply sealed containers got released. Before much
the principle to the facts given below and select the could happen, the local municipal authorities
most appropriate answer. managed to contain the disaster. The
authorities filed a suit against UCIL for the
71. LEGAL PRINCIPLE : A statement is costs that were incurred in decontamination.
defamatory in nature if it is injurious to a However, later it was realized that the clean-
person's reputation and if the statement has up by the authorities could have been done
been published. withoul spending as much resources and the
FACTUAL SITUATION : Rudra had been damage was not that significant. UCIL argued
dating a girl named Kiara for three weeks. that it would pay only part of the amount
But he had introduced himself to heras Ricky demanded by the authorities, which could
Thakur (who is one of Rudra's friends) and he have dealt with the contamination.
continued to be Ricky for the rest of their
relationship. But ultimately the relationship
ended badly and Kiara being upset and angry

Previous Years
CLAT & AILET Papers Page 273
DECISION : (d) Satwik is not liable as by the time Prateek
(a) UCIL is liable only to the extent of borrowed money from Abbas, the
contamination caused. lt does not need partnership was no more in existence
to pay the authorities the entire amount
demanded by them. 74. LEGAL PRINCIPLE : Everybody is under a
(b) The authorities are entitled to the whole legal obligation to take reasonable care to
sum, as UCIL shall be held liable for all avoid act or omission which he can foresee
the repercussions of their act even if they would injure his neighbour, the neighbour for
had exercised due care. this purpose is any person whom he should
(c) UCIL can plead that the escape of the have in his mind as likely to be affected by
gas had been caused by a storm and not his act.
due to its own negligence. lt was an FACTUAL SITUATION : Krish, while driving
inevitable accident. a car at a high speed in a crowded road,
(d) The municipal authorities should have knocked down a cyclist. The cyclist died on
analyzed the damage first before jumping
the spot with a lot of blood spilling around,
into action. lt was due to their own
Lekha, a pregnant woman passing by,
negligence because of which they had to
suffered from a nervous shock, leading to
shell out more than required.
abortion. Lekha filed a suit against Krishnan
claiming damages.
73. LEGAL PRINCIPLE : A partner is liable for
DECISION :
the debts incurred by the other partners in
(a) Krish will be liable, because he owed a
the course of Partnership.
duty of reasonable care to everybody on
FACTUAL SITUATION : Satwik and Prateek
the road including Lekha.
enter into a partnership to, produce a film,
(b) Krish will not be liable, because he could
wherein Satwik also directs the movie. The not have foreseen Lekha suffering from
movie bombed at the box office. nervous shock as a result of his act.
Consequently, they run into financial (c) Krish will be liable to Lekha because he
difficulties and the partnership ends. Prateek failed to drive carefully.
goes to Abbas to borrow some money, which (d) None of the above.
Abbas understands is for-repaying the debts
from the partnership. Prateek takes the money 75. LEGAL PRINCIPLE : The occupier of a
and absconds to Malibu. Abbas sues Satwik premise owes a duty of care to all his invitees
for the amount. Decide and visitors.
DEClSlON : FACTUAL SITUATION : Lalit was running a
(a) Satwik is liable to return the money as it dairy from his house. People used a part of
was his partner, Prateek, who directed his farm as shortcut to get to a nearby railway
the movle. station. Lalit who did not approve of this, put
(b) Abbas has been negligent in not properly up a notice that “Trespassers will be
enquiring the purpose for which Prateek prosecuted”. However since a number of
borrowed the money. Satwik is not liable
these people were also his customers he
to pay him back according to the principle
tolerated them. One day a person who was
of contributory negligence
using this short cut was attacked by a bull
(c) Satwik is not liable as prateek absconded
belonging to the farm. The injured person filed
with the money instead of using it to
a suit against him.
payoff the debts in the partnership.

Previous Years
Page 274 CLAT & AILET Papers
DECISION : FACTUAL SITUATION : Parliament enacted
(a) Lalit is not liable in view of the clear notice a law, which according to a group of lawyers
against trespassers. is violating the fundamental rights of traders.
(b) Lalit is liable for having kept a bull on his A group of lawyers files a writ petition
farm. challenging the Constitutional validity of the
(c) Lalit is not liable to the people other than statute seeking-relief to quash the statute and
his customers. further direct Parliament to enact a new law.
(d) Lalit is liable because in fact he allowed DECISION :
the people to use his premises. (a) The court can quash the existing law if it
violates fundamental rights but cannot
76. LEGAL PBINCIPLE : A master shall be liable direct Parliament to make a new law.
for the acts of his servants done in the course (b) The court can quash existing law if it
of employment. violates fundamental rights and can direct
FACTUAL SITUATION : PUL, a public sector Parliament to make a new law.
undertaking, is operating a number of bus (c) No writ would lie against Parliament, as
services for its employees in Pune. These the court has no authority to direct
buses are quite distinct in their appearance Parliament to enact or re-enact a law.
and carry the board “for PUL employees only”. (d) The court cannot quash the law as
M, a villager from neighbouring state, was reasonable restrictions can be put on the
waiting for a regular bus in one of the bus fundamental rights.
stops in Pune. A bus belonging to PUL
78. LEGAL PRINCIPLE : When one person
happened to stop nearby and number of
signifies to another his willingness to do or
people got into the bus. M, without realizing
abstain from doing anything, with a view to
that it was PUL bus, got into the bus and
obtaining the assent of that person to such
soon-thereafter, the bus met with an accident
an act or abstinence, he is said to have made
due to driver's negligence. M, along with
a proposal.
several others, was injured in the accident.
FACTUAL SITUATION : Ram sends a
M seeks to file a suit against PUL claiming
telegram to Sohan, writing : “Will you sell
damages.
me your Rolls Royce car? Telegram the
DECISION :
lowest cash price.” Sohan also replied by
(a) M will succeed, because he got into the
telegram: “Lowest price for car is Rs. 20 lakh."
bus without realizing that it was PUL bus.
(b) M will not succeed, because it was for Ram immediately sent his consent through
him to find out whether it was a public telegram stating “l agree to buy the cat for
transport. Rs. 20 lakh asked by you.”, Sohan refused
(c) M will succeed, because the driver was to sell the car
anyhow duty-bound to drive carefully. (a) He cannot refuse to sell the car because
(d) PUL is not liable as the bus met with an the contract has already been made.
accident due to drive’s negligence. (b) He can refuse to sell the car because it
was only invitation to offer and not the
77. LEGAL PRINCIPLE : Only Parliament or real offer.
State Legislatures have the authority to enact (c) lt was not a valid offer because willingness
laws on their own. No law made by the State to enter into a contract was absent.
(d) lt was not a valid contract as offer and
can take away a person's fundamental right.
acceptance is conveyed through
telegram.

Previous Years
CLAT & AILET Papers Page 275
79. LEGAL PRINCIPLE : A person is said to be DECISION :
of sound mind for the purpose of making a (a) X cannot be prosecuted because he had
contract if, at the time when he makes it, he actually no knowledge about the new
is capable of understanding it and of forming notification issued two days ago
a rational judgement as to its effect upon his (b) X cannot be prosecuted because
interests. ignorance of fact is excusable
FACTUAL SITUATION : Mr. X who is usually (c) X can be prosecuted because ignorance
of sound state of mind, but occasionally of of law is not excusable
unsound state of mind, enters into a contract (d) X’s liability would depend on the discretion
with Mr. Y when he was of unsound state of of the court.
mind. Mr. Y having come to know about this
81. LEGAL PRINCIPLE : Any direct physical
fact afterwards, wants to file a suit against
interference with goods in somebody’s
Mr. X.
possession without lawful justification is
DECISION :
called trespass of goods.
(a) Mr. X cannot enter into contract because
FACTUAL SITUATION : Z purchased a car
he is of unsound state of mind when he
from a person who had no title to it and sent
entered into contract.
it to a garage for repair. X believing wrongly
(b) Mr. X can enter into contract but the
burden is on the other party to prove that that the car was his, removed it from the
he was of unsound state of mind at the garage. Has X committed any offence?
time of contract. DECISION :
(c) Mr. X can enter into contract but the (a) X cannot be held responsible for trespass
burden is on Mr. X to prove that he was of of goods as he was under a wrong belief.
sound state of mind at the time of (b) X can be held responsible for trespass of
contract. goods.
(d) Contract with a person of unsound mind (c) Z has no right over the car as he purchased
is void. it from a person who has no title over it.
(d) None of the above.
80. LEGAL PRINCIPLE : Ignorance of Fact is
excused but ignorance of law is not an excuse 82. LEGAL PRINCIPLES :
to criminal liability. 1. Bigamy is not permitted under Hindu law.
FACTUAL SITUATION : X was a passenger 2. A Hindu of sound mind can adopt a child
of the gender they don’t already have a
from Zurich to Manila in a Swiss Plane. When
child of.
the plane landed at the Airport of Bombay on
3. Only the child's father, mother or guardian
22 November 2014 it was found on searching
has the capacity to give the child up in
the X carried 34 kg of Gold Bars on his person
adoption, under Hindu law.
and that he had not declared it in the ‘Manifest
FACTUAL SITUATION : Ramesh comes
for Transit’. On 24 Novemeber 2014, the
from a family where from the past two
Government of India has issued a notification
generations; all male members have had two
modifying its earlier exemption, making it
wives. He also wants to continue this tradition
mandatory now that the gold must be
and thus he married Suman first and then
declared in the “Manifest” of the aircraft.
Tania. Ramesh has a son, Gunjan,with
Suman. However, with Tania, he is childless.
He thus wishes to adopt some children. Rajat
is the child of Ramesh’s brother. However,

Previous Years
Page 276 CLAT & AILET Papers
Rajat’s parents decide to go off to Dubai for 5 (c) Raja is liable as you don’t expect
years and they leave their son under the care anything better from a gangster.
of Ramesh’s sister. Ramesh and Tania ask (d) Raja is not liable as he was confused as
for Rajat in adoption from his sister who to whom he should be returning the
agrees and Rajat is adopted. Later, Ramesh property to.
and Suman want to adopt a daughter as well
and they do adopt a girl-Sara-from an 84. LEGAL PRINCIPLES : The state shall make
orphanage. However, at the time of signing special laws for the upliftment of citizens of
the adoption deed, Ramesh was dead drunk. the country, and these laws can be made for
Ramesh and his entire family member are the benefit of any specific caste, class or sex
Buddhists. Keeping in mind the given of people living in the society.
principles and facts, solve the questions. Is FACTUAL SITUATION : The state of Hindu
Rajat’s adoption a valid adoption ? Pradesh comes out with a law, which provided
DECISION : for reservation to Muslims in all government
(a) No, because Ramesh already has a son. and government aided institutions. This
(b) No, because Ramesh's sister has no law is challenged in the High Court of
authority to give Rajat up in adoplron. Hindu Pradesh, as being arbitrary and
(c) Yes, it is a valid adoption because contrary to the established laws. Can the
Ramesh has no child with Tania ano challenge be successful ?
Ramesh's sister is Rajat's guardian for 5 DECISION :
years at least. (a) Yes, since people from other religions
(d) Both (a) and (b) would also start making such demand’s
which would jeopardize the unity and
83. LEGAL PRINCIPLES : Whoever dishonestly integrity of the country.
takes away any property from the possession (b) No, since the state has the right to make
of another, with an intention of such taking special laws for the upliftment of the
away, without his permission is liable for theft. citizens of the country.
FACTUAL SITUATION : Raja, a famous (c) Yes, since the state has not been
gangster, moves into an apartment in mandated to make reservation, based on
Kankurgachi, Calcutta. There, he discovers a person's religion.
that the previous owner of the apartment had (d) No, since the Government cannot neglect
left behind a pair of beautiful ivory handled the minorities.
combs. Mesmerized by their beauty and
85. LEGAL PRINCIPLE : The master principal
confused as to whom he should be returning
is liable for all acts done by his duly
them to, he decides to retain them and starts appointed servant/agent for all acts done by
using them. The previous owner of the combs him lawfully in the course of his employment.
gets to know; this and registers an FIR for FACTUAL SITUATION : A, B, C and D
theft against Raja. ls Raja liable ? carried on a business in partnership. While
DECISION : making a deal with another company, B bribed
(a) Raja is liable for theft as he failed to return the clerk there. ls the partnership firm
the property even when he knew it was vicariously liable ?
someone else’s property. DECISION :
(b) Raja is not liable as he is not taken it (a) No, as bribing is not in course of
away from anyone else's possession and employment of the partners.
there was no dishonest intention. (b) Yes, as partners are agent of the firm.

Previous Years
CLAT & AILET Papers Page 277
(c) Yes, as B can be said to have implied 88. LEGAL PRINCIPLE : Nobody shall unlawfully
authority for the same. interfere with a person’s use or enjoyment of
(d) No, as this act was not authorised by the land, or some right over, or in connection with
others. it. The use or enjoyment, envisaged herein,
should be normal and reasonable taking into
86. LEGAL PRINCIPLE : A contract which is account surrounding situation.
impossible to perform becomes void. FACTUAL SITUATION : Jogi and Prakash
were neighbours in a residential locality.
FACTUAL SITUATION : Surender agreed to
Prakash started a typing class in a part of
deliver a specific quality of rice to Sonakshi
his house and his typing sound disturbed Jogi
identified by both of them. Before delivery,
who could not put up with any kind of
the rice was burnt by short circuit. ls Surender
continuous noise. He filed a suit against
discharged from the performance of the Prakash.
contract? DECISION :
DECISION : (a) Prakash is liable, because he should not
(a) Surender is discharged from performance have started typing class in his house.
as the subject matter of the contract is (b) Prakash is liable, because as a
destroyed. neighbour, he should have realised Jogi’s
(b) Surender is discharged from performance delicate nature.
as the subject matter has been (c) Prakash is not liable, because typing
specifically identified. sound did not disturb anyone else other
(c) Surender is not discharged from than Jogi
performance as he can procure rice from (d) None of the above.
other sources. 89. LEGAL PRINCIPLES :
(d) None of the above. 1. Whoever causes death by doing an act
with the intention of causing death or with
87. LEGAL PRINCIPLE : Whoever dishonestly the intention of causing such bodily injury
misappropriates or converts to his own use as is likely to cause death or with the
any movable property is guilty of criminal knowledge that he is likely by such act
misappropriation of property. to cause death commits the offence of
FACTUAL SITUATION : A finds a culpable homicide.
government promissory note belonging to Z, 2. Mens rea and actus reus must concur to
bearing a blank endorsement. A knowing that result in a crime which is punishable by
the note belongs to Z, pledges it with a banker the law.
as security for a loan, intending to restore it FACTUAL SITUATION : A and B went for
to Z at a future time. Has A committed shooting. A knows Z to be behind a bush. B
criminal misappropriation ? does not know it. A induces B to fire at the
DECISION : bush. B fires and kills Z. Has an offence been
(a) Yes since he deprived Z from using his committed ?
property and used it for his own use. DECISION :
(b) No, since he intended to return the (a) A had mens rea but no actus reus. B had
actus reus but no mens rea. No one is
property to Z in the future.
guilty.
(c) No, it is theft and not criminal
(b) A induced B to fire at the bush with the
misappropriation. knowledge that Z is there. A is guilty
(d) Yes since he deprived Z from using his c.culpable homicide but B is not guilty of
property. any offence.
(c) Both A and B are guilty.
(d) None of the above.

Previous Years
Page 278 CLAT & AILET Papers
90. LEGAL PRINCIPLES : (d) No, the shopkeeper is guilty as he was
1. The crime of kidnapping involves taking the one who circulated the counterfeit note
someone away from the custody of their to Roshni.
lawful guardian.
3. The crime abduction involves inducing or 92. LEGAL PRINCIPLES :
forcing somebody to go away from some 1. To constitute a punishable criminal
place against their will. offence, guilty intention must accompany
FACTUAL SITUATION : A steals B's slave. an illegal act.
ls it a crime ? 2. Criminal mischief means causing damage
DECISION : to public property intentionally or with the
I. Kidnapping ll. Abduction lll. Neither knowledge that harm may occur.
REASON : FACTUAL SITUATION : Neel being a
(A) Slavery is illegal. Shahrukh Khan fan went for the premier of
(B) A has taken him away from B's lawful Happy New Year. As usual, he carried his
custody. pen-knife, a gift from his dead mother. At the
(C) A has forced somebody to go with him security check, impatient of waiting in the
against his will. queue, Neel slunk past the guards and the
DECISION : metal detector when no one was watching.
(a) l (B) (b) ll (C) Later, he was apprehended in the hall and
(c) III (A) (d) I (A) charged for mischief and possession of a
weapon when it was expressly forbidden.
91. LEGAL PRINCIPLE : Whoever delivers to DECISION :
another person as genuine any counterfeit (a) Neel is not criminally liable since he had
currency which he knows to be counterfeit, no intention to commit mischief.
but which that other person is not aware of at (b) Liable for possession of the weapon since
the time when he received it, is guilty of it was expressly forbidden and mere
counterfeiting currency. possession was enough; although he
FACTUAL SITUATION : While returning might not be liable for mischief as he did
home one day, Roshni realizes that the local not do anything.
shopkeeper has given her a fake note ot Rs. (c) Neel is not liable since the pen knife has
1,000. Disappointed, she goes to the same an emotional value and rather the guards
shop and buys cosmetics worth Rs. 600. She should be punished for the security
then passes the same fake note to the breach.
shopkeeper. The shopkeeper while inspecting (d) Liable for both possession of weapon and
the note finds out that it is fake. ls Roshni criminal mischief since he slunk past the
guilty ? guards which shows his intention to
DECISION : commit the crime.
(a) No, as she was merely attempting to
return the note to the same shopkeeper 93. LEGAL PRINCIPLE :
who gave her the note. 1. Everyone has a right to defend their life
(b) No, she is not guilty of any offence as and property against criminal harm
neither did she manufacture the note nor provided it is not possible to approach
did she circulate it with a view to deceive public authorities and more harm than is
the public. necessary has been caused to avert the
(c) Yes, as she attempted to pass on a note danger.
which she knew was counterfeit. 2. Nothing is an offence which is done in
the exercise of the right of private defence.

Previous Years
CLAT & AILET Papers Page 279
FACTUAL SITUATION : The accused found to commit it, is an attempt to commit the
the deceased engaged in sexual intercourse offence which is by itself an offence.
withhis 15 years old daughter. The accused FACTUAL SITUATION : A wanted to kill B
assaulted the deceased on the head with a and had therefore gone to the market to buy
spade which resulted in his death. Accused explosives to plant in his house. After A has
claimed private defence and the prosecution planted the bomb, he felt guilty and went
claimed that the sexual intercourse was with back to remove the bomb but while he was
the consent of the daughter. Here, doing so, B saw him and called the police.
DECISION : Can A be held liable ?
(a) Accused is entitled to the right of private DECISION :
defence since the girl was only 15 years (a) Yes, because he has done something
old. more than mere preparation.
(b) Accused exceeded the right of private (b) No, because B did not die.
defence. (c) Yes, because there existed a mala fide
(c) Accused is not entitled to private defence intention.
as the intercourse was consensual. (d) No, because he had removed before
(d) Accused is not entitled to private defence anything could happen.
as the right of private defence is available
for defending one’s life and property only. 96. The Supreme Court of India constituted _____
Bench in December 2014.
94. LEGAL PRINCIPLES : Necessity knows no (a) Constitution (b) Arbitration
law, and any person facing danger may do (c) Social Justice (d) Fast-Track
all that is necessary to avert the same till he
can take recourse to public authorities. 97. Who is the author of the book “lndian
FACTUAL SITUATION : Akshay, a law Parliamentary Diplomacy - Speaker’s
abiding citizen decided to remove the weed Perspective” ?
of corruption from India society. One day, (a) Sumitra Mahajan
confronted with a bribing official, Akshay (b) Somnath Chaterjee
decided to teach him a lesson and punched (c) Meira Kumar
him on his face. Akshay (b) Manohar Joshi
(a) Can plead defence of necessity as he was
being bribed which is a crime. 98. Which of the following recently became 123rd
(b) Cannot plead defence of necessity as State party to the International Criminal Court?
there was no necessity to act in the (a) lsrael (b) Palestine
manner he acted. (c) Jordan (d) Bangladesh
(c) Can plead defence of necessity as aware
99. Which body has launched the “Group of
and vigilant citizenry forms the basis of a
Friends Against Terrorism” ?
good democracy.
(a) United Nations (b) European Union
(d) Can plead defence of necessity as there
(c) ASEAN (d) SAARC
was no time to take recourse to public
authorities. 100. Which one of the following Committee was
constituted to review environmental law in
95. LEGAL PRINCIPLES : the country ?
1. Preparation to commit an offence is not (a) Subramanian Committee
an offence. (b) Kasturi Ranjan Committee
2. After one has finished preparation to (c) Madhav Nair Committee
commit an offence, any act done towards (d) Ullas Karanth Committee
committing the offence with the intention
Previous Years
Page 280 CLAT & AILET Papers
101. Which South-East Asian country has recently consciousness, where we become aware that
banned surrogacy services to end its we live with other people. Finally, we develop
flourishing rent-a-womb industry? a sense of values, which is our overall
(a) Singapore (b) India estimation of our worth in the world.
(c) Thailand (d) Vietnam
Which one of the following would be the best
102. The Third National Lok Adalat which disposed completion of this passage?
of 56000 cases in a single day was held under (a) The sum total of all these developments
the chairmanship of we call the self-image or the self-concept.
(a) Justice H. L. Dattu (b) This estimation of worth is only relative
(b) Justice T. S. Thakur to our value system.
(c) Justice Jagdish Singh Khehar (c) Therefore, our social consciousness is
(d) Justice Anil R Dave dependent on our sense of values.
(d) The sum total of living with other people
103. In which of the following cases, the Supreme and developing a sense of values makes
Court, recently, held that demand for dowry us a total person.
can be made at any time and not necessarily
before marriage? 107. Violence against racial and religious minority
(a) State of uttarakhand v. Bhim Singh & Anr groups increased sharply throughout the
(b) State of Bihar v. Arnesh Kumar & Anr country last year, despite a slight decline in
(c) State ot Rajasthan v. Prem Kumar state wide figures. Compiling incidents from
(d) State of Punjab v. Kans Raj & Ors police departments and private watchdog
groups, the County Human Relations
104. National Green Tribunal ruled _____ old diesel Committee reported almost 500 hate crimes
vehicles cannot ply on Delhi roads. in the year, up from only 200 last year. It was
(a) 15 years the first increase since the committee began
(b) 10 years to report a yearly figure six years ago. The
(c) 20 years lower state wide figures are probably in error
(d) 5 years due to underreporting in other countries;
underreporting is the major problem that state
105. Supreme Court struck down which Section surveyors face each year.
of the lnformation Technology Act 2000, in All of the following, if true, would support the
Shreya Singhal v. U. O. I, as unconstitutional? conclusion or the explanation of the
(a) 66 B (b) 66 A discrepancy in the state and country figures
(c) 66 C (d) 66 EXCEPT:
(a) The number of hate crimes and those
SECTION - D : REASONING resulting in fatalities has increased in
neighbouring states.
106. At birth we have no self-image. We cannot (b) Anti-immigration sentiment was fanned
distinguish anything from the confusion of this year by an anti-immigration ballot
light and sound around us. From this referendum.
beginning of no-dimension, we gradually (c) Many law-abiding members of minority
begin to differentiate our body from our groups are fearful or distrustful of the
environment and develp a sense of identity, police.
with the realisation that we are a separate (d) All of the countries in the state have active
and independent human being. We then begin private watchdog groups that carefully
to develop a conscience, the sense of right monitor hate crimes.
and wrong. Further, we develop social

Previous Years
CLAT & AILET Papers Page 281
108. In the last three years, the numbers of arrests yield less than the catch of five years ago,
for burglary and robbery in Sandy Beach has and less than one quarter of the total of ten
declined by more than 30 percent. At the years ago. The cod has almost disappeared.
same time, the city has reduced the size of The numbers of fisherman in Newfoundland
its police force by 25 percent. and New England have declined, and their
Which one of the following helps to resolve
yearly earnings are now at an all-time low.
an apparant discrepancy in the information
Yet radar has made fishing methods more
above?
(a) Neighbourhood Watch programs have efficient than ever.
always been active in Sand Beach. Which one of the following identifies most
(b) The number of reported burglaries and clearly a faulty assumption in the reasoning
robberies in Sandy Beach has increased of this passage?
in the last three years. (a) The argument fails to consider that the
(c) Compared to other cities in the state, decline in the catch may be due to factors
Sandy Beach has one of the lowest crime other than pollution.
rates. (b) The argument assumes that the waters
(d) Many of the residents of Sandy Beach off Newfound land are representative of
have installed expensive security systems all the American oceans.
in their homes. (c) The pollution of the sea may have been
caused by natural as well as human
109. To be admitted to Bigshot University, you forces.
must have a 3.5 grade-point average (GPA) (d) The argument does not allow for the
and a score of 800 on the admission test, a possibility that the catch may increase
3.0 GPA and a score of 1,000 on the in size in the next five years.
admission test, or a 2.5 GPA and a score of
1,200 on the admission test. A sliding scale 111. A number of lawsuits have been brought
exists for the other scores and GPAs. against popular singing groups charging that
suicidal themes in their songs have led to
Which one of the following is inconsistent teenage suicides. So far, the courts have
with the above?
found that the lyrics are protected under the
(a) Jagan was admitted with a 2.7 GPA and
Constitution. But what if this should change
a score of 1,100 on the admission test.
and a court decides that suicidal themes in
(b) No student with a score less than 800 on
popular songs are dangerous? In fact, the
the admission test and a 3.4 GPA will be
songs that have been charged so far are anti-
admitted.
(c) More applicants had a GPA of 3.5 than suicide; they present sardonically the self-
had a GPA of 2.5. destructive behaviour of drinking, drugs, and
(d) Some of the students with a score of less escape by death. They describe a pitiful state
than 1,200 on the admissions test and a of mind, but they do not endorse it.
GPA of less than 2.5 were admitted. Blaming suicide on the arts is nothing new.
In the late eighteenth centrury, Goethe’s
110. Only 75 years ago, the best fishing in the popular novel The Sorrows of Young Werther
world was the Grand Banks of the North was said to be the cause of a rash of suicides
Atlantic. But now overfishig and man’s in imitation of the novel’s hero. If we begin to
pollution have decimated the area. There will hold suicide in books or music responsible
be no fishing industry in the Americas in a for suicides in real life, the operas of Verdi
few years. The waters off Newfoundland now and Puccini will have to go, and Romeo and

Previous Years
Page 282 CLAT & AILET Papers
Juliet and Julius Caesar will disappear from Direction (Q. 113 - 118) : Each group of questions
the high school reading lists. is based on a set of conditions. Choose the response
that most accurately and completely answers each
Which one of the following is an assumption question.
necessary to the author s argument? A science student has exactly four flasks-1, 2, 3
(a) A lyric presenting suicide in a favourable and 4-originally containing a red, a blue, a green
light should not have Constitutional and an orange chemical, respectively. An experiment
protection. consists of mixing exactly two of these chemicals
(b) Literature or music cannot directly together by completely emptying the contents of
influence human behaviour one of the flasks into another of the flasks. The
(c) Freedom of speech is most threatened following conditions apply;
by our personal freedom The product of an experiment cannot be used in
(d) The audience, not the performer, is further experiments.
responsible for the audience’s actions. Mixing the contents of 1 and 2 produces a red
chemical. Mixing the contents of 2 and 3 produces
112. Archaeologists have come to the support of an orange chemical.
Arctic anthropologists. A small minority of Mixing the contents of 3 with the contents of either
anthropologists assert that stone Age tribes 1 or 4 produces a blue chemical.
of the Arctic domesticated wolves and trained Mixing the contents of 4 with the contents of either
them to haul sleds. Excavations have 1 or 2 produces a green chemical.
recently found evidence to support this claim.
Archaeologists have found wolf bones near 113. lf the student performs exactly one
the site of a Stone Age village. They have experiment, which one of the following could
also found walrus bones that might have been be the colours of the chemicals in resulting
used on primitive sleds. The small minority three non-empty flasks ?
of anthropologists believe that their theories (a) blue, blue, green
have been proved. (b) blue, orange, orange
(c) blue, orange, red
Which one of the foliowing is true of the (d) green, green, red
evidence cited in the paragraph above ? 114. lf the student performs ex a c t l y t w o
(a) lt conclusively contradicts the experiments, which one of the following
anthropologists’ conclusions. could be the colours of the chemicals in
(b) lt neither supports nor refutes the the resulting two non-emptv flasks ?
anthropologist’ conclusions positivety.
(a) blue, blue (b) blue, orange
(c) lt conclusively supports only a part of the
(c) blue, red (d) green, red
anthropologists’ conclusions.
(d) lt supports the anthropologists’ 115. lf the student performs exactly one
conclusions authoritatively. experiment and none of the resulting three
non-empty flasks contains a red chemical,
which one of the following could be the colours
of the chemicals in the three flasks ?
(a) blue, blue, green
(b) blue, green, green
(c) blue, green , orange
(d) blue, orange, orange

Previous Years
CLAT & AILET Papers Page 283
116. lf the student performs exactly one Direction (Q. 122 - 124) : Read the following
experiment and exactly one of the resulting information carefully and answer the questions given
three non-empty flasks contains a blue below.
chemical, which one of the following must be The city K is 30 km to the southeast of Z while Y is
the colours of the chemicals in other two 50 km to the northwest of K. Also, H is 38 km to
flasks ? the southeast of Y. L lies in the direct route between
(a) both green Y and K and its distance from H is 14 km. G also
(b) both orange lies on this route and is exactly midway between L
(c) both red and Y.
(d) one green and one red 122. A car starting from K at 9 a.m. and running at
a constant speed towards Y reaches H at
117. If the students will perform exactly two 9.24 a.m. and then reaches G at
experiments and after the first experiment (a) 9.18 a.m. (b) 10.16 a.m.
exactly one of the resulting three non-empty (c) 10.36 a.m. (d) 10.42 a.m.
flasks contains an orange chemical, then in
the second experiment the student could mix 123. If M is 1 km to the southeast of L, then it is
together the contents of flasks exactly midway between
(a) 1 and 2 (b) 1 and 3 (a) H and L (b) Y and K
(c) 2 and 3 (d) 3 and 4 (c) H and Z (d) None of these

118. lf the student performs exactly one experiment 124. The distance from G to H is
and none of the resulting three none-empty (a) 26 km (b) 24 km
flasks contains an orange chemical, then the (c) 12 km (d) 16 km
student must have mixed the
(a) flask 1 with flask 2 Direction (Q. 125 - 128): Each question contains
(b) flask 2 with flask 4 four arguments of three sentences each. Choose
(c) flask 2 with one of the other flasks the set in which the third statement is a logical
(d) flask 4 with one of the other flasks conclusion of the first two.
125. (1) Some bikes are mopeds. All mopeds are
Direction (Q, 119 - 121): Read the following scooters. Some bikes are scooters.
information carefully and answer the questions given (2) All children are hairs. No hairs are red.
below. No children are red.
In a joint family of seven persons L, M, N, O, P, Q (3) No pencil is pen. Some pens are
and R, two are married couple. ‘R’ is a housewife markers. Some pencils are markers.
and her husband is a lawyer. ‘N’ is the wife of ‘M’, ‘L’ (4) Every man has a wife. All wives are
is an engineer and is the grand-daughter of ‘R’ and devoted. No devoted has a husband.
‘O’ is the father-in-law of ‘N’, a doctor, and father of
‘P’, a professor. ‘Q’ is L’s brother and M’s son. (a) (1), (2), and (3) (b) (1) and (2)
(c) (3) and (2) (d) (1), (2), (3) and (4)
119. How is P related to M ?
(a) Son (b) Brother 126. (1) No moon is not red. AII stars are moon.
(c) Daughter (d) Uncle All stars are red.
(2) All doors are open. No open is outdoors.
120. How is Q related to O ? All door are not outdoors.
(a) Grandfather (b) Uncle (3) No Japanese can fire. All Chinese are
(c) Grandson (d) brother books. Japanese and Chinese can fight.
(4) No A is B. No B is C. No A is C.
121. Who is M’s father ? (a) (1) only (b) (2) only
(a) O (b) R (c) N (d) P (c) (1) and (2) only (d) (4) only
Previous Years
Page 284 CLAT & AILET Papers
127. (1) All envelopes are rectangles. All 131. Every player will become a champ.
rectangles are rectangular. All envelopes I. Rajesh is a player.
are rectangular. ll. Rajesh will become champ.
(2) Some things are smart. Some smart lll. Rajesh is not a player.
things are tiny. Some things are tiny. lV. Rajesh will not become a champ.
(3) Learneds are well read. Well read know. (a) l, lV (b) lV, I
Learneds know. (c) III, IV (d) IV, III
(4) Dieting is good for health. Healthy foods
Direction (Q. 132 - 135) : The question given below
are rare. Dieting is rare.
has a statement followed by two courses of action
(a) (4) only (b) (3) only
numbered I and II. You have to assume everything
(c) Both (1) and (3) (d) All of these
in the statement to be true. Then decide which of
128. (1) Shahrukh is an actor. Some actors are the two suggested course of action logically follows
for pursuing.
pretty. Shahrukh is pretty.
Mark answer as :
(2) Some executives are soldiers. All soldiers
(a) lf only l follows
are patriotic.
(b) lf only ll follows
(3) All cricketers are patriotic. Some (c) ll both l and ll follows
executives are soldiers. Some executives (d) lt neither l follows nor ll follows
are patriotic.
(4) All actors are pretty. Shahrukh is not an 132. Statement:
actor. Shahrukh is not pretty. The number of people who die on the roads
(a) (4) only (b) (2) only every year is so alarming that the numbers
(c) (1) only (d) (2) and (3) every year are also close to the numbers
which can be attributed to the major diseases
Direction (Q. 129 - 131) : Each question has a main
statement, followed by four statements labelled. I, of the world.
II, III and IV. Choose the ordered pair of statement, Course of Action :
where the first statement implies the second, and I. There should be an active campaign for
the two statements are logically consistent with the sensitising people towards road safety
main statement. norms.
II. There should be an increased emphasis
129. Whenever Vijay reads late into the night, his on enforcing speed limits, road safety
grandfather reprimands him. rules and traffic management on roads
I. His grandfather does not reprimand Vijay. across the world.
II. Vijay reads late into the night.
III. Vijay reads early in the morning. 133. Statement:
IV. Vijay’s grandfather reprimands him in the An unacceptable number of children die
morning during the first year of their lives. The high
(a) lll,lV (b) II,IV incidence of infant death is a major cause for
(c) I, II (d) none of the above concern for the health ministry.
Course of Action :
130. Either Amir is angry, or he shows mock anger. I. All government hospitals should be
l. Amir shows mock anger. privatized to improve health care facilities.
ll. Amir is angry. ll. Government should commit higher levels
lll. Amir does not show mock anger. of their budget to health services.
lV. Amir is not angry.
(a) III, II only (b) IV, I only
(c) I, II only (d) lll, ll and lV, I

Previous Years
CLAT & AILET Papers Page 285
134. Statement: 137. Which one of them is lying about another
The cream of India’s Cricket team is likely to person’s tendencies ?
retire in the next three years leaving a vacuum (a) Akshay
which the lndian Cricket team is going to (b) Himanshu
struggle to overcome. (c) Anuj and Himanshu
Course of Action : (d) Anuj
l. The BCCI should start to induct Going around the village, you come across three
youngsters into the team and start to give people. One of them is a dentist, one is a barrister
them exposure to pressure situations. and one is a professor. You want to know who is
II. There should be a rotation policy adopted who.
for senior players in order to prolong their Peter: I am not a professor. Shina is not a professor.
careers and keep them injury free. Matt: Peter is not a barrister. Shina is a professor.
Shina: Peter is not a dentist. I am not a professor.
135. Statement:
The lack of employment and income during 138. Which of the following is true ?
economic depression leads to an increase (a) Shina is the professor
in the crime rates. (b) Peter is the dentist
Course of Action : (c) Matt is the barrister
I. The government should provide an (d) None of the above
unemployment compensation as done in
advanced countries. Further, you come across three women, one of
ll. The government should encourage greater whom is an excellent singer. You start questioning
economic activity and increase the them, when you notice that Minaxi is wearing a flower
number of industries in order to reduce in her hair.
crime rates. Madhuri: I am not the singer. The singer wears a
flower in her hair.
Direction (Q. 136 - 140): Read the questions below Minaxi: I am the singer. The singer is amongst us.
very carefully and choose the correct answer. Jaya: Madhuri is the singer. Minaxi is not the singer.
On an island ‘Neverland’ the inhabitants always
answer any question with two sentences - one of 139. Who is the singer ?
which is always true and the other always false. (a) Madhuri
Perhaps due to this peculiar habit, there’s been a (b) Minaxi
high rate of suicides on the island. As a doctor, you (c) Jaya
have to identify potentially suicidal people and (d) Cannot be ascertained
counsel them. You know that all people who are
suicidal feel that the life is futile. On questioning You want to expand your horizons and decide to go
three inhabitants, these are the answers you get: to the village of “Where is Who” which is deep inside
Anuj: Himanshu is suicidal. I am not suicidal. the island. You come the border of “Kya Kya” and
Himanshu : I do not want to die. Akshay does not see a road sign. One leads to left and the other
want to die. right. Threre are no other roads. You ask the
Akshay: Life is futile. I am suicidal. inhabitants.
Maroof : I do not speak to the strangers. I am new
136. Who among the three is suicidal ? to these parts.
(a) Akshay Nafish : Take road to the right. I am married to
(b) Himanshu Ayesha.
(c) Himanshu and Akshay Ayesha : I am not Nafish’s wife.
(d) None of these Maroof is not new to these parts.

Previous Years
Page 286 CLAT & AILET Papers
140. Which one of the following is true ? (a) Rs. 550 (b) Rs. 580
(a) The road to the right leads to “Where is (c) Rs. 540 (d) Rs. 570
Who”?
(b) The road to the left leads to “Where is 146. Three maths classes: X, Y and Z take an
Who”? algebra test. The average score of class X is
(c) Nafish is married to Ayesha 83. The average score of class Y is 76. The
(d) None of the above average score of class Z is 85. The average
score of class X and Y is 79 and average
SECTION - E : MATHEMATICS score of class Y and Z is 81. What is the
average score of classes X, Y and Z ?
141. A piece of string is 40 centimetres long. lt is (a) 81.5 (b) 80.5 (c) 83 (d) 78
cut into three pieces. The longest piece is 3
times as long as the middle-sized and the 147. You can collect rubies and emeralds as many
shortest piece is 23 centimetres shorter than as you can. Each ruby is worth Rs. 4 crore
the longest piece. Find length of the shortest and each emerald is worth of Rs. 5 crore.
piece (in cm). Each ruby weighs 0.3 kg and each emerald
(a) 27cm (b) 5 cm weighs 0.4 kg. Your bag can carry at the most
(c) 4 cm (d) 9 cm 12 kg. What you should collect to get the
maximum wealth?
142. Fresh grapes contain 90% water by weight (a) 20 rubies and 15 emeralds
while dried grapes contain 20% water by (b) 40 rubies
weight. What is the weight of dry grapes (c) 28 rubies and 9 emeralds
available from 20 kg of fresh grapes ? (d) none of these
(a) 2 kg (b) 2.4 kg
(c) 2.5 kg (d) 2.6 kg 148. A man has 9 friends: 4 boys and 5 girls. In
how many ways can he invite them, if there
143. A group of men decided to do a job in 8 days. have to be exactly 3 girls in the invitee’s list?
But since 10 men dropped out every day, the (a) 320 (b) 160 (c) 80 (d) 200
job got completed at the end of the 12th day.
How many men were there at the beginning? 149. Ten points are marked on a straight line and
(a) 165 (b) 175 11 points are marked on another straight line.
(c) 80 (d) none of these How many triangles can be constructed with
vertices from among the above points?
144. In a race of 200 m run, A beats S by 20 m (a) 495 (b) 550 (c) 1045 (d) 2475
and N by 40 m. If S and N are running a race
of 100 m with exactly same speed as before, 150. Number of students who have opted for the
then by how many metres will S beat N ? subjects A, B and C are 60, 84 and 108
(a) 11.11 m (b) 10 m respectively. The examination is to be
(c) 12 m (d) 25 m conducted for these students such that only
the students of the same subject are allowed
145. Total expenses of a boarding house are partly in one room. Also the number of students in
fixed and partly varying linearly with the each room must be same. What is the
number of boarders. The average expense per minimum number of rooms that should be
boarder is Rs. 700 when there are 25 boarders arranged to meet all these conditions?
and Rs. 600 when there are 50 boarders. (a) 28 (b) 60 (c) 12 (d) 21
What is the average expense per boarder
when there are 100 boarders ?

Previous Years
CLAT & AILET Papers Page 287
AILET Question Paper 2015
SECTION - A : ENGLISH

Directions (Qs. 1-9): Read the following passage carefully and then answer the questions that follow.

Rural manual workers comprise the single largest occupational category in lndia. ln 1991, according to the
National Commission on Rural Labour, 60 percent of the workers in rural India were manual workers and
they numbered more than 160 million. The changes in the working and living conditions of rural labourers
are thus central to changes in the welfare of the rural population and of the country as a whole. The
structure and working of rural labour markets in India is complex; as is well known, there is great diversity
across regions and across segments of the labour market. This article brings together an interesting body
of research that seeks to understand and explain the types of changes that have accrued in the structure
of rural labour markets over the last few decades.

The 1980s were characterised by an explosion of the rural labour force, slow employment growth in agriculture
and a rise in the share of non-agricultural employment. The decade was also characterized by a growing
casualisation of the work force (for a relative rise in casual employment as opposed to regular employment).

At the same time, it was a period when agricultural wages increased in real terms and when income
poverty declined. There was what may be called "the tension between the estimated decline in poverty on
the one hand, and the slow growth of agricultural employment and increased casualisation of the labour
force on the other. Some of the trends in the development of rural labour over for this period are a source of
concern. These include, as Radhakrishnan and Sharma note, the continuous widening of the gap between
labour productivity in agricultural and non-agricultural occupations, the burgeoning mass of rural casual
workers who have no social security safety net, and the increasing number of women employed at very low
wages in agriculture. Another matter for concern, one that emerges from a desegregation of data on rural
unemployment by age groups, is that the incidence of unemployment is higher for persons in the age group
of 15-29 than for any other age group in others words, unemployment is typically high among new entrants
to the workforce.

ln her review of trends in wages, employment and poverty, Sheila Bhalla shows that the real wages of
agricultural labourers stagnated from the time of independence to the mid 1970s and then began to rise in
all parts of the country. This was also the period in which the incidence of rural poverty began to decline.
The rise in wages was not limited to the more prosperous agricultural zones, and Bhalla argues that the
movement in real wages was co-related with the increase in the share of non-agricultural employment in
total employment. As wages in non-agricultural work are typically higher than wages in agriculture, the
expansion of non-farm work could also explain some of the decline in rural poverty. In the 1990s, the
improvement in real wages and the decline in poverty were reversed while agricultural employment expanded.
Economic development all over the world has been associated with a rise in the share of employment in the
secondary and tertiary sectors of the economy and a fail in the share of the agricultural sector. In India,
changes in the composition of the rural workforce in the 1980s points to a "structural retrogression"

1. Give an appropriate title to the passage


(a) The complex labour markets in lndia
(b) Matters of concern in rural agriculture
(c) The agricultural and non- agricultural sectors: Changing perspective
(d) The Changing Structure of Rural labour market
Previous Years
Page 288 CLAT & AILET Papers
2. The author does not say
(a) ln 1991, about 3/5 of the workers in rural India were manual workers
(b) The decade of 1980s was characterised by a relative rise in casual employment as opposed to
regular employment
(c) There is no gender bias among workers in agriculture sector
(d) Wages in agricultural employment have been lesser than those in non-agricultural employment
in the 1990s

3. Why is the increasing gap between labour productivity in agricultural and non-agricultural occupations
a cause of concern' according to Radhakrishnan and Sharma ?
(a) This would increase the wages of agricultural sector
(b) This would read to pressure on both the agricultural and non-agricultural sectors as whichever
sector expands at the expense of the other, there would be increased labour pressure on that
sector and lesser economic development in the other
(c) This would indirectly mean a pressure on agricultural sector in terms of higher wages
(d) This would indirectly mean a pressure on non-agricultural sector in terms of higher wages

4. How does Bhalla explain the fact that the real wages of agricultural labourers began to rise in all
parts of the country after mid-70s ?
(a) Economic development increased for both the sectors across the world
(b) Economic liberalization in India led to the development of the agricultural sector
(c) The agricultural labourers became rich due to successive good crops during the time of Green
Revolution
(d) There was a mass outflow of agricultural labourers into the non-agricultural Sectors and the
remaining labourers. Who were lesser in number, now apportioned the existing agricultural
sector wealth, getting richer individually

5. What is the most important problem in understanding the condition of rural labour markets in India?
(a) The rural labour markets are uneconomic in nature
(b) The rural labour markets are very complex and there exists great diversity across regions and
across segments of such markets
(c) The rural labourers are a reticent lot; not forthcoming with their problems, not very open to
suggestions on how to improve their lifestyle
(d) They are lazy and want doles from the government without undertaking any viable productive
activities

6. What sort of tension exists between the decline of poverty and the slow growth of agricultural
employment and the increased casualisation of the labour force ?
(a) A decline in rural poverty is only possible when there is increased agricultural employment and
lower casualisation of the labour force
(b) The tension exists in the fact that an increased casualisation of the labour force would increase
Poverty
(c) When there is a decline in poverty there should be faster growth of agricultural employment and
the decreased casualisation of the labour force'
(d) Both (b) and (c)

7. What sort of Passage is this ?


(a) Political (b) Social commentary (c) Economic (d) Philosophical

Previous Years
CLAT & AILET Papers Page 289
8. The author does not say which of the following statements in the passage?
(a) The National Commission on Rural Labour gives data on the activities of rural labourers in lndia
(b) Economic development basically means a rise in the share employment in the agricultural
sector at the expense of the secondary and the tertiary sectors
(c) In India, employment was higher among the new entrants to the workforce in the 1980s
(d) Real wages of the agricultural labourers started showing an upward trend from the 1970s

9. Why are changes in the working and living conditions of rural manual workers of utmost significance
to the country as a whole ?
(a) Rural workers migrate a lot to the cities, adding to the already burgeoning population of these
places and so any improvement in their living conditions which would stall this trend would
benefit
(b) The rural workers live in abject poverty and a change in their working and living conditions is
therefore very crucial
(c) They form the bulk of the rural workers and so any change in their living standards augurs well for
the country as a whole
(d) Both (a) and (b)

Directions (Q. 10 - Q. 12): For the word given at the top of each table, match the dictionary definitions on
the left
(a, b, c, d) with their corresponding usage on the right (e, f, g, h). Out of the four possibilities given below
the table select the one that has all the usages correctly matched.

10. Run down

Dictionary Definition Usage


a Trace e My cat was run down by a bus.
b Criticize f Have you run down those addresses
asked for last week?
Suraj is running for president of the
c Hit with a car g
club.
d Champaign for a government position h My father runs everyone down.

(a) ah, bf, ce, dg (b) ah, be, cf, dg (c) ag, bf, ce, dh (d) af, bh,ce, dg

11. Wind up

Dictionary Definition Usage


a Finish e She just think it’s a big wind-up
b To tighten the spring of (a clockwork f He wound up the toy top and set it on
mechanism) floor.
To become nervous, tense, etc; If he doesn’t get his act together, he is
c g
excited going to wind up in prison.
The kids always get wound up to when
d An act or instance of teasing h
uncle Ronnie comes over.

(a) ae, bg, cf, dh (b) ae, bf, cg, dh (c) ag, bf, ch,de (d) ah, bg, cf, de

Previous Years
Page 290 CLAT & AILET Papers
12. Run off

Dictionary Definition Usage


Would you mind running off 20 copies
a Make leave e
of this document for me?
b Reproduce f You shouldn’t swim where the dirty
water runs off into the ocean.
c Leave quickly g Why did you run off after the dinner?
The new government is trying to run
d To flow h
the criminals off.

(a) ah, be, cg, df (b) af, bh,cg, de (c) af, bh,ce, dg (d) ag, be, ch, df

Directions (Q. 13 - Q. 17) : Read the following newspaper report and put the verbs in brackets into the
most suitable form. lndicate your choice in the provided box.

A woman______ (13) ______ (take) to hospital after her car collided with a lorry near Noida yesterday. She
______ (14) ______ (allow) home later after treatment. The road ______ (15) _________ (block) for an hour
after the accident, and traffic had to ______ (16) ______ (divert). A police inspector said afterwards: The
woman was lucky. She could ______ (17) ______ (kiil).

13. (a) is taken (b) took (c) was taken (d) take

14. (a) allow (b) is allowed (c) allowed (d) was allowed

15. (a) was blocked (b) block (c) is blocked (d) be blocked

16. (a) diverted (b) be diverted (c) was diverted (d) is diverted

17. (a) was killed (b) have been killed (c) killed (d) kill

18. In which of the following clusters, all the words mean 'ignoring existence of God' ?
(a) Pantheism, Agnosticism, Secularism (b) Atheism, Agnosticism, Secularism
(c) Deism, Secularism, Agnosticism (d) Atheism, Deism, Secularism

19. Which set of words are only nouns?


(a) Pompous, ridiculous, photographic (b) Penance, science, porous
(c) Analysis, praxis, thesis (d) Poisonous, vocalize, stupidity

20. Which set of words are only adjectives?


(a) Ridiculous, native, psychologise (b) Astronomy, sympathy, privacy
(c) Nation, action, privacy (d) Chemical, mathematical, french

21. In which set each word is a noun, adjective and verb also?
(a) Delegate, defeat, temporary, tertiary (b) Chronic, incumbency, parent, proponent
(c) Topic, alacrity, android, auditory (d) Square, precipitate, collect, free

Previous Years
CLAT & AILET Papers Page 291
Direction (Qs. 22 - 26): Find the odd one out.

22. (a) wary (b) gullible (c) credulous (d) naive

23. (a) gusto (b) verve (c) burst (d) zest

24. (a) beatific (b) diatribe (c) blithe (d) ecstatic

25. (a) trappings (b) orifice (c) egress (d) vent

26. (a) spectre (b) apparition (c) hoodlum (d) phantom

Direction (Qs. 27 - 35): Fill in the blanks with the most appropriate word.

27. You_________mad if you think I'm going to lend you any more money.
(a) should be (b) are supposed to be (c) must be (d) ought to be

28. Who _________was coming to see me this morning?


(a) you said (b) did you say (c) did you say that (d) you did say

29. I notified _________ I had changed my address.


(a) with the Bank (b) the Bank that (c) in the Bank (d) to the Bank that

30. lf I_________a more reliable car, l_________to Surat rather than fly.
(a) had; would drive (b) would have; would drive
(c) had; had driven (d) would have had; would drive

31. I first met Sohan on a beach________Kochi. I later found out that he had been a carpenter and a
dustman,________other things'
(a) by; among (b) near; between (c) by; between (d) near; among

32. 'When did you last hear_________Don?' 'He phoned me just this morning, He's coming to Delhi
next week, so we agreed_________ a time and place to meet.'
(a) from; on (b) about; on (c) from; at (d) of; to

33. Perhaps because something in us instinctively distrusts such displays of natural fluency, some
readers approach John Updike's fiction with_______.
(a) suspicion (b) bewilderment (c) veneration (d) recklessness

34. Despite the mixture's_________nature, we found that by lowering its temperature in the laboratory,
we could dramatically reduce its tendency to vaporize.
(a) volatile (b) resilient (c) insipid (d) acerbic

35. l_________you can swim so well and I can't'


(a) hate (b) hate it that (c) hate that (d) hate it

Previous Years
Page 292 CLAT & AILET Papers
SECTION - B : GENERAL KNOWLEDGE

36. Which folk dance India has made it to the Guinness Book of World Records as the largest folk
dance in the world ?
(a) Bihu (Assam) (b) Garba (Gujarat)
(c) Nati (Himachal Pradesh) (d) Bhangra (Punjab)

37. Which e-commerce company has made its maiden entry in the Thomson Reuters top 100 Global
Innovators list 2015 ?
(a) Amazon (b) Snapdeal (c) ebay (d) Alibaba

38. Which of the following groups of international institutions are also known as 'Bretton Woods Sisters'?
(a) United Nations and Bank of International Settlement
(b) IMF and World Bank Group
(c) IMF and WTO
(d) IMF, IBRD and International Trade Organisation

39. Which of the following variables is not part of the 'lmpossible Trinity' conundrum ?
(a) Capital Control (b) Exchange Rate (c) Fiscal Policy (d) Monetary Policy

40. The measurement of poverty-line in lndia is based on the criteria of


(a) Nature of employment (b) Dwelling houses
(c) Level of education (d) Calorie consumption

41. Who termed Paris deal on climate change as 'Climate Justice' ?


(a) Ban-Ki-Moon (b) Barack Obama (c) Narendra Modi (d) Francois Hollande

42. What is R2 - D2 ?
(a) lt is an astromech droid that extinguishes fires' repairs spaceships and assists the lead characters
in the Star Wars film
(b) lt is a new atomic particle discovered
(c) lt is vaccine against Ebola
(d) lt is a video game

43. Tunisian mediators of the so called National Dialogue Quartet won the Nobel Peace Prize 20l5.
Which of the following is/are part of Quartet?
I. The Tunisian General Labour Union (UGTT)
II. The Tunisian Confederation of Industry, Trade and Handicrafts (UTlCA)
III. The Tunisian Human Rights League (LTDH)
IV. The Tunisian Order of Lawyers

The correct answer is


(a) l and ll (b) ll and lll (c) l, ll and lV (d) l, ll, lll and lV

44. The Employee's Provident Fund organisation has won the National Award on e-governance 2015-16
for launching
(a) Unique ldentification Number (b) Universal Account Number
(c) Online Pension scheme (d) Online complaint portal

Previous Years
CLAT & AILET Papers Page 293
45. Which is the World,s first airport to fully run on solar power ?
(a) Vancouver (US) (b) Dubai (UAE) (c) Kochi (lndia) (d) Zurich (Switzerland)

46. Government has introduced Start-up Scheme to promote new enterprises' Such enterprises has
been awarded_________years tax holiday.
(a) 1 (b) 2 (c) 3 (d) 5

47. Consider the following statements in respect of PAN (effective from January 1,2016)
I. Quoting PAN is mandatory for cash payments made to settle hotel bills or for buying foreign
travel tickets of an amount above Rs 50,000
II. The PAN requirement for non-luxury cash transactions is Rs 2 lakh
III. PAN for making post office deposit of over Rs 50,000 has been dispensed with.
lV. PAN is mandatory on purchase of immovable property of Rs 10 lakh
The correct answer is
(a) l and ll (b) l, ll and lV (c) ll, lll and lV (d) l, ll, lll and lV

48. How many firms, recently, have got licence from RBI for establishing Payment Banks ?
(a) 10 (b) 11 (c) 12 (d) 13

49. Government has allowed_________FDI in operations of white level ATMs.


(a) 100% (b) 74% (c) 51% (d) 4e%

50. Which of the following pairs of rivers have been interlinked formally for the first time in lndia ?
(a) Betwa - Ken (b) Son - Ken (c) Ganga - Jhelum (d) Godavari - Krishna

51. Which of the following feature films from India has been declared as the official entry for 88th Oscar
Awards ?
(a) Queen (b) Court (c) Mary Kom (d) Kaaka Muttai

52. Why the sidereal day is 4 minutes shorter than the solar day ?
(a) The Earth is round in shape. (b) The Earth revolves around the Sun'
(c) The Earth rotates on its axis. (d) The Earth has an atmosphere'

53. Which is the only industrialised country of the world that did not ratify the Kyoto Protocol?
(a) Russia (b) United States (c) Japan (d) France

54. The 8th BRICS Summit (2016) is proposed to be held in


(a) Brazil (b) lndia (c) China (d) South Africa

55. Which of the following statements is incorrect about foreign investment in India ?
(a) Foreign Portfolio lnvestment (FPls) can invest up to 5.1% in multi-brand retail
(b) FPl up to 49% through automatic route is permissible across sectors
(c) 49% FPI in Brownfield pharma projects is allowed
(d) Foreign retailers are not able to have direct management control of an lndian venture

56. An lndian car maker recently changed the name of its newly launched car because the name was
similar to an epidemic
(a) Tata Motors (b) Maruti Suzuki
(c) Mahindra & Mahindra (d) Honda Motors

Previous Years
Page 294 CLAT & AILET Papers
57. The first telecommunication company to launch pan lndia 4G mobile services
(a) BSNL (b) Vodafone (c) Reliance Jio (d) Bharti Airtel

58. WTO has recently approved membership of which country ?


(a) Zimbabwe (b) Sweden (c) Afghanistan (d) Sri Lanka

59. FM broadcast band lies in_________.


(a) HF (b) SHF (c) UHF (d) VHF

60. What percentage of Employees Provident Fund can be invested in equity shares?
(a) 0% (b) 5%
(c) 10% (d) Minimum 5% and maximum 15%

61. Liquid crystals are used In


(a) Pocket Calculator (b) Display Devices (c) Wrist watches (d) All of the above

62. Which country recently allowed women to vote in local elections for the first time ?
(a) lran (b) lraq (c) lndonesia (d) Saudi Arabia

63. 'Brass Plate Subsidiaries' are usually set up


(a) For the purpose of Income tax avoidance (b) In the metal sector
(c) To take advantage of subsidies (d) As cross border investment

64. Global fast-food chain_________has maximum number of restaurants across the world.
(a) KFC (b) Subway (c) McDonalds (d) Starbucks

65. Who has been honoured by the Rajiv Gandhi Khel Ratna Award 2015 ?
(a) Rohit Sharma (b) Jitu Rai (c) Sania Mirza (d) Deepika Pallikal

66. Which of the following countries has recently declared a 'state of economic emergency' ?
(a) Bangladesh (b) Germany (c) Burundi (d) France

67. Section 80 TTA of the Income Tax Act pertains to interest earned on
(a) Saving Bank Accounts (b) Fixed Deposits
(c) Recurring deposit (d) Government saving schemes

68. Who is the chairperson of the Film and Television Institute of India (FTII) ?
(a) Suneel Darshan (b) Pankaj Dheer
(c) Gajendra Chauhan (d) Prashant Pathrabe

69. MNREGA program provides 100 days employment in rural areas. The Government during September
2015 decided to provide additional_________days employment in drought affected areas'
(a) 20 (b) 25 (c) 40 (d) 50

70. United Nations has appointed Anupam Kher as the ambassador of


(a) 'Let the Girl Live' campaign (b) 'He for She' campaign
(c) 'Share Humanity' campaign (d) 'Youth Now' campaign

Previous Years
CLAT & AILET Papers Page 295
SECTION - C : LEGAL APTITUDE

Directions (Qs. 71 - 82): Given below is a statement of legal principle followed by a factual situation. Apply
the principle to the facts given below and select the most appropriate answer.

71. LEGAL PRINCIPLE : A reasonable classification having nexus with the object sought to be achieved
is not violative of Article 14 or Article 16 of the Constitution of lndia.

FACTUAL SlTUATlON : 'X' is a male teacher in a Women's college, who applied for the post of
principal of that College. His candidature was rejected on the basis of the Government's policy of
appointing only women as Principal of a women's college. 'X' challenges the policy on the ground of
discrimination. Whether the challenge is sustainable?

DECISION :
(a) Yes, because rejection of X's candidature amounts to sexual discrimination and deprivation of
opportunity.
(b) No, the rejection does not amount to discrimination since it is a reasonable classification
permissible under the Constitution.
(c) No, because the policy of appointment of only lady Principal in a women's college is a reasonable
classification having a nexus with the object sought to be achieved.
(d) Yes, because the policy is violative of the guarantee of equality before law under Article 14 of the
Constitution.

72. LEGAL PRINCIPLE : Any institution or body can be a 'State' if it is created under the Constitution
or a statute; or if it is substantially financed by the Government; or the Government holds its share
capital.

FACTUAL SITUATION : K approached the High court by filing a writ petition against the Board for
Control of Cricket in lndia (BCCI). The argument advanced was that BCCI is a 'State' within the
meaning of Article 12 of the constitution of lndia. The question is whether the argument is acceptable?

DECISION :
(a) Yes, because the Board has monopoly on cricket in India.
(b) No, because the monopoly on cricket is neither State conferred nor State protected.
(c) No, because the control of the government on BCCI, if any, is only regulatory.
(d) No, because neither the Board is created under a statute nor any part of share capital of the
Board is held by the government and no financial assistance is given by the government to the
Board.

73. LEGAL PRINCIPLE : A suit shall be instituted in the court within whose jurisdiction the cause of
action arises; or the defendant actually and voluntarily resides or carries on business, or personally
works for gain.

FACTUAL SITUATION : 'Y' carries on business in Mumbai. 'Z' carries on business in Delhi. 'Z' buys
goods of 'Y' in Mumbai through his agent and request 'Y' to deliver them at Delhi. Accordingly, 'Y'
delivered the goods at Delhi. But he did not get the price of the goods delivered in Delhi. Therefore,
he intends to move the Civil Court for recovery of amount from 'Z'. Which court may 'Y' approach ?

Previous Years
Page 296 CLAT & AILET Papers
DECISION :
(a) 'Y' may institute the suit either at Delhi where Z carries on business or at Mumbai where the
cause of action arose.
(b) 'Y' may institute the suit at Delhi where 'Z' carries on business.
(c) 'Y' may institute the suit simultaneously at Delhi where 'Z' caries on business and at Mumbai
where the cause of action arose.
(d) 'Y' may institute the suit at Mumbai where the cause of action arose.

74. LEGAL PRINCIPLE : The acceptance of an offer will be valid only if it is made in the way it was
expected to be made.

FACTUAL SITUATION : There was a telephonic discussion between 'J' and 'K' for negotiating the
sale of the shop of former to the latter. Upon reaching an agreement as to the price of the shop of 'J'
at Rs 20 lakh, 'J' told 'K' to send a letter to him within two weeks confirming that she wishes to buy
the shop for the price finalized. Two days thereafter, 'K' gave her acceptance to 'J' over telephone but
sent the letter of confirmation after lapse of one month. ls 'J' bound by acceptance of 'K' ?

DECISION :
(a) Yes, because the acceptance was conveyed within two weeks over telephone and it was followed
by a letter of acceptance as stipulated.
(b) No, because although the acceptance over telephone was conveyed in time but not in the mode
specified and the letter of acceptance was also not sent within two weeks.
(c) No, because sale of immovable property cannot be finalized online; neither any acceptance can
be given over phone. Hence, the entire negotiation is invalid.
(d) Yes, because no law can compel the purchaser to give his acceptance through the mode
prescribed by the vendor.

75. LEGAL PRINCIPLE : A power conferred by a statute cannot be withdrawn by a subordinate legislation.

FACTUAL SITUATION : The Cinematograph Act conferred powers upon the District Magistrate
(DM) to grant license subject to the control of the government. The government framed Rules under
the said Act. The effect of these Rules was that the licensing power stood transferred to the Government
itself and the District Magistrate was rendered powerless. Whether such Rules are valid ?

DECISION :
(a) The licensing power was granted by the Cinematograph Act. Any withdrawal or transfer thereof
was possible only through an Amending Act and not by any Rules made under the Parent Act.
(b) Although the legislature has conferred power upon the DM to grant license but the government
being the implementing agency might find it unfeasible. Therefore, the government rightly withdrew
it from the DM.
(c) The Rules are valid since these are framed under the Parent Act in order to better implement it.
(d) The Rules are valid since the DM under the Parent Act was not independent but subject to the
control of Government'

76. LEGAL PRINCIPLE : Clause (1) of Article 15 of the Constitution of lndia prohibits the State from
discriminating between citizens on the ground only of religion, race, caste, sex, place of birth or any
of them.

Previous Years
CLAT & AILET Papers Page 297
FACTUAL SITUATION : The admission Rules of an Engineering College located in XYZ State of
India provided that no capitation fee shall be charged from the residents of the XYZ State but the
non-residents shall be required to pay capitation fee. Whether the Rules are violative of Article 15 (1)
of the Constitution ?

DECISION :
(a) yes, because Article 15 (1) prohibits discrimination between citizens on the ground only of
religion, race, caste, Sex, place of birth or any of them.
(b) yes, because Article 15(1) prohibits discrimination on the basis of place of birth which impliedly
includes place of residence.
(c) yes, because Article 15 (1) prohibits discrimination between citizens on the ground only of
religion, race, caste, sex, place of birth and the provision suffers from causus omissus and
"place of residence" is inadvertently omitted.
(d) No, because Article 15 (1) does not prohibit discrimination based on the place of residence.

77. LEGAL PRINCIPLE : ln the employer - employee relationship, the employer is held liable for all the
wrongs committed by his employees in the course of employment.

FACTUAL SITUATION : David was employed as a Driver in ABC & Co over the past 15 years and
has been appreciated by the General Manager for his hard work and sincerity. He has been rewarded
by the company for his accident free record. David's younger brother wanted to join the same
company as a driver. He obtained a Learner's Licence, joined a Driving School and was learning
driving during the last three months. He was on the verge of completion of the training and appear for
the Driving test. He wanted to have more practice before the test and requested his brother David for
using the Company's car for two days. David also allowed him to use the office car for the practice.
While he was practising driving, a truck came from the wrong side, hit the company's car driven by
David's brother, which in turn hit a pedestrian and injured him. The pedestrian sues the company for
damages.

DECISION :
(a) The Company is not liable as it was driven by David's brother
(b) The Company is liable as David allowed his brother to drive the car
(c) David's brother is personally liable
(d) The Company can shift the responsibility on to the truck driver

78. LEGAL PRINCIPLE : No person shall be deprived of his life or personal liberty except according to
procedure established by law and Civil Courts have coercive powers to compel attendance of witness
only within its local territory.

FACTUAL SITUATION : Puchu, a resident of Faridabad was summoned by the Delhi High Court as
a witness in a civil case regarding wrongful possession of immovable property filed by Amu against
Kichu. He refused to appear before the court due to his office job. He was prosecuted by the court.
ls he liable ?

DECISION :
(a) He is not liable because he is not the resident of Delhi.
(b) He is not liable because he has fundamental right under Article 21 of personal liberty.
(c) He is liable because he is called as a witness in a civil trial and it is a procedure established by
law.
(d) He is not liable because he has no interest in the suit property.

Previous Years
Page 298 CLAT & AILET Papers
79. LEGAL PRINCIPLE : Article 19(1) (g) of the Constitution of India guarantees to all citizens the right
to practice any profession, or to carry on any trade, occupation and business but Article 19 (6)
empowers the State to impose reasonable restrictions on this right in the interest of public.
FACTUAL SITUATION : Having experienced acute shortage of labour for agricultural purpose due to
engagement of agricultural labourer in manufacture of Bidis, the state Government enacted a law to
prohibit such engagement of agricultural labour in the manufacture of Bidis. Whether the law violates
the constitutional provisions ?
DECISION :
(a) No, because the law in a reasonable restriction in the interest of public as if labourers would not
be available for agricultural purposes there can be shortage of food grains and wastage of crops.
(b) No, because Bidis are harmful for health of people so any law preventing people from engaging
in manufacture of Bidis is in the interest of public.
(c) Yes, because the law imposes an unreasonable restriction as it indirectly makes the two
sectors (manufacture of Bidis and agriculture) alternative options for the labourers where as
some people would like to work in both of these.
(d) Yes, because the object sought to be achieved by this law is to keep sufficient labour supply for
agricultural purpose, which could have been easily achieved by restraining the employment of
agricultural labour in Bidi manufacturing during the agricultural season only. Absolute restriction
amounts to withdrawal of the right. Hence, the law is unconstitutional.

80. LEGAL PRINCIPLE : When a person unlawfully intervenes in the chattel of another person by which
the latter is deprived of its use, the former commits the tort of conversion. And nobody shall enrich
himself at other's cost.
FACTUAL SITUATION : X, a patient suffering from fibroids in her uterus approached KLM Medical
Institute. X was suggested to undergo surgery to remove the fibroids from her uterus. The operation
was successfully performed and X was discharged after few days. one of the researchers of the
KLM lnstitute discovered some rare and unique cells in the fibroids of x and using these cells, the
laboratory of KLM developed some life-saving drugs and earned rupees twenty crore from a leading
lnternational Pharma Company. When X came to know about it, she claimed five crore from the
Institute.
DECISION :
(a) KLM lnstitute need not share its income with X because X far from being deprived of the use of
her fibroids was actually benefitted by its removal.
(b) KLM lnstitute need not share its income with X because the medical institute instead of destroying
the waste fibroids of X conducted research on its own and invented new life-saving drugs.
(c) KLM lnstitute must share its income with X because KLM could not have achieved its success
without the fibroids of X.
(d) KLM lnstitute must share its income with X on moral grounds.

81. LEGAL PRINCIPLE: A judgment which binds only the parties to a suit in which the judgment was
passed is called judgment in personam; whereas a judgment which binds all men irrespective of
whether they were party to suit or not is known as judgment in rem.
FACTUAL SITUATION : "Judgment of a competent court determining contractual obligations of the
parties to a contract is an example of judgment in personam; but a judgment of a competent court
declaring a party to be insolvent is an example of judgment in rem." Comment on the correctness of
this statement.

Previous Years
CLAT & AILET Papers Page 299
DECISION :
(a) The statement is incorrect because a judgment relating to contract is a judgment in rem as it
binds both the parties to the suit as well as the strangers. But a judgment relating to insolvency
applies only to the person who has been adjudged to be an insolvent; hence it is a judgment in
personam.
(b) The Statement is wrong as both the judgments are judgments in rem as both bind not only the
parties to the suit but also others.
(c) The Statement is wrong as both the judgments are judgments in personam as both the judgments
bind not only the parties to the suit but not the others.
(d) The statement is correct.

82. LEGAL PRINCIPLE : According to Article 20 (1) of the Constitution, no person shall be convicted of
any offence except for violation of the law in force at the time of the commission of the act charged
as an offence, nor be subjected to a penalty greater than that which might have been inflicted under
the law in force at the time of the commission of the offence.

FACTUAL SITUATION : 'P' was charged with an offence punishable with imprisonment for a term of
one year. The Magistrate convicted him and awarded him a punishment of one year imprisonment.
While 'P' was undergoing the sentence, the law under which 'P' was convicted came to be amended
and the punishment for the offence of which 'P' was convicted was reduced to six months. The
defense filed an application to the Magistrate for review of sentence and to commute it to six
months. Can the application be allowed ?

DECISION :
(a) No, because penal laws only have prospective application.
(b) No, because a penal statute cannot be given retrospective effect.
(c) No, since at the time of coming into force of the amended law, 'P' was already suffering the
sentence and had not completed the full term. Hence, his case should not be dealt under the
new law.
(d) Yes, because retrospective application of criminal law if it is beneficial to the accused is not
against Article 20 (1) of the Constitution.

Directions (Qs. 83 - 89) : The following questions consist of two statements, one labelled as, Assertion'
and the other as 'Reason'. Read both the statements carefully and answer using the codes given below.

(a) Both A and R are true and R is the correct explanation of A


(b) Both A and R are true but R is not the true explanation of A
(c) A is true but R is false
(d) A is false but R is true

83. Assertion (A) : The entries in the three legislative lists are not always set out with scientific
precision.
Reason (R) : The entries are not powers but are only fields of legislation.

84. Assertion (A) : No action lies for mere damage caused by some act which does not violate a
legal right.
Reason (R) : An action lies for interference with another's legal right even where it causes no
actual damage.

Previous Years
Page 300 CLAT & AILET Papers
85. Assertion (A) : The parties to the contract must be competent to contract othenrvise it will be a
void contract.
Reason (R) : All wagering agreements are void.

86. Assertion (A) : Custom to have force of law must be followed from time immemorial.
Reason (R) : Custom represents common consciousness of the people.

87. Assertion (A) : An accused person cannot be forced to give his thumb impression.
Reason (R) : An accused person cannot be compelled to be a witness against himself.

88. Assertion (A) : ln federalism, there is division of powers between the centre and the States.
Reason (R) : The legislation is not invalid merely because it incidentally encroaches upon matters
which have been assigned to another organ.

89. Assertion (A) : lnternational law is not law at all.


Reason (R) : International law has no judicial system to enforce the law by applying sanctions.

90. The constitution (One hundredth Amendment) Act, 2015 amended the________ Schedule of the
Constitution to give effect to an agreement entered into by India and_________on acquiring and
transferring of territories between the two countries.
(a) Ninth; Pakistan (b) Second; Pakistan (c) First; Bangladesh (d) First; China

91. Who is the CEO of NITI Aayog ?


(a) Sindhushree Khullar (b) Arvind Panagariya
(c) Bikesh Debroy (d) Amitabh Kant

92. Allahabad High Court has held that the___________of deceased Government employees are eligible
for appointment on compassionate ground.
(a) wife (b) daughter (c) mother (d) married daughter

93. Which party has won the general elections of Spain in spite of its worst result since 1989 ?
(a) People's Party (b) Pedro Sanchez's Socialist party
(c) Popular Union (d) Ciudadanos Party

94. Pension Fund Regulatory and Development Authority has started using________ for online registration
under the National pension system scheme.
(a) PAN Card (b) passport (c) Aadhar Card (d) Driving License

95. When a vehicle is financed by a bank, what kind of charge does the bank have over the primary
security ?
(a) Pledge (b) Hypothecation (c) Assignment (d) Lien

96. Under which Act can action be taken against wilful defaulters of banking loans ?
(a) Under Section 420 of IPC
(b) SEBI Act
(c) Banking Regulation Act
(d) Securitisation and Reconstruction of Financial Assets and Enforcement of Security Interest Act
(SARFAESI Act)

Previous Years
CLAT & AILET Papers Page 301
97. Which of the following court / tribunal ordered levy of an environment compensatory charge on
commercial vehicles not bound for the capital yet passing through Delhi ?
(a) Supreme Court of India (b) Delhi High Court
(c) National Green Tribunal (d) None of these

98. Parliament has enacted which of the following Legislation in 2015 - 16 ?


I. The Juvenile Justice (Care and Protection of Children) Act
II. The Election Laws (Amendment) Act
III. The Scheduled Caste and Scheduled Tribes (Prevention of Atrocities) Amendment Act
IV. Commercial Courts, Commercial Division and Commercial Appellate Division of High Courts Act

(a) l and lll (b) lV only (c) l and lV (d) l, ll, lll and lV

99. ln the following Sates more than 60% seats are in reserved category in educational institutions
against 50% cap as fixed by the Supreme Court of India
I. Karnataka II. Tamil Nadu III. Odisha IV. Bihar
The correct answer is :
(a) only ll (b) l and ll (c) ll and lV (d) I' ll and lll

100. According to SEBI norms, a person found guilty of indulging in unfair trade practices shall be liable
to a penalty of
I. Rs. 25 crore
II. Three times the amount of profits made out of such practices,.....

The correct answer is :


(a) only l (b) only Il (c) either I or Il (d) neither I nor Il

101. The minimum wages in_________are the highest in the northern region.
(a) Rajasthan (b) Dethi (c) Punjab (d) Haryana

102. The correct sequence in ascending order of their creation of the following international institution is:
I. WTO II. GATT III. UNCTAD IV. NAFTA

(a) ll, lll, l, lV (b) lll, ll, l, lV (c) lV, ll, l, lll (d) l, ll, lll, lV

103. Government of India has decided to establish_________in selected High Courts.


(a) Tax Division (b) SC/ST Division
(c) Commercial Division (d) Economic Offence Division

104. 'Zero Rating' is a recent term used in


(a) lnsurance (b) Credit Rating (c) Energy Efficiency (d) Net Neutrality

105. Which Article of the Constitution of India was used to impose President Rule in Uttarakhand and
placing the Assembly under suspended animation in March 2016 ?
(a) Article 102 (b) Articte 143 (c) Article 356 (d) Arricle 365

Previous Years
Page 302 CLAT & AILET Papers
SECTION - D : REASONING

Directions (Qs. 106 - 109) : A passage is given below followed by several inferences. You have to examine
each inference separately in the context of the passage and decide upon its degree of truth or falsity. Mark
your answer as:
(a) lf the inference is 'definitely true'
(b) lf the inference is 'probably true'
(c) lf the 'data provide in inadequate'
(d) lf the inference is 'probably false'
Investors today have more investment options than were available just a few years ago. Choice in any
decision-making is good in so far it provides variety, differentiation and benchmarking. lt could also, however,
at times lead to clutter and "noise" if the options are mostly similar and undifferentiated. To make sense of
this choice conundrum, it is imperative for an investor to define objective - both returns and digestible risk
and then identify the possible options. The investor also needs to select the mix and regularly monitor that
objectives and investment outcomes remain aligned. Sounds simple, but can present the most confounding
situation which multiplies with the quantum of wealth.
106. lnvestors need to critically evaluate the risk of each investment option.

107. Present day investors need to use their judgement more critically before investing.

108. Multiple investment options of similar types helps in making better investment decisions.

109. In the past, investors were generally guided by the fund managers.

Directions (Qs. 110 -115): Read the following short passages and answer the questions that follow each
passage.

110. For some women, the cost of giving birth can be an unexpectedly a large burden. The average
normal birth now costs Rs. 3,200 and a birth with complications can cost thousands of rupees
more. Of women in the primary child-bearing age range of eighteen to twenty-four, who account for
about 40 percent of all births in this country annually, more than 25 percent have no health care
insurance to pay maternity costs.
lf the above statements are true, which one of the following must also be true ?
(a) Each year, about 75 percent of all births in this country are to women who have health care
coverage of maternity costs.
(b) Each year, about 60 percent of all births in this country are to women who are younger than
eighteen or older than twenty-four.
(c) For an average birth, health care insurance pays about 75 percent of Rs. 3,200.
(d) In this country, about 75 percent of the women who do not have health care coverage of maternity
costs are younger than eighteen or older than twenty-four.

111. Products sold under a brand name used to command premium prices because, in general, they
were superior to non-brand rival products. Technical expertise in product development has become
so widespread, however, that special quality advantages are very hard to obtain these days and
even harder to maintain. As a consequence, brand-name products generally neither offer higher
quality nor sell at higher prices' Paradoxically, brand names are a bigger marketing advantage than
ever.

Previous Years
CLAT & AILET Papers Page 303
Which of the following, if true, most helps to resolve the paradox outlined above?
(a) Brand names are taken by consumers as a guarantee of getting a product as good as the best
rival product.
(b) Consumers recognised that the quality of products sold under invariant brand names can drift
over time
(c) In the acquisition of one corporation by another, the acquiring corporation is interested more in
acquiring the right to use certain brand names than in acquiring existing production facilities.
(d) In the earlier days when special quality advantages were easier to obtain than are now, it was
also easier to get new brand names established.

112. The extent to which a society is really free can be gauged by its attitude towards artistic expression.
Freedom of expression can easily be violated in even most outwardly democratic of societies.
When a government's Arts Council withholds funding from a dance performance that its members
deem "obscene", the voice of a few bureaucrats have in fact censored the work of the choreographer,
thereby committing the real obscenity of repression.

Which of the following, if true, would most seriously weaken the argument above?
(a) Members of government Arts Council are screened to ensure that their beliefs reflect those of
the majority,
(b) The term obscenity has several different definitions that should not be used interchangeably, for
rhetorical effect.
(c) Withholding financial support for a performance is not the same as actively preventing or inhibiting
it.
(d) The Council's decision could be reversed if the performance were altered to conform to public
standards of appropriateness.

113. lt is not generally realised that when a court upholds or invalidates legislation or executive action, it
neither approves nor condemns any legislative policy, nor is if concerned with the wisdom or expediency
of the administrative action. lt merely determines whether the legislation or executive action is in
conformity with or contrary to the provisions of the Constitution. lt discharges the function of guarding
the Constitution, no more, no less. Judicial activism is to be properly understood in the context of
the extent and the vigour and the readiness with which the courts exercise their power of judicial
review. When courts actively perform an interventionist role, we witness the phenomenon of judicial
activism. When the judiciary exercises self-restraint in exercising the power of judicial review and
limits its role, there is absence of judicial activism. But the pendulum of judicial review is never static
and judicial activism, or lack of it, is a variable phenomenon.

Which one of the following, if true, would defeat the case presented above ?
(a) The judiciary is plagued with the Public lnterest Litigations against some or the other forms of
administrative action.
(b) Often, the judiciary is seen to be making disparaging remarks about the executive, pulling up
officials for lapses in conduct, many a time condemning them.
(c) The judicial review exercises that the judiciary often undertakes is what comprises judicial
activism.
(d) Judicial activism is not mandatory under the Constitution. lt is for the judiciary to decide whether
it wants to intervene in a certain issue or not.

Previous Years
Page 304 CLAT & AILET Papers
114. Traditionally, decision-making by managers that is reasoned and step-by-step has been considered
preferable to intuitive decision-making. However, a recent study found that top managers used
intuition significantly more than did most middle or lower-level managers. This confirms the alternative
view that intuition is actually more effective than careful, methodical reasoning.

The conclusion above is based on which of the following assumptions ?


(a) Methodical, step-by-step reasoning is inappropriate for making many real-life management
decisions.
(b) Top managers have the ability to use either intuitive reasoning or methodical, step-by-step
reasoning in making decisions.
(c) The decisions made by middle and lower-level managers can be made as easily by using
methodical reasoning as by using intuitive reasoning.
(d) Top managers are more effective at decision-making than middle or lower-level managers.

115. Banker: By transferring income to a retirement account at our bank, people can save money by
delaying payment of taxes.
Accountant: That plan won't actually save money because the taxes will have to be paid sometime
in the future when the money is withdrawn.
Which one of the following best explains the conflict between the Banker and the Accountant ?
(a) The Banker is primarily concerned with recruiting new customers for the bank but the Accountant
is not.
(b) The Accountant misunderstands the application of the tax laws.
(c) The Banker and the Accountant disagree on the application of the term "save".
(d) Retirement accounts are nothing more than a tax shelter, which the Parliament intends to cut
out of next year's tax amendment.

Direction (Qs. 116 - 120) : Read the following information carefully and answer the questions given below.
i. Five friends A, B, C, D and E travelled to five different cities of Chennai, Kolkata, Delhi, Bengaluru
and Hyderabad by different modes of transport viz. bus, train, aeroplane, car and boat from Mumbai.
ii. The person who travelled to Delhi did not travel by boat.
iii. C went to Bengaluru by car and B went to Kolkata by aeroplane.
iv. D travelled by boat whereas E travelled by train.
v. Mumbai is not connected by bus to Delhi and Chennai.

116. Which of the following combinations of person and mode is not correct ?
(a) A - Bus (b) D - Boat (c) C - Car (d) E - Aeroplane

117. Which of the following combination is true for A ?


(a) Delhi - Bus (b) Chennai - Bus (c) Hyderabad - Bus (d) Hyderabad - Car

118. Which of the following combinations of place and mode is not correct ?
(a) Hyderabad - Train (b) Hyderabad - Bus (c) Chennai - Boat (d) Delhi - Train

119. The person travelling to Hyderabad went by which of the following modes ?
(a) Train (b) Bus (c) Boat (d) None of these

120. Who among the following travelled to Delhi ?


(a) D (b) A (c) E (d) None of these

Previous Years
CLAT & AILET Papers Page 305
Direction (Qs. 121 - 125) : Read the following information carefully and answer the questions given below.
All the streets of a city are either perpendicular or parallel to one another. The streets are all straight.
Streets N, O, P, Q and R are parallel to one another. Streets S, T, U, V, W, X and Y are horizontally parallel
to one another,
i. Street N is 1 km east of Street O.
ii. Street O is % km west of Street p.
iii. Street Q is 1 km west of Street R.
iv. Street S is 1/2 km south of Street T.
v. Street U is 1 km north of Street V.
vi. Street W is 1/2 km north of Street X.
vii. Street W is 1 km south of Street Y.
121. lf W is parallel to U and W is 1/2 km south of V and 1 km north of T, then which two streets would
be 1 & 1/2 km apart ?
(a) U and W (b) V and S (c) V and T (d) W and V

122. Which of the following possibilities would make two streets coincide ?
(a) X is 1/2 km north of U (b) P is 1 km west of E
(c) O is 1/2 km east of N (d) R is 1/2 km east of O

123. Street R is between O and P, then the distance between p and e is


(a) 1/2 km (b) 1 km (c) 1.5 km (d) 1.25 km

124. R is between O and P, then which of the following is false ?


(a) Q is 1.75 km west of N (b) P is less than 1 km km from Q
(c) R is less than 1 km km from N (d) Q is less than 1 km km from O

125. Which of the following is necessarily true ?


(a) R and O intersect (b) Q is 2 km west of O
(c) Y is 1.5 km north of X (d) O is at least 2 km west of N

Direction (Qs. 126-129) : Study the following information carefully and answer the questions given below.
A family consists of six members H, l, J, K, L and M. There are two married couples. I is a doctor and father
of L. M is grandfather of J and is a contractor. K is grandmother of L and is a housewife. There is one doctor,
one contractor, one nurse, one housewife and two students in the family.
126. What is the profession of H ?
(a) Doctor (b) Student (c) Doctor or Nurse (d) Nurse

127. Who is the husband of H ?


(a) l (b) M . (c) J (d) None of these

128. Which of the following are two married couples ?


(a) LK, JM (b) MK, lH (c) MK, JH (d) MK, IL

129. Who is the sister of L ?


(a) J (b) H (c) K (d) Information insufficient

130. lf Saturday falls four days after today, which is 6th January, on what day did the 1st of December of
previous year fall ?
(a) Sunday (b) Monday (c) Tuesday (d) Wednesday

Previous Years
Page 306 CLAT & AILET Papers
131. At what angle the hands of a clock are inclined when it is half past 8 ?
(a) 70° (b) 80° (c) 75° (d) 60°

Direction (Qs. 132 - 135): Find the missing numbers/letters in following series.
132. 3, 20, 63, 144, 275, ?
(a) 468 (b) 461 (c) 467 (d) 469

133. 113, ?,164, 215, 283, 368


(a) 132 (b) 130 (c) 129 (d) 128

134. AYBZC, DWEXF, GUHVI, ?, MQNRO


(a) LSJTL (b) JSKLT (c) JSKTL (d) LTSKY

135. 21 A, X2D, V6G, T21J, R88M, ?


(a) P440P (b) N2670S (c) N2676S (d) P445P

Directions (Qs. 136 -140): A fact situation and result is presented. Numbered statements follow the result.
Each statement has to be separately evaluated in relation to the fact-situation and result' Evaluate these
statements with the following sequences of decisions in the order of a, b, c and d. The first of these that
you cannot eliminate is the correct answer.
(a) lf the statement is inconsistent with, or contradicts, the fact situation, the result, or both together. lf
so Choose (a).
(b) lf the statement present a possible adequate explanation of the result.
(c) lf the statement is deducible from something in the fact-situation, or the result, or both together.
(d) lf the statement either supports or weakens a possible explanation of the result.
Situation : Major X, an able officer in the Kapistan Army, failed to receive a promotion for eight years.
Then he had been reassigned to a military supply depot in Khawalpindi, despite his university
training in engineering and electronics, and his remarkably high performance ratings from his
commanding officers. X had never been an active member of any political party as a youth nor
as an adult, yet neither had he given the party or his superiors any cause to doubt his absolute
loyalty to Kapistan. X's brother-in-law had been a diplomat in the Kapistan government until
his death in 1971 in a plane crash on Koviet soil while he was working in the Kapistan Embassy
in Kosco. X had always assumed that the mishap was indeed an accident, until his friend Y,
a middle-level officer in the lnterior Ministry, broadly hinted that the plane crash had been an
act of sabotage. Soon after the talk with Y, X visited his sister, the diplomat's widow, in her
Kosco apartment. During the visit she asked X several questions that struck him as strange
and inappropriate. As he was leaving her apartment, she asked X to wear her late husband's
scarf and to return to his hotel by way of a certain park. Bewildered, but not wishing to offend
his sister, X obeyed her odd instructions.
Result: Two months later, X received a promotion and was made Commander of the Kapistan missile
division in Kahore.
136. X's sister had arranged for her husband's fatal accident.

137. X's sister was an operative in the Kapistan Secret Police, and her husband had been acting as a spy
before his death.

138. X's promotion and reassignment to the Kosco Army Headquarters came as a result of Y's
recommendations.
Previous Years
CLAT & AILET Papers Page 307
139. For X, a missile base is a more highly prized assignment than a supply depot.

140. X's failure to win a promotion for eight years was the result of the clerical error in his records.

SECTION - E : MATHEMATICS

141. A die is rolled twice. What is the probability that sum of the numbers on the two faces is 5 ?
(a) 5/12 (b) 1/9 (c) 1/6 (d) 5/36

142. Twenty five workers were employed to complete a compound wall in 12 days. Five workers left after
working for 4 days. The remaining 20 workers completed the work. In how many days the total job
was completed ?
(a) 15 days (b) 16 days (c) 14 days (d) 18 days

143. In a garden, there are B rows and 10 columns of papaya trees. The distance between the two trees
is 2 metres and a distance of one metre is left from all sides of the boundary of the garden. The
length of the garden is
(a) 24 metres (b) 14 metres (c) 20 metres (d) 18 metres

144. A man can row 14 km/h in still water. ln the stream flowing with the speed of 10 km/h he takes 4
hours to move with the stream and come back. Find the distance he rowed the boat.
(a) 11.71 km (b) 13.71 km (c) 14.71 km (d) 12.71 km

145. What will be the difference in simple and compound interest on Rs. 2, 000 after three years at the
rate of 10 percent per annum ?
(a) Rs. 60 (b) Rs. 42 (c) Rs. 62 (d) Rs. 40

146. A man covers a certain distance between his house and office on a Scooter. Having an average
speed of 30 km/hr, he reaches office late by 10 minutes. However, with a speed of 40 km/hr, he
reaches his office 5 minutes earlier. The distance between his house and office is
(a) 30 km (b) 10km (c) 20 km (d) 40 km

147. A box contains Rs, 56 in the form of coins of one rupee, 50 paisa and 25 paisa' The number of 50
paisa coins is double the number of 25 paisa coins and four times the number of one rupee coins.
The number of 50 paisa coin in the box is
(a) 14 (b) 16 (c) 32 (d) 64

148. The price of 7 bananas is equal to the cost of 3 kiwis The price of 2 kiwis is equal to the cost of 1
banana and 5 chikoos. lf Rambo has just enough money to buy 30 chikoos, then how many bananas
Rambo could buy with the same amount ?
(a) 22 (b) 20 (c) 2s (d) 11

149. ln a certain class, 72% of the students prefer cold coffee and 44% prefer fruit juice' lf each of them
prefers cold coffee or fruit juice and 48 likes both, the total number of students in the class is
(a) 240 (b) 200 (c) 300 (d) 250

150. The average price of 10 pens is Rs. 12 while the average price of 8 of these pens is Rs. 11.75. Of the
remaining two pens, if the price of one pen is 60% more than the price of the other, what is the price
of each of these two pens ?
(a) Rs. 12, Rs. 14 (b) Rs. 5, Rs. 7.50 (c) Rs. 8, Rs. 12 (d) Rs. 10, Rs. 16

Previous Years
Page 308 CLAT & AILET Papers
1il|ilililililililt|
llIIIIII:TIIIIIITTTIIIITTI:rIIrIIrIIIl
rtt
I
- Etrraar^rr
ADMISSION TEST - 2017
I
I
!r
I Exam. :
Date of Exam. (Sunday) Cente/s Name _
(Sunday) May, 2017 i
rl lDate
I lDuration : Minutes
7th
|noll ruo. | Center's Name : |

,otrln sheet No. :-_li


Duration
Minutes 90 Roll No.
IlI Marks : 150
Max. Marks
lMax. 150
I

OMR Sheet No.


'
li
Il | Birth |

I| | Date of Birth
' -lr li
Tffii *"=o::'::""^'='
I i
Question Booklet
ll
No. l-
I i-la-l- Sl. No.
nr-
I
a rrr6t'Ftl ffil
INSTRUCilONS rO CANptpATES I Zb67U
r^-Fr^lrra
25879 .;A
li A l lrAlA r
-FA
!

r r lr
i . No clarification on the question paper can be sought. Answer the questions as they are. I-
!{AtaalariliaaIiananlt:aa,,aati^^^^.l^^^^..^Lr^^..-.-..rL.^-.
1

I 2. There are 150 multiple choice objective type questions. Answer ALL the questions.
! S. fach question carries ONE mark. Totat marks are 150. I
I +. ffrere is no negative marking. I
i S. provided,
Candidates have to indicate the correct answer by darkening one of the four r."rponr",
!
I with HB Pencil in the OMR Answer Sheet.
I
I Example : Forthe question, "where is the Taj Mahal located ?", the correct answer is (b). i
candidate has to darken the corresponding circle as indicated below , I
! The(a) Kolkata (b)Agra I
i (c) Bhopat (d) Delhi
T
Itl Right Method Wrong Merhods I
-
! @o@@ @@@@ @@@@ @oc@
o. Answering tne questions by any method
-I be considered incorrect and no marks willother than the method indicated above snall !
be awarded for the same.
I
i 7. lt/ore than one response to a question shail be counte d as wrong.
; 8. The candidate shall not write anything on the OMR Answer Sheet other than the details -!I
I required and in the spaces provided for.
I 9. After the Test is over, the candidate has to return the test booklet along with the OMR --
I Answer Sheet to the invigilator.
"-'-I
! tO. fne use of any unfair meins by' any'-candidate shall result in the cancellation of his/her
I candidature. -
! t L to
ttp"rsonation is an offence and the candidate, apart from disqualification,
face criminal prosecution.
I "---
"'-r haue
'' may
I
!
it, :9tflP,-g"dgets like mobile ghones, pagers
permitted. inside the Test Gentre/Hall.
or calculators are stricrty not
I
i ^ I
;13.
_
The candidates shall not leave the hall before the Test is over. I
lrrrrrrrrrrrrrrrrrrrr-rrrrrrrrrrrrrrl
llllllllllllll'll llll

BREAK-UP OF MARKS

s"&r8,S{ Ssuniect Marks Q. Nos. Page No.

A English 35 1-35 3-9

B General Knowledge 35 36-70 10- 13

c LegalAptitude 35 71 - 105 14 -22

D Reasoning 35 106-140 23-29

E Mathematics 10 141 -150 30

t"t*** 150

-*
1tililililililil||||

1. Cashed up, aspirational and a marketer's dream, the hoi polloi are on the march,
writes Bridie Smith.
Choose the antonym of the word hoi polloi.
(a) Middle class (b) Aristocracy (c) Masses (d) Working class
2. There's a lot in life that can leave you nonplussed.
Choose the antonym of the word nonplussed.
(a) dumbfound (b) disconcert (c) astound (d) unperturbed
3. Stay away from the touristy Pioneer Square historic district and most of its cheeseball
clubs, at least on nights and weekends,
Choose the meaning of the word cheeseball.
(a) Expensive (b) Notorious
(c) Inforior in quality (d) Stylish

4, lt was marvellous to see how gingerly, the little beasts footed it in sueh places.
Choose the synonym of the word glngerly.
(a) roughly (b) assiduously (c) r"ashty (d) carelessly
5. She's also an up-and-coming fiction writer with a penchant for the dark and surreal.
Choose the meaning of the word surreal.
(a) Bizarre (b) Haunting (c) Edsy (d) Satanic
Directions (Q. 6 and O. 7) : Match the ideas in (i) and (ii) by using the words in the
brackets.

6. l. (even though) (with)


11.
lll. (seeing that) lV. (even if)
(i) (ii)
1, I took my raincoat and umbrella a. because Davies is its
captain,
2, I stillwon't be able to get to a meeting b. despite a problem in
at 8.30 digging the foundations.
3. The team is likely to do well in this season c, as weather forecast
was so bad.
4. The building work is still on schedule _ d, whether or not I catch
an earlier train.
(a) lll-1-c, lv-2-d, ll.3-a, l-4-b (b) l'1-c, ll-2-d, lll-3-a, lv-4-b
(c) l-2-d, lll-4-b, ll-3-a, lV-1-c (d) lV-3-a, lll-2-b, ll-4-c, l-1-d
-9.
ilililillllllllllll

7. L (due to) (in order to)

ilt. (in order that) (so as to)

We have decided not to go on because there had recentlY

holiday this Year been a strike bY Postalworkers.

The council have Planted trees at to give Dave somewhere


the side of the road ._- private to studY before his exams.

The parcel has been delaYed because we want to save


money for a new car.

We've put a table and chair in the in an attempt to reduce traffic


spare bedroom
(a) l-1 -a,ll-2-b,lll-3-c, lv-4-d
(b) lV-1 -a,lll-2-b,ll-3-c, l-4-d
(c) I-4-a, ll-3-c, lll-2-b, l-1-a
(d) ll-1-c, lV-2-d, l-3-a, lll-4-b

Directions (Q. g and Q. g) : Complete the sentences on the right with appropriate compound
nouns related to the two-word verbs used in the sentences on the left.

ll. Telling off


lV. downpour
The teacher told me off for handing The children have a secret

in my homework late. at the bottom of the garden.

My mind flashed back to the time Iwas caught in a sudden


when lwas living in Stockholm. and got soaked through.

The escaped prisoners crept into an My father gave me a good


old barn and hid out until it got dark. for knocking down his Prize roses.
There are a number of in the
The rain was Pouring down as we
got out of the taxi. film to the time before the robbery.

(a) l-1 -a,ll-2-b,lll-3-c, lV-4-d


-
(b) ll-1-c, l-2-d, lll-3-a, lv-4-b
(c) ll-1 -c,l-2-d,lV-3-a, lll-4-b
(d) l-4-d, ll-3-c, lll-2-b, lV-1-a
+
ilililililililililt|

9. l. setbacks ll. Tip-offs


lll. passers-by lV. Upbringing
1.The police were waiting for the thieves. a. Harry had a very strict_
and was
Someone must have tipped them off. glad to move away from his
parents.
2. As I passed by her house, I could see b. The engine fault was the latest of
people dancing in the front room. several in the development
of the car.
3. She was born and brought up in c. We received several_ that
central London. there would be an attempted
break-out at the prison.
4. The injury has set back his chances d. The man was leaning out of the
of being fit to play in the final. window, shouting at _in the
street below.
(a) l-4-b, ll-1-c, lll-2-d, lV-3-a (b) ll-3-c, lll-2-b, lV-1-a, l-4-d
(c) ll-1-c,l-2-d,lV-3-a, llt-4-b (d) l-3-b, ll-4-c, lll-1-d, lV-2-a

Directions (O. 10 and Q. 12) : Observe the following sentences where same word in
different form is used keeping the seRse of the sentence same. ln the following questions,
the sentences have some element of similarity. You have to find out the similarity and
choose the option which is odd one out.

10. (a) Such jokes do not give me any amusement.


Such jokes do not amuse me.
(b) He put up a brave fight.
He fought bravely.
(c) Her failure disappointed her.
She failed and it disappointed her.
(d) The boy gave a prompt answer.
The boy answered promptly.

11. (a) Getting up he walked away.


He got up and walked away.
(b) Fortunately, nothing has happened.
It was fortunate that nothing has happened.
(c) You must take rest to get well.
You must take some rest othenrrrise you can't get well.
(d) He made a promise. He kept it.
He not only made a promise but also kept it.
llllllllllllllllllll

12. (a) Mother said to Geeta, "Where are you going ?"
Mother asked Geeta where she was going'
(b) Radhika said, "Can you tell me how to reach there ?" "YeS", said Ram'
he could.
Radhika asked if he could tell her how to reach there. Ram replied that
(c) All could hear the mournful wails of his wife'
Themournfu|wailsofhiswifecou|dbeheardbyal|.
(d) she said to her husband, "Did you like my dance yesterday ?"
she asked her husband if he had liked her dance the last day.
: Choose the sentence which is incorrect grammatioally.
Direction (o. 13 and Q. l4)

13. (a) Sita is more intelligent and wise than Rita'


(b) The wisest man that ever lived made mistakes'
(c) | doubt if Raviwill come.
(d) The flowers smelt sweet.

14, (a) | am looking fonrvard to receiving your reply'


(b) Eaeh boy and every girl was given rewards'
(c) | can run as fast, if Rot faster, than you'
(d) I can reproduce this lesson word for word'
of
Direcilon (o. 15 - Q. 17) : choose the alternative which best expresses the meaning
the idiom/Phrase in the question.

15. Pell-mell
(a) In hurried disorder
(b) GaietY and excitement
(c) ExtremelY worried
(d) None of these

16, To have one's heart in one's boots


(a) To get angry
(b) To be deePlY dePressed
(c) To be frightened
(d) To keeP a secret

17. To give one's ears


(a) To listen carefullY
(b) To act in a foolish waY
(c) To make a guess
(d) To make almost anY sacrifice
ililililililililil|
Direction (O. 1B - O. 20) : Rearrange the given six sentences K, L, M, N, O and p in the
proper sequence so as to form a meaningful paragraph and then answer the questions
that follows.
(K) The subjugation or enslavement of the people and countries is fast becoming a
thing of the past.
(L) Since then the progress in science and technology and in other branches of
knowledge has resulted in the betterment of the conditions of life in many countries.
(M) The lndustrial Revolution in England about the middle of the eighteenth century
was history's great leap fonruard.
(N) All this is on credit of modern civilisation.
(o) society has been progressing and improving for thousands of years.
(P) The span of human life has been enlarged, the condition of the tabouring classes in
many countries has improved, knowledge and education have made great strides,
the treatment of children has become more humane, feats of engineering have done
wonders, democracy and freedom have been granted to most countries of tfre world,
and a very much larger percentage of people have become entitled to the good
things of life.

18. Which sentence should come fifth in the paragraph ?


(a) P (b) K (c) N (d) L
19. Which sentence should come first in the paragraph ?
(a) o (b) M (c) P (d) K
20. Which sentence should come third in the paragraph ?
(a) N (b) M (c) P (d) L
Direction (O. 21 - O. 24) : Find out the sentence/s which is/are written correcly.
a. Charles is a gifted footballer, but up to now he didn't play well in international matches.
b. We've been staying with Paul and Jenny until last weekend.
c. He just heard the news and was rushing home to teil his family.
d. When I grow up I'm going to be an astronaut.

21. (a) a, d (b) c (c) d (d) b, c


a. Have you ever been to the opera when you lived in Milan ?
b. The man who broke the window wants to see you.
c. I won't be able to meet you next week. I will stay in London for a few days.
d. My mother who is in her seventies enjoys hill walking.

22. (a) b, d (b) a, b,c (c) b (d) d


-7-
ililillillllllllllll

a. Applications have risen this year by as much as 50%.


b. There are anumber of reasons I don't like him, but his meanness is the main.
c. I felt confident to pass my driving test.
d. I haven't got enough cash on me for paying the bill.
23. (a) a, d (b) b, c (c) d (d) a
a. Although they played well, but they never looked like winning.
b. Despite the snow was stillfalling heavily, she went out.
c. lf Schumacher were to win today he would become world champion.
d. She got low grades for her exams, therefore she had to retake them to get into
college.

24. (a) b, d (b) c (c) c, d (d) d

Direction (O.25 - O.35) : Fill in the blanks with appropriate words.

Under a zero-hour contract, a party described as a'casualworker' has a contract with an


employer, but the contract may be of little benefit as its provisions do not (25) any
obligation upon the employer to offer work, and similarly, there is no obligation for the
-
worker to accept it. A 'casual worke/ is the correct term to define workers who do not have
(26)_ hours of work but instead supply services on an irregular, flexible basis when
the employer needs them. A'worker' is not, therefore, an'employee'. This is an important
distinction, as employees enjoy significantly better protection under the law.

Although workers do have basic rights, for example, the right to a national minimum
(27\_, the right not to be unlawfully discriminated against and the right to sick pay,
but they are denied other important rights. For example, casualworkers do not have the
same rights when it comes to issues such as maternity or paternity pay, (28) pay in
the event that there is not enough work available and the right not to be unfairly
(2g) _. These contracts are very unpopular with trade unions, who argue that
employers are using this type of contract as a way of denying workers the benefits which
they would otherwise be entitled to under current employment (30)
As a result of public criticism, the Eurona government held an independent review to
analyse the use of these contracts. In June 2016, Eurona Business Secretary,Zoro,
concfuded that zero-hour contracts were not to be declared (31) ------, as he considered
that many groups of people benefited from them. However, he did change the law in order
to prevent employers from banning casualworkers from working elsewhere when bound
by a zero-hour contract. This is not the encl of the story, however. The use of zero-hour
contracts is extremely popular in the food and retail (32) of the Eurona economy.
ilililililil||ilil|

For example, McDonald's and Burger King have recently disclosed that they use this
type of contract for over 80% of their workers in the Eurona and another high-street giant,
Sports Direct, has admitted that 90% of its workers are under this type of contract. However,
the latter (gg) _
a legal challenge in November 2016 when Tera Phera filed a claim
against the company in a Eurona Employment Tribunal. Tera Phera claimed that although
she was contracted as a casual worker for Sports Direct, she had the same responsibilities
and duties as an employee but without receiving the benefits of an employee. A (34)
was reached according to which Sports Direct is now required at the time of advertising
for future zero-hours staff to expressly state that the role does not (35)work. The
company must also produce clear written policies setting out what sick pay and paid
holiday their zero-hours staff are entitled to.

25. (a) impose (b) compel (c) force (d) impress

26. (a) fixed (b) secure (c) firm (d) stable

27. (a) salary (b) wage


(c) earnings (d) remuneration

28. (a) let go (b) redundancy


(c) termination (d) release

29. (a) dismissed (b) discharged


(c) terminated (d) laid off

30. (a) statutes (b) codes


(c) legislations (d) acts
31. (a) unlawful (b) illegal
(c) illegitimate (d) felonious

32. (a) zones (b) regions


(c) sectors (d) areas

33. (a) suffered (b) bore


(c) endured (d) faced
34. (a) agreement (b) settlement
(c) Deal (d) Decision

35. (a) provide (b) entitle


(c) guarantee (d) offer
-9
ililililtilllllllll

36. Which country recently decided to quit commonwealth group ?


(a) Maldives (b) Fiii (c) Nigeria (d) Zimbabwe

37. The official mascot of the 2o1B FIFA World cup is _--- who wears
---.
(a) Zabivaka, glasses (b) Willie, glasses
(c) Zaumi, Russia's kit (d) Zabivaka, flag of Russia

38. ln India, three women had held the post of Deputy Governor of RBl. Which of the
following is not amongst them ?
(a) K.J. Udeshi (b) Usha Thorat
(c) Dr. lsher J. Ahluwalia (d) Shyamala Gopinath

39. World's largest refugee camp, which was in news recently, is located

(a) Dadaab, KenYa (b) Zaalari, Syria


(c) Yida, Sudan (d) Katumba, Tanzania

40. "Greenmail" is a tYPe of


(a) Currency ManiPulation (b) Takeover Defence
(c) CorPorate Communication (d) Eco- friendly device

41. Which of the following labels are not owned by Yum Brands ?
(a) KFC (b) Pizza Hut (c) Burger King (d) Taco Bell

42. has become the first bank in lndia to introduce software robotics in
power banking.
(a) HDFC (b) lclcl (c) SBI (d) Yes Bank

43. is not a component of the Li Keqiang Index.


(a) Bank Loans (b) Export
Consumption
(c) Electricity (d) Raitway freight

44. Train travel insurance in India is available at a premium of_.-- with effect
from October7,2016.
(a) 1 rupee (b) 2 rupees paisa
(c) 92 (d) 1 Paisa

45. Which of the following banks overtook Wells Fargo as the most valuable bank in
USA ?
(a) J.S. Morgan Chase (b) HSBC
(c) Citigroup (d) Bank of America
-10-
ilililillllilllllll
46. Nobel prize in Economics for 2016 has been given to Oliver Hart and Bengt
Holmstrom for their contribution to
(a) Contract Theory
(b) lmPerfect ComPetition

[:] :ffi:-ff:3" state intervention in market economv

47. Why a small pouch containing silica gel is often found in bottles of medicine in
tablet or Powder form ?
bacteria
(a) tt kills (b) lt kills germs and spores
moisture
(c) lt absorbs (d) lt acts as a preservative

4g. The Union Government has decided to withdraw the benefit of subsidised LPG to
the customers having income above
(a) Rs.7.5 lakh (b) Rs. 10lakh (c) Rs. 12.5 lakh (d) Rs. 15lakh

49. What is beaufort Scale ?


(a) Scale to measure wind speed (b) Scale to measure ocean waves
(c) scale to measure sound waves (d) scale to measure depth

50. Navtej Sarna who is recently appointed as an Ambassador of


Ambassador of earlier.
(a) Canada, lsrael (b) USA, lsrael
(c) USA, Canada (d) USA, France

51. What is the primary objective of the Governments in opting for deliberate devaluation
of currency ?
(a) Boosts imPort (b) Boosts Internal Trade
(c) Boosts exPorts (d) Increase gold reserves

SZ. for States and for Union Territories is the new limit
of flexible funO decided by the government in the Centrally Sponsored Schemes in
order to give more freedom of operation in these schemes.
(a\ 25o/",30"/o (b) 207",30"/o (c) 10%,35% (d) 50%'20%
53. Which of the following banks recently designated P. V. Sindhu and K. Srikanth as
brand ambassadors ? -
(a) Canara Bank (b) sBl
(c) PNB (d) Bank of Baroda

54. Which of the following are optical illusions ?


(a) Twinkle of stars in the sky (b) Rainbow
(c) Auroras (d) All of these
-11-
ililtilllilllllllll

55. Which of the following are Nocturnal in nature ?

(a) Kiwi (b) Red Panda (c) catfish (d) All of these

56. is the world's lightest material developed by lsRo scientists.


(a) Silica Aerogel (b) Carbon Aerogel
(c) Graphite Aerogel (d) Fiberol

57. Wi-Fi uses in order to transmit data whereas Li - Fi uses


in order to transmit data.
(a) radio waves, laser (b) radio waves, visible light
(c) electromagnetic waves, laser (d) infrared rays, radio waves

5g. is used for constructing filament of the World's thinnest light bulb ?
(a) Graphite (b) Graphene (c) Carbon(d) Tungsten
ROSS to
59. Which of the following companies has hired World's first robot lawyer
assist its vaiious teams in legal research ?
wardwell
(a) Davis Polk and (b) cravath, swaine and Moore
(c) Baker Hostetler (d) Latham and watkins

60.

(a) Russia (b) US (c) China (d) UK

61. The scientists from have developed a catalyst to render the drinking
water E. Colifree'
(a) Indian lnstitute of science Education and Research
(b) Tata Institute of Fundamental Research
(c) National Chemical LaboratorY
(d) lndian lnstitute of Sciences

62. India has launched its latest communication satellite GSAT - 18 from
(a) Turkey (b) French Guiana (c) Russia (d) Japan

63. ln which country the world's first baby was born from a new procedure that combines
the DNA of three PeoPle ?
(a) Canada (b) England (c) Australia (d) US

64. International Day of Happiness is observed on and --...-is


World in World Happiness Report 2017
adjudged as the happiest country in the
released by UN.
(a) February 20, Denmark (b) March 20, Norway
(c) February 21, Norway (d) March 21, lndia
-12-
riltililillillllllll

65. Which of the following seas of Antarctica has been declared as world's largest marine
protected area?
(a) Amundsen Sea
(b) Bellingshausen Sea
(c) Weddell Sea
(d) Ross Sea

Ms. from has been crowned Miss International20l6.


(a) Kylie Verzosa, Venezuela
(b) Kylie Verzosa, PhiliPPines
(c) Edymar Martlnez, Venezuela
(d) Edymar Martinez, Australia

67. has developed the concept of Human Development Index in


(a) Mehboob-ul-Haq, 1990
(b) Avinash Dixit, 1999 -.
(c) Jagdish Bhagwati, 1980
(d) Amartya Sen, 1970

68. The position of the Earth in its orbit, when it is at its greatest distance from the Sun
causing summer in the Northern Hemisphere is
(a) Apogee
(b) Perigee
(c) Aphelion
(d) Perihelion

69. Indiawon medals in 2017 Special Olympics World Winter Games


held in Austria.
(a) 73 (b) 1o
(c) 12 (d) 63

70. Recently a huge Dead Zone is discovered in


(a) Gulf of Mexico
(b) Bay of Bengal
(c) Coasts of Namibia
(d) Baltic Sea

-1$

(
1ilil]illllllllllll

followed by a
Directions (o. 71 - e. 79) : Given below is a statement of legal principle
the most appropriate
factual situation. nppry tre rjrinciple to the facts given below and select
answer.
parties. But
Legal principle: Contract is an agreement freely entered into between theis voidable at
contract
when consent to un'Igi&;"ni i. o"ot"ineo oy ulclu.e influence, the
the option of the party whose consent was so obtalneo.
Anurag since the
Factual situation: The pragya had been worked for a business man Pragya purchased
2000, (aged 21)
age of 18, working ior u luaOg".it Anrtag's businesses. ln
and he asked Pragya
a flat. ln 2005, vrr. Anum6;5 business ilas facing financial difficul[ies,
In July
t" ofi"i ,tneitrat ui tir.ri"n"iil security aqainst in overdraft facility for the.business'
take indepenq9nl
of il.,ui v"bi, ine nanrc solicitors wrote'to Fragya, advising that
she should
bd;t;a;i"6 oetore putting rrer property yp securitylorthe debt' The bank also notified
e"s:a
FAgt; that the guarantee was unli'miied in both time and financial amount. Having
discussed tne ariiib.ri"t *itr-Anurag,
pragya.was unaware of the extent of the
upon, and that his own
borrowing, out waJis"zured that her mortg"ag.e w6lto not be called
pr"p"rti"i'*fri.f, weie atso used as gec-uiity would-be looked Mr' at first. A charge was
executed ouer. tne Fragya's property in Auglst ?099 In 2009, Anr{ag's business
Pragya..Pragla
*irii.i" riquioation undir," nunil formartyitating
de-mandeg Fr. 60,24,912trom
that Mr. Anurag had induced her to enter
raised the defence of unOue influence -
influence which
i"ioin" u-gieement, ano tn" ng1r rrao tutLknovilledge/notice of ihis undue
Pragya' Bank is
should set aside iii" ninro. right to eniorce the dibt recovery against
contending that there is no undue influence'

71. whether the consent to offer the flat as financial security obtained through undue
influence ?
(a) No because pragya was not forced by Anurag to offer her flat as a security.
(b) No, because pragya was an educated and adult employee of Anurag and she
knew what she was doing.
(c) yes, an employer/employee relationship was capable of d.eveloping into such
a retatioi;tidffi ,n|rrj lnftren"e. Ttiere was no benefit to Pragya in the
agr""r"ni. tf'" lack of benefit to one party was evidence enough.
(d) No, an employer/e.mployee relationship is not capable of developing into a
relationship for undue influence.
Decide
72. Irrespective of your answer to Q. 71 , assume it is a case of undue influence.
whether the bdnk has done enougfr to allay concerns of undue influence ?
(a) The bank had not made all reasonable steps to allay themselves of the concerns
t-'
iegliOing undue influence. The fact that; on adviceirom the bank, the defendant
did not.-"6ri"i"i"pen;"nt advice, snorito have been taken as confirmation of
undue influence'
(b) yes, the Bank has advised Pragya that s.heshould take independent legal advice
.,beforeputtingherpropertyupassecurityforthedebt.
(c) Pragya has a duty to be aware of the consequences of her act.
that the guarantee was unlimited
iOi eunL l-'a$ done elougfr as it had notified Pragya
in both time and financial amount'

-14-

S:
ilililililIilililil|

73. Legal Principte: The acceptance must be absolute and unqualified, leaving no
ground for doubt or uncertainty. lf the acceptance is conditional, no valid contract is
formed, and the offer can be withdrawn at any moment till the absolute acceptance
has taken place within reasonable time of such offer.
Factual Situation: Delhi Government conducted an auction for the sale of license
of wine shop. X offered the highest bid which was provisionally accepted "...subject
to the confirmation of Chief Commissioner who may reject any bid without assigning
any reasons." Since X failed to deposit the required amount, Chief Commissioner
rejected the bid. The government held X liable for the difference between the bid
offered by him and the highest bid accepted in re-auction, and commenced
proceedings for the recovery of the sum. lt was contended on behalf of the government
of Delhi that X was under a legal obligation to pay the difference as it was due to his
default that a resale of the excise shop was ordered and hence X was liable for the
deficiency in price and allexpenses of such resale which was caused by his default.
Decide, giving reason, whether X is liable to make payment to the Delhi Government
(a) No, X is not liable to make payment as the shop was sold to the highest bidder.
(b) X is liable to pay because the Government of Delhi has to conduct re-auction
and also suffered loss in the sale of the shop.
(c) X is liable because his bid was accepted but he failed to deposit the required
amount on time.
(d) No, contract for sale was not complete till the bid was confirmed by the Chief
Commissioner and till such confirmation : the bidder was entitled to withdraw
the bid.

74. Legaf Principle: Employers/Principles are vicariously liable, under the respondeat
superior doctrine, for negligent acts or omissions by their employees/agents in the
course of employmenVagency. A servanVagent may be defined as any person
employed by another to do work for him on the terms that he, the servanVagent, is to
be subject to the control and directions of his employer/principal in respect of the
manner in which his work is to be done.
Factual Situation: A motor car was owned by and registered and insured in the
name of A (wife) but was regarded by her and her husband (B) as "our car." B used
it to go to work, and A for shopping at the weekends. B told A that if ever he was
drunk and unfit to drive through, he would get a sober friend to drive him or else
telephone her to come and fetch him. On the day in question the husband telephoned
the wife after work and told her that he was going out with friends. He visited a
number of public houses and had drinks. At some stage, he realised that he was
unable to drive safely and asked a friend, C, to drive. C drove them to other public
houses. After the last had been visited C offered the three friends (X, Y and Z) a lift
and they got in, together with B who was in a soporific condition. C then proceeded,
at his own suggestion, to drive in a direction away from the B's home to have a
meal, On the way, due to C's negligent driving, an accident occurred in which both
B and C were killed and the other friends got injured. X, Y andZ brought an action
against the wife both in her personal capacity and as administratrix of the husband's
estate. Decide whether A is liable.
-15-
llllllllllilllllllll

Decision:
(a) Yes, she was vicariously liable for the negligent driving of C as the principle of
vicarious liability was to put responsibility on to the person, namely, in the case
of a motor car, the owner, who ought in justice to bear it, and that in the case of
a "family ca/'the owner was responsible for the use of it by the other spouse.
(b) No, C had not been the wife's agent in driving the husband about as he had
been doing at the time of the accident. To fix vicarious liability on the owner of a
motor car in a case such as the present, it must be shown that the driver was
using it for the owner's purposes under delegation of a task or duty.
(c) No, because this is a case of volenti non fit injuria as X, Y and Z voluntarily took
the lift knowing that C was also drunk.
(d) No, because C was not employed by A to drive her husband back to the home
on the day of accident.

75. Legal Principle:


1 . Battery is the intentional causation of harmful or otfensive contact with another's
person without that person's consent.
2. When lawfully exercising power of arrest or some other statutory power a police
officer had greater rights than an ordinary citizen to restrain another.

Factual Situation: Two police officers on duty in a police car observed two women
in the street who appeared to be soliciting forthe purpose of prostitution. One of the
women was known to the police as a prostitute but the other, X, was not a known
prostitute. When the police officers requested X to get into the car for questioning
she refused to do so and instead walked away from the car. One of the officers, a
policewoman, got out of the car and followed X in order to question her regarding
her identity and conduct and to caution her, if she was suspected of being a prostitute,
in accordance with the approved police procedure for administering cautions for
suspicious behaviour before charging awoman with being a prostitute. X refused to
speak to the policewoman and walked away, whereupon the policewoman took
hold of X's arm to detain her. X then swore at the policewoman and scratched the
officer's arm with her fingernails. X was convicted of assaulting a police officer in
the execution of her duty. She appealed against the conviction, contending that
when the assault occurred the officer was not exercising her power of arrest and
was acting beyond the scope of her duty in detaining X by taking hold of her arm.
The police contended that the officer was acting in the execution of her duty when
the assault occurred because the officer had good cause to detain X for the purpose
of questioning her to see whether a caution for suspicious behaviour should be
administered. Decide whether the police officer is liable for battery.
-16-
ililililrililllllll

Decision:
(a) X is liable for trespass on the person of a police otficer while performing her
officialduty.
(b) The policewoman had not been exercising her power of arrest when she detained
X, and since in taking hold of the appellant's arm to detain herthe policewoman's
conduct went beyond acceptable lawful physical contact between two citizens,
hence the officer's act constituted a battery on X and that she had not been
acting in the execution of her duty when the assault occurred.
(c) The fact that the reason the police otficer detained X was to caution her regarding
her suspicious behaviour render the officer's conduct lawful if in detaining her
she used a degree of physicalcontact that went beyond lawful physical contact
as between two ordinary citizens.
(d) The police otficer was on duty and performing her duty in the regular course of
the work so is not liable for battery.

76. Legal Principle:


1. A careless person becomes liable for his negligence when he owed a duty of
care to others.
2. Volenti non fit iniura is defence to negligence.

Factual Situation : K was a friend of L and was teaching her to drive. Prior to such
an arrangement K had sought assurances from L that appropriate insurance had
been purchased in the event of accident. On the third day, L was executing a simple
manoeuvre at slow speed when she panicked which resulted in the car crashing
into a lamp-post injuring K. L was subsequently convicted of driving without due
care and attention. L denied liability to pay compensation to K on the ground of
votenti non fit injuria and also that she was just learning to drive and was not in
complete control of the vehicle. Decide.

Decision:
(a) L is liable as the defence of volenti non fit injurawas not applicable. Secondly,
thatthe duty of care owed by a learner drivertothe public (including passengers)
was to be measured against the same standard that would be applied to any
other driver.
(b) L is not liable as K voluntarily accompanied her.
(c) L is not liable as she is just learning to drive and duty of care rests upon the
instructor.
(d) L is not liable as a learner driver do not owe a duty of care towards public in
general and towards the passenger in specific.
-17-
ililtililililfiilil
77. Legal Principle : Article 19(1) (d) of the Constitution of India guarantees to all citizens
the right to move freely throughout the territory of lndia. But at the same time, Article
19(5) empowers the State to impose reasonable restrictions on the freedom of
movement on the ground of interest of general public.

Factual Situation: Wearing of helmet is made compulsory for all two-wheeler riders
by a law enacted by the State. The constitutionality of the law is questioned before
the High Court on the ground that it violates Article 19(1Xd) of the petitioner. Will the
petitioner succeed ?

Decision:
(a) Yes, because the restriction is not reasonable and no interest of general public
is protected by this law.
(b) No, because the restriction is reasonable as it intends to protect interest of general
public by preventing loss of lives of citizen of lndia.
(c) Yes, because freedom of movement is a fundamental right of every citizen of
lndia and the State cannot take it away by way of legislation but has to amend
the Gonstitution to take away the fundamental rights.
(d) No, because the freedom of movement will not be violated by the impugned
legislation

78. Legal Principle : An unlawful interference with a person's use or enjoyment of


land or some right over or in connection with it is a nuisance in tort. The fact that the
plaintiff "came to the nuisance" by knowingly acquiring property in the vicinity of the
defendant's premises is not a defense to nuisance. However, an act cannot be a
nuisance if it is imperatively demanded by public convenience. Thus, when the
. public welfare requires it, a nuisance may be permitted for special purposes.

Factual Situation : D owned and occupied an estate about two miles from RAF
Wittering, an operational and training base for Harrier Jump Jets. D claimed that
they suffered severe noise disturbance every time the Harrier pilots carried out training
circuits: an average of 70 times a day. D alleged that the noise nuisance constituted
a very serious interference with their enjoyment of their land. D instituted judicial
proceedings against the'defendants, the Ministry of Defence (MoD), damages
amounting to Rs. 1,00,00,000.

The MoD denied liability and raised defence that the Harrier training was undertaken
for the public benefit and that they had prescriptive right over the land as D had
bought their property at a time when RAF Wittering was already established so he
cannot claim compensation as he already knew about existence of RAF Wittering
near his property.

-18-
|il]l1lllllllllll
Decision:
(a) D is not entitled to compensation as the training of pilots is a public welfare
activitY.
(b) The Harrier training is not an ordinary use of land and that although there w?s q
'
public benefit to the continued training of Harrier pilots, D should not be required
to bear the cost of the public benefit.
(c) D is not entitled to compensation as his property is two miles away from the
training base.
(d) D will not get compensation as the training activity 99"t not amountto nuisance
'' and D had full knbwledge about the training activities when he purchased the
land.

79. Legal PrinciPle:


1. Negligence is the absence of care on the part of one party which results in
some damage to the other Party.
2. Generally, a person is under no duty to control another to prevent his doing
damage to a third Party.
3. The foreseeability test basically asks whether the person causing the iniury
should have reasonably foreseen the general consequences that would result
because of his or her conduct.
4. Statutory authority implies that an act is done by a person to fulfil his duty imposed
by the Siate. Statutory authority is a valid defence under the law of torts.
Factual Situation: Ten borstal trainees were working on an island in a harbour in
the custody and under the control of three officers. During the night, seven of them
escaped. liwas claimed that at the time of the escape the officers lad retired to bed.
The ieven got on board a yacht, moored off the island and set it in motion. They
collided with another yacht, the property of X and damaged it. X sued the Home
office for the amount bt tne damage. Decide whether on the facts pleaded in the
statement of claim the Home Office, its servants or agents owed any duty of care to
X capable of giving rise to a liability in damages with respect to the detention of
persilns undeigoing sentences of borstal training or with respect to the manner in
wnicfr such persons were treated, employed, disciplined, controlled or supervised
whilst undergoing such sentences.

Decision:
(a) The Home office is not liable as they are performing statutory duty and has
immunity from liability in negligence'
(b) The trainees are liable and not the Home Officers as the injury to X's property
could not be reasonably foreseen by the officers.
(c) The fact that the immediate damage to the property of X was caused by the acts
of third persons, the trainees, prevent the existence of a duty on the part of the
officers towards X.
(d) The taking by the trainees of the nearby yacht and the causing of damage to the
other yacht wnicn bebnged to X ought to have been foreseen by the borstal
officers as likely to occur if they failed to exercise proper control or supervision;
in the particular circumstances, the officers prima facie owed a duty of care to X.

-19-
llllllilllffllllll

Directions (O. 80 - O. 86) : The following questions consist of two statements, one labelled
as'Assertion' and the other as 'Reason'. Read both the statements carefully and answer
using the codes given below.
(a) Both A and R are true and R is the correct explanation of A
(b) Both A and R are true but R is not the true explanation of A
(c) A is true but R is false
(d) A is false but R is true

80. Assertion (A) : The framers of the Indian Constitution were keen to preserve the
democratic values to which Indians had attached the highest importance in their
struggle forfreedom.
Reason (R) : The Constitution describes lndia as a Union of States, thereby implying
the indestructible nature of its unity.

81. Assertion (A) : The "basic features" of the Constitution cannot be amended by
exercising the power of Amendment under Article 368.
Reason (R) : Though Fundamental Rights, as such are not immune from
Amendmenten bloc, particular Rights or Parts thereof may be held as "basic feature."

82. Assertion (A) : Legislations which gives a wide power to the executive to select
cases for special treatment, without indicating the policy, may be set aside as violative
of equality.
Reason (R) : Article 14 of the Constitution provides that the State shatt not deny to
any person equality before law or equal protection of laws within the territory of
India.

83. Assertion (A) : lf the Constitution is flexible it may be playing at the whims and
caprices of the ruling party.
Reason (R) : The framers of the Indian Constitution were keen to avoid excessive
rigidity.

84. Assertion (A) : A person suffering from AIDS can be restricted in his movements by law.
Reason (R) : Where policy is laid down, discretion is not of reasonableness.

85. Assertion (A) : lf a convict is prepared to give an interview to journalists and


video-graphers, the facility should be allowed to the latter.
Reason (R) : Position of a person sentenced to death is not inferior to that of a
citizen.

86. Assertion (A) : A files a false and frivolous civil suit against B and B wins the case,
he cannot file another civil suit for damages and costs.
Reason (R): In a civil suit, damage of reputation is absent because the action is
tried in public and if person wins his case then his reputation will be re-established.
ililI]Iililil ililt|

87. A minority community may reserve upto seats for members of its own
community in an educational institution established and administered by it even if
the institution receives aid from the State.
(a) 50% (b)40% (c) 33% (d) 27%

88. After Delhi and Goa, which is the third Indian State to have child - friendly court ?
(a) West Bengal (b) Karnataka
(c) Telangana (d) Tamil Nadu

89. Which of the following statements are true regarding the Constitution (One Hundred
and First Amendment) Act, 2016 ?
l. GST Act is a comprehensive direct tax on manufacture, sale and consumption
of goods and services throughout lndia.
ll. The Act will transform India into a corRmon market, harmonising myriads of State
and Central levies into a national goods and services tax which is expected to
boost manufacturing and reduce corruption.
lll. lt provides for Constitution of a Goods and Services Tax Council by inserting
Article 297A in the Constitution.
lV. GST will be based on input tax credit method.
(a) l, lland lll lV
(b) ll, llland lV
(c) l, ll and (d) All of the above

90.CentralVigilanceCommissionershal|holdofficeforatermof-yearS
from the date on which he enters upon his office.
(a) 4 (b) 3 (c) 2 (d) 1

91. ln Youth Bar Association V. Union of lndia, the Supreme Court directed to upload
copies of FIR within hours on police website.
, (a) 48 (b)12 (c) 2a @) 48-72
92. India's first court-annexed mediation centre was established in
(a) Delhi Nadu
(b) Tamil (c) Chennai (d) Karnataka
93. Which is the first country to allow voting through mobile phones ?
(a) Estonia (b)Finland France (c) (d) Germany

94. All students graduating from academic year _ onwards need to clear the
All India Bar Examination in order to practice law in India.
(a) 2007 - 08 (b) 2009 - 10 (c) 2008 - 0e (d) 2010 - 1 1

95. "The people have a right, an inalienable, indisputable, indefeasible, divine right to
-
that most dreaded and envied kind of knowledge I mean, of the character and
conduct of their rulers." Who said this ?
(a) Abraham Lincoln (b) John Adams
(c) Martin Luther (d) Mahatma Gandhi

-21-
ililllilllllll ll llll

96. John Doe order in India is known as


(a) Ashok Kumar Order (b) DMCA Order
(c) Take Down Order (d) Injunction

97. In 2016, the Supreme Court has clarified that the "Third Gender" will include(s)
(a)Transgender (b) Gays and Lesbians
Bisexuals
(c) (d) All of these

98. New Zealand Government has granted legal status of a person to a


after a legal battle of 140 Years'
(a) National Park (b) Tree
(c) Mountain (d) River

99. Which of the following Articles of the Cbnstitution of India authorize the Chief Justice
of India to request a ietired Judge to join the Supreme Court ?
(a) 12e (b) 128 (c) 127 (d) 126

100. The'Public Trust'the doctrine is related to


(a) Charitable Institution (b) Environment
(c) Journalism (d) Public Offices

101. The Delhi High court has asked Facebook to ban children below
years of age from creating an account.
(a) 10 (b) 13 (c) 16 (d) 18

lO2. Which one of the following decided to set up a website, which will compare the
World's Constitutions by themes online "to arm drafters with a better tool for
' Constitution design and writing" ?
(a) UN (b) Google (c) Microsoft (d) Amnesty

103. Mental Health Care Bill 2016 seeks to


1. Decriminalise suicide
2. Prohibit electro'convulsive therapy
3. Provide right to make Advance Directives
4. Provide for seclusion and solitary confinement in few cases
(a) 1, 2 and 4 (b) 2, 3 and 4 (c) 1, 2 and 3
(d) All of the above

104. What is the new minimum wage of non-agricultural, unskilled workers of the Central
Government as announced ny tne Union Government on August 30, 2016 ?
(a) Rs. 350 Per daY (b) Rs. 250 Per daY
(c) Rs. 246 per daY (d) Rs. 400 per day

105. Maternity Benefits (Amendment) Bill, 2016 provides weeks of full paid
mdternity leave to women who work in organised sector.
(a) 12 (b) 26 (c) 24 (d) 32
ilililillililililil|

Directions (O. 106 - Q. 108) : Read the following case and answer the questions.
Sakshi is the Vice-President in an electrical equipment company in Delhi. One day, her
subordinate Bhaskar requested that Kishen, a project manager, be transferred to the
Mumbaioffice from the Delhi office. In Mumbai, Kishen would work alone as a researcher.
Bhaskar gave the following reasons for his request "Kishen is known for fighting frequently
with colleagues of his team. He cannot accept criticism and feels rejected and get hostile.
He is overbearing and is generally a bad influence on the team."
Sakshi called upon Samir, another project manager and sought further information on
Kishen. Samir recalled that a former colleague, Lalit (who was also Kishen's former boss)
has made few remarks on Kishen in his appraisal report. ln his opinion, Kishen was not fit
for further promotion as he was emotionally unstable to work in groups in spite of the fact
that he had seven years of work experience. Lalit has described Kishen as too authoritative
to work under anyone. Lalit has further told Samir that Kishen had an ailing wife and an
old mother, who does not want to stay with his wife.
Consider the following solutions to the problem mentioned above
L
Sakshi should transfer Kishen to Mumbai office.
ll'
Sakshi should try and verify the facts from other sources as well.
lll.
Kishen should be sacked.
lV'
Kishen should be demoted.
V'
Sakshi should suggest Kishen to visit a family counsellor.

106. Which of the following would be the most appropriate sequence of decisions in
terms of immediacy starting from immediate to a long term solution ?
(a) ll, V, I (b) ll, |il, lv (c) ll, V, lV (d) lt, l, v
107. Sakshi sought an appointment with Lalit to find out ways to help Kishen. Lalit is of
the view that the company's responsibility is restricted to the workplace and it should
not try to address the personal problems of employees. lf Sakshi has to agree to
Lalit's view, which of the solutions presented in the previous question would be
weakened ?
(a) Only lll (b) Only lV (c) Only V (d) Only ll
108. Which of the following statements, if true would weaken the decision to sack Kishen
the most ?
(a) Another article published in the magazine, Caventers Quaterly, highlighted that
employee's problem at home affect their performance at work.
(b) In the latest issue of a reputed Journal, Xanders Business Review, it was
published that most top managers, find it difficult to work in a group.
(c) lt was published in Santers Management Review, another reputed Journalthat
individuals who cannot work in teams find it difficult to adjust to a new location.
(d) Bhaskar was of the opinion that emotionally unstable persons, find it difficult to
get back to normal working life.
ilililllllllllllill

Directions (Q. 1Og O. 1 13) : Read the following information carefully and answer the
-
questions. A word arrangement machine, when given an input line of words, rearranges
tirem following a particular rule in each step. The following is the illustration of the input
and the steps of arrangement.
Input - Diksha cannot but feel happy for him
Step | : but cannot Diksha happy feel him for
Step ll : cannot but feel happy Diksha for him
Step lll : but cannot happy feel him for Diksha
Step lV : happy cannot but him feel Diksha for
questions
and so on for subsequent steps. You have to find out the logic and answer the
given below.

1Og. lf Step V read "weeks of tepid slothful and weak ideas", then what would be Step lV
read ?
(a) ideas and tepid weeks of weak slothful
(b) of weeks and slothful tepid ideas weak
(c) ideas weeks and tepid of weak slothful
(d) none of these

1 10. lf Step I read "it was the name bestowed upon him", then what would be the
arrangement for SteP Vll ?
(a) it was him the name bestowed upon
(b) upon the him was it bestowed name
(c) bestowed it was the name upon him
(d) none of these

111. lf Step Vl reads "workers must take a stand against working", then what will be the
last word of SteP lll ?
(a) must (b) take
(c) a (d) none of these

112. lf Step lll reads "the best waY of promoting our sports", then what will be the
arrangement of the inPut ?
(a) best the sports way of our promoting
(b) our promoting the best sports way of
(c) sports best the of way our promoting
(d) of our best the way sports promoting

113. lf the given input is "it is good approach with care", then what will be Step lV ?
(a) care with a good approach is it (b) approach is a care good it with
(c) a good approach it is with care (d) with approach it is a good care
I il|l1il ||ilil il ilr
Direction (O. 114 - Q. 118) : Read the following information carefully and answer the
questions given below.
Nine players - G, H, l, J, K, L, M, N and O - have to be put in three teams. Each team will
consist of three players and each player will appear only once in a team. The teams must
be arranged according to the following conditions.
i'
I and N must be in the same team.
ii.
K and L must be in the same team.
iii. O and J cannot be in the same team.
iv. M must be in the second team.
v.
Either J or M or both must be in the team with H.

114. Which of the following cannot be true ?


(a) M is in 1st team and H is in the 3rd
(b) O is in 3rd team
(c) | is in 2nd team
(d) H is in the 3rd team

1 15. All of the following could be in the same team as K, except


(a) G (b) J (c) | (d) M
1 16. lf J and K are in the 3'd team, which of the following players must be in 2"d ?
(a) L (b) | (c) G (d) H

117. Which of the following players could be in a team together ?


(a) K, l, M (b) N, l, J (c) G, J, M (d) G, L, O
118. The 3'd team could consist of the following except
(a) G, H, J (b) K, L, J (c) K, L, O (d) G, l, J
Direction (O. 119 - Q. 122) : Read the following information carefully and answer the
questions given below.
A couple decided to organize a party and invited a few of their friends. Only the host and
hostess were sitting at the opposite ends of a rectangular table with three persons on
each side. The pre-requisite of the sitting arrangement was that each person must be
seated so that at least on one side he/she has a person of the opposite sex. Romeo is
opposite to Sonakshi, who is not the hostess. Ruchir has a woman on his right and is
sitting opposite to a woman. Jia is sitting to the hostess's right, next to Vikram. One person
is seated between Mahima and Rajni, who is not the hostess. The men were Romeo,
Ruchir, Vikram and Rajnikant while the women were Mahima, Rajni, Sonakshi and Jia.

1 19. The eighth person present, Rajnikant must be


l. The host
ll. Seated to Sonakshi's right
lll. Seated opposite to Rajni
(a) Only lll (b) I and ll (c) Only I (d) Only ll
-25-
llllllilllilllllllll

120. lf each person is placed directly opposite to his/her spouse, which of the following
pairs must be married ?
(a) Ruchir and Mahima (b) Rajni and Rajnikant
(c) Mahima and Vikram (d) Ruchir and Jia

121 . Which of the following persons is definitely not seated next to a person of same sex ?
(a) Mahima (b) Sonakshi
(c) Rajnikant (d) Romeo

122. lf Ruchir would have exchanged his seat with a person sitting four places to his left,
which of the following would have been true after the exchange ?
l. No person of the opposite sex was seated between two persons of same sex
ll. One side of the table consisted entirely of persons of the same sex
lll' Either the host or the hostess changed their seats
(a) Only I (b) Only ll (c) land ll (d) lland lll

129. Starting from a point M, Rohit walked 18 m towards South. He turned to his left and
walked 25 m. He then turned to his left and walked 25 m. Then he turned to his left
and walked 18 m. He again turned to his left and walked 35 m and reached a point P.
What is the direction of P in respect of M ?
(a) North (b) East
(c) West (d) South-East

124. Rashmi starts walking towards North. After walking 15 m she turns towards South.
After walking 20 m, she turns towards East and walks 10 m. She then turns towards
North and walks 5 m. How far is she from her original position and in which direction ?
(a) 10 m, East. (b) 10 m, West
(c) 10 m, North (d) 10 m, South

125. Suraj reached the coffee shop 20 minutes before 8 : 50 h, and he came 30 minutes
before Suman who reached 40 minutes late. What was the scheduled time of meeting ?
(a) 8:05h (b) 8:20h (c) 8:09h (d) 8:10h

126. A bus for New Delhi leaves every two and half hour from Kashipur Bus Stand. An
announcement was made at the bus stand that the bus for New Delhi has left
40 minutes ago and the next bus will leave at 18 h. At what time the announcement
was made ?
(a) 16: 10 h (b) 16: 15 h
(c) 17:05 h (d) 17:20 h

-26-
ilililililililil||||

127. lf Kriti says, "Amrita's father Amar is the only son of my father-in-law, Arjun." Then
how is Babita, who is the sister of Amrita, related to Arjun ?
(a) Wife (b) Niece
(c) Grand-daughter (d) Sister-in-law

128. Marc is Robin's mothe/s father. Marc has three brothers. One of them has a grandson
Anurag. Krish is the son of Anurag. Krish is related to Robin as
(a) Uncle (b) Nephew
(c) Brother (d) Cousin

129, 'A+B'meansAisthefatherof B,'A-B'meansthatAisthewifeof B,'AxB'meansthat


A isthe brotherof B and'A+ B'meansthatA isthedaughterof B, lf 'S +Tx U +Z',
which of the following is true ?
(a) S is the daughter of U
(b) Z is the aunt of S
(c) S is the aunt of Z
(d) S and Z are cousins

Dlroctlon (Q. 130 and Q. 131) : Select the odd one out.

130, (a) ZYAB (b) TSGH .(c) ONLM (d) UTFH

131. (a) JIHG (b) ROP0 (c) WXUV (d) UTSR

Direction (O. 132 and Q. 133) : Find out the wrong number.

132. 5, 7.5, 11.25, 17.5,29.75, 50, 91.25


(a) 2e.75 (b) 17.5 (c) e1.25 (d) 50

133. 10, 13, 26,37,51, 85, 154


(a) 154 (b) 51 (c) 26 (d) 13

Dlrectlon (O. 134 and Q. 135) : Find out the missing number/words.

1g4. sT39, UV43, WX47,?


@) YZaT (b) YZ52
(c) YZ51 (d) Yz50

135. 13 (168) 13, 14 (181) 13, 15 (?) 13


(a) 1eo (b) 1e5 (c) 1e0 (d) 1e4
ililillilllllllllll

Directions (o. 136 - o. 140) : Read the following short passages and choose the best
answers to the questions that follow each passage.

136. Columnist: Senators who are not firmly committed to a core set of beliefs will formulate
new campaign strategies and moot points before each upcoming election' Senators
are elected to serve multiple terms in office only if they formulate new campaign
strategies and moot points before each upcoming election. Therefore, senators who
are not firmly committed to a core set of beliefs are elected to serve multiple terms in
office.
Which one of the following displays a flawed pattern of reasoning most closely
parallelto that in the columnist's argument ?
(a) Nuclear proliferation will accelerate only if there is widespread access to fissile
material. Widespread access to fissile material goes hand in glove with
increased regional instability. Therefore, nuclear proliferation will not accelerate
unless there is an increase in regional instability'
(b) lf health care costs continue to rise, foreign currency markets will devalue the
Euro. But health care costs will not continue to rise. Therefore, foreign currency
markets will not devalue the Euro.
(c) A loyal person is a likeable person, for a loyal person always stands by her
friends and a likeable person afso always stands by her friends'
(d) A well-constructed house includes code-compliant framing, wiring, and
insulation. A house is marketable only if it is well-constructed. Therefore, a
marketable house includes code-compliant framing, wiring, and insulation.

1gZ. Cuisine from the north of China is one of the most popular kinds of food in India. But
this is not the case for Southern Chinese cuisine. ln fact, in a recent survey, Southern
Chinese cuisine was voted one of the least popular styles of cooking in the country.
Which one of the following, if true, most helps to resolve the apparent paradox
described above ?
(a) Southern Chinese cuisine frequently incorporates obscure ingredients that are
difficult to locate, while the ingredients necessary to prepare Northern Chinese
cuisine are easily found in most Indian grocery stores'
(b) lmmigrants from all parts of China brought their local cuisines with them when
they came to India in great numbers in the seventeenth and eighteenth centuries.
(c) Northern Chinese cuisine has recently become more popular in Indiathan the ltalian
cuisine, which for many years was this country's favourite internationalfood.
(d) Northern Chinese cuisine is more time consuming and labour intensive to
prepare than Southern Chinese cuisine.
|iltiltltililltiltl

138. A growing number of ecologists have begun to recommend lifting the ban on the
hunting of leopards, which are not an endangered species, and on the international
trade of leopard's skin. Why, then, do I continue to support the protection of leopards ?
For the same reason, that I oppose the hunting of people. Admittedly, there aretar
too many human beings on this planet to qualify us for inclusion on the list of
endangered species. Still, I doubt the same ecologists endorsing the resumption of
leopard hunting would use that fact to recommend the hunting of human beings.
Which one of the following, if true, would most weaken the author's argument ?
(a) Human being might, in fact, be placed on the list of endangered species.
(b) Despite the growing number of ecologists supporting a repeal of the ban on
leopard hunting, most will still support it.
(c) The international ban on leopard hunting was instituted before leopards became
an endangered species.
(d) Leopards, now dangerously overpopulated, cannot be supported by their
ecosystems.

139. Experts musicologists believe that Beethoven wrote his last piano sonata in 1824,
three years before his death. However, the manuscript of a piano sonata was recently
discovered that bears Beethoven's name and dates from 1825. Clearly, the experts
are mistaken because not every piece that Beethoven wrote was catalogued in his
lifetime, and it is known that Beethoven continued to compose untiljustweeks before
his death.
The reasoning in the argument is most vulnerable to which of the following criticisms ?
(a) That either of two things could have occurred independently is taken to show
that those two things could not have occurred simultaneously.
(b) An inconsistency that, as presented, has more than one possible resolution is
treated as though only one resolution is possible
(c) Establishing that a certain event occurred is confused with having established
the cause of that event.
(d) A claim that has a very generalapplication is based entirely on evidence from a
narrowly restricted range of cases.

140. Some people fear that our first extra-terrestrial visitors will not be the friendly aliens
envisaged in popular science fiction movies, but rather hostile invaders bent on
global domination. This fear is groundless. Any alien civilisation that makes it to our
planet must have acquired the wisdom to control war, or it would have destroyed
itself long before contacting us.
The author bases the argument above on which of the following assumptions ?
(a) Our planet will have contact with extra-terrestrial visitors at some time in the
future.
(b) lnterstellar travel is unworkabte except in a society much more technologically
advanced than ours,
(c) A civilisation that has learned to controlwar on its own planet will not wage war
on another.
(d) Alien civilisations are more morally advanced than those on Earth.
ilililllllllllillll

was spent-to make the land


141. A garden iS rectangular in shape. A sum,of Rs.,1,000 garden.is 50 m' lf lhe length
usable at the iut" oiZS paisa p|im', The breadth of the
in rupees for making
of the garoenlrlnJreas"o nv 2o r, *hai*itt n" the expeiditure
the larid usable, at the same rate per m2?
(a) Rs. 1,250 (b) Rs. 1,220 (c) Rs' 1,400 (d) Rs' 2'250
the Rrolyct
142. The sum of ages of a mother and daughter is 45 years.', FiverTears.ago'
i"", four times tne motnlr's age afthat time' The present age of the
of their
mother "g"i
is
(a) 38 years (b) 37 years (c) 36 years (d) 39 years

143. Today is Monday in a leap year. The day after 68 days will be
(a)Friday.(b)Monday(c)Thursday(d)Saturday
in 4 hours' lt covers the same
144. A streamer goes downstream !1om one port toanother
disrance ,plitiiil'iil #r".
d iitne spebd of stream is 2 km/h, the distance between
the two ports is
(a) 80 km (b) 81 km (c) 70 km (d) 71 km
parallelto the railway line
145. A train running at the rate of 49 kmlq passes.a man riding
in the same direction at 2b km/tl in +a'seconds. Find
the length ofihe train in metres.
(a) 100 m (b) 200 m (c) 150 m (d) 250 m

146. Reema, Ruchi and Richa can make a painting jl


1 5, 20 and€0 days, respectively'

They unoertire io mir,e a p"inting ior hs, 81oI me share of Reema exceeds that of
Ruchi by
(a) Rs.91 (b) Rs. e2 (c) Rs.90 (d) Rs.8e
males ' 50% population is
147 . Population of a city is 2, 96,000 out of which 1 , 66,000 are
women who are literate is
literate. ltT}%males are literite,then the number of
(a) 48,000 (b) 66,400 (c)32,200 (d) 31'800
But it was later found that
148. The average weight of lAstudents was calculated as 71'
instead of 56 and of
the weight ;f o;; student nqd ilen-wrongty entered as 42
anotheias 7+ instead of 32. The correct average is
(a) 75 (b) 80 (c) 68 (d) 6e

149. Fouryears ago, the ages of Ritu and Reenawere in the ratio 5:6 respectively'
be 8 : 9. What is the sum
Eight years ii6m now the respective ritio of their ages will
of their ages at Present ?
(a) 52 years (b) 50 years (c) 60 years (d) 62 years
less and it had been sold
150. A shirt was sold at a profit of 15%, lf its cost had been 5%
of the shirt.
for Rs. zr res-sltn; iG profit would have been 10%. Find the cost
(a)Rs.210(b)Rs.22O(c)Rs.1O0(d)Rs.200
1 ililililtilil il ill
SPACE FOR ROUGH WORK

-31-
illillllllllllll llll

SPACE FOR ROUGH WORK

-32-
SET – I (2020)
SECTION – A : ENGLISH

Directions (Q. 1 – Q. 5): Each set of questions in this section is based on the passage. The
questions are to be answered on the basis of what is stated or implied in the passage. For
some of the questions, more than one of the choices could conceivably answer the question.
However, you are to choose the best answer; that is, the response that most accurately and
completely answers the questions.

In order to understand the development of Gangetic Valley plains, scholars have


traditionally relied primarily on evidence from historical documents. However, such
documentary sources provide a fragmentary record at best. Reliable accounts are very scarce
for many parts of Northern India prior to the fifteenth century, and many of the relevant
documents from the fourteenth and fifteenth centuries focus selectively on matters relating to
cultural or commercial interests.
Studies of fossilized pollens preserved in peats and lake muds provide an additional
means of investigating vegetative landscape change. Details of changes in vegetation
resulting from both human activities and natural events are reflected in the kinds and
quantities of minute pollens that become trapped in sediments. Analysis of samples can
identify which kinds of plants produced the preserved pollens and when they were deposited,
and in many cases the findings can serve to supplement or correct the documentary record.
For example, analysis of samples from a bay in Jammu has revealed significant
patterns of cereal-grain pollens beginning by about fourth century. The substantial clay
content of the soil in this part of Jammu makes cultivation by primitive tools difficult.
Historians thought that such soils were not tilled to any significant extent until the
introduction of the wooden plough to India in the seventh century. Because cereal cultivation
would have required tiling of the soil, the pollens evidence indicates that these soils must
indeed have been successfully tilled before the introduction of the new plough.
Another example concerns flax cultivation in Jammu, one of the great linen-producing
areas of India during the sixteenth century. Some aspects of linen production in Jammu are
well documented, but the documentary record tells little about the cultivation of flax, the
plant from which linen is made, in that area. The record of sixteenth-century linen production
in Jammu, together with the knowledge that flax cultivation had been established in India
centuries before that time, led some historians to surmise that this plant was being cultivated
in Jammu before the sixteenth century. But pollens analyses indicate that this is not the case;
flax pollens were found only in deposits laid down since the sixteenth century.
It must be stressed, though, that there are limits to the ability of the pollen record to
reflect the vegetative history of the landscape. For example, pollen analysis cannot identify
the species, but only the genus or family, of some plants. Among these is turmeric, a
cultivated plant of medicinal importance in India. Turmeric belongs to a plant family that also
comprises various native weeds, including Brahma Thandu. If Turmeric pollen were present
in a deposit it would be indistinguishable from that of uncultivated native species.

1. The phrase “documentary record” (para 2 and 4) primarily refers to -


(A) articles, books, and other documents by current historians listing and
analyzing all the available evidence regarding a particular historical period.

1
(B) government and commercial records, maps, and similar documents produced
in the past that recoded conditions and events of that time.
(C) documented results of analyses of fossilized pollen.
(D) the kinds and qualities of fossilized pollen grains preserved in peats and lake
muds.

2. The passage indicates that pollen analyses have provided evidence against which one
of the following views?
(A) In certain parts of Jammu, cereal grains were not cultivated to any significant
extent before the seventh century.
(B) Cereal grain cultivation began in Jammu around fourth century.
(C) In certain parts of India, cereal grains have been cultivated continuously since
the introduction of the wooden plough.
(D) Cereal grain cultivation requires successful tilling of the soil.

3. The passage indicates that prior to the use of pollen analysis in the study of the history
of the Gangetic Valley plains, at least some historians believed which one of the
following?
(A) Turmeric was not used as a medicinal plant in India until after the sixteenth
century.
(B) Cereal grain was not cultivated anywhere in India until at least the seventh
century.
(C) The history of the Gangetic Valley plains during the fourteenth and fifteenth
centuries was well documented.
(D) The beginning of flax cultivation in Jammu may well have occurred before the
sixteenth century.

4. Which of the following most accurately describes the relationship between the second
paragraph and the final paragraph?
(A) The second paragraph describes a view against which the author intends to
argue, and the final paragraph states the author’s argument against that view.
(B) The second paragraph proposes a hypothesis for which the final paragraph
offers a supporting example.
(C) The final paragraph qualifies the claim made in the second paragraph.
(D) The final paragraph describes a problem that must be solved before the
method advocated in the second paragraph can be considered viable.

5. Which one of the following most accurately expresses the main point of the passage?
(A) While pollen evidence can sometimes supplement other sources of historical
information, its applicability is severely limited, since it cannot be used to
identify plant species.
(B) Analysis of fossilized pollen is a useful means of supplementing and in some
cases correcting other sources of information regarding changes in the
Gangetic Valley plains.
(C) Analysis of fossilized pollen has provided new evidence that the cultivation of
such crops as cereal grains, flax, and turmeric had a significant impact on the
Gangetic Valley plains.
(D) Analysis of fossilized pollen has proven to be a valuable tool in the
identification of ancient plant species.

2
Directions (Q. 6 – Q. 8): In each of the following questions, a word is highlighted.
Choose the word which is a synonym of the highlighted word.

6. The systematic vilification of facts and expertise, the violent abnegation of diverse
thought, the constant blasts of paranoia-stoking crime reports and patriotic sound bites
on an inescapable news network—could this be more now?
(A) indulgence (B) denial
(C) acceptance (D) adoption

7. Her 2014 autobiography, A Fighting Chance, and recent stump speeches are
festooned in pep club spirit and folksy blandishments, cloying bits of business that
have attached themselves to her life story.

(A) cajolery (B) roughness


(C) criticism (D) bully

8. Were other international trade negotiations to be put back because of the virus — for
example those being conducted between London and Washington — Britain’s
government could start to look obdurate about the Brexit talks.

(A) sensitive (B) illusive


(C) flexible (D) callous

Directions (Q. 9 – Q. 11): Complete the following sentences with an appropriate irregular
verb and one of these phases.

(i) (ii)
I. Make a. From the jeweler
II. Choose b. on the label
III. Carry c. To the players
IV. Give d. To represent India
V. Print e. At today’s meeting
VI. Damage f. On the boulevard
VII. Take g. In the storm

9. The road repairs ……………………. might delay traffic.


(A) I–b (B) VII - c
(C) VI - g (D) III - f

10. All the bijouterie …………….. have now been recovered.


(A) VII – a (B) III - f
(C) VI - a (D) III - a

3
11. The admonition ……………………. about their behaviour on the pitch was ignored.

(A) I–a (B) VII - b


(C) VI - g (D) IV - c

Directions (Q. 12 – Q. 13): Following are the questions based on the same words used
as different parts of speech. Choose the correct matches.

12. Back

1. Noun a. The back portion of the


house is in dilapidated
condition.
2. Adverb b. In a coalition government a
number of parties back the
single largest party to form
the government.
3. Adjective c. There is a road at the back of
this theater.
4. Verb d. She has come back from
America.

(A) 1-a, 2-b, 3-c, 4-d (B) 1-d, 2-c, 3-b, 4-a
(C) 1-c, 2-d, 3-a, 4-b (D) 1-b, 2-a, 3-d, 4-c

13. Near

1. Adjective a. Draw near while I speak to


you.
2. Verb b. There is a mango tree near
our house.
3. Adverb c. Lajwanti is a near relative of
mine.
4. Preposition d. I am nearing the end of the
given work.

(A) 1-c, 2-d, 3-a, 4-b (B) 1-d, 2-c, 3-b, 4-a
(C) 4-c, 1-b, 2-d, 3-a (D) 3-c, 2-b, 1-a, 4-d

Directions (Q. 14 – Q. 16): Observe the following sentences where some changes are made
in the sentence but keeping the sense of the sentence same. In the following questions, the
sentences have some element of similarity. You have to find out the similarity and choose the
option which is odd one out.

4
14. (A) His behaviour displeased his officers.
His officers were displeased at his behaviour.
(B) One must respect one’s elders.
One’s elders must be respected.
(C) I said, “Do not speak of the past.”
I advised him not to speak of the past.
(D) A crash radio message was handed over to me.
They handed over a crash radio message to me.

15. (A) I don’t expect to see him back here.


I don’t expect that I will see him back here.
(B) In spite of his poverty, he is satisfied.
He is poor but he is satisfied.
(C) He gave them not only food but some money also.
Besides food, he gave him some money also.
(D) Escaping arrest, he ran away.
He ran away in order to escape arrest.

16. (A) The teacher was strict but always loving.


Though the teacher was strict, she was always loving.
(B) He is very rich and can buy a car.
He is so rich that he can buy a car.
(C) He ran hard but missed the bus.
Although he ran, he missed the bus.
(D) You know what my errand is.
You know my errand.

Direction (Q. 17 – Q. 22): Choose the sentence which is correct grammatically.

17. (A) “Mr. Sharma has conveyed his heart-felt thanks to the Principal of the school
and its management for their support to the cause of children with special
needs.”
(B) “The government was adviced to take immediate steps”.
(C) This colt will make a good mare.
(D) Is your mother the executrix of this deed?

18. (A) You have fallen in bad company.


(B) Please run through the book.
(C) He set every thing to naught.
(D) He is calling you a bad name.

19. (A) All of the reptiles lay eggs.


(B) Waiting in the queue for half an hour, Jay suddenly realised that he had left his
wallet at home.
(C) ‘Do you think Meena will remember your birthday?’ ‘I suspect not.’
(D) The coffee in this coffee shop is the best one in town.

20. (A) He has a shave every morning, but you wouldn’t think he had.
(B) Supposing you don’t get the job – what will you do then?
(C) The strikes were mainly concerned about working conditions.

5
(D) She is quite younger than me.

21. (A) Because I’d lost my watch, so I was late for the meeting.
(B) She lives in 38 Middle Street.
(C) He’d left his papers all across the room.
(D) There have been many problems with the new bridge.

22 (A) I had to go into work even though I was feeling terrible.


(B) Competition entries must be received until 12.00 on 10 August.
(C) Why on earth were you waving that knife around? What were you thinking
about?
(D) Do you care if I smoke a cigar?

Direction (Q. 23 – Q. 26): Choose the alternative which best expresses the meaning of
the idiom/ phrase in the question.

23. Purple Patch


(A) a dark period (B) a period of success
(C) safe way to royalty (D) mending old ties

24. French Leave


(A) an insulting defeat (B) a time of revelry
(C) leave without permission (D) a great deception

25. Hang out to dry


(A) to desert one in troubling situation (B) to ridicule
(C) a time of truce (D) be critical of

26. Down to the wire


(A) digging deep (B) no chance of error
(C) in great details (D) until the last moment

Directions (Q. 27 – Q. 29): Replace the underlined word/words with one of these two-
or three-word verbs in an appropriate form.

27. If I tell you the secret, you must promise not to tell anyone else.
(A) let in on (B) bring in
(C) put down to (D) shoot down

28. They’re going to suffer a lot of criticism for increasing bus fares by so much.
(A) put down to (B) gather up
(C) come in for (D) put up with

29. It’s best to attribute his bad mood to tiredness and just forget it.
(A) flick through (B) put down to
(C) take on (D) see through

Direction (Q. 30 – Q. 35): Choose the best word to fill in the blanks

6
30. Owing to the combination of its proximity and ………………. atmosphere, Mars is the
only planet in our solar system whose surface details can be discerned from the Earth.
(A) viscous (B) ossified
(C) rarefied (D) copious
31. Using the hardships of the Ganguli family as a model, Jhumpa Lahiri’s The Namesake
effectively demonstrated how one clan’s struggles epitomized the …………. experienced
by an entire community.
(A) reticence (B) quiescence
(C) verisimilitude (D) tribulation
32. The Pyramid of Giza is more than just ……………. edifice; this imposing structure was
built to create a chirping echo whenever people clap their hands on the staircase.
(A) a venerable (B) a specious
(C) a prosaic (D) a humble
33. Some wealthy city-dwellers become enchanted with the prospect of trading their hectic
schedules for a bucolic life in the countryside, and they buy property with a pleasant view
of farmland – only to find the stench of the livestock so ………….. that they move back
to the city.
(A) bovine (B) atavistic
(C) olfactory (D) noisome
34. When Sheena brought home the irascible puppy, her more quiescent dogs were rattled by
their new …………….. housemate.
(A) languid (B) bellicose
(C) diminutive (D) phlegmatic
35. Despite the vast amount of time Tarun dedicated to learning six different languages, he
was ……………… communicator; his mastery of vocabulary and grammar failed to
redress his inability to redress his inability to construct cogent prose.
(A) an astute (B) a prolific
(C) a maladroit (D) a florid

SECTION – B : GENERAL KNOWLEDGE

36. Bru Settlement is regarding Bru refugee crisis and it was entered between:
(A) Central Government, Tripura, Mizoram and Reang Tribe
(B) Central Govt. and Indian Coffee Trade Association
(C) Tripura, Mizoram and Reang Tribe
(D) Central Government, Indian Coffee Trade Association and Reang Tribe

37. Which Movie won the Best Picture in 92nd Academy Awards 2020?
(A) Joker (B) Parasite

7
(C) Jojo Rabbit (D) Once Upon a Time in Hollywood

38. Which State in India tops the Sustainable Development Goal (SDG) India Index 2019?
(A) Kerala (B) Gujarat
(C) Andhra Pradesh (D) Himachal

39. Which of the following statements is not correct about the Blue Dot Network (BDN)?
(A) Blue Dot network is an initiative to grade the infrastructure projects in terms of
debt, environment safety, labour standards, etc.
(B) The Blue Dot Network project is being led by USA. Japan and Australia.
(C) BDN offer public funds or loans for the project.
(D) It is expected to serve as a globally recognised evaluation and certification system
with a focus on the Indo-Pacific region.

40. Who among the following is not a recipient of Padma Vibhushan Award 2020?
(A) Sushma Swaraj (B) George Fernandez
(C) Arun Jaitley (D) Teejan Bai

41. Which of the following from India is included in the SCO list of eight wonders of the
world?
(A) Soho House (B) Statue of Unity
(C) Signature Bridge (D) Ajanta Caves

42. Which of the following statements on Bond Yields is /are true?


I. Yield from Government Bond is always lower than yields from corporate bonds.
II. When the economy is in recession, the Bond Yields usually tends to decrease.
III. Government Bonds are known as Treasury Bills in India.
IV. When the Bond price increases then Bond Yield decreases.

(A) II, IV (B) I, III


(C) I, II, III (D) I, II, III, IV

43. Which of the following first-ever talked about the concept of ‘Sustainable Development’?
(A) Earth Summit 1992 (B) Kyoto Protocol
(C) Paris Summit (D) Brundtland Report

44. In Global Democracy Index 2019, which of the following criteria is the main reason for
India’s sliding ranking?
(A) Electoral process and pluralism
(B) Civil liberties
(C) Functioning of government
(D) Political participation

45. From which of the following sites, the female skeleton 16614 is recovered whose DNA
sequencing has revealed that the people in the Indus Valley Civilization have an
independent origin?
(A) Ganveriwala (B) Mohenjo-Daro

8
(C) Dholavira (D) Rakhigarhi

46. Tishreen Revolution of 2019 is regarding:


I. Corruption
II. unemployment
III. inefficient public services
V. to stop Iranian intervention in Iraq

(A) I, II (B) II, III


(C) I, IV (D) I, II, III, IV

47. The term Cytokine Storm is related to which of the following?


(A) Earth’s Weather (B) Human body’s Immune system
(C) Atomic energy (D) Space Exploration

48. What is the main reason of people’s protest in Hong Kong in 2019-20?
(A) Unemployment (B) Economic slow down
(C) Extradition law (D) labour laws

49. Which of the following is/are not true about the Organization of Islamic Cooperation
(OIC)?
I. India is an observer to OIC.
II. Only Muslim majority states are members to the body.
III. It is the second largest inter-governmental organization after the United
Nation.
IV. The recent summit of the OIC was held in Mecca, Saudi Arabia.

(A) I, II, III, IV (B) I, II, III


(C) I, II (D) III, IV

50. Who won the Ramon Magsaysay Award 2019 for Journalism?
(A) Ravish Kumar (B) Dibang
(C) Sudhir Chaudhary (D) Rahul Kanwal

51. Who is the writer of the patriotic song Saare Jahan Se Accha ?
(A) Allama Mohd. Iqbal (B) Rabindra Nath Tagore
(C) Bankim Chandra Chatterjee (D) Kavi Pradeep
52. Apple has started a credit card named “Apple Card” in collaboration with which of the
following companies?
(A) MasterCard (B) CitiBank
(C) American Express (D) Goldman Sachs

53. India has signed the Singapore Convention on Mediation in 2019. It is regarding -
(A) Cross-border river disputes
(B) Cross-border commercial disputes
(C) Terrorism
(D) War crimes

54. Consider the following statements:

9
I. The Administer, appointed by the President, is the constitutional head of the
Union Territory.
II. The relationship of the Union Territories with the Central Government is a part of
the federal structure.
Which of the abovementioned statements is/are correct?
(A) I only (B) II only
(C) I, II (D) None

55. Where is the Nine Dash Line located?


(A) South China Sea (B) North Atlantic Ocean
(C) Arctic Ocean (D) East Philippine Sea

56. What is the duration of Solar Cycle?


(A) 100 years (B) 6 months
(C) 11 years (D) 12 months

57. Which city in the World is declared the Most Liveable City in 2019?
(A) Melbourne (B) Vienna
(C) Sydney (D) Osaka

58. The Central Board of Secondary Education has collaborated with which of the following
companies for developing Artificial Intelligence (AI) based tools for learning in schools?
(A) IBM (B) Google
(C) Intel (D) Wipro

59. Who has said “How dare you? You have stolen my dreams, my childhood with your
empty word.”?
(A) Malala Yousafzai (B) Greta Thunberg
(C) Jaden Anthony (D) Martinez

60. Sound cannot travel through


(A) Gas (B) Liquid
(C) Metal (D) Vacuum

61. The chemical component which is found in most of the virus is -


(A) Protein (B) Lipids
(C) DNA (D) RNA

62. Ajanta Caves depicts paintings and sculptures from which of the following?
(A) Panchtantra Tales (B) Animals
(C) Jataka Tales (D) dance forms

63. In which of the following countries, a law to criminalize fake news came into effect in
October 2019?
(A) Singapore (B) Finland
(C) China (D) India

64. ‘Singularity’ has been in the news in the context of the Black Hole image. It refers to
which of the following?

10
(A) Gravitational waves generated by the merger of Black Holes.
(B) The boundary around a Black Hole, beyond which no light or other radiation can
escape.
(C) It is a stage in the life-cycle of a star.
(D) It is the centre of the Black Hole, which contains a huge mass in an infinitely small
space, where density and gravity become infinite.

65. Who has been elected as the Speaker of the 17th Lok Sabha?
(A) Rjiv Gauba (B) Om Birla
(C) Adhir Ranjan Chowdhary (D) Sumitra Mahajan

66. Who among the following became youngest ever elected Member of Parliament in India?
(A) Dushyant Chautala (B) Tejasvi Surya
(C) Chandrani Murmu (D) Raksha Khadse

67. Who has been appointed India’s first Lokpal?


(A) Justice (retd.)Pinaki Chandra Ghose (B) Shri. Suresh Mathur
(C) Justice (retd.) A. D. Koshal (D) Justice (retd.) A.K. Sikri

68. Who has been declared the winner of the Nobel Peace Prize, 2019?
(A) Abhijit Banerjee
(B) Abdel Fattah el-Sisi, President of Egypt
(C) Greta Thunberg, Climate Activist
(D) Abiy Ahmed Ali, the Prime Minister of Ethiopia

69. The Neelakurinji flower, which grows after a period of 12 years, grows in which of the
following areas?
(A) The Andaman and Nicobar Islands (B) Lakshadweep
(C) The Western Ghats (D) Arunachal Pradesh

70. Which of the following Statements correctly relates to ‘Pink Tax’?


(A) It is a tax legally imposed on women in some countries.
(B) It is a tax imposed legally, the proceeds of which go for the development of
women.
(C) It is the name of the phenomenon wherein women pay more for women
specific goods and services.
(D) It is a kind of tax relief given for women specific products.

SECTION – C : LEGAL APTITUDE

Directions: Given below is a statement of legal principle followed by a factual situation.


Apply the principle to the facts given below and select the most appropriate answer.

Directions (Q. 71 – Q. 74): Apply the legal principles to the facts given below and select
the most appropriate answer.

Legal Principles:

11
1. Negligence is a legal wrong that is suffered by someone at the hands of another who
has a duty to take care but fails to take proper care to avoid what a reasonable person
would regard as a foreseeable risk.
2. The test of liability requires that the harm must be a reasonably foreseeable result of
the defendant's conduct, a relationship of proximity must exist and it must be fair, just
and reasonable to impose liability.
3. The claimant must prove that harm would not have occurred 'but for' the negligence
of the defendant.
4. Duty of care is a legal obligation which is imposed on an individual requiring
adherence to a standard of reasonable care while performing any acts that could
foreseeably harm others.
5. Conversations between a doctor and patient are generally confidential but there are
few exceptions.

71. A company called KLM, manufacturers of electrical equipment, was the target of a
takeover by ABS Industries. KLM was not doing well. In March 2019, KLM had issued a
profit warning, which had halved its share price. In May 2019, KLM's directors made a
preliminary announcement in its annual profits for the year up to March. This confirmed
that the position was bad. The share price fell again. At this point, ABS had begun buying
up shares in large numbers. In June 2019, the annual accounts, which were done with the
help of the accountant Dinesh, were issued to the shareholders, which now included ABS.
ABS reached a shareholding of 29.9% of the company, at which point it made a general
offer for the remaining shares, as the City Code's rules on takeovers required. But once it
had control, ABS found that KLM's accounts were in an even worse state than had been
revealed by the directors or the auditors. It sued Dinesh for negligence in preparing the
accounts and sought to recover its losses. This was the difference in value between the
company as it had and what it would have had if the accounts had been accurate. Which
of the following answers in incorrect?

(A) No duty of care had arisen in relation to existing or potential shareholders. The
only duty of care the auditor`s owed was to the governance of the firm.
(B) Dinesh is not liable as it is a case of pure economic loss in the absence of
contractual agreements between parties.
(C) There are circumstances where an auditor will owe a duty of care in respect of
reports produced. These are conditional that at the time the report is prepared
it is known by the auditors that the results are for a specific class and for a
specific purpose.
)D( An ability to foresee indirect or economic loss to another person as the result
of a defendant’s conduct automatically impose on the defendant a duty to take
care to avoid that loss.

72. In 2005, the local council of Delhi approved building plans for the erection of a block
of apartments. The approved plans showed the base wall and concrete foundations of the
block to be three feet or deeper to the approval of local authority. The notice of approval
said that the bylaws of the council required that notice should be given to the council both
at the commencement of the work and when the foundations were ready to be covered by
the rest of the building work. The council had the power to inspect the foundations and

12
require any corrections necessary to bring the work into conformity with the bylaws, but
was not under an obligation to do so.
The block of apartments was finished in 2006. The builder (who was also the owner)
granted 99-year leases for the apartments, the last conveyance taking place in 2010. In
2017 structural movements occurred resulting in failure of the building comprising cracks
in the wall, sloping of the floors and other defects. In 2019, the plaintiffs who were
lessees of the apartments filed cases for negligence against the builder and the council.
The plaintiffs claimed that the damage was a consequence of the block having been built
on inadequate foundations, there being a depth of two feet six inches only as against the
three feet or deeper shown on the plans and required under the bylaws. The plaintiffs
claimed damages in negligence against the council for approving the foundations and/or
in failing to inspect the foundations. Decide whether the council owed a duty of care to
the claimants in respect of the incorrect depth of the foundations laid by the third-party
builder?
(A) The Council is not liable for damages to the plaintiff as failing to inspect would not
render the council liable unless it was considered that it had failed to properly exercise
its discretion to inspect and that they had failed to ensure proper compliance with
building regulations.
(B) The Council is liable for negligence as they failed to inspect the foundation.
(C) There is no negligence in building the apartments as there is minor difference
between a foundation which is three feet deep and a foundation which is two feet six
inches deep.
(D) The tenets has a duty to inspect the property properly before entering into such a long
lease agreement.

73. Soman was the student of PRQ University. He met Pamela in a youth festival and fell in
love with her. However, Pamela was not interested in having any serious relationship
with Soman. Due to this, Soman went into emotional crisis and started consulting a
psychologist in the PRQ Memorial Hospital. In October 2018, Soman murdered Pamela.
Pamela’s parents contended that only a short time prior, Soman had expressed his
intention to murder their daughter to his therapist, Dr. Surana, a psychologist employed
by the University. They further alleged that Dr. Surana had warned campus police of
Soman’s intentions, and that the police had briefly detained him, but then released him.
Pamela’s parents filed a case of negligence against the Police Department and the
University officials on two grounds: the failure to confine Soman, in spite of his
expressed intentions to kill Pamela, and failure to warn Pamela or her parents. Defendants
maintained that they owed no duty of care to the victim, and were immune from suit.
Which of the following is incorrect?

(A) The police did not have the requisite proximity or special relationship with family of
Pamela, sufficient to impose a duty to warn her of Soman’s intention.
)B( The public policy favoring protection of the confidential character of patient
psychotherapist communications must yield to the extent to which disclosure is
essential to avert danger to others. The protective privilege ends where the public peril
begins.
)C( The therapists and Regents of University are liable for breach of duty to exercise
reasonable care.

13
)D( Soman only once expressed the desire to kill Pamela. Such kinds of feelings are
normal in any mentally ill patient. Moreover, information received during a
counselling session is confidential in nature and so therapists cannot reveal it to the
parents of Pamela.

74. R, T and U were watchmen in Skypark Society. They were on night shift and began
vomiting after drinking tea. They went to the SEM Hospital and complained to the nurse
about it. The nurse thought they were vomiting because of alcohol they had been drinking
earlier in the evening. However, the nurse reported it to the medical officer who refused
to examine them and said that they needed to go home and contact their own doctors.
They returned to their workplace, where U’s condition deteriorated. U died of arsenic
poisoning five hours later on way to hospital.

U’s wife brought a claim of negligence against the Hospital administration. She argued
that the hospital was negligent in not identifying that U had been poisoned, and the doctor
should therefore have seen to him when they first approached the hospital. The hospital
denied they were negligent, and in any event said they did not cause his death. Decide.

(A) The hospital is not liable for negligence because even if the patient was examined five
hours earlier to the death he would have died anyways. The test of causation was not
satisfied. The Hospital did not cause U’s death – But for the defendant’s negligence,
U would have died anyways.
(B) It was highly possible that the doctor would have identified U’s condition as arsenical
poisoning, and therefore U would have received the treatment he needed to survive.

(C) Where there are a number of possible causes, the claimant must still prove the
defendant's breach of duty caused the harm or was a material contribution.

)D( Both (A) & (C)

75. Legal Principle: Generally, the owner of the property has a duty to maintain his property
so as to make it reasonably safe for use. However, the occupier also owes a duty to take
such care as is reasonable to see that the visitor is reasonably safe in using the premises
for the purposes for which he is invited or permitted by the occupier to be therein.

Facts: Sheila is a painter. She went to her friend Ruchi’s house for meeting her. Sheila
requested to use the bathroom and injured her right hand on a broken water faucet handle.
Sheila filed a personal injury action for hand injuries suffered alleging that Ruchi failed to
warn her that her bathroom fixtures were cracked and dangerous. Ruchi says she had
complained to the landlord about the broken handle so the landlord is liable. Decide
whether the Sheila’s injury the proximate cause of Ruchi's negligence?

(A) A licensee or social guest was obliged to take the premises as he or she found them,
and the possessor of the premises owed a duty only to refrain from wanton or wilful
injury.
(B) The landlord is liable as Ruchi had complained to the landlord about the broken
handle and it is the duty of the landlord to get the repair work done.

14
(C) Ruchi is not liable as the use of toilet is not the purposes for which Sheila was invited
or permitted by the occupier to be therein.
(D) Ruchi owes a duty to warn of a dangerous condition so the guest can take special
precautions, like the host would, when they come in contact with it.

76. Legal Principle: Intimidation involves a threat to do something unlawful or


'illegitimate'; it must be intended to coerce the claimant to take or not take certain
action.

Facts: Hari, a skilled draughtsman and employee of the Overseas Airways


Corporation (OAC), resigned his membership of the Association of Engineering and
Shipbuilding Draughtsmen (AESD), a registered trade union. It was agreed between
OAC and AESD (among others) that no strike or lockout should take place and
disputes should be handled by arbitration. He resigned from his union, the Association
of Engineering and Shipbuilding Draftsman (AESD), after a disagreement. The
Corporation and AESD had a contract that stipulates that the employer will only hire
workers from a specific union and those workers can only remain with that employer
while they are a part of the union so AESD threatened a strike unless Hari resigned
also from his job or was fired. Corporation suspended Hari and, after some months,
dismissed him with one week's salary in lieu of notice. Hari brought an action for
damages alleging that he was the victim of a tortious intimidation. Decide.

(A)The union was guilty of the tort of intimidation. It was unlawful intimidation to
use a threat to break their contracts with their employer as a weapon to make him do
something which he was legally entitled to do but which they knew would cause loss
to Hari.
(B)The Union was not guilty of intimidation as no unlawful means were used to
induce Corporation to terminate his contract of service.
(C) There was a contract between Union and Corporation that stipulates that the
employer will only hire workers from a specific union and those workers can only
remain with that employer while they are a part of the union so the Union is not liable.
)D( Hari cannot claim damages as he was paid one week’s salary in lieu of notice.

77. Legal Principles:

1. A deceit occurs when a misrepresentation is made with the express intention of


defrauding a party, subsequently causing loss to that party.
2. “Misrepresentation” means and includes— the positive assertion, in a manner not
warranted by the information of the person making it, of that which is not true,
though he believes it to be true; any breach of duty which, without an intent to
deceive, gains an advantage of the person committing it, or any one claiming
under him, by misleading another to his prejudice, or to the prejudice of any one
claiming under him; causing, however innocently, a party to an agreement, to
make a mistake as to the substance of the thing which is the subject of the
agreement.

15
Fact: XY Company in its prospectus stated that the company was permitted to make
engines that were powered by electricity, rather than by fuel. In reality, the company did
not possess such a right as this had to be approved by the Government Board. Gaining the
approval for such a claim from the Board was considered a formality in such
circumstances and the claim was put forward in the prospectus with this information in
mind. However, the claim of the company for this right was later refused by the Board.
The individuals who had purchased a stake in the business, upon reliance on the
statement, brought a claim for deceit against the defendant’s business. Decide.

(A) The company is liable for false representation as their claims were eventually turned
out to be false.
(B) The company is liable as their false statements has resulted in causing loss to the
shareholders.
(C) The company is not liable as the statement in its prospectus was simply incorrect and
not fraudulent.
(D) The shareholders should have collected as much information regarding the company
as possible before purchasing a stake in it.

78. Legal Principles:

1. Private nuisance is a continuous, unlawful and indirect interference with the use or
enjoyment of land, or of some right over or in connection with it.
2. Generally, nuisances cannot be justified on the ground of necessity, pecuniary
interest, convenience, or economic advantage to a defendant.
3. A person is liable if he can reasonably foresee that his acts would be likely to injure
his neighbour.
4. In cases of nuisance, the court may grant an injunction restricting the nuisance from
occurring in the future when the loss could not adequately compensated.

Facts: Tina purchased a house in an estate which was adjacent to a functioning, in use,
cricket field. The members of Super Eleven Cricket Club used to play Cricket in that field
for over 70 years. After Tina moved into the property, cricket balls began to fly over the
field’s protective barrier and into the Tina’s property. Tina complained, which caused
Super Eleven Cricket Club to erect a chain link fence. This improved matters as less balls
were now flying onto the Tina’s property but it did not fully solve the issue as some still
got through. The club offered Tina to pay for any damage done or injuries received as a
result of the balls landing onto her land, including fixing any broken windows and
similar. Tina, however, refused all of the club’s offers and filed a case against the
members of the Club alleging nuisance and negligence and requested court to grant an
injunction to prevent the club from playing cricket on their ground. Tina argued that even
though the club offered to make good any damage and that there had been no injuries, she
was not able to use her garden when matches were being played for fear of being struck
by a cricket ball. Decide.

(A) The members of Club are not liable as Tina was aware about the activities of the
Cricket Club and had willingly purchased the property.

(B) The members of the Club are liable for nuisance and court should pass an order of
compensation as the injury is small and could be compensated in terms of money.

16
Also, public interest considerations outweighed the private rights of the plaintiff and
therefore a remedy of damages was sufficient in the circumstances.

(C) The members of the Club are liable for nuisance and court should pass an order of
injunction. The plaintiff’s right to enjoyment of her property outweighs the right of
the members of the Club to play cricket.

(D) The Club is not liable as they have already taken sufficient measures to mitigate the
effects of their act and are ready and willing to do so in future too.

79. Legal Principles:

1. According to rule of strict liability, any person who for his own purposes brings on his
lands and collects and keeps there anything likely to do mischief if it escapes, must
keep it in at his peril, and, if he does not do so, is prima facie answerable for all the
damage, irrespective of fault, which is the natural consequence of its escape in respect
of the non-natural use of land.
2. A person is liable if he can reasonably foresee that his acts would be likely to injure
his neighbour.

Facts: PN was the owner of a gas pipe which passed under the surface of an old railway
between Ramnagar and Kotpur. XY was the local council which was responsible for a
water pipe which supplied water to a block of flats in the nearby Shining Apartment
Complex. A leak developed which was undetected for some time. The water collected at
an embankment which housed PN’s high pressure gas main. The water caused the
embankment to collapse and left the gas main exposed and unsupported. This was a
serious and immediate risk and PN took action to avoid the potential danger. They then
sought to recover the cost of the remedial works. PN argued that the XY Council was
liable for negligence under strict liability.

(A) The Council is liable under strict liability rule as the damage is not remote as it
was possible for the Council to reasonably foresee a leakage which would
eventually lead to collapse of the gas main.
)B( The escape of water as a result of leak is sufficient to make the Council liable.
)C( The Council is not liable as PN should have been careful in detecting the leak
earlier. They cannot shift the blame on the Council.
(D) The Council is not liable under rule of strict liability for the damage as the
Council’s use was neither a non- natural nor dangerous use of the land.

80. Legal Principles:


1. When the negligent act of two or more person results in the same damage, it is
called composite negligence. The liability in such a case is joint and several of the
tort-feasers.
2. A person is liable if he can reasonably foresee that his acts would be likely to
injure someone.
3. The foreseeability of the type of damage is a pre-requisite of liability.
4. The claimant must prove that harm would not have occurred 'but for' the
negligence of the defendant.

17
Facts: Zara filed a civil suit against five drug manufacturing companies. Zara’s
mother took synthetic estrogen while pregnant with her. As a result of receiving the
drug in-utero, Zara developed cancer as an adult. The drug was manufactured by the
Defendants, five major drug companies and by about 195 other companies not named
in the suit. The Defendants together produced 90% of the drug. Zara is unable to
identify which company produced the actual drug her mother took. Decide whether
the Defendants only can be held liable for Zara’s cancer.

(A) No, as the industry responsible for the production of this drug is large, so
holding only the defendants responsible is not correct.
)B( Yes, as defendants joins a substantial share of the manufactures into the
lawsuit, the chances of the actual tort-feasor escaping liability is greatly
reduced.
)C( No, as defendants can be made liable if he can reasonably foresee that his acts
would be likely to injure someone.
)D( No, Zara is not entitled to any damages as the drugs her mother was
administered were needed at that time and her mother had taken the medicines
voluntarily. Zara developed cancer after so many years and she must prove
that harm would not have occurred 'but for' the negligence of the defendants.

81. Legal Principles:


1. Any intentional false communication, either written or spoken, that harms a
person's reputation; decreases the respect, regard, or confidence in which a
person is held; or induces disparaging, hostile, or disagreeable opinions or
feelings against a person.
2. The statement must tend to lower the claimant in the estimation of right-
thinking members of society.
3. A mere vulgar abuse is not defamation.
4. Defamation encompasses both written statements, known as libel, and spoken
statements, called slander.
5. A public official or public figure can recover damages for defamation on a
matter of public concern only if he proves that the speaker acted with actual
malice.

Facts: In 2018, a police officer, Suresh Singh, shot and killed Dayal. After the officer was
convicted of culpable homicide not amounting to murder, Dayal's family retained a lawyer,
Kaushal, to represent them in civil litigation against the officer. In a magazine called Indian
Opinion, the Anti-Communist Society accused Kaushal of being a “Naxalite” and a
“Communist-fronter” because he chose to represent clients who were suing a law
enforcement officer. Because the statements contained serious inaccuracies, Kaushal filed a
libel action against the editors of the magazine. Decide whether he will succeed.
(A) No, as it is mere vulgar abuse.
(B) The statement may be defamatory but the editors of the magazine are not liable unless
Kaushal is able to establish actual malice in making the statement.
(C) No, as the statement does not tend to lower Kaushal in the estimation of right-thinking
members of society and print media is known for such type of journalism.

(D) Yes, as statement is defamatory and Kaushal is not required to establish actual malice
to successfully bring a claim of defamation.

18
82. Legal principle: A contract may, in some circumstances, be discharged by a breach
of contract. Where there exists a breach of condition this will enable the innocent
party the right to repudiate the contract (bring the contract to an end) in addition to
claiming damages.

Facts: In April, Sagar Tour &Travels agreed to employ Hiten as his courier for three
months from 1 June 2020, to go on a trip around the European continent. On 11 May,
Sagar Tour & Travels wrote to say that Hiten was no longer needed. On 22 May,
Hiten sued Sagar Tour & Travels for breach of contract. Sagar Tour & Travels
argued that Hiten was still under an obligation to stay ready and willing to perform till
the day when performance was due, and therefore could commence no action before
June 1, 2020.

(A) A breach of contract by renouncing the duty to perform the future obligation
does not render the party liable immediately to a suit of action for damages by
the injured party.

)B( The renunciation of a contract of future conduct by one party immediately


dissolves the obligation of the other party to perform the contract.
)C( Hiten has suffered no harm and the offer can be revoked anytime before June
1, 2020 so he cannot claim any damages.
)D( A contract for future conduct do not constitutes an implied promise that, in the
meantime, neither party will prejudice the performance of that promise.

83. Legal Principle: An agreement between two private parties that creates mutual legal
obligations. A contract can be either oral or written.

Facts: Mr. Ram and Mr. Rahim were work colleagues who had an arrangement
regarding shared lifts to work. Rahim would drive his motorbike and Ram would ride
pillion in return for a weekly sum of money. Unfortunately both were killed in a road
traffic accident and the wife of Mr. Ram made a claim for damages against the estate
of Mr. Rahim. However Rahim’s insurance policy did not cover pillion passengers
and as his estate had no assets or money to satisfy the judgment, Mrs. Ram pursued
the Motor Insurance Bureau (MIB).

The MIB have an agreement whereby accidents and consequential claims would be
satisfied by the Government in circumstances where the driver has no relevant policy
of insurance. However the rules covering this situation require Mr. Ram was carried
for “hire or reward”. Mrs. Ram argues that there was a contract in place between Ram
and Rahim for the lifts to work. Decide whether there was a contract so as to make
Mrs. Ram eligible for claim from MIB.

(A) Notwithstanding the regular payment of money in return for the lift, it was not
a legal obligation as to create a contract. There were no terms as to how long
this was to last, what would happen in default of payment or the availability of
transport, or anything written down so as to at least make their intention clear.
)B( Yes, there was clearly an offer of transport and this was accepted. In addition,
the consideration exchanged by the parties was the service of transport and the
money paid by Ram.

19
)C( Yes, the practice of agreements between colleagues sharing a lift to work (or
“car-pooling”) is an accepted and wide spread practice. Parties will usually
agree that one will take their car and in return the others will make a
contribution towards the petrol costs.
)D( No, unless the wife of Ram can show that the accident happened due to rash
and negligent driving of Rahim.

84. Legal Principles:


1. Offer is a proposal made by one person to another to do an act or abstain from
doing it. The person who makes the offer is known as the promisor or offeror and
the person to whom an offer is made is known as the promisee or the offeree.
2. A contract comes into being by the acceptance of an offer. When the person to
whom the offer is made signifies his consent thereto, the proposal is said to be
accepted and the parties are at consensus ad idem regarding the terms of the
agreement.

Facts: Mr. Kumar visited the Holiday Bliss Hotel. He had not made an in advance
booking and upon arrival requested a room for the night. He signed the register and
there was no mention at that stage of any other terms or conditions that might impact
upon his stay at the hotel. During the course of his stay Mr. Kumar discovered that
someone had broken into his room and stolen certain property including a coat.
Kumar filed a case of negligence on the Hotel administration. Nevertheless, the Hotel
sought to rely upon an exclusion clause that was placed in the bedroom the claimant
stayed in. This stated that the hotel would not accept liability for lost or stolen items
belonging to customers. Decide whether the exclusion clause that was displayed in the
bedroom constituted a valid term of the contract.

(A) The contract was made when Kumar signed the register at the reception and so
the acceptance of the offer mean acceptance of all the terms of the offer.
)B( There is a valid contract between Kumar and Hotel and the Hotel has taken
reasonable steps to bring exclusion clause to Kumar’s attention in the room.
)C( Terms must be brought to the attention of the customer, consumer or party
against whom they are trying to be enforced at the moment the contract was
entered into. Kumar was not given notice of this exclusion clause until he had
already entered into the contract and therefore it was unenforceable against
him.

(D) Though the terms must be brought to the attention of the customer, consumer
or party against whom they are trying to be enforced at the moment the
contract was entered into. But, such type of clauses are generally part of all
contracts and customers should be aware of such exclusion of liability clauses.

85. Legal Principles:

1. When one person signify to another person his willingness to do or not do


something (abstain) with a view to obtain the assent of such person to such an act
or abstinence, he is said to make a proposal or an offer.
2. The communication of the offer is complete when it comes to the knowledge of
the person to whom it is made.

20
3. Communication of acceptance is complete when it is put in the course of
transmission to him as to be out of the power of the acceptor to withdraw the same
and when it comes to the knowledge of the proposer.
4. In case of the proposer, the communication of the acceptance is complete when he
puts such acceptance in the course of transmission.
5. The communication in case of the acceptor is complete when the proposer
acquires knowledge of such acceptance.
6. An offer may be revoked at any time before the communication of its acceptance
is complete as against the proposer, but not afterwards.

Facts: G offered to sell the L fleeces of wool for a certain price. G requested that the
response be made by post. This letter was misdirected by G so that it was not received
for 3 days after it was sent. L decided to accept the offer and responded on the same
day. This was posted on the 5th July but not received until the 9th July. However, G
decided on the 8th July that as he had not received a response so decided to sell the
wool to someone else. But L argued that a contract had been created as he had
accepted their offer. Decide.

(A) No contract was formed as L accepted the offer even before he received the
offer by post. Contract will be entered upon when L had received the offer and
then written to him saying that the terms were agreed.
)B( The contract was entered on the 9th September when the acceptance was
received by G.
)C( G is free to withdraw his offer before 9th September.
)D( The contract was entered on the 5th September when the acceptance was
posted, not when it was received.

86. Legal Principles:

1. In order to convert a proposal into a promise the acceptance must be absolute and
unqualified.
2. Acceptance must be expressed in some usual and reasonable manner, unless the
proposal prescribes the manner in which it is to be accepted. If the proposal
prescribes a manner in which it is to be accepted, and the acceptance is not made
in such manner, the proposer may, within a reasonable time after the acceptance is
communicated to him, insist that his proposal shall be accepted in the prescribed
manner, and not otherwise; but, if he fails to do so, he accepts the acceptance.

Facts: TUV’s employee performing installation on Motorola’s premises was injured due
to the negligence of Motorola employees. TUV had executed a purchase order that
contained an indemnity form. First purchase order contained indemnity provision which
narrowly applied to damages caused by the negligence of TUV’s employees. It attached a
broader indemnity form page which would make TUV also responsible for the negligence
of Motorola employees in connection with the work. This page was marked VOID.
Amendment 2 to Purchase order contained same provision and attached same additional
indemnity form which this time was not marked VOID. But also contained the additional
language that “acceptance should be executed on acknowledgement copy which should
be returned to the buyer.” Employee was injured several months before the
acknowledgement copy of the second purchase order was executed, but was in the course
of performing work related to the second purchase order. Decide whether TUV liable

21
under the broader indemnity provision.

(A) TUV is liable as workers have sustained injury during the course of their employment
even though TUV has not accepted the amended purchase order as it did not execute the
acknowledgement.

(B) Motorola’s amendment gave a suggested mode of acceptance which did not preclude
TUV’s acceptance by another method. TUV accepted when TUV undertook performance
of the work called for by the amendment with the “consent and acquiescence” of
Motorola.

(C) TUV is not liable under the broader indemnity provision as it did not execute the
acknowledgement copy until several months after the employee sustained his injury.

(D) TUV is not liable as commencement of work was acceptance to the first purchase
order in which the broader indemnity provision was marked as void.

87. Legal Principles:

1. A contract can become void when: It is unfairly one-sided; it goes against public
policy; its subject matter is illegal; it is impossible to perform; it unfairly restricts one
side's actions (such as the right to work); one of the parties is not legally competent to
enter into a binding contract.
2. A contract is void as against public policy if: (1) it is a contract by the defendant to
pay the plaintiff for inducing a public official to act in a certain manner; (2) it is a
contract to do an illegal act; or (3) it is a contract that contemplates collusive bidding
on a public contract.

Facts: BR Industries, a company manufacturing drills, machine parts and components


thereof and a purchaser of subcontract work from other suppliers, won the bid from the
HLK Company to supply certain parts to it at a specified price. BR industries then
contracted with SU Co. to supply the parts under the contract for a much lower price. BR
Industries then intended to keep the difference between the amount it billed the HLK
Company and the amount SU Co. charged for the parts. BR Industries initiated an action
for breach of contract when SU Co. failed to complete the order. In its defense, SU Co.
asserts that the contract is void as against public policy because Defendant turned a profit
of 84.09% on anvils, 39.13% on holder primers and 68.33% on plunger supports. Did
plaintiff receive too much compensation deeming it unconscionable and against public
policy?

(A) The contract is void as against public policy as it is a contract that


contemplates collusive bidding on a public contract.
(B) The Contract is void as it unfairly one sided.
)C( Relative values of the consideration in a contract between business men at
“arms-length” without fraud will not affect the validity of the contract.
)D( Relative values of the consideration in a contract between business men will
affect the validity of the contract as it amounts to abuse of dominance and
unconscionable.

22
88. Legal Principles:
1. Offer is a proposal made by one person to another to do an act or abstain from
doing it. The person who makes the offer is known as the promisor or offeror and
the person to whom an offer is made is known as the promisee or the offeree.
2. A contract comes into being by the acceptance of an offer. When the person to
whom the offer is made signifies his consent thereto, the proposal is said to be
accepted and the parties are at consensus ad idem regarding the terms of the
agreement.

Facts: In Dec. 2019, a convicted murderer who was sentenced to death escaped from
the custody of Ramesh, a prison official. Ramesh later offered a reward of INR
50,000 to anyone who captured the fugitive and returned him to the authorities. In
Jan. 2020, without knowledge or notice of the reward, Sunil captured the fugitive and
took him to Ramesh's jail house. Ramesh refused Sunil's demands for the reward
money. Sunil filed a case against Ramesh to recover the reward. Ramesh alleges that
there is no contract between Ramesh and Sunil.

(A) A mere offer or promise to pay did not give rise to a contract. Rather, the
assent or meeting of two minds gave rise to a contract, and therefore it was not
complete until the offer was accepted. Having notice or knowledge of the
existence of the reward when he captured the fugitive is essential to his right
to recover the reward offered by Ramesh.
)B( The act of capturing the fugitive was acceptance of the offer of reward through
conduct and so a valid contract is entered upon.
)C( There was an offer by Ramesh and acceptance by Sunil and it is immaterial
whether Sunil had notice or knowledge of the existence of the reward when he
captured the fugitive.
)D( Such an offer, like the reward here, could be accepted by anyone who
performs the service called for, when the acceptor knows that it has been made
and acts in performance of it.

89. Which of the following States have passed a Bill providing for life imprisonment and fine
up to INR 5 Lakhs against accused in case of mob lynching leading to death of the
victim?
(A) Madhya Pradesh (B) Rajasthan
(C) Uttar Pradesh (D) Haryana

90. The strength of Judges in Supreme Court of India has been increased from 31 to ………. .
(A) 34 (B) 38
(C) 33 (D) 35

91. In September 2019, High Court of …………. has held that the right to have access to the
internet is part of fundamental right to education and right to privacy under Article 21 of
the Constitution of India.
(A) Delhi (B) Madhya Pradesh
(C) Kerala (D) Mumbai

23
92. Which of the following statements is/are correct regarding Delimitation Commission?
I. It determines the number and boundaries of constituencies to make population
of all constituencies nearly equal.
II. The orders of Delimitation Commission have the force of law and can be
challenged in a court of law.
III. Constitution of India has put a freeze on fresh delimitation until 2025.
IV. The constitution has also capped the number of Lok Shaba & Rajya Sabha
seats to a maximum of 550 & 250 respectively

(A) I, II, III, IV (B) I, II, III


(C) II, III (D) I, IV

93. The Enemy Property Act applies to the property of which of the following?
(A) people who took citizenship of China & Pakistan
(B) people who took citizenship of Bangladesh, China & Pakistan
(C) people who took citizenship of Bangladesh & China
(D) people who took citizenship of China & Myanmar

94. Epidemic Diseases Act, 1897 provides for which of the following powers to prevent the
spread of an epidemic disease?

I. Inspection of person
II. detention of persons
III. penalties for disobeying provisions of the Act under Section 188, 269 & 271
Indian Penal Code
IV. legal protection of implementing officers

(A) I, II, III, IV (B) I, II, III


(C) I, II, IV (D) I, III, IV

95. Jammu and Kashmir Reorganisation (Adaptation of State Laws) Second Order 2020
under J&K Civil Services (Decentralization and Recruitment) Act provides for which of
the following:
I. Anyone who has lived in the Union Territory of J&K for at least 10 years is a
domicile.
II. Anyone who has studied there for at least seven years and appeared in Class X or
Class XII examinations in any school in the region is a domicile.
III. Anyone who is registered as a migrant by the Relief and Rehabilitation
Commissioner (Migrants) in the Union territory of Jammu and Kashmir is a
domicile.
IV. A domicile of the UT shall be eligible for appointment to any post with a pay
scale of not more than Level-4 under the UT or any local authority within the
territory.

(A) II, III (B) I, II, III


(C) I, IV (D) II, III, IV

24
96. Which of the following statements regarding Anti-defection Law is/are incorrect?

I. Grounds of disqualification on ground of defection are provided in the 10th


Schedule to the Constitution.
II. If 1/3rd of the members of the political party defect from it than it is not defection.
III. Decision of the Speaker regarding disqualification of a member under Anti-
Defection Law cannot be challenged in a court of law.
IV. A legislator is deemed to have defected if he disobeys the directives of the party
leadership on a vote.

(A) I, II, (B) I, II, IV


(C) II, III (D) I, II, III, IV

97. Which of the following statements is/are correct regarding the Essential Commodities
(Amendment) Ordinance, 2020?
I. The Ordinance seeks to increase competition in the agriculture sector and
enhance farmers’ income.
II. The Ordinance requires that imposition of any stock limit on certain specified
items must be based on price rise.
III. The provisions of the Ordinance regarding the regulation of food items and the
imposition of stock limits will apply to any government order relating to the
Public Distribution System or the Targeted Public Distribution System.
IV. Recently, sanitizers, masks and oxygen cylinders are included under the Act
till June 30, 2020.

(A) I, II, III (B) I, II


(C) III, IV (D) only I

98. Recently, the Supreme Court of India has held that women officers are also entitled to
Permanent Commission in ……………… .
(A) Indian Navy (B) Indian Army
(C) Indian Air Force (D) all of these

99. In February 2020, the Supreme Court has held that reservations in promotion, in
government jobs, is not a fundamental right and refused to give directions to provide
reservations to the government of which of the following States?
(A) Uttarakhand (B) Jharkhand
(C) Kerala (D) Delhi

100. Which of the following statements is incorrect regarding SC and ST (Prevention of


Atrocities) Amendments Act?
(A) No arrest can be made without prior permission.
(B) No anticipatory bail can be granted to the accused under the Act.
(C) The Police must file an FIR and arrest the accused on receiving the complaint.
(D) The offences under the Act are cognizable.

25
101. Which of the following statements is/are correct regarding the Citizenship
(Amendment) Act 2019?
I. The Act does not apply to tribal areas of Tripura, Mizoram, Assam and
Meghalaya but apply to all other states and UTs in India.
II. The Act provides that that no order of cancellation of registration shall be
passed unless the Overseas Citizen of India Cardholder has been given a
reasonable opportunity of being heard.
III. The Act increases the period of naturalisation for such group of persons from
six years to eight years.
IV. The Act applies to those who were “forced or compelled to seek shelter in
India due to persecution on the ground of religion”. It aims to protect such
people from proceedings of illegal migration.

(A) I, IV (B) II, III, IV


(C) I, III, IV (D) II, IV

102. Which of the following statements is/are correct regarding preventive detention?
I. National Security Act as well as Article 22, Constitution of India provide for
preventive detention in certain cases
II. Maximum duration in case of preventive detention can be till 12 months
III. In case of preventive detention, accused has the right to be informed about the
grounds of arrest at the time of arrest.
IV. In case of preventive detention, accused has no right to be represented by a
lawyer.

(A) I, II (B) III, IV


(C) I, III, IV (D) I, II, IV
103. Which of the following countries joined the International Criminal Court (ICC) in
2019?
(A) Turkey (B) Malaysia
(C) Myanmar (D) India

104. The Citizenship (Amendment) Act grants citizenship to the Hindus, Christians, Sikhs,
Buddhist, Jains and Parsis — from ……………………….who had arrived in India before
…………..
(A) Afghanistan, Pakistan and Bangladesh; 31 December 2014
(B) Afghanistan, Sri Lanka and Bangladesh; 31 December 2014
(C) Myanmar, Pakistan and Tibet; 31 December 2011
(D) Afghanistan, Pakistan and Myanmar; 31 December 2011
105. Which of the following countries have abolished flogging as a form of punishment in
April 2020?
(A) Indonesia (B) Maldives
(C) Saudi Arabia (D) Singapore

26
SECTION- D : REASONING

Direction (Q. 106 – Q. 112): Read the following information carefully and answer the
questions given below.

Sonu has to deliver seven parcels containing different items– Medicine, Book, Stationery,
Grocery, Cosmetics, Clothe and Chocolate – to houses on different floors of a building, one
parcel each to a house. The seven floors of the building are numbered consecutively 1
through 7. Delivery of parcels to houses must meet the following conditions:

i. Medicine is delivered on a lower numbered floor than Grocery.


ii. Clothe is delivered at one level below the floor at which Book is delivered.
iii. Stationery is delivered at first floor or else seventh floor.
iv. Chocolate is delivered at fourth floor.

106. There can be at most how many floors between the floor at which Medicine is
delivered and the floor at which Grocery is delivered?
(A) Two
(B) Three
(C) Four
(D) Five

107. If Clothe is delivered at floor 1, which of the following could be true?


(A) Book is delivered at a floor one below than the floor at which Grocery is
delivered.
(B) Book is delivered at a floor one below than the floor at which Cosmetics is
delivered.
(C) Medicine is delivered at a floor one below than the floor at which Stationery is
delivered.
(D) Cosmetics is delivered at a floor one below than the floor at which Book is
delivered.

108. Which one of the following is an acceptable assignment of parcels to the floors in the
building, in order from floor 1 through 7?
(A) Medicine, Clothe, Cosmetics, Chocolate, Book, Grocery, Stationery
(B) Stationery, Cosmetics, Chocolate, Clothe, Book, Medicine, Grocery
(C) Stationery, Clothe, Book, Chocolate, Grocery, Medicine, Cosmetics
(D) Medicine, Cosmetics, Grocery, Chocolate, Clothe, Book, Stationery
109. If Cosmetics is delivered at floor 2, which one of the following must be true?
(A) Medicine is delivered at a lower numbered floor than Chocolate.
(B) Chocolate is delivered at a lower numbered floor than Grocery.
(C) Grocery is delivered at a lower numbered floor than Stationery.
(D) Clothe is delivered at a lower numbered floor than Grocery.

27
110. If Cosmetics is delivered on floor 5, which one of the following is a pair of parcels
that could be delivered, not necessarily in the order given, at floors whose numbers
are consecutive to each other?
(A) Book, Grocery
(B) Clothe, Chocolate
(C) Stationery, Grocery
(D) Book, Stationery

111. If Book is delivered at one floor below the floor at which Medicine is delivered, then
which one of the following must be true?
(A) Cosmetics is delivered on fifth floor.
(B) Medicine is delivered on third floor.
(C) Stationery is delivered on first floor.
(D) Clothe is delivered on fifth floor.

112. It must be true that the lowest numbered floor on which


(A) Book can be delivered on third floor.
(B) Medicine can be delivered on second floor.
(C) Cosmetics can be delivered on third floor.
(D) Grocery can be delivered on second floor.

Directions (Q. 113 – Q. 116): Read the following information carefully to answer the
following questions.
I. ‘X + Y’ means that ‘X is the mother of Y’.
II. ‘X ̶ Y’ means that ‘X is the sister of Y’.
III. ‘X × Y’ means that ‘X is the father of Y’.
IV. ‘X ÷ Y’ means that ‘X is the brother of Y’.

113. If it is given ‘B + D × M ÷ N’, then how is M related to B?


(A) grand-daughter (B) son
(C) grand-son (D) daughter

114. Which of the following represent ‘J is the son of F’?


(A) J÷R̶T×F (B) J+R̶ T×F
(C) J÷R̶N×F (D) none of these

115. Which of the following represent ‘R is the niece of M’?


(A) M÷K×T̶ R (B) M ̶ J+R̶ N
(C) R̶ M×T÷W (D) M+T×K÷R

116. If it is given ‘B × D ̶ M’, then which of the following is true?


(A) B is the brother of M (B) B is the sister of M
(C) B is the uncle of M (D) B is the father of M

Direction (Q. 117 – Q. 121): Read the given information carefully and answer the

28
following questions.

Vidya, Umesh and Tanu are sitting around a circular table. Arun, Bina and
Chetan are sitting around the same table but two of them are not facing centre
(they are facing the direction opposite to centre). Vidya is second to the left of
Chetan. Umesh is second to the right of Arun. Bina is third to the left of Tanu.
Chetan is second to the right of Tanu. Arun and Chetan are not sitting together.

117. Which of the following is not facing centre?


(A) Bina & Arun (B) Chetan & Vidya
(C) Bina & Chetan (D) Arun & Umesh

118. Which of the following is the position of Tanu in respect of Bina?


(A) third to the right (B) second to the right
(C) third to the left (D) third to the left & right, both

119. What is the position of Vidya in respect of Chetan?


(A) second to the right (B) third to the left
(C) fourth to the right (D) fourth to the left

120. Which of the following statement is correct?


(A) Arun, Bina and Chetan are sitting together
(B) Vidya, Umesh and Tanu are sitting together
(C) Only two people are sitting between Vidya and Tanu
(D) Those who are not facing centre are sitting together

121. What is the position of Arun in respect of Umesh?


(A) second to the left (B) second to the right
(C) third to the right (D) none of these

Directions (Q. 122 – Q. 123): In each of the following questions is given a statement
followed by two conclusions. You have to assume everything in the statement to be true,
then consider the two conclusions together and decide which of these logically follows
beyond reasonable doubt from the given information.

Give answer
(A) if only Conclusion I follows
(B) if only Conclusion II follows
(C) if both Conclusions I and II follow
(D) if neither Conclusion I nor II follows

122. Statement – The use of non-conventional sources of energy will eliminate the energy
crisis in the world.

Conclusion I – Modern technology is gradually replacing the conventional source of


energy.
Conclusion II – The excessive exploitation of environment has led to the depletion of
conventional source of energy.

29
123. Statement – Computer advertisements now fill magazine pages but the real computer
revolution in India is taking place quietly and is a likely organisation of Government.

Conclusion I – Both the Central and State Government are computerizing rapidly.
Conclusion II –The Government does not fill the magazine pages with its computer
advertisements.

Direction (Q. 124 – Q. 125): Each of these questions consists of a pair of words bearing
a certian relationship. From among the options, find out the one that best illustrates a
similar relationship.

124. LOM : NMK : : PKI : ?


(A) RIG (B) RIH
(C) SHG (D) RHG

125. ABCD : OPQR : : WXYZ : ?


(A) EFGH (B) STUV
(C) KLMN (D) QRST

Direction (Q. 126 – Q. 127): Find the one that does not belong to the group.

126. (A) GMS (B) EKQ


(C) JOU (D) LRX

127. (A) RARCOT (B) NIATCRU


(C) BACGEBA (D) ILBJARN

128. If in a certain code language ‘TEMPERATURE’ is written as ‘BZQDYXVBNXZ’,


then how will ‘RAMP’ be written in that language?
(A) XVQD (B) XVDQ
(C) XQVD (D) XDVQ

129. In a certain coding system, ‘JUNE’ is written as ‘PQRS’ and ‘AUGUST’ is written as
‘WQFQMN’. How will ‘GUEST’ be written in that coding language?
(A) FQTMN (B) FPSMN
(C) FQSMN (D) FQSNM

130. A clock is set right at 5 am. The clock loses 16 minutes in 24 hours. What will be the
right time when the clock indicates 10 pm on the 3rd day?
(A) 11:15 pm (B) 11 pm
(C) 12 pm (D) 12:30 pm

131. The minute hand of a clock overtakes the hour hand at intervals of 65 minutes of the
correct time. How much does a clock gain or lose in a day?
10 10
(A) 10 143 minutes (gain) (B) 10 143 minutes (loss)
10 10
(C) 9 143 minutes (gain) (D) 9 143 minutes (loss)

30
132. If a day before yesterday was Tuesday, then what day of the week will it be on a day
after tomorrow?
(A) Monday (B) Wednesday
(C) Friday (D) Saturday

133. If 15th August, 2017 was Tuesday, then what day of the week was it on 17th
September, 2017?
(A) Sunday (B) Saturday
(C) Friday (D) Thursday

134. Tia and Renu both are walking away from point ‘X’. Tia walked 3 m and Renu
walked 4 m from it, then Tia walked 4 m North of X and Renu walked 5 m South of
Tia. What is the distance between them now?
(A) 9.5 m (B) 9m
(C) 16 m (D) 11.40 m

135. Tanaya travelled 4 km straight towards South. He turned left and travelled 6 km
straight, then turned right and travelled 4 km straight. How far is he from the starting
point?
(A) 8 km (B) 10 km
(C) 12 km (D) 18 km

Directions (Q. 136 – Q. 140): Read the following short passages and choose the best
answers to the questions that follow each passage.

136. Therapists have just completed an extensive study of recently delinquent juveniles in
order to determine which factors contributed most to the delinquency. The researchers
found that in a great majority of the cases of delinquency, the parents met, on average,
fewer than 4 times per week with their children. From this data, the therapists have
determined that a failure to spend time together with the children is a major factor
leading to delinquency.

Which one of the following, if true, would cast the most doubt on the researcher’s
hypothesis?
(A) Parents and children in the study who reported that they met each other more
than four times per week also indicated that they tended to perceive their
relationships as healthy.
(B) In many cases, people in unhappy family tend to express their displeasure by
avoiding contact with each other when possible.
(C) Most of the families in the study who were unable to meet often with each
other worked outside of the home.
(D) Many families who have a long and strong family bonding met each other
fewer than four times per week.

137. Several coaching centres have recently switched at least partially from study material
written by hand on printer paper from study material written on a computer and sent
electronically with no use of paper at all. Therefore, less printer paper will be used as
a result of these changes than would have been used if these coaching centres had
continued to use handwritten study materials.

31
Which of the following, if true, most strengthens the argument above?
(A) The amount of printer paper needed to explain the electronic study material is
less than the amount that would have been used for handwritten material.
(B) Coaching centres that used more printer paper were more likely to switch to
electronic materials than centres that used less printer paper.
(C) Some of the Institutes that have switched at least partially to electronic study
material still primarily used printer paper for other operations.
(D) More printer paper was used to create manuals for the use of electronic study
materials than was used to write handwritten materials.

138. In visual media, it’s possible to induce viewers to project their feelings onto
characters on the screen. In one study, a camera shot of a girl’s face was preceded by
images of a still river. The audience thought the girl’s face registered contentment.
When the same girl’s face was preceded by images of a mighty and violent tsunami
wave, the audience thought the girl’s face registered fear. Media news teams must be
careful to avoid such manipulation of their viewers.

Which of the following is best supported by the information in the passage?


(A) The technique for manipulating audiences described in the passage would also
work in an audio program that played dramatic music.
(B) Audiences should strive to be less gullible.
(C) Images of a still river engendered feelings of happiness in the audiences.
(D) The expression on the woman’s face was, in actuality, blank.

139. Although measuring the productivity of outsourced workers is a complex endeavor.


Company X, which relies heavily on outsourced workers, must find ways to assess
the performance of these workers. The risks to a company that does not review the
productivity of its human resources are simply too great. Last year, Company Y was
forced into receivership after its productivity declined for three consecutive
quarters.

The clauses in the italics in the above paragraph, play which of the following roles in
the argument above?
(A) The first clause in italics express a position, and the second warns against the
adoption of that position.
(B) The first clause in italics represents the author’s conclusion, and the second
supports the conclusion with an analogy.
(C) The first clause in italics states the author’s premises, and the second states the
author’s conclusion.
(D) The first clause in italics provides background information, and the second
offers evidence to contradict that information.

140. A telephonic poll conducted in two states asked respondents whether they get
adequate water during summers. Ninety-nine percent of respondents said their houses
were having running water through-out the day. The pollsters published their findings,
concluding that ninety-nine percent of all homes in India have adequate water for use.

Which of the following most accurately describes a questionable technique employed


by the pollsters in drawing their conclusions?

32
(A) The pollsters conducted the poll by telephone, thereby relying on the veracity
of respondents.
(B) The pollsters never defined the term “adequate” in terms of a specific quantity
of water.
(C) The pollsters didn’t visit respondent’s houses in person, so no measure of
adequacy of water during summers in a subject’s house was actually made.
(D) The poll assumes conditions in the two states are representative of the entire
country.

SECTION – E : MATHEMATICS

141. If the third and the tenth term of an arithmetic progression are 14 and 56, respectively,
then the arithmetic mean of the first 15 terms of the arithmetic progression is

(A) 40 (B) 44
(C) 80 (D) 34

142. Ronnie has a bag containing 7 red and 9 green balls. From it, he draws out 6 balls
simultaneously at random. The probability that 4 of them are red and the rest are
green is
24 45
(A) (B)
143 286
155 302
(C) (D)
2001 1005

143. City A is connected to City B by four highways and City B is connected to City C by
three highways. Kesari wants to travel from City A to City C via City B. In how many
ways, Kesari can do it?
(A) 4 (B) 7
(C) 10 (D) 12

144. The number of integral values of x satisfying the inequation |x| < 4 is
(A) 7 (B) 8
(C) 9 (D) 10

145. If the price of each book goes up by ₹ 5, then Asha can buy 20 books less for ₹ 1200.
Find the original price and the number of books Asha could buy at the original price.
(A) ₹ 25, 50 (B) ₹ 20, 60
(C) ₹ 15, 80 (D) ₹ 10, 90

146. If 2x = 7y = 14z, then find the value of z in terms of x and y.


(A) x + y/ x - y (B) xy/ x + y
(C) x + y + xy (D) xy – (x + y)

147. Bhiku borrowed some money from Dhanraj to admit his daughter in a reputed Law
School. He agreed to pay the interest-free loan of ₹ 60, 000 in a monthly installments
which increased by a constant amount. After the 20th installment, he found that he has
paid ¾ of the loan. If the entire loan was cleared this way in 25 installments, find out
the value of the first installment.
(A) ₹ 1560 (B) ₹ 1067
(C) ₹ 1680 (D) ₹ 1987

33
148. A vessel contains an 80% alcohol solution. 20% of the solution was removed and
replaced with water. If this process is repeated, find the percentage of alcohol which
remains in the solution.
(A) 64% (B) 51.2%
(C) 50.6% (D) 45%

149. Arsh takes up a job, wherein each working day he is given a target. For each day, he
meets the target he is paid ₹ 105 and for each day he does not meet the target he is
paid ₹ 18 less. If by the end of a month, he is paid total of ₹ 2988, then how many
days does he meets the target?
(A) 21 (B) 12
(C) 10 (D) 14

150. A fruit vendor purchases two varieties of oranges at the rates of 12 oranges for ₹ 18
and 18 oranges for ₹ 12. He mixes the two varieties in the ratio of 2 : 3 and sells the
mixed stock at a price of ₹ 144 for 10 dozens. What percentage of profit or loss does
he make?
(A) 26 ¾% (B) 34¼%
(C) 20% (D) 28%

*****

34

You might also like